Вы находитесь на странице: 1из 337

ADONIS

I. BASIC CONCEPTS OF A CONTRACT OF SALE

1) TEODORO ACAP vs. CA

FACTS:

TeodoroAcap has been a tenant of a portion of land of Lot No. 1130


HinigaranCadastre since 1960. Said lot was formerly owned by Spouses Vasquez and
LorenzaOruma, which upon their death was inherited by Felixberto. In 1975, Felixberto sold
the lot to CosmePido. Acap remained to be a registered tenant of the said land and
religiously paid his leasehold rentals to Pido and thereafter, upon his death, to his widow
Laurenciana. On 1981, Pido’s wife and children executed a notarized document
denominated “Declaration of Heirship and Waiver of Rights” of the land in favor Edydelos
Reyes. Delos Reyes alleged that he and Acap entered into an oral lease agreement
whereby Acap undertook to pay him 10 cavans of rice per year as lease rental. From
1983 onwards Acap refused to pay further lease rentals. In defense, Acap denied having
entered in an oral lease agreement with delos Reyes and that he did not recognize his
ownership over the land. As a matter of fact he alleged that he continued to pay
Laurenciana, Pido’s wife. Delos Reyes filed a suit of recovery of possession against Acap
and for the payment of rentals accruing to him as owner of the said lot. Trial court rendered
decision in favor of delos Reyes ruling that there was a perfected sale between heirs of
Pido and delos Reyes over the said lot and ordered Acap to deliver possession of
the same to delos Reyes. Upon appeal, CA affirmed the lower court’s decision. Hence, this
petition.

ISSUE:

WON waiver of hereditary rights in this case, constitutes a contract of sale.

HELD:

No. The Court noted that an asserted right or claim to ownership or a real right
over a thing arising from a juridical act, however justified, is not per se sufficient to give rise
to ownership over the res. That right or title must be completed by fulfilling certain
conditions imposed by law. Hence, ownership and real rights are acquired only
pursuant to a legal mode or process. While title is the juridical justification, mode is the
actual process of acquisition or transfer of ownership over a thing in question. Under
Article 712 of the Civil Code, modes of acquisition may either be original or
derivative. Original modes of acquisition include occupation, acquisitive prescription, law or
intellectual creation. Derivative modes of acquisition on the other hand include
succession mortis causa and tradition as a result of certain contracts such as sale,
barter, donation, assignment or mutuum. In the instant case, the Court determined
whether delos Reyes acquired ownership over the lot in question through any of the modes
mentioned. It was ruled that he had not acquired ownership by virtue of sale, as opposed to
the ruling of both RTC and CA. The execution of the heirs of Pido the Declaration of
Heirship and Waiver of Rights was held to be not tantamount to sale. Such declaration
is only one whereby heirs adjudicate and divide the estate left by the decedent among them
as they see fit. The Court further noted that waiver of hereditary rights is different from sale

SALES CASE DIGEST COMPILATION 1


of hereditary rights. Sale of hereditary rights presupposes an existence of a contract of
sale whereas waiver of hereditary rights is an abdication or intentional relinquishment of a
known right with a knowledge of its existence and intention to relinquish it in favor of
other persons who are co-heirs in the succession. As delos Reyes is a stranger to
the succession of CosmePido, he cannot claim ownership over the lot on the sole basis of
the document executed. Hence, private respondent delos Reyes had not acquired
ownership over Lot 1130 and consequently had no right to exact lease rentals from
petitioner Acap.

2) Polytechnic University vs. CA

FACTS:

Petitioner National Development Corp., a government owned and controlled


corporation, had in its disposal a 10 hectares property. Sometime in May 1965,
private respondent Firestone Corporation manifested its desire to lease a portion of it
forceramic manufacturing business. On August 24, 1965, both parties entered into a
contract of lease for a term of 10 years renewable for another 10 years. Prior to the
expiration of the aforementioned contract, Firestone wrote NDC requesting for an extension
of their lease agreement. It was renewed with an express grant to Firestone of the
first option to purchase the leased premise in the event that it was decided "to
dispose and sell the properties including the lot..."Cognizant of the impending expiration
of the leased agreement, Firestone informed NDC through letters and calls that it
was renewing its lease. No answer was given. Firestone's predicament
worsened when it learned of NDC's supposed plans to dispose the subject property in
favor of petitionerPolytechnic University of the Philippines. PUP referred to
Memorandum Order No. 214 issued by then President Aquino ordering the transfer
of the whole NDC compound to the National Government. The order of
conveyance would automatically result in the cancellation of NDC's total obligation in
favor of the National Government.Firestone instituted an action for specific performance to
compel NDC to sell the leased property for its favor.

ISSUE:

WON there is a valid sale between NDC and PUP.

HELD:

Yes. A contract of sale, as defined in the Civil Code, is a contract where one
of the parties obligates himself to transfer the ownership of and to deliver a
determinate thing to the other or others who shall pay therefore a sum certain in
money or its equivalent. It is therefore a general requisite for the existence of a valid and
enforceable contract of sale that it be mutually obligatory, i.e., there should be a
concurrence of the promise of the vendor to sell a determinate thing and the
promise of the vendee to receive and pay for the property so delivered and
transferred. The Civil Code provision is, in effect, a "catch-all" provision which
effectively brings within its grasp a whole gamut of transfers whereby ownership of a
thing is ceded for a consideration. All three (3) essential elements of a valid sale,
without which there can be no sale, were attendant in the "disposition" and "transfer"
of the property from NDC to PUP -consent of the parties, determinate subject matter,
and consideration therefore. Consent to the sale is obvious from the prefatory clauses
of Memorandum Order No. 214 which explicitly states the acquiescence of the

SALES CASE DIGEST COMPILATION 2


parties to the sale of the property. Furthermore, the cancellation of NDC's
liabilities in favor of the National Government constituted the "consideration" for the sale.

3) Manila Container vs. PNB

FACTS:

Petitioner was the owner of 8,015 square meters of parcel of land located in
Mandaluyong City, Metro Manila. To secure a P900,000.00 loan it had obtained from
respondent Philippine National Bank, petitioner executed a real estate mortgage over the lot.
Respondent PNB later granted petitioner a new credit accommodation. On August 5, 1982,
respondent PNB filed a petition for extrajudicial foreclosure of the real estate mortgage and
sought to have the property sold at public auction. After due notice and publication, the
property was sold at public action where respondent PNB was declared the winning bidder.
Petitioner sent a letter to PNB, requesting it to be granted an extension of time to
redeem/repurchase the property. Some PNB personnel informed that as a matter of policy,
the bank does not accept “partial redemption”. Since petitioner failed to redeem the property,
the Register of Deeds cancelled TCT No. 32098 and issued a new title in favor of PNB.

Meanwhile, the Special Asset Management Department (SAMD) had prepared a statement
of account of petitioner’s obligation. It also recommended the management of PNB to allow
petitioner to repurchase the property for P1,574,560.oo. PNB rejected the offer and
recommendation of SAMD. It instead suggested to petitioner to purchase the property for
P2,660,000.00, in its minimum market value. Petitioner declared that it had already agreed
to SAMD’s offer to purchase for P1,574,560.47 and deposited a P725,000.00.

ISSUE:

WON petitioner and respondent PNB had entered into a perfected contract for
petitioner to repurchase the property for respondent.

RULING:

No. The SC affirmed the ruling of the appellate court that there was no perfected
contact of sale between the parties.

A contract is meeting of minds between two persons whereby one binds himself, with
respect to the other, to give something or to render some service. Under 1818 of the Civil
Code, there is no contract unless the following requisites concur:

1. Consent of the contracting parties;

2. Objection certain which is the subject matter of the contract;

3. Cause of the obligation which is established.

Contract is perfected by mere consent which is manifested by the meeting of the offer and
the acceptance upon the thing and causes which are to constitute the contract. Once
perfected, the bind between other contracting parties and the obligations arising therefrom
have the form of law between the parties and should be complied in good faith. The absence
of any essential element will negate the existence of a perfected contract of sale.

SALES CASE DIGEST COMPILATION 3


The court ruled in Boston Bank of the Philippines vsManalo:

“A definite agreement as to the price is an essential element of a binding agreement to sell


personal or real property because it seriously affects the rights and obligations of the parties.
Price is an essential element in the formation of a binding and enforceable contract of sale.
The fixing of the price can never be left to the decision of one of the contracting parties. But
a price fixed by one of the contracting parties, if accepted by the other, gives rise to a
perfected sale.”

In the case at bar, the parties to the contract is between Manila Metal Container Corporation
and Philippine National Bank and not to Special Asset Management Department. Since the
price offered by PNB was not accepted, there is no contract. Hence it cannot serve as a
binding juridical relation between the parties.

4) Cruz vs. Fernando

FACTS:

In 1983, Cruz executed a Kasunduan with the Gloriosos for the consideration of the
rear portion of a 223 sq m lot. The Kasunduan provides that the lot will be sold at a P40 per
sq m. That the portion of the lot to be sold is the rear portion of it. That upon selling, the Cruz
will transfer their house from the front portion to the rear portion of the land once it is bought.
That they will have a right of way from the front portion going to the back end of the lot. The
Cruz never gave anything to the Gloriosos for there was an alleged failure to have the land
surveyed. Due to non payment, the Gloriosos instead sold the whole lot (back and rear
portion) to the Fernandos.

In 1994, after repeated demands, the Fernandos filed a case in court for accionpubliciana
demanding the Cruz to vacate the lot and to pay a rental of P500.00. The RTC ruled in favor
of the Fernandos. The CA affirmed the RTC ruling.

ISSUE:

WON what transpired between the Cruzes and the Gloriosos was a contract of sale.

HELD:

No. The absence of a specific manner of payment in the terms and conditions of the
contract makes it a contract to sell. Ownership was never transferred to the Cruzes. This is
because the manner of payment of the purchase price is an essential element before a valid
and binding contract of sale can exist. Although the Civil Code does not expressly state that
the minds of the parties must also meet on the terms or manner of payment of the price, the
same is needed, otherwise there is no sale. Also, the Cruzes never transferred their house
from the front portion to the rear portion of the lot. It was evident in the contract that they will
transfer the house to the rear portion once they were able to buy it.

The SC also ruled that the Fernandos were not buyers in bad faith. There was no
consummated sale between the Cruzes and the Gloriosos. In a contract to sell, there being
no previous sale of the property, a third person buying such property despite the fulfillment
of the suspensive condition such as the full payment of the purchase price, for instance,
cannot be deemed a buyer in bad faith and the prospective buyer cannot seek the relief of
reconveyance of the property. There is no double sale in such case. Title to the property

SALES CASE DIGEST COMPILATION 4


will transfer to the buyer after registration because there is no defect in the owner-seller’s
title per se, but the latter, of course, may be sued for damages by the intending buyer.

5) Delpher Trades Corp vs. IAC

FACTS:

In 1974, Delfin Pacheco and his sister, Pelagia Pacheco, were the owners of 27,169
square meters of real estate identified as Lot No. 1095, Malinta Estate, in the Municipality of
Polo (now Valenzuela), Province of Bulacan (now Metro Manila) which is covered by
Transfer Certificate of Title No. T-4240 of the Bulacan land registry.

On April 3, 1974, the said co-owners leased to Construction Components


International Inc. the same property and providing 'that during the existence or after the term
of this lease' the lessor should he decide to sell the property leased shall first offer the same
to the lessee and the latter has the priority to buy under similar conditions.On August 3,
1974, lessee Construction Components International, Inc. assigned its rights and obligations
under the contract of lease in favor of Hydro Pipes Philippines, Inc. with the signed
conformity and consent of lessors Delfin Pacheco and Pelagia Pacheco. The contracts of
lease, as well as the assignment of lease were annotated at the back of the title, as per
stipulation of the parties.

On the ground that it was not given the first option to buy the leased property
pursuant to the proviso in the lease agreement, respondent Hydro Pipes Philippines, Inc.,
filed an amended complaint for reconveyance of Lot. No. 1095 in its favor under conditions
similar to those whereby Delpher Trades Corporation acquired the property from Pelagia
Pacheco and Delfin Pacheco.

ISSUE:

WON the "Deed of Exchange" of the properties executed by the Pachecos on the
one hand and the Delpher Trades Corporation on the other was meant to be a contract of
sale.

HELD:

No. The "Deed of Exchange" of property between the Pachecos and Delpher Trades
Corporation cannot be considered a contract of sale. There was no transfer of actual
ownership interests by the Pachecos to a third party. The Pacheco family merely changed
their ownership from one form to another. The ownership remained in the same hands.
Hence, the private respondent has no basis for its claim of a right of first refusal under the
lease contract.

6) Toyota Shaw vs. CA

FACTS:

Luna L. Sosa and his son, Gilbert, went to purchase a yellow Toyota Lite Ace from
the Toyota office at Shaw Boulevard, Pasig (petitioner Toyota) on June 14, 1989 where they
met Popong Bernardo who was a sales representative of said branch. Sosa emphasized

SALES CASE DIGEST COMPILATION 5


that he needed the car not later than June 17, 1989 because he, his family, and a
balikbayan guest would be using it on June 18 to go home to Marinduque where he will
celebrate his birthday on June 19. Bernardo assured Sosa that a unit would be ready for
pick up on June 17 at 10:00 in the morning, and signed the "Agreements Between Mr. Sosa
&Popong Bernardo of Toyota Shaw, Inc.,” a document which did not mention anything about
the full purchase price and the manner the installments were to be paid. Sosa and Gilbert
delivered the down payment of P100,000.00 on June 15, 1989 and Bernardo accomplished
a printed Vehicle Sales Proposal (VSP) No. 928 which showed Sosa’s full name and home
address, that payment is by "installment," to be financed by "B.A.," and that the "BALANCE
TO BE FINANCED" is "P274,137.00", but the spaces provided for "Delivery Terms" were not
filled-up.

When June 17 came, however, petitioner Toyota did not deliver the Lite Ace. Hence,
Sosa asked that his down payment be refunded and petitioner Toyota issued also on June
17 a Far East Bank check for the full amount of P100,000.00, the receipt of which was
shown by a check voucher of Toyota, which Sosa signed with the reservation, "without
prejudice to our future claims for damages." Petitioner Toyota contended that the B.A.
Finance disapproved Sosa’s the credit financing application and further alleged that a
particular unit had already been reserved and earmarked for Sosa but could not be released
due to the uncertainty of payment of the balance of the purchase price. Toyota then gave
Sosa the option to purchase the unit by paying the full purchase price in cash but Sosa
refused.

The trial court found that there was a valid perfected contract of sale between Sosa
and Toyota which bound the latter to deliver the vehicle and that Toyota acted in bad faith in
selling to another the unit already reserved for Sosa, and the Court of Appeals affirmed the
said decision.

ISSUE:

Was there a perfected contract of sale between respondent Sosa and petitioner
Toyota?

HELD:

No. The Supreme Court granted Toyota’s petition and dismissed Sosa’s complaint
for damages because the document entitled “Agreements Between Mr. Sosa &Popong
Bernardo of Toyota Shaw, Inc.,” was not a perfected contract of sale, but merely an
agreement between Mr. Sosa and Bernardo as private individuals and not between Mr. Sosa
and Toyota as parties to a contract.

There was no indication in the said document of any obligation on the part of Toyota
to transfer ownership of a determinate thing to Sosa and neither was there a correlative
obligation on the part of the latter to pay therefor a price certain. The provision on the down
payment of P100,000.00 made no specific reference to a sale of a vehicle. If it was intended
for a contract of sale, it could only refer to a sale on installment basis, as VSP No.928
executed on June 15, 1989 confirmed. The VSP also created no demandable right in favor
of Sosa for the delivery of the vehicle to him, and its non-delivery did not cause any legally
indemnifiable injury.

7) Nabus vs. Pacson

FACTS:

SALES CASE DIGEST COMPILATION 6


The spouses Bate and Julie Nabus were the owners of parcels of land with a total
area of 1,665 square meters, situated in Pico, La Trinidad, Benguet, duly registered in their
names under TCT No. T-9697 of the Register of Deeds of the Province of Benguet. The
property was mortgaged by the Spouses Nabus to the Philippine National Bank (PNB), La
Trinidad Branch, to secure a loan in the amount of P30,000.00.

On February 19, 1977, the Spouses Nabus executed a Deed of Conditional Sale
covering 1,000 square meters of the1,665 square meters of land in favor of respondents
Spouses Pacson for a consideration of P170,000.00, which was duly notarized on February
21, 1977.

Pursuant to the Deed of Conditional Sale, respondents paid PNB the amount of
P12,038.86 on February 22, 19776 andP20,744.30 on July 17, 19787 for the full payment of
the loan.

On December 24, 1977, before the payment of the balance of the mortgage amount
with PNB, Bate Nabus died. On August 17, 1978, his surviving spouse, Julie Nabus, and
their minor daughter, Michelle Nabus, executed a Deed of Extra Judicial Settlement over the
registered land covered by TCT No. 9697. On the basis of the said document, TCT No. T-
177188 was issued on February 17, 1984 in the names of Julie Nabus and Michelle Nabus.

Meanwhile, respondents continued paying their balance, not in installments of


P2,000.00 as agreed upon, but in various, often small amounts ranging from as low as
P10.009 to as high as P15,566.00,10 spanning a period of almost seven years, from March
9, 197711 to January 17, 1984.12. There was a total of 364 receipts of payment. The
receipts showed that the total sum paid by respondents to the Spouses Nabus was
P112,455.16,14 leaving a balance of P57,544.84.

During the last week of January 1984, Julie Nabus, accompanied by her second
husband, approached Joaquin Pacsonto ask for the full payment of the lot. Joaquin Pacson
agreed to pay, but told her to return after four days as his daughter, Catalina Pacson, would
have to go over the numerous receipts to determine the balance to be paid. When Julie
Nabus returned after four days, Joaquin sent her and his daughter, Catalina, to Atty.
Elizabeth Rillera for the execution of the deed of absolute sale. Since Julie was a widow with
a minor daughter, Atty. Rillera required Julie Nabus to return in four days with the necessary
documents, such as the deed of extrajudicial settlement, the transfer certificate of title in the
names of Julie Nabus and minor Michelle Nabus, and the guardianship papers of Michelle.
However, Julie Nabus did not return.

Getting suspicious, Catalina Pacson went to the Register of Deeds of the Province of
Benguet and asked for a copy of the title of the land. She found that it was still in the name
of Julie and Michelle Nabus. After a week, Catalina Pacson heard a rumor that the lot was
already sold to petitioner Betty Tolero.

On March 28, 2008, respondents Joaquin and Julia Pacson filed with the Regional
Trial Court of La Trinidad, Benguet (trial court) a Complaint for Annulment of Deeds, with
damages and prayer for the issuance of a writ of preliminary injunction. Julie and Michelle
Nabus alleged that respondent Joaquin Pacson did not proceed with the conditional sale of
the subject property when he learned that there was a pending case over the whole
property. Joaquin proposed that he would rather lease the property with a monthly rental of
P2,000.00 and apply the sum ofP13,000.00 as rentals, since the amount was already paid to
the bank and could no longer be withdrawn. Hence, he did not affix his signature tothe
second page of a copy of the Deed of Conditional Sale.26 Julie Nabus alleged that in March

SALES CASE DIGEST COMPILATION 7


1994, due to her own economic needs and those of her minor daughter, she sold the
property to Betty Tolero, with authority from the court.

Betty Tolero put up the defense that she was a purchaser in good faith and for value.
She testified that it was Julie Nabus who went to her house and offered to sell the property
consisting of two lots with a combined area of 1,000square meters. She consulted Atty.
Aurelio de Peralta before she agreed to buy the property. She and Julie Nabus brought to
Atty. De Peralta the pertinent papers such as TCT No. T-17718 in the names of Julie and
Michelle Nabus, the guardianship papers of Michelle Nabus and the blueprint copy of the
survey plan showing the two lots. After examining the documents and finding that the title
was clean, Atty. De Peralta gave her the go-signal to buy the property

ISSUE:

WON the Deed of Conditional Sale is a Contract of Sale.

HELD:

No. The Court holds that the contract entered into by the Spouses Nabus and
respondents was a contract to sell, not a contract of sale.

A contract of sale is defined in Article 1458 of the Civil Code, thus:


Art. 1458. By the contract of sale, one of the contracting parties
obligates himself to transfer the ownership of and to deliver a determinate
thing, and the other to pay therefor a price certain in money or its equivalent.
xxx
Sale, by its very nature, is a consensual contract because it is
perfected by mere consent. The essential elements of a contract of sale are
the following:
a) Consent or meeting of the minds, that is, consent to
transfer ownership in exchange for the price;
b) Determinate subject matter; and
c) Price certain in money or its equivalent.

Under this definition, a Contract to Sell may not be considered as a


Contract of Sale because the first essential element is lacking. Furthermore, It
is not the title of the contract, but its express terms or stipulations that
determine the kind of contract entered into by the parties. In this case, the
contract entitled “Deed of Conditional Sale” is actually a contract to sell

Unfortunately for the Spouses Pacson, since the Deed of Conditional Sale executed
in their favor was merely a contract to sell, the obligation of the seller to sell becomes
demandable only upon the happening of the suspensive condition. The full payment of the
purchase price is the positive suspensive condition, the failure of which is not a breach of
contract, but simply an event that prevented the obligation of the vendor to convey title from
acquiring binding force.

SALES CASE DIGEST COMPILATION 8


II. STAGES IN THE LIFE OF A CONTRACT OF SALE
1) San Miguel Properties vs. Sps. Huang

FACTS:

San Miguel Properties offered two parcels of land for sale and the offer was made to
an agent of the respondents. An “earnest-deposit” of P1 million was offered by the
respondents and was accepted by the petitioner’s authorized officer subject to certain terms.
Petitioner, through its executive officer, wrote the respondent’s lawyer that because ethe
parties failed to agree on the terms and conditions of the sale despite the extension granted
by the petitioner, the latter was returning the “earnest-deposit”.

The respondents demanded execution of a deed of sale covering the properties and
attempted to return the “earnest-deposit” but petitioner refused on the ground that the option
to purchase had already expired.

A complaint for specific performance was filed against the petitioner and the latter
filed a motion to dismiss the complaint because the alleged “exclusive option” of the
respondents lacked a consideration separate and distinct from the purchase price and was
thus unenforceable; the complaint did not allege a cause of action because there was no
“meeting of the mind” between the parties and therefore the contact of sale was not
perfected.

The trial court granted the petitioner’s motion and dismissed the action. The
respondents filed a motion for reconsideration but were denied by the trial court. The
respondents elevated the matter to the Court of Appeals and the latter reversed the decision
of the trial court and held that a valid contract of sale had been complied with.

ISSUE:

WON there was a perfected contract of sale between the parties

RULING:

No. The decision of the appellate court was reversed and the respondents’ complaint
was dismissed.It is not the giving of earnest money , but the proof of the concurrence of all
the essential elements of the contract of sale which establishes the existence of a perfected
sale. The P1 million “earnest-deposit” could not have been given as earnest money because
at the time when petitioner accepted the terms of respondents’ offer, their contract had not
yet been perfected. This is evident from the following conditions attached by respondents to
their letter.

The first condition for an option period of 30 days sufficiently shows that a sale was
never perfected. As petitioner correctly points out, acceptance of this condition did not give
rise to a perfected sale but merely to an option or an accepted unilateral promise on the part
of respondents to buy the subject properties within 30 days from the date of acceptance of
the offer. Such option giving respondents the exclusive right to buy the properties within the
period agreed upon is separate and distinct from the contract of sale which the parties may
enter. All that respondents had was just the option to buy the properties which privilege was
not, however, exercised by them because there was a failure to agree on the terms of
payment. No contract of sale may thus be enforced by respondents.

Even the option secured by respondents from petitioner was fatally defective. Under
the second paragraph of Art. 1479, an accepted unilateral promise to buy or sell a

SALES CASE DIGEST COMPILATION 9


determinate thing for a price certain is binding upon the promisor only if the promise is
supported by a distinct consideration. Consideration in an option contract may be anything of
value, unlike in sale where it must be the price certain in money or its equivalent. There is no
showing here of any consideration for the option. Lacking any proof of such consideration,
the option is unenforceable.

In the present case, the parties never got past the negotiation stage. The alleged
“indubitable evidence” of a perfected sale cited by the appellate court was nothing more than
offers and counter-offers which did not amount to any final arrangement containing the
essential elements of a contract of sale. While the parties already agreed on the real
properties which were the objects of the sale and on the purchase price, the fact remains
that they failed to arrive at mutually acceptable terms of payment, despite the 45-day
extension given by petitioner.

SALES CASE DIGEST COMPILATION 10


III. ESSENTIAL CHARACTERISTICS OF A CONTRACT

1) Quijada vs. CA

FACTS:

On April 5, 1956, Trinidad Quijada and her sisters executed a deed of conditional
donation in favor of the Municipality of Talacogon, the condition being that the land shall be
used exclusively for the construction of a provincial high school. Trinidad remained in
possession of the land. On July 29, 1962, Trinidad sold the land to respondent
RegaladoMondejar. In 1980, the heirs of Trinidad, herein petitioners, filed a complaint for
forcible entry against the respondent. In 1987, the proposed campus did not materialize, and
the Sangguniang Bayan enacted a resolution donating back the land to the donor. In the
meantime, respondent Mondejar conveyed portions of the land to the other respondents. On
July 5, 1988, petitioners filed a complaint for quieting of title, recovery of possession and
ownership of the land.

ISSUE:

WON the sale between Trinidad and Regalado is valid considering the capacity of
the vendor to execute the contract in view of the conditional deed of donation.

RULING:

Yes. However, The donor may have an inchoate interest in the donated property
during the time that ownership of the land has not reverted to her. Such inchoate interest
may be the subject of contracts including a contract of sale. In this case, however, what the
donor sold was the land itself which she no longer owns. It would have been different if the
donor-seller sold her interests over the property under the deed of donation which is subject
to the possibility of reversion of ownership arising from the non-fulfillment of the resolutory
condition.

Sale, being a consensual contract, is perfected by mere consent, which is manifested


the moment there is a meeting of the minds as to the offer and acceptance thereof on three
(3) elements: subject matter, price and terms of payment of the price. Ownership by the
seller on the thing sold at the time of the perfection of the contract of sale is not an element
for its perfection. What the law requires is that the seller has the right to transfer ownership
at the time the thing sold is delivered. Perfection per se does not transfer ownership which
occurs upon the actual or constructive delivery of the thing sold. A perfected contract of sale
cannot be challenged on the ground of non-ownership on the part of the seller at the time of
its perfection; hence, the sale is still valid.

The consummation, however, of the perfected contract is another matter. It occurs


upon the constructive or actual delivery of the subject matter to the buyer when the seller or
her successors-in-interest subsequently acquires ownership thereof. Such circumstance
happened in this case when petitioners who are Trinidad Quijada's heirs and successors-in-
interest became the owners of the subject property upon the reversion of the ownership of
the land to them. Consequently, ownership is transferred to respondent Mondejar and those
who claim their right from him. Article 1434 of the New Civil Code supports the ruling that the
seller's "title passes by operation of law to the buyer." This rule applies not only when the
subject matter of the contract of sale is goods, but also to other kinds of property, including
real property.

SALES CASE DIGEST COMPILATION 11


2) Laforteza vs. Machuca

FACTS:

Roberto Laforteza and Gonzalo Laforteza, Jr., in their capacities as attorneys-in-fact


of Dennis Laforteza, entrered into a MOA (Contract to Sell) with Alonzo Machuca over a
house and lot registered in the name of the late Francisco Laforteza. Machuca was able to
pay the earnest money but however failed to pay the balance on time. Upon a request of an
extension of time, Machuca informed petitioner heirs that the balance was already covered,
but petitioners refused to accept the balance and told Machuca that the subject property is
no longer for sale. The petitioners contend that the Memorandum of Agreement is merely a
lease agreement with “option to purchase”; hence, it only gave the respondent a right to
purchase the subject property within a limited period without imposing upon them any
obligation to purchase it. And since the respondent’s tender of payment was made after the
lapse of the option agreement, his tender did not give rise to the perfection of a contract of
sale.

ISSUE:

WON the tender of payment after the lapse of the option agreement gave rise to the
perfection of a contract of sale.

HELD:

Yes. It did. A perusal of the Memorandum Agreement shows that the transaction
between the petitioners and the respondent was one of sale and lease.

A contract of sale is a consensual contract and is perfected at the moment there is a


meeting of the minds upon the thing which is the object of the contract and upon the price.
From that moment the parties may reciprocally demand performance subject to the
provisions of the law governing the form of contracts. In the case at bench, all the elements
of a contract of sale were thus present.

3) Villanueva vs. PNB

FACTS:

The Special Assets Management Department (SAMD) of the Philippine National


Bank (PNB) issued an advertisement for the sale of certain PNB properties in
Calumpang, General Santos City, including Lots 17 and 19 with advertisedfloor prices
of P1,409,000.00 and P2,268,000.00 respectively.Villanueva offered to purchase the lots
for P3,677,000.00. He also manifested that he was depositing P400,000.00 to show
his good faith but with the understanding that said amount may be treated as part of the
payment of the purchase price only when his offer is accepted by PNB. At the bottom of
said letter there appears an unsigned marginal note stating that P400,000.00 was
deposited into Villanueva's account with PNB-General Santos Branch.PNB forwarded the
letter of Villanueva to Ramon Guevara, Vice President, SAMD. Guevara informed
Villanueva that only Lot No. 19 is available and that the asking price therefor is
P2,883,300.00. Guevara further wrote:If our quoted price is acceptable to you,
please submit a revised offer to purchase. Sale shall be subject to our Board of
Director's approval and to other terms and conditions imposed by the Bank on sale of
acquired assets. Instead of submitting a revised offer, Villanueva merely inserted at the
bottom of Guevara's letter a marginal note, which reads:CONFORME:PRICE OF
P2,883,300.00 (downpayment of P600,000.00 and the balance payable in two (2) years at
quarterly amortizations.)Villanueva paid P200,000.00 to PNB which the latter issued

SALES CASE DIGEST COMPILATION 12


areceipt to acknowledge receipt of the "partial payment deposit on offer to purchase."
On the dorsal portion of Official Receipt No. 16997, Villanueva signed a typewritten note,
stating:This is a deposit made to show the sincerity of my purchase offer with
the understanding that it shall be returned without interest if my offer is not
favorably considered or be forfeited if my offer is approved but I fail/refuse to push
through the purchase.Thereafter, however, Guevara wrote Villanueva that SAMD is
deferring negotiations with him over said property and returning his deposit of
P580,000.00.Undaunted, Villanueva attempted to deliver postdated checks covering the
balance of the purchase price but PNB refused the same.

Thus Villanueva filed a Complaint for specific performance which the RTC granted
anchoring its judgment on the finding that there existed a perfected contract of sae
between PNB and Villanueva.PNB appealed to the CA which reversed and set aside
the decision, stating that in the case at bench, consent, in respect to the price and
manner of its payment, is lacking. The record shows that appellant, thru Guevara's July
6, 1990 letter, made a qualified acceptance of appellee's letter-offer dated June 28,
1990 by imposing an asking price of P2,883,300.00 in cash for Lot 19. The letter dated July
6, 1990 constituted a counter-offer (Art. 1319, Civil Code), to which appellee made a new
proposal, i.e., to pay the amount of P2,883,300.00 in staggered amounts, that is,
P600,000.00 as downpayment and the balance within two years in quarterly
amortizations.CA held that a qualified acceptance, or one that involves a new
proposal, constitutes a counter-offer and a rejection of the original offer (Art.
1319). Consequently, when something is desired which is not exactly what is
proposed in the offer, such acceptance is not sufficient to generate consent because
any modification or variation from the terms of the offer annuls the offer.

ISSUE:

WON there was a perfected Contract of Sale between respondents PNB and herein
petitioner Villanueva.

HELD:

No. Contracts of sale are perfected by mutual consent whereby the seller obligates
himself, for a price certain, to deliver and transfer ownership of a specified thing or right
to the buyer over which the latter agrees. Mutual consent being a state of mind, its
existence may only be inferred from the confluence of two acts of the parties: an
offer certain as to the object of the contract and its consideration, and an
acceptance of the offer which is absolute in that it refers to the exact object and
consideration embodied in said offer. While it is impossible to expect the acceptance
to echo every nuance of the offer, it is imperative that it assents to those points in
the offer which, under the operative facts of each contract, are not only material but
motivating as well. Anything short of that level of mutuality produces not a contract
but a mere counter-offer awaiting acceptance. More particularly on the matter of the
consideration of the contract, the offer andits acceptance must be unanimous both on the
rate of the payment and on its term. An acceptance of an offer which agrees to the rate
but varies the term is ineffective.Tracing the transactions and letters between
Villanueva and PNB, it can be said thatthere was no perfected contract of sale between the
parties. The first letter of PNB stating that only Lot 19 was available was certainly not an
acceptance but a mere counter-offer. Further, such counter-offer imposed two more
conditions that Villanueva submit a revised offer to purchase based on the new price
and that such sae of property be approved by the Board of Directors. However,
Villanueva’s reply to said counter-offer was not an acceptance but a further counter-
offer since he qualified his acceptance proposing a two-year payment.

SALES CASE DIGEST COMPILATION 13


4) Fule vs. CA and Belarmino

FACTS:

Petitioner Gregorio Fule, a banker by profession and a jeweler, acquired a 10-


hectare property in Tanay, Rizal. In 1984, he met private respondent Dr. Ninevetch Cruz
who was interested on the said lot. It so happened that at the time, petitioner had also
shown interest in buying a pair of emerald-cut diamond earrings owned by Dr. Cruz.

Subsequently, an agreement for the barter of the jewelry and the Tanay property
ensued. Petitioner prepare the documents of the deed of absolute sale while Dr. Cruz
attended to the safekeeping of the jewelry. Dr. Cruz got the earrings from her safety deposit
box and handed it to petitioner, who, when asked if those were alright, nodded and took the
earrings. Two hours after, petitioner Fule alleged that the earrings he received were fake. He
filed a complaint to declare the sale of property null and void on the ground of fraud and
deceit on the part of the respondent.

ISSUE:

WON the contract of sale should be nullified on the ground of fraud.

HELD:

No. There is fraud when, through the insidious words or machinations of one of the
contracting parties, the other is induced to enter into a contract which, without them, he
would not have agreed to.

The records, however, are bare of any evidence manifesting that private respondents
employed such insidious words or machinations to entice petitioner into entering the contract
of barter. Neither is there any evidence showing that Dr. Cruz induced petitioner to sell his
Tanay property or that she cajoled him to take the earrings in exchange for said property. On
the contrary, Dr. Cruz did not initially accede to petitioner’s proposal to buy the said jewelry.
It was in fact petitioner who resorted to machinations to convince Dr. Cruz to exchange her
jewelry for the lot. On account of the petitioner’s work as a banker-jeweler, it can be rightfully
assumed that he was an expert on matters regarding gems. He had the intellectual capacity
and the business acumen as a banker to take precautionary measures to avert such a
mistake, considering the value of both the jewelry and his land; that the finger of suspicion of
switching the genuine jewelry for a fake inevitably points to him.

There were no legal bases for the nullification of the contract of sale. Ownership over
the parcel of land and the pair of emerald-cut diamond earrings had been transferred to Dr.
Cruz and petitioner, respectively, upon the actual and constructive delivery thereof.

5) Vda. De Ape vs. CA

FACTS:

Cleopas Ape died in 1950 and left a parcel of land (Lot 2319) to his 11 children. The
children never formally divided the property amongst themselves except through hantal-
hantal whereby each just occupied a certain portion and developed each.

SALES CASE DIGEST COMPILATION 14


On the other hand, the spouses Lumayno were interested in the land so they started
buying the portion of land that each of the heirs occupied. On 11 Apr 1973, one of the
children, Fortunato, entered into a contract of sale with Lumayno. In exchange of his lot,
Lumayno agreed to pay P5,000.00. She paid in advance P30.00. Fortunato was given a
receipt prepared by Lumayno’s son in law (Andres Flores). Flores also acted as witness.
Lumayno also executed sales transactions with Fortunato’s siblings separately.

In 1973, Lumayno compelled Fortunato to make the the delivery to her of the
registrable deed of sale over Fortunato’s portion of the Lot No. 2319. Fortunato assailed the
validity of the contract of sale. He also invoked his right to redeem (as a co-owner) the
portions of land sold by his siblings to Lumayno. Fortunato died during the pendency of the
case.

ISSUE:

WON there was a valid contract of sale

HELD:

No. Fortunato was a “no read no write” person. It was incumbent for the the other
party to prove that details of the contract was fully explained to Fortunato before Fortunato
signed the receipt.

A contract of sale is a consensual contract, thus, it is perfected by mere consent of


the parties. It is born from the moment there is a meeting of minds upon the thing which is
the object of the sale and upon the price. Upon its perfection, the parties may reciprocally
demand performance, that is, the vendee may compel the transfer of the ownership and to
deliver the object of the sale while the vendor may demand the vendee to pay the thing sold.
For there to be a perfected contract of sale, however, the following elements must be
present: consent, object, and price in money or its equivalent.

For consent to be valid, it must meet the following requisites:

(a) it should be intelligent, or with an exact notion of the matter to which it refers;

(b) it should be free and

(c) it should be spontaneous. Intelligence in consent is vitiated by error; freedom by


violence, intimidation or undue influence; spontaneity by fraud.

Lumayno claimed that she explained fully the receipt to Fortunato, but Flores’
testimony belies it. Flores said there was another witness but the other was a maid who also
lacked education. Further, Flores himself was not aware that the receipt was “to transfer the
ownership of Fortunato’s land to her mom-in-law”. It merely occurred to him to explain the
details of the receipt but he never did.

SALES CASE DIGEST COMPILATION 15


ERIK
6) ANTONIO R. CORTES vs. CA

FACTS:

For the purchase price of P3,700,000.00, private respondent Villa Esperanza


Development Corporation (Corporation) as buyer, and Antonio Cortes (Cortes) as seller,
entered into a contract of sale over the subject lots located at Baclaran. On various dates in
1983, the Corporation advanced to Cortes the total sum of P1,213,000.00.

Sometime in September 1983, the parties executed a deed of absolute sale


containing several terms (DP=P2.2M, P1.5M payable in 1 year). Said Deed was retained by
Cortes for notarization. The parties agreed that the Corporation would pay the balance of the
downpayment when Cortes delivers the TCTs.

In 1985, the Corporation filed a case for specific performance seeking to compel
Cortes to deliver the TCTs and the original copy of the Deed of Absolute Sale. According to
the Corporation, despite its readiness and ability to pay the purchase price, Cortes refused
delivery of the sought documents. On the other hand, Cortes claimed that the owner’s
duplicate copy of the three TCTs were surrendered to the Corporation and it is the latter
which refused to pay in full the agreed down payment. He added that portion of the subject
property is occupied by his lessee who agreed to vacate the premises upon payment of
disturbance fee. However, due to the Corporation’s failure to pay in full the sum of
P2,200,000.00, he in turn failed to fully pay the disturbance fee of the lessee who now
refused to pay monthly rentals.

RTC rescinded the sale and directed Cortes to return to the Corporation the amount
of P1,213,000.00, plus interest. CA reversed the RTC decision and directed Cortes to
execute a Deed of Absolute Sale conveying the properties and to deliver the same to the
Corporation together with the TCTs, simultaneous with the Corporation’s payment of the
balance of the purchase price of P2,487,000.00.

ISSUE:

WON the parties are compelled to perform their respective obligations to the
Contract of Sale.

HELD:

CA correctly ordered the parties to perform their respective obligation in the contract
of sale. The contract of sale in question gave rise to a reciprocal obligation of the parties.
Reciprocal obligations are those which arise from the same cause, and which each party is a
debtor and a creditor of the other, such that the obligation of one is dependent upon the
obligation of the other. They are to be performed simultaneously, so that the performance of
one is conditioned upon the simultaneous fulfillment of the other.

Article 1191 of the Civil Code, states:

ART. 1191. The power to rescind obligations is implied in reciprocal ones, in case
one of the obligors should not comply with what is incumbent upon him.

SALES CASE DIGEST COMPILATION 16


As to when said failure or delay in performance arise, Article 1169 of the same Code
provides that –
ART. 1169
xxxx
In reciprocal obligations, neither party incurs in delay if the other does not comply or
is not ready to comply in a proper manner with what is incumbent upon him. From
the moment one of the parties fulfills his obligation, delay by the other begins.

In the case at bar, the stipulation in the Deed of Absolute Sale was that the
Corporation shall pay in full the P2,200,000.00 down payment upon execution of the
contract. However, Cortes admitted that he agreed that the Corporation’s full payment of the
sum of P2,200,000.00 would depend upon his delivery of the TCTs of the three lots. In fact,
his main defense is that, he performed what is incumbent upon him by delivering to the
Corporation the TCTs and the carbon duplicate of the Deed of Absolute Sale, but the latter
refused to pay in full the down payment.

By agreeing to transfer title upon full payment of P2,200,000.00, Cortes’ impliedly


agreed to deliver the TCTs to the Corporation in order to effect said transfer. Since Cortes
did not perform his obligation to have the Deed notarized and to surrender the same
together with the TCTs, the trial court erred in concluding that he performed his part in the
contract of sale and that it is the Corporation alone that was remiss in the performance of its
obligation. Actually, both parties were in delay. Considering that their obligation was
reciprocal, performance thereof must be simultaneous. The mutual inaction of Cortes and
the Corporation therefore gave rise to a compensation morae or default on the part of both
parties because neither has completed their part in their reciprocal obligation.

Thus, Cortes is yet to deliver the original copy of the notarized Deed and the TCTs,
while the Corporation is yet to pay in full the agreed down payment of P2,200,000.00. This
mutual delay of the parties cancels out the effects of default, such that it is as if no one is
guilty of delay.

7) Almocera vs Ong

FACTS:

Johnny Ong tried to acquire from Andre T. Almocera and First Builder Multi-Purpose
Cooperative (FBMC) a "townhome" in Cebu City. As reflected in a Contract to Sell, the
selling price of the unit was P3,400,000.00 pesos. Out of the purchase price, he was able to
pay the amount of P1,060,000.00.

In trying to recover the amount he paid as down payment for the townhouse unit,
Johnny Ong filed a complaint for Damages against Andre T. Almocera and FBMC alleging
that they were guilty of fraudulent concealment and breach of contract when they sold to him
a townhouse unit without divulging that the same, at the time of the perfection of their
contract, was already mortgaged with the Land Bank of the Philippines (LBP), with the latter
causing the foreclosure of the mortgage and the eventual sale of the townhouse unit to a
third person.

In their Answer, Almocera and FBMC denied liability claiming that the foreclosure of
the mortgage on the townhouse unit was caused by the failure of Johnny Ong to pay the
balance of the price of said townhouse unit.

ISSUE:

SALES CASE DIGEST COMPILATION 17


WON the respondent’s refusal to pay the balance of the purchase price is justified.

HELD:

The respondent is justified in refusing to pay the balance of the contract price.

It cannot be disputed that the contract entered into by the parties was a contract to
sell. The contract was denominated as such and it contained the provision that the unit shall
be conveyed by way of an Absolute Deed of Sale, together with the attendant documents of
Ownership – the Transfer Certificate of Title and Certificate of Occupancy – and that the
balance of the contract price shall be paid upon the completion and delivery of the unit, as
well as the acceptance thereof by respondent. All these clearly indicate that ownership of the
townhouse has not passed to respondent.

The unit shall be completed and conveyed by way of an Absolute Deed of Sale
together with the attendant documents of Ownership in the name of the BUYER – the
Transfer Certificate of Title and Certificate of Occupancy within a period of six (6) months
from the signing of Contract to Sell.

From the terms of the contract, it is clear that petitioner and FBMC had the obligation
to complete the townhouse unit within six months from the signing of the contract. Upon
compliance therewith, the obligation of respondent to pay the balance of P2,400,000.00
arises. Upon payment thereof, the townhouse shall be delivered and conveyed to
respondent upon the execution of the Absolute Deed of Sale and other relevant documents.

The evidence adduced shows that petitioner and FBMC failed to fulfill their obligation
-- to complete and deliver the townhouse within the six-month period. With petitioner and
FBMC’s non-fulfillment of their obligation, respondent refused to pay the balance of the
contract price. Respondent does not ask that ownership of the townhouse be transferred to
him, but merely asks that the amount or down payment he had made be returned to him.

The contract subject of this case contains reciprocal obligations which were to be
fulfilled by the parties, i.e., to complete and deliver the townhouse within six months from the
execution of the contract to sell on the part of petitioner and FBMC, and to pay the balance
of the contract price upon completion and delivery of the townhouse on the part of the
respondent.

In the case at bar, the obligation of petitioner and FBMC which is to complete and
deliver the townhouse unit within the prescribed period, is determinative of the respondent’s
obligation to pay the balance of the contract price. With their failure to fulfill their obligation
as stipulated in the contract, they incurred delay and are liable for damages. They cannot
insist that respondent comply with his obligation. Where one of the parties to a contract did
not perform the undertaking to which he was bound by the terms of the agreement to
perform, he is not entitled to insist upon the performance of the other party.

8) FERNANDO A. GAITE vs Fonacier

FACTS:

Isabelo Fonacier executed a ‘Deed of Assignment’ in favor of Fernando Gaite as his


true and lawful attorney-in-fact so that the latter may enter into a contract for the exploration
and development of the mining claims owned by Fonacier. Gaite executed a general
assignment conveying the development and exploitation of said mining claims to Larap Iron

SALES CASE DIGEST COMPILATION 18


Mines owned by him.

Fonacier decided to revoke the ‘Deed of Assignment’ to which Gaite assented on the
condition that Fonacier is to pay him P75,000 for the 24,000 metric tons of iron lodes already
extracted and to retain the company name Larap Iron MInes. Fonacier already paid P10,000
leaving a balance of P65,000 which, as agreed by them, is to be derived from the local sale
of Iron ore made by Larap Iron Mines. On December 8, 1954 , Fonacier issued a security
bond to secure payment of balance with Far Eastern Surety and Insurance Co. But the
surety providedthat liability to the company will only attach when there had been actual sale
of iron ore by Larap Iron Mines for an amount of not les than P65,000 and that the bond wil
automatically expire on December 8 1955.

No sale of the iron ore was made thereafter. Gaite failed to pay Fonacier the balance
and the surety company refused to pay contending that the bond expired automatically.

Gaite instituted the present case. Fonacier argued that the payment of the P65,000
balance was subject to the condition that it would be paid out of the first sale of the iron ore
by Larap Mines which did not happen.

ISSUE:

WON the obligation of Fonacier to pay Gaite the balance of P65,000 was
extinguished because the iron ore was not sold within a year.

HELD:

The shipment or local sale of the iron ore is not a condition precedent (or
suspensive) to the payment of the balance of P65,000.00, but was only a suspensive period
or term. That the sale did not occur within a period of one year did not extinguish the
obligation of Fonacier to pay Gaite the balance of P65,000 because it does not seem to be
the intention of the parties to the contract. The Court looked into several circumstances
which lead them to conclude that the sale of the iron ore is but a suspensive term. First, the
words of the contract express no contingency in the buyer's obligation to pay.

Second, in the usual course of business, an onerous contract is most likely preferred
by the parties in a sale. Nothing is found in the record to evidence that Gaite desired or
assumed to run the risk of losing his right over the ore without getting paid for it, or that
Fonacier understood that Gaite assumed any such risk. This is proved by the fact that Gaite
insisted on a bond a to guarantee payment of the P65,000.00, an not only upon a bond by
Fonacier, the Larap Mines & Smelting Co., and the company's stockholders, but also on one
by a surety company; and the fact that appellants did put up such bonds indicates that they
admitted the definite existence of their obligation to pay the balance of P65,000.00.

Assuming that there could be doubt whether by the wording of the contract the
parties indented a suspensive condition or a suspensive period (dies ad quem) for the
payment of the P65,000.00, the rules of interpretation would incline the scales in favor of
"the greater reciprocity of interests", since sale is essentially onerous. The Civil Code of the
Philippines, Article 1378, paragraph 1, in fine, provides:

“If the contract is onerous, the doubt shall be settled in favor of the greatest
reciprocity of interests.”

There can be no question that greater reciprocity obtains if the buyer' obligation is
deemed to be actually existing, with only its maturity (due date) postponed or deferred, that if
such obligation were viewed as non- existent or not binding until the ore was sold.

SALES CASE DIGEST COMPILATION 19


The only rational view that can be taken is that the sale of the ore to Fonacier was a
sale on credit, and not an aleatory contract where the transferor, Gaite, would assume the
risk of not being paid at all; and that the previous sale or shipment of the ore was not a
suspensive condition for the payment of the balance of the agreed price, but was intended
merely to fix the future date of the payment.

9) Buenaventura vs. CA

FACTS:

Defendant spouses Leonardo Joaquin and Feliciana Landrito are the parents of
plaintiffs Consolacion, Nora, Emma and Natividad as well as of defendants Fidel, Tomas,
Artemio, Clarita, Felicitas, Fe, and Gavino, all surnamed JOAQUIN.

The plaintiffs sought to have certain deeds of Sale of Real Property executed by
Spouses Leonardo and Feliciana in favor of some of their children, the herein defendants.
They allege that the said deeds are fictitious for lack of consideration or if there was
consideration, such was grossly inadequate. They further alleged that the deeds of sale
were not refelective of the true intent of parties and that it was only a deliberate conspiracy
designed to deprive the plaintiffs of their share in the inheritance.

RTC ruled in favor of the defendants and CA affirmed the ruling of RTC.

ISSUE:

WON The sale should be declared null and void for lack of consideration or if there
was a consideration it was grossly inadequate?

HELD:

Petitioners failed to show that the prices in the Deeds of Sale were absolutely
simulated. To prove simulation, petitioners presented Emma Joaquin Valdoz’s testimony
stating that their father, respondent Leonardo Joaquin, told her that he would transfer a lot to
her through a deed of sale without need for her payment of the purchase price. The trial
court did not find the allegation of absolute simulation of price credible. Petitioners’ failure to
prove absolute simulation of price is magnified by their lack of knowledge of their respondent
siblings’ financial capacity to buy the questioned lots. On the other hand, the Deeds of Sale
which petitioners presented as evidence plainly showed the cost of each lot sold. Not only
did respondents’ minds meet as to the purchase price, but the real price was also stated in
the Deeds of Sale. As of the filing of the complaint, respondent siblings have also fully paid
the price to their respondent father.

Petitioners ask that assuming that there is consideration, the same is grossly
inadequate as to invalidate the Deeds of Sale.

Art. 1355. Except in cases specified by law, lesion or inadequacy of cause shall not
invalidate a contract, unless there has been fraud, mistake or undue influence.

Art. 1470. Gross inadequacy of price does not affect a contract of sale, except as
may indicate a defect in the consent, or that the parties really intended a donation or some
other act or contract.

Petitioners failed to prove any of the instances mentioned in Articles 1355 and 1470

SALES CASE DIGEST COMPILATION 20


of the Civil Code which would invalidate, or even affect, the Deeds of Sale. Indeed, there is
no requirement that the price be equal to the exact value of the subject matter of sale. All the
respondents believed that they received the commutative value of what they gave.

10) EQUATORIAL REALTY DEVELOPMENT, vs. MAYFAIR THEATER, INC

FACTS:

In 1967 ,Mayfair entered into a 20 year lease contract with Carmelo and Bauermann,
Inc. (Carmelo) for a portion of the latter’s 2 storey building which Mayfair used as a
moviehouse. Another lease contract was executed in 1969 with the same period of 20 years.
The contract contained an agreement of a right of first refusal in favor of Mayfair. However,
in 1978 or before the end of the lease period, Carmelo sold the property to Equatorial
Realty. Mayfair filed a complaint to annul the sale between Carmelo and Equatorial which
was eventually granted by the Supreme Court (mother case). The ruling is based on the fact
that no right of ownership was transferred from Carmelo to Equatorial in view of a patent
failure to deliver the proeprty to the buyer. The decision to rescind the contract became final
and executory.

Five months after the finality of the decision and after Mayfair submitted a Motion for
Execution, Equatorial filed an action for the collection of a sum of money against Mayfair,
claiming payment of rentals or reasonable compensation for the defendant’s use of the
subject premises after its lease contracts had expired.

ISSUE:

Is Equatorial entitled to the rental payments upon the expiration of the contract of
lease between Mayfair and Carmelo?

HELD:

Rental is a civil fruit of ownership which only the ‘owner is entitled to receive. In the
mother case, it was already decided that ownership of the property did not pass on to
Equatorial because there was no delivery. Ownership is a real right that can be transferred
not only by contract but also by tradition or delivery. The execution of a public instrument of
sale is considered constructive delivery but may be negated by the failure fo the vendee to
take actual possession of the land.

In the case at bar, Mayfair and not Equatorial continued to be in actual possession of
the property. The filing of the the complaint by Mayfair served as a hindrance to the actual
delivery of the property.

Petitioner also cannot invoke Art 1146 of the Civil Code providing for reciprocal
performance because with the rescission of the contract in the mother case there arose the
obligation to return the things which were the object of the contract , together with their fruits
, and the price with its interest (Art 1385 Civil Code).

Furthermore, the general principle that ‘a rescissible contract is valid until rescinded’
is not applicable in the case because ownership was not vested to petitioner .

The sale to Equatorial may have been valid from inception, but it was judicially
rescinded before it could be consummated. Petitioner never acquired ownership, not
because the sale was void, as erroneously claimed by the trial court, but because the sale

SALES CASE DIGEST COMPILATION 21


was not consummated by a legally effective delivery of the property sold.

11) San Lorenzo Development Corp. vs CA

FACTS:

In 1986, spouses Lu allegedly sold through a verbal contract 2 parcels of land in Sta.
Rosa Laguna to Pablo Babasanta at 15 pesos/sqm. This sale was in part influenced by
Pacita Lu’s being indebted to Babasanta. Babasanta made a downpayment of 50k to Pacita
Lu, evidenced by a memorandum reciept. Here thereafter made other payments totalling
200k. Sometime after that, Babasanta asked the Lus for a reduction in the price, and the Lus
refused. Because of this, Babasanta allegedly backed out of the sale and asked that the
original loan be carried out. In 1989, Babasanta wrote to the Lus to demand the execution of
a deed of sale in his favor so that he could pay the full price, but the Lus refused.
Subsequently, Babasanta learned that the lands in question were sold to San Lorenzo Devt
Co.

In 1990 the SLDC intervened in the case, claiming that they were buyers in good
faith, since from the time that the land was offered to them and up to the time that they
bought it, they didn’t know that the lands were ‘sold’ to Babasanta. They also state that after
they had paid a total of 632k, the Lus executed a Deed of Absolute Sale with Mortgage in its
favor. Babasanta’s claims were not annotated on the titles, and furthermore, SLDC moved to
possess the property after execution of the deed, unlike Babasanta who never exercised any
form of ownership.

ISSUE:

Who has better claim to the lands?

HELD:

A contract to sell, and not a contract of sale, was perfected between Lu and
Babasanta.

The Court concluded that the contract between the Lus and Babasanta was a
contract to sell. In this case, the acts of the spouses Lu signfy that they never intended to
transfer ownership to Babasanta. This was evident in Babasanta’s repeated requests to
have the deed executed in his favor, all of which the Lus refused. The Lus could also have
executed the Deed when they received the partial payment, thus the receipt should be
considered at best, a perfected contract to sell. Even if the 50k downpayment was made in
partial payment of the lands, the Lus still reserved the ownership of the property until the
payment of the full purchase price. In contracts to sell, the payment of the purchase price
acts as a suspensive condition which, if not fulfilled, prevents the obligation of the vendor to
convey to the buyer.

Sale as mere title, not a mode.

Even if we assume that the contract between the Lus and Babasanta was a
perfected contract of sale, ownership of the land could not have passed to Babasanta in the
absence of delivery, since in a contract of sale, ownership is transferred to the vendee only
upon the delivery of the thing sold.

The perfection of a contract is different from its consummation. In relation to the

SALES CASE DIGEST COMPILATION 22


acquisition and transfer of ownership, it should be noted that sale is not a mode, but merely
a title. A mode is the legal means by which dominion or ownership is created, transferred or
destroyed, but title is only the legal basis by which to affect dominion or ownership.

Under Article 712 of the Civil Code, "ownership and other real rights over property
are acquired and transmitted by law, by donation, by testate and intestate succession, and in
consequence of certain contracts, by tradition." Contracts only constitute titles or rights to the
transfer or acquisition of ownership, while delivery or tradition is the mode of accomplishing
the same. Therefore, sale by itself does not transfer or affect ownership; the most that sale
does is to create the obligation to transfer ownership. It is tradition or delivery, as a
consequence of sale, that actually transfers ownership.

Ownership transferred though delivery

Explicitly, the law provides that the ownership of the thing sold is acquired by the
vendee from the moment it is delivered to him in any of the ways specified in Article 1497 to
1501.The word "delivered" should not be taken restrictively to mean transfer of actual
physical possession of the property. The law recognizes two principal modes of delivery, to
wit: (1) actual delivery; and (2) legal or constructive delivery.

1. Actual delivery – consists in placing the thing sold in the control and possession of the
vendee

2. Constructive delivery :

a. execution of public document evidencing the sale

b. symbolical tradition (delivery of keys to the place)

c. traditio longa manu or by mere consent or agreement if the movable cannot be


transferred

to possession of buyer at the time of sale

d. traditio brevi manu or if the buyer had already possessed the object before the
sale

e. traditio constitutum possessorium or when the seller remains in possession of the

propertyin a different capacity.

Respondent Babasanta did not acquire ownership by the mere execution of the
receipt by Pacita Lu acknowledging receipt of partial payment for the property. For one, the
agreement between Babasanta and the Spouses Lu, though valid, was not embodied in a
public instrument. Hence, no constructive delivery of the lands could have been effected. For
another, Babasanta had not taken possession of the property at any time after the perfection
of the sale in his favor or exercised acts of dominion over it despite his assertions that he
was the rightful owner of the lands.

SALES CASE DIGEST COMPILATION 23


12) JOSE B. AZNAR vs RAFAEL YAPDIANGCO

FACTS:

In 1959, Teodoro Santos advertised his Ford Fairlane 500 as for sale in 2
newspapers. An ‘L. de Dios’ inquired about the car, claiming to be the nephew of a Vicente
Marella, who wanted to buy the car. Teodoro sent his son Irineo to meet with Marella. At this
meeting, Marella agreed to buy the car for 14.7k on the understanding that the price would
be paid only after the car had been registered in his name. A deed of sale for the car was
subseuquently executed in favor of Marella, and the car was eventually registered in
Marella’s name.

However, when Irineo returned to ask for the purchase price, Vicente expained that
he was short 2k pesos. He suggested that Irineo accompany L. de Dios (and another
unidentified person) to his sister and get the money. In the meantime, he asked for the
registrations papers so as to show them to his lawyer. Irineo trusted Marella, gave him the
documents and accompanied L. de Dios to the house of the sister. They rode in the Ford
car, and Irineo gave the car keys to the unidentified third person so he could drive them
there. Eventually it was revealed that no sister lived in the house, and by the time Irineo
realized the scam, L. de Dios, Marella, and their car was gone. Father and son quickly
reported to the police. On the same day Marella was able to sell the car to Jose Aznar for
15k. While Aznar was registering the car, the police came and confiscated the car. Aznar
filed for replevin, while Santos intervened.

At the end of the trial, the lower court rendered a decision awarding the disputed
motor vehicle to the intervenor- appellee, Teodoro Santos. In brief, it ruled that Teodoro
Santos had been unlawfully deprived of his personal property by Vicente Marella, from
whom the plaintiff-appellant traced his right. Consequently, although the plaintiff- appellant
acquired the car in good faith and for a valuable consideration from Vicente Marella, the said
decision concluded, still the intervenor-appellee was entitled to its recovery on the mandate
of Article 559 of the New Civil Code which provides:

ART. 559. The possession of movable property acquired in good faith is equivalent to
title. Nevertheless, one who lost any movable or has been unlawfully deprived thereof, may
recover it from the person in possession of the same.

If the possessor of a movable lost or of which the owner has been unlawfully
deprived, has acquired it in good faith at a public sale, the owner cannot obtain its return
without reimbursing the price paid therefor.

Aznar appealed, saying that Article 1506 was applicable and not 559. Under 1506, a
buyer who buys from a seller who has voidable title, the buyer acquires good title to the
goods provided he bought them in good faith, for value and without notice of seller’s defect
of title.

ISSUE:

Who has better right to the car?

HELD:

Vicente Marella had no title at all.

SALES CASE DIGEST COMPILATION 24


Vicente Marella did not have any title to the property under litigation because the
same was never delivered to him. He sought ownership or acquisition of it by virtue of the
contract. Vicente Marella could have acquired ownership or title to the subject matter thereof
only by the delivery or tradition of the car to him.

Under Article 712 of the Civil Code, ownership is not transferred by contract merely
but by tradition or delivery. Contracts only constitute titles or rights to the transfer or
acquisition of ownership, while delivery or tradition is the mode of accomplishing the same.

In the case on hand, the car in question was never delivered to the vendee by the
vendor as to complete or consummate the transfer of ownership by virtue of the contract. It
should be recalled that while there was indeed a contract of sale between Vicente Marella
and Teodoro Santos, the former, as vendee, took possession of the subject matter thereof
by stealing the same while it was in the custody of the latter's son.

Thus, Article 1506 has no applicability as Marella doesn’t have any title to the car, let
alone a voidable title.

On Irineo giving the car keys to the 3rd person - this was not a ‘delivery’ in the eyes
of the law, as the act or surrendering a thing must be coupled with the intent to deliver the
thing. Irineo only gave the keys to the person so they could drive to the bogus address.

SALES CASE DIGEST COMPILATION 25


IV. DISTINIGUISHED FROM OTHER
TRANSACTIONS/CONTRACT
1) INOCENCIA YU DINO and her HUSBAND doing business under the trade name
"CANDY CLAIRE FASHION GARMENTS", petitioners,

vs.

COURT OF APPEALS and ROMAN SIO, doing business under the name "UNIVERSAL
TOY MASTER MANUFACTURING", respondents.

(G.R. No. 113564, June 20, 2001)

FACTS:

Petitioners and respondent Sio entered into a contract whereby the latter would
manufacture for the petitioners 20,000 pieces of vinyl frogs and 20,000 pieces of vinyl
mooseheads at P7.00 per piece in accordance with the sample approved by the petitioners.
These frogs and mooseheads were to be attached to the shirts petitioners would
manufacture and sell.

Respondent Sio delivered in several installments the 40,000 pieces of frogs and
mooseheads. Petitioner fully paid the agreed price. However, petitioners subsequently
returned to respondent a number of pieces of frogs and mooseheads for failing to comply
with the approved sample.

Petitioners then demanded from the respondent a refund of the purchase price of the
returned goods in the amount of P208,404.00. As respondent Sio refused to pay, petitioners
filed an action for collection of a sum of money.

ISSUE:

WON there was a contract for a piece of work?

HELD:

"Art. 1467. A contract for the delivery at a certain price of an article which the vendor
in the ordinary course of his business manufactures or procures for the general market,
whether the same is on hand at the time or not, is a contract of sale, but if the goods are to
be manufactured specially for the customer and upon his special order, and not for the
general market, it is a contract for a piece of work."

"Art. 1713. By the contract for a piece of work the contractor binds himself to execute
a piece of work for the employer, in consideration of a certain price or compensation. The
contractor may either employ only his labor or skill, or also furnish the material."

A contract for a piece of work, labor and materials may be distinguished from a
contract of sale by the inquiry as to whether the thing transferred is one not in existence and
which would never have existed but for the order of the person desiring it. In such case, the
contract is one for a piece of work, not a sale. On the other hand, if the thing subject of the
contract would have existed and been the subject of a sale to some other person even if the
order had not been given then the contract is one of sale.

The contract between the petitioners and respondent stipulated that respondent
would manufacture upon order of the petitioners 20,000 pieces of vinyl frogs and 20,000

SALES CASE DIGEST COMPILATION 26


pieces of vinyl mooseheads according to the samples specified and approved by the
petitioners. Respondent Sio did not ordinarily manufacture these product but only upon order
of the petitioners and at the price agreed upon. Clearly, the contract executed by and
between the petitioners and the respondent was a contract for a piece of work.

2) COMMISSIONER OF INTERNAL REVENUE, petitioner,vs.THE COURT OF


APPEALS, THE COURT OF TAX APPEALS and ATENEO DE MANILA
UNIVERSITY, respondent(271 SCRA 605, G.R. No. 115349, April 18, 1997)

FACTS:

ADMU Institute of Philippine Culture is engaged in social science studies of


Philippine society and culture. Occasionally, it accepts sponsorships for its research
activities from international organizations, private foundations and government agencies.

On July 1983, CIR sent a demand letter assessing the sum of P174,043.97 for
alleged deficiency contractor’s tax. Accdg to CIR, ADMU falls under the purview of
independent contractor pursuant to Sec 205 of Tax Code and is also subject to 3%
contractor’s tax under Sec 205 of the same code. (Independent Contractor means any
person whose activity consists essentially of the sale of all kinds of services for a fee
regardless of whether or not the performance of the service calls for the exercise or use of
the physical or mental faculties of such contractors or their employees.)

ISSUES:

1) WON ADMU is an independent contractor hence liable for tax?

2) WON the acceptance of research projects by the IPC of ADMU a contract of sale
or a contract for a piece of work?

HELD:

1) Hence, to impose the three percent contractor’s tax on Ateneo’s Institute of


Philippine Culture, it should be sufficiently proven that the private respondent is
indeed selling its services for a fee in pursuit of an independent business.

2) Records do not show that Ateneo’s IPC in fact contracted to sell its research
services for a fee. In the first place, the petitioner has presented no evidence to
prove its bare contention that, indeed, contracts for sale of services were ever
entered into by the private respondent. Funds received by the Ateneo de Manila
University are technically not a fee. They may however fall as gifts or donations
which are tax-exempt. Another fact that supports this contention is that for about 30
years, IPC had continuously operated at a loss, which means that sponsored funds
are less than actual expenses for its research projects.

In fact, private respondent is mandated by law to undertake research activities to


maintain its university status. In fact, the research activities being carried out by the IPC is
focused not on business or profit but on social sciences studies of Philippine society and
culture. Since it can only finance a limited number of IPC’s research projects, private
respondent occasionally accepts sponsorship for unfunded IPC research projects from
international organizations, private foundations and governmental agencies. However, such
sponsorships are subject to private respondent’s terms and conditions, among which are,
that the research is confined to topics consistent with the private respondent’s academic

SALES CASE DIGEST COMPILATION 27


agenda; that no proprietary or commercial purpose research is done; and that private
respondent retains not only the absolute right to publish but also the ownership of the results
of the research conducted by the IPC.

3) CELESTINO CO & COMPANY, petitioner,vs.COLLECTOR OF INTERNAL


REVENUE, respondent.(99 Phil 841, G.R. No. L-8506, August 31, 1956)

FACTS:

Celestino Co & Company is a duly registered general co-partnership doing business


under the trade name of “Oriental Sash Factory”.

From 1946 to 1951 it paid percentage taxes of 7% on the gross receipts of its sash,
door and window factory. However in 1952 it began to claim liability only to the contractor’s 3
per cent tax (instead of 7 per cent) under section 191 of the same Code. It alleges primarily
that it is an ordinary contractor, presenting as evidence letters, sketches and price
quotations sent by the manager to four customers who allegedly made special orders to
doors and window from the said factory. It contended that it does not manufacture ready-
made doors, sash and windows for the public but only upon special order of its select
customers.

ISSUES:

1) WON Co & Co. is a manufacturer or contractor? MANUFACTURER, hence still


liable for 7% tax.

2) WON Is Co & Co.’s business a matter of contract of sale or contract of piece of


work? SALE.

HELD:

1) The important thing to remember is that Celestino Co & Company habitually


makes sash, windows and doors, as it has represented in its stationery and advertisements
to the public. That it “manufactures” the same is practically admitted by appellant itself. The
fact that windows and doors are made by it only when customers place their orders, does
not alter the nature of the establishment, for it is obvious that it only accepted such orders as
called for the employment of such material-moulding, frames, panels-as it ordinarily
manufactured or was in a position habitually to manufacture. The Oriental Sash Factory
does nothing more than sell the goods that it mass- produces or habitually makes; sash,
panels, mouldings, frames, cutting them to such sizes and combining them in such forms as
its customers may desire.

2) Art. 1467. A contract for the delivery at a certain price of an article which the
vendor in the ordinary course of his business manufactures or procures for the general
market, whether the same is on hand at the time or not, is a contract of sale, but if the goods
are to be manufactured specially for the customer and upon his special order, and not for the
general market, it is contract for a piece of work.

When it accepts a job that requires the use of extraordinary or additional equipment,
or involves services not generally performed by it-it thereby contracts for a piece of work —
filing special orders within the meaning of Article 1467. The orders herein exhibited were not
shown to be special. They were merely orders for work — nothing is shown to call them
special requiring extraordinary service of the factory.

SALES CASE DIGEST COMPILATION 28


Oriental Sash Factory did not merely sell its services to Teodoro & Co. It sold
materials ordinarily manufactured by it — sash, panels, mouldings — to Teodoro & Co.,
although in such form or combination as suited the fancy of the purchaser. Such new form
does not divest the Oriental Sash Factory of its character as manufacturer. Neither does it
take the transaction out of the category of sales under Article 1467 above quoted, because
although the Factory does not, in the ordinary course of its business, manufacture and keep
on stock doors of the kind sold to Teodoro, it could stock and/or probably had in stock the
sash, mouldings and panels it used therefor.

Supposing for the moment that the transactions were not sales, they were neither
lease of services nor contract jobs by a contractor. But as the doors and windows had been
admittedly “manufactured” by the Oriental Sash Factory, such transactions could be, and
should be taxed as “transfers” thereof under section 186 of the National Revenue Code.

4) ENGINEERING & MACHINERY CORPORATION, petitioner, vs. COURT OF


APPEALS and PONCIANO L. ALMEDA, respondents.(252 SCRA 156, G.R. No.
52267, January 24, 1996)

FACTS:

Pursuant to the contract dated September 10, 1962 between petitioner and private
respondent, the former undertook to fabricate, furnish and install the air-conditioning system
in the latter’s building along Buendia Avenue, Makati in consideration of P210,000.00.
Petitioner was to furnish the materials, labor, tools and all services required in order to so
fabricate and install said system. The system was completed in 1963 and accepted by
private respondent, who paid in full the contract price.

Private respondent sold the building to the National Investment and Development
Corporation (NIDC). The latter took possession of the building but on account of NIDC’s
noncompliance with the terms and conditions of the deed of sale, private respondent was
able to secure judicial rescission thereof. The ownership of the building having been decreed
back to private respondent, he re-acquired possession sometime in 1971. It was then that he
learned from some NIDC employees of the defects of the air-conditioning system of the
building.

Private respondent filed on May 8, 1971 an action for damages against petitioner
with the then CFI. The complaint alleged that the air-conditioning system installed by
petitioner did not comply with the agreed plans and specifications. Petitioner assails that the
prescriptive period of six months (Article 1571, CC) has set in regarding the responsibility of
a vendor for any hidden faults or defects in the thing sold. Private respondent said that their
contract was not a contract of sale but a contract for a piece of work under Article 1713 of
the Civil Code. Thus, in accordance with Article 1144 (1) of the same Code, the complaint
was timely brought within the ten-year prescriptive period.

Trial court ruled that the complaint was filed within the ten-year prescriptive period
although the contract was one for a piece of work, because it involved the “installation of an
air-conditioning system which the defendant itself manufactured, fabricated, designed and
installed.” CA affirmed.

ISSUE:

Is a contract for the fabrication and installation of a central air-conditioning system in


a building, one of “sale” or “for a piece of work”? – piece of work (thus, the action has not yet

SALES CASE DIGEST COMPILATION 29


prescribed)

HELD:

Article 1713 of the CC defines a contract for a piece of work thus:

“By the contract for a piece of work the contractor binds himself to execute a piece of
work for the employer, in consideration of a certain price or compensation. The
contractor may either employ only his labor or skill, or also furnish the material.”

A contract for a piece of work, labor and materials may be distinguished from a
contract of sale by the inquiry as to whether the thing transferred is one not in existence and
which would never have existed but for the order of the person desiring it. In such case, the
contract is one for a piece of work, not a sale. On the other hand, if the thing subject of the
contract would have existed and been the subject of a sale to some other person even if the
order had not been given, then the contract is one of sale.

To Tolentino, the distinction between the two contracts depends on the intention of
the parties. Thus, if the parties intended that at some future date an object has to be
delivered, without considering the work or labor of the party bound to deliver, the contract is
one of sale. But if one of the parties accepts the undertaking on the basis of some plan,
taking into account the work he will employ personally or through another, there is a contract
for a piece of work.

Clearly, the contract in question is one for a piece of work. It is not petitioner’s line of
business to manufacture air-conditioning systems to be sold “off-the-shelf.” Its business and
particular field of expertise is the fabrication and installation of such systems as ordered by
customers and in accordance with the particular plans and specifications provided by the
customers. Naturally, the price or compensation for the system manufactured and installed
will depend greatly on the particular plans and specifications agreed upon with the
customers.

5) COMMISSIONER OF INTERNAL REVENUE vs. ARNOLDUS CARPENTRY


SHOP, INC. and COURT OF TAX APPEALS (159 SCRA 199, G.R. No. 71122,
March 25, 1988)

FACTS:

Arnoldus Carpentry Shop, Inc. is a domestic corporation involved in "preparing,


processing, buying, selling, exporting, importing, manufacturing, trading and dealing in
cabinet shop products, wood and metal home and office furniture, cabinets, doors, windows,
etc., including their component parts and materials, of any and all nature and description".
These were sold locally and exported abroad. For this business venture, private respondent
kept samples or models of its woodwork on display from where its customers may refer to
when placing their orders.

Sometime in March 1979, the examiners of the petitioner Commissioner of Internal Revenue
conducted an investigation of the business tax liabilities of private respondent. As per the
examination, the examiners classified private respondent as an “other independent
contractor”. As a result thereof, the examiners assessed private respondent for deficiency
tax. Against this assessment, private respondent filed a protest. In the protest letter, private
respondent's manager maintained that the carpentry shop is a manufacturer and therefor
entitled to tax exemption under Section 202 (e) of the National Internal Revenue Code.

SALES CASE DIGEST COMPILATION 30


The CIR considered the private respondent a contractor and not a manufacturer.
Private respondent appealed to the Court of Tax Appeals alleging that the decision of the
Commissioner was contrary to law and the facts of the case. Then, the Court of Tax Appeals
held that private respondent was a manufacturer.

ISSUE:

WON the private respondent is a manufacturer and not a contractor under Article
1467 of the New Civil Code. – YES.

HELD:

The private respondent is a manufacturer. The SC agreed with the decision of the
CTA.

Article 1467 states that a contract for the delivery at a certain price of an article which
the vendor in the ordinary course of his business manufactures or procures for the - general
market, whether the same is on hand at the time or not, is a contract of sale, but if the goods
are to be manufactured specially for the customer and upon his special order, and not for the
general market, it is a contract for a piece of work.

As can be clearly seen from the wordings of Art. 1467, what determines whether the
contract is one of work (and thus classifying private respondent as a contractor) or of sale
(which would classify private respondent as a manufacturer) is whether the thing has been
manufactured specially for the customer and upon his special order." Thus, if the thing is
specially done at the order of another, this is a contract for a piece of work. If, on the other
hand, the thing is manufactured or procured for the general market in the ordinary course of
one's business, it is a contract of sale.

The findings of the Court of Tax Appeals show that the private respondent had a
ready stock of its shop products for sale to its foreign and local buyers. As a matter of fact,
the purchase orders from its foreign buyers showed that they ordered by referring to the
models designed by private respondent. Even purchases by local buyers for television
cabinets were by orders for existing models. With regard to the television cabinets, private
respondent presented three witnesses who testified that samples of television cabinets were
designed and made by private respondent, from which models the television companies
might choose, then specified whatever innovations they desired. If found to be saleable,
some television cabinets were manufactured for display and sold to the general public. In the
case of private respondent’s other woodwork products, these were manufactured without
previous orders. Samples were displayed, and if in stock, were available for immediate sale
to local and foreign customers.

The aforementioned considerations were what precisely moved the respondent Court
of Tax Appeals to rule that 'the fact that [private respondent] kept models of its products
indicate that these products were for sale to the general public and not for special orders.
These attendant facts reveal habituality of the production for the general public.

6) INCHAUSTI AND CO., plaintiff-appellant,vs.ELLIS CROMWELL, Collector of


Internal Revenue, defendant-appellee.(20 Phil 345, G.R. No. 6584, October 16,
1911)

SALES CASE DIGEST COMPILATION 31


FACTS:

Inchausti is engaged in the business of buying and selling at wholesale hemp, both
for its own account and on commission.

It is customary to sell hemp in bales which are made by compressing the loose fiber
by means of presses, covering two sides of the bale with matting, and fastening it by means
of strips of rattan; that the operation of bailing hemp is designated among merchants by the
word “prensaje.”

In all sales of hemp by Inchausti, the price is quoted to the buyer at so much per
picul, no mention being made of bailing. It is with the tacit understanding that the hemp will
be delivered in bales. The amount depends under the denomination of “prensaje” or the
baled hemp.

CIR made demand in writing upon Inchausti for the payment of the sum of P1,370.68
as a tax of one third of one per cent on the sums of money mentioned as aggreagate sum
collected as prensaje or the baled hemp. Inchausti paid upon protest, contending that the
collected amount is illegal upon the ground that the said charge does not constitute a part of
the selling price of the hemp, but is a charge made for the service of baling the hemp.

ISSUE:

WON the baled hemp constitutes a contract of sale or a contract for a piece of work?
Contract of sale, hence Inchausti is chargeable for the amount of tax based on the baled
hemp and not on the raw hemp sold alone.

HELD:

TEST OF EXISTENCE: The distinction between a contract of sale and one for work,
labor, and materials is tested by the inquiry whether the thing transferred is one no in
existence and which never would have existed but for the order of the party desiring to
acquire it, or a thing which would have existed and been the subject of sale to some other
person, even if the order had not been given.

It is clear that in the case at bar, the hemp was in existence in baled form before the
agreements of sale were made, or, at least, would have been in existence even if none of
the individual sales here in question had been consummated. It would have been baled,
nevertheless, for sale to someone else, since, according to the agreed statement of facts, it
is customary to sell hemp in bales. When a person stipulates for the future sale of articles
which he is habitually making, and which at the time are not made or finished, it is
essentially a contract of sale and not a contract for labor. It is otherwise when the article is
made pursuant to agreement. If the article ordered by the purchaser is exactly such as the
plaintiff makes and keeps on hand for sale to anyone, and no change or modification of it is
made at the defendant’s request, it is a contract of sale, even though it may be entirely made
after, and in consequence of, the defendant’s order for it.

Baling was performed for the general market and was not something done by plaintiff
which was a result of any peculiar wording of the particular contract between him and his
vendee. One whose exposes goods for sale in the market must have them in marketable
form. The hemp in question would not have been in that condition if it had not been baled.
the baling, therefore, was nothing peculiar to the contract between the plaintiff and his
vendee. It was precisely the same contract that was made by every other seller of hemp. It
resulted simply in the transfer of title to goods already prepared for the general market.

SALES CASE DIGEST COMPILATION 32


Baling is performed before the sale. The purchaser of hemp owes to the seller
nothing whatever by reason of their contract except the value of the hemp delivered. That
value, that sum which the purchaser pays to the vendee, is the true selling price of the
hemp, and every item which enters into such price is a part of such selling price. By force of
the custom prevailing among hemp dealers in the Philippine Islands, a purchaser of hemp in
the market, unless he expressly stipulates that it shall be delivered to him in loose form,
obligates.

ERWIN M.

7.) ANDRES QUIROGA vs PARSONS HARDWARE CO.


G.R. L-11491 August 23, 1918

FACTS:
On January 24, 1911, a contract was entered into by and between Andres Quiroga
and J. Parsons, to whose rights and obligations the present defendant later subrogated
itself. The contract stipulated that Don Andres Quiroga grants the exclusive right to sell his
beds in the Visayan Islands to J. Parsons.

Quiroga files a case against Parsons for allegedly violating the following stipulations:
not to sell the beds at higher prices than those of the invoices; to have an open
establishment in Iloilo; itself to conduct the agency; to keep the beds on public exhibition,
and to pay for the advertisement expenses for the same; and to order the beds by the dozen
and in no other manner. With the exception of the obligation on the part of the defendant to
order the beds by the dozen and in no other manner, none of the obligations imputed to the
defendant in the two causes of action are expressly set forth in the contract. But the plaintiff
alleged that the defendant was his agent for the sale of his beds in Iloilo, and that said
obligations are implied in a contract of commercial agency. The whole question, therefore,
reduced itself to a determination as to whether the defendant, byreason of the contract
hereinbefore transcribed, was a purchaser or an agent of the plaintiff for the sale of his beds.

ISSUE: Whether or Not the contract is a contract of agency or of sale.

RULING:
In order to classify a contract, due regard must be given to its essential clauses. In
the contract in question, what was essential, as constituting its cause and subject matter, is
that the plaintiff was to furnish the defendant with the beds which the latter might order, at
the price stipulated, and that the defendant was to pay the price in the manner stipulated.
The price agreed upon was the one determined by the plaintiff for the sale of these beds in
Manila, with a discount of from 20 to 25 per cent, according to their class. Payment was to
be made at the end of sixty days, or before, at the plaintiff's request, or in cash, if the
defendant so preferred, and in these last two cases an additional discount was to be allowed
for prompt payment. These are precisely the essential features of a contract of purchase and
sale. There was the obligation on the part of the plaintiff to supply the beds, and, on the part
of the defendant, to pay their price. These features exclude the legal conception of an
agency or order to sell whereby the mandatory or agent received the thing to sell it,
and does not pay its price, but delivers to the principal the price he obtains from the
sale of the thing to a third person, and if he does not succeed in selling it, he returns
it. By virtue of the contract between the plaintiff and the defendant, the latter, on receiving

SALES CASE DIGEST COMPILATION 33


the beds, was necessarily obliged to pay their price within the term fixed, without any other
consideration and regardless as to whether he had or had not sold the beds.
It would be enough to hold, as we do, that the contract by and between the defendant and
the plaintiff is one of purchase and sale, in order to show that it was not one made on the
basis of a commission on sales, as the plaintiff claims it was, for these contracts are
incompatible with each other. But, besides, examining the clauses of this contract, none of
them is found that substantially supports the plaintiff's contention. Not a single one of these
clauses necessarily conveys the idea of an agency. The words commission on sales used in
clause (A) of article 1 mean nothing else, as stated in the contract itself, than a mere
discount on the invoice price. The word agency, also used in articles 2 and 3, only
expresses that the defendant was the only one that could sell the plaintiff's beds in the
Visayan Islands. With regard to the remaining clauses, the least that can be said is that they
are not incompatible with the contract of purchase and sale.

8.) PUYAT & SONS, INC. V. ARCO AMUSEMENT CO.

FACTS :
Arco Amusement was engaged in the business of operating cinematographs.
Gonzalo Puyat & Sons Inc. (GPS) was the exclusive agent in the Philippines for the Starr
Piano Company. Desiring to equip its cinematograph with sound reproducing devices, Arco
approached GPS, through its president, GIl Puyat, and an employee named Santos. After
some negotiations, it was agreed between the parties that GPS would order sound
reproducing equipment from Starr Piano Company and that Arco would pay GPS, in addition
to the price of the equipment, a 10% commission, plus all expenses such as freight,
insurance, etc. When GPS inquired Starr Piano the price (without discount) of the
equipment, the latter quoted such at $1,700 FOB Indiana. Being agreeable to the price (plus
10%commission plus all other expenses), Arco formally authorized the order. The following
year, both parties agreed for another order of sound reproducing equipment on the same
terms as the first at $1,600 plus 10% plus all other expenses. Three years later, Arco
discovered that the prices quoted to them by GPS with regard to their first 2 orders
mentioned were not the net prices, but rather the list price, and that it had obtained a
discount from Starr Piano. Moreover, Arco alleged that the equipment were overpriced.
Thus, being its agent, GPS had to reimburse the excess amount it received from Arco.

ISSUE:
Whether or Not there was a contract of agency, not of sale

HELD :
NO. The letters containing Arco's acceptance of the prices for the equipment are
clear in their terms and admit no other interpretation that the prices are fixed and
determinate. While the letters state that GPS was to receive a 10% commission, this does
not necessarily mean that it is an agent of Arco, as this provision is only an additional price
which it bound itself to pay, and which stipulation is not incompatible with the contract of
sale. It is GPS that is the exclusive agent of Starr Piano in the Philippines, not the agent of
Arco. It is out of the ordinary for one to be the agent of both the seller and the buyer. The
facts and circumstances show that Arco entered into a contract of sale with GPS, the
exclusive agent of Starr Piano. As such, it is not duty bound to reveal the private
arrangement it had with Starr Piano relative to the 25% discount. Thus, GPS is not bound to
reimburse Arco for any difference between the cost price and the sales price, which
represents the profit realized by GPS out of the transaction.

SALES CASE DIGEST COMPILATION 34


V. DISTINGUISHED FROM AGENCY TO SELL
1.)KER & CO, LTD. LINGAD
G.R. No. L-20871 APRIL 30,1971

FACTS :

Ker and Co, Ltd. was assessed by then Commissioner of Internal Revenue Domingo
the sum of P 20,272.33 as the commercial broker’s percentage tax surcharge and
compromise penalty. There was a request on the part of Ker for the cancellation of such
assessment which request was turned down. As a result, it filed a petition for review with the
Court of Tax Appeals. The CTA held that Ker is taxable except as to the compromise penalty
of P 500, the amount due from it being fixed at P 19, 772.33.
Such liability arose from a contract of Ker with the United States Rubber International
(USRI). The former being referred to as the distributor and the latter specifically designated
as the company. The contract was to apply to transactions between the former and Ker, as
distributor from July 1, 1948 to continue in force until terminated by either party giving to the
other 60 days’ notice. The shipments would cover products for consumption in Cebu, Bohol,
Leyte, Samar, Jolo, Negros Oriental and Mindanao except the province of Davao, Ker as
distributor being precluded from disposing such products elsewhere than in the above
places unless written consent be obtained from the company. Ker as distributor is required
to exert every effort to have the shipment of the products in the maximum quantity and to
promote in every way the sale thereof.
(Crucial stipulation: The company shall form time to time consign to Ker and Ker will receive,
accept and/or hold upon consignment the products specified under the terms of this
agreement in such quantities as in the judgment of company may be necessary.
It is further agreed that this agreement does not constitute Ker the agent or legal
representative of the company for any purpose whatsoever.)

ISSUE :
Whether or Not the contract between Ker and the USRI is a contract of sale.

HELD :
No. By taking the contractual stipulations as a whole and not just the disclaimer, it
would seem that the contract between them is a contract of agency. That the petitioner Ker
& Co., Ltd. is, by contractual stipulation, an agent of U.S. Rubber International is borne out
by the facts that:
1. petitioner can dispose of the products of the Company only to certain persons or entities
and within stipulated limits, unless excepted by the contract or by the Rubber Company;
2. it merely receives, accepts and/or holds upon consignment the products, which remain
properties of the latter company; 3. every effort shall be made by petitioner to promote in
every way the sale of the products (Par. 3); that sales made by petitioner are subject to
approval by the company;
4. on dates determined by the rubber company, petitioner shall render a detailed report
showing sales during the month;
5. the rubber company shall invoice the sales as of the dates of inventory and sales report
(Par. 14); that the rubber company agrees to keep the consigned goods fully insured under
insurance policies payable to it in case of loss;
6. upon request of the rubber company at any time, petitioner shall render an inventory of
the existing stock which may be checked by an authorized representative of the former
7. upon termination or cancellation of the Agreement, all goods held on consignment shall
be held by petitioner for the account of the rubber company until their disposition is provided
for by the latter

SALES CASE DIGEST COMPILATION 35


The National Internal Revenue Code defined “Commercial broker” as “all persons, other
than importer, manufacturers, producers or bona fide employees who, for compensation or
profit, sell or bring about sales or purchase of merchandise for other persons or being
proposed buyers and sellers together” and also includes commission merchants such as Ker
in this case.
The mere disclaimer in a contract that an entity like Ker is not “the agent or legal
representative for any purpose whatsoever” does not suffice to yield the conclusion that it is
an independent merchant if the control over the goods for resale of goods consigned is
pervasive in character.

2. )SCHMID & OBERLY, INC. vs. RJL MARTINEZ


G.R. No. 75198 October 18, 1988

FACTS:
RJL Martinez Fishing Corporation is engaged in deep-sea fishing. In the course of its
business, it needed electrical generators for the operation of its business. Schmid and
Oberly sells electrical generators with the brand of “Nagata”, a Japanese product. D. Nagata
Co. Ltd. of Japan was Schmid’s supplier. Schmid advertised the 12 Nagata generators for
sale and RJL purchased 12 brand new generators. Through an irrevocable line of credit,
Nagata shipped to the Schmid the generators and RJL paid the amount of the purchase
price. (First sale = 3 generators; Second sale = 12 generators).
Later, the generators were found to be factory defective. RJL informed the Schmid that it
shall return the 12 generators. 3 were returned. Schmid replaced the 3 generators subject of
the first sale with generators of a different brand. As to the second sale, 3 were shipped to
Japan and the remaining 9 were not replaced.
RJL sued the defendant on the warranty, asking for rescission of the contract and that
Schmid be ordered to accept the generators and be ordered to pay back the purchase
money as well as be liable for damages. Schmid opposes such liability averring that it was
merely the indentor in the sale between Nagata Co., the exporter and RJL Martinez, the
importer. Asmere indentor, it avers that is not liable for the seller’s implied warranty against
hidden defects, Schmid not having personally assumed any such warranty.

ISSUES:
1) WON the second transaction between the parties was a sale or an indent transaction
.

2) Even is Schmid is merely an indentor, may it still be liable for the warranty?

HELD:
1.)INDENT TRANSACTION
An indentor is a middlemen in the same class as commercial brokers and commission
merchants. A broker is generally defined as one who is engaged, for others, on a
commission, negotiating contracts relative to property with the custody of which he has no
concern; the negotiator between other parties, never acting in his own name but in the name
of those who employed him; he is strictly a middleman and for some purpose the agent of
both parties. There are 3 parties to an indent transaction, (1) buyer, (2) indentor, and (3)
supplier who is usually a non-resident manufacturer residing in the country where the goods
are to be bought. The chief feature of a commercial broker and a commercial merchant is
that in effecting a sale, they are merely intermediaries or middle-men, and act in a certain
sense as the agent of both parties to the transaction.

SALES CASE DIGEST COMPILATION 36


RJL MARTINEZ admitted that the generators were purchased “through indent order.” RJL
admitted in its demand letter previously sent to SCHMID that 12 of 15 generators “were
purchased through your company, by indent order and three (3) by direct purchase.” The
evidence also show that RJL MARTINEZ paid directly NAGATA CO, for the generators, and
that the latter company itself invoiced the sale and shipped the generators directly to the
former. The only participation of Schmid was to act as an intermediary or middleman
between Nagata and RJL, by procuring an order from RJL and forwarding the same to
Nagata for which the company received a commission from Nagata.
Sale vs. Indent Transaction:
The essence of the contract of sale is transfer of title or agreement to transfer it for a price
paid or promised. If such transfer puts the transferee in the attitude or position of an owner
and makes him liable to the transferor as a debtor for the agreed price, and not merely as an
agent who must account for the proceeds of a resale, the transaction is a sale.

3 evidences pointing to fact that Schmid is merely an indentor:


a. the Quotation and the General Conditions of Sale on the dorsal side thereof do not
necessarily lead to the conclusion that NAGATA CO., was the real seller of the 12
generators.

b. When RJL complained to SCHMID, it immediately asked RJL to send the defective
generators to its shop to determine what was wrong. SCHMID informed NAGATA about the
complaint of RJL. After the generators were found to have factory defects, SCHMID
facilitated the shipment of three (3) generators to Japan and, after their repair, back to the
Philippines.
c. the letter from NAGATA CO. to SCHMID regarding the repair of the generators
indicated that the latter was “within the purview of a seller.”

2) YES, under its contractual obligations it may be liable. But in this case, Schmid did not
warrant the products.
Even as SCHMID was merely an indentor, there was nothing to prevent it from voluntarily
warranting that twelve (12) generators subject of the second transaction are free from
anyhidden defects. In other words, SCHMID may be held answerable for some other
contractual obligation, if indeed it had so bound itself. As stated above, an indentor is to
some extent an agent of both the vendor and the vendee. As such agent, therefore, he may
expressly obligate himself to undertake the obligations of his principal.
Notably, nowhere in the Quotation is it stated therein that SCHMID did bind itself to answer
for the defects of the things sold. Balagtas testified initially that the warranty was in the
receipts covering the sale. Nowhere is it stated in the invoice that SCHMID warranted the
generators against defects. He again changed his mind and asserted that the warranty was
given verbally. Hence, RJL has failed to prove that SCHMID had given a warranty on the 12
generators subject of the second transaction.

3.)VICTORIAS MILLING CO. vs CA and CONSOLIDATED SUGAR CO.


[G.R. No. 117356. June 19, 2000]

FACTS:
St. Therese Merchandising (STM), who regularly bought sugar from Victorias Milling
Co. (VMC), was issued Shipping List/Delivery Receipts (SLDRs) by the latter as proof of
purchases for bags of sugar. Thereafter, STM sold to Consolidated Sugar Co. (CSC) its
rights in one of the SLDRs. CSC communicated to VMC that it had been authorized by STM
to withdraw the sugar covered by SLDR. Enclosed in the letter were a copy of SLDR and a
letter of authority from STM authorizing CSC "to withdraw for and in our behalf the refined
sugar covered by SLDR”. CSC surrendered the SLDR to VMC’s warehouse and was
allowed to withdraw sugar but after several bags were released, it was later on refused to
allow further withdrawals of sugar. CSC communicated to VMC to allow it to withdraw sugar

SALES CASE DIGEST COMPILATION 37


because the SLDR had been “sold and endorsed” to it by STM. VMC contended that it could
not allow any further withdrawals of sugar against SLDR because STM had already
withdrawn sugar covered by cleared checks. CSC filed complaint against VMC. VMC
contended that it had no privity of contract with CSC, the dealings between it and STM were
part of a series of transactions involving only one account or one general contract of sale
because CSC was an agent of STM. CSC countered that the sugar purchases involving
SLDR were separate and independent transactions.

ISSUE: Whether or not CSC was an agent of STM.

HELD:
No. CSC was a buyer of the SLDR form, and not an agent of STM.
CSC was not subject to STM's control. The question of whether a contract is one of sale or
agency depends on the intention of the parties as gathered from the whole scope and effect
of the language employed. That the authorization given to CSC contained the phrase "for
and in our (STM's) behalf" did not establish an agency. CSC communicated to VMC that the
SLDR had been “sold and endorsed” to it by STM. The use of the words "sold and
endorsed" means that STM and CSC intended a contract of sale, and not an agency.
The basis of agency is representation. On the part of the principal, there must be an actual
intention to appointor an intention naturally inferable from his words or actions; and on the
part of the agent, there must be an intention to accept theappointment and act on it, and in
the absence of such intent, there is generally no agency. One factor which most clearly
distinguishes agency from other legal concepts is control; one person - the agent - agrees to
act under the control or direction of another - the principal. Indeed, the very word "agency"
has come to connote control by the principal. The control factor, more than any other, has
caused the courts to put contracts between principal and agent in a separate category.

SALES CASE DIGEST COMPILATION 38


VI. DISTINGUISHED FROM DACION EN PAGO
1.)LO VS. KJS
G.R. No. 149420, October 8, 2003

FACTS: Respondent KJS ECO-FORMWORK System Phil., Inc. is a corporation engaged in


the sale of steel scaffoldings, while petitioner Sonny L. Lo, doing business under Sans
Enterprises, is a building contractor. Lo ordered scaffolding equipment from KJS worth
P540,425.80. He paid a down payment in the amount of P150,000.00. The balance was
made payable in ten monthly instalments.
KJS delivered the scaffoldings to Lo. Lo was able to pay the first two monthly instalments.
His business, however, encountered financial difficulties and he was unable to settle his
obligation to KJS despite oral and written demands made against him.
Lo and KJS executed a Deed of Assignment whereby Lo assigned to KJS his receivables in
the amount of P335,462.14 from Jomero Realty Corporation.
However, when KJS tried to collect the said credit from Jomero Realty Corporation, the latter
refused to honor the Deed of Assignment because it claimed that Lo was also indebted to it.
Subsequently, KJS sent a letter to Lo demanding payment of his obligation, but he refused
to pay claiming that his obligation had been extinguished when they executed the Deed of
Assignment.
Consequently, KJS filed an action for recovery of a sum of money against Lo before the
RTC. Lo argued that his obligation was extinguished with the execution of the Deed of
Assignment of credit. KJS, for its part, presented the testimony of its employee, Almeda
Baaga, who testified that Jomero Realty refused to honor the assignment of credit because it
claimed that Lo had an outstanding indebtedness to it.
RTC dismissed the complaint on the ground that the assignment of credit extinguished the
obligation. CA reversed the decision. In finding that the Deed of Assignment did not
extinguish the obligation of the petitioner to the respondent, the CA held that (1) petitioner
failed to comply with his warranty under the Deed; (2) the object of the Deed did not exist at
the time of the transaction, rendering it void pursuant to Article 1409 of the Civil Code; and
(3) petitioner violated the terms of the Deed of Assignment when he failed to execute and do
all acts and deeds as shall be necessary to effectually enable the respondent to recover the
collectibles.
ISSUE: Whether or not the deed of assignment extinguished Lo’s obligations.

HELD:
No. An assignment of credit is an agreement by virtue of which the owner of a credit,
known as the assignor, by a legal cause, such as sale, dacion en pago, exchange or
donation, and without the consent of the debtor, transfers his credit and accessory rights to
another, known as the assignee, who acquires the power to enforce it to the same extent as
the assignor could enforce it against the debtor.
Corollary thereto, in dacion en pago, as a special mode of payment, the debtor offers
another thing to the creditor who accepts it as equivalent of payment of an outstanding debt.
In order that there be a valid dation in payment, the following are the requisites
There must be the performance of the prestation in lieu of payment (animo solvendi) which
may consist in the delivery of a corporeal thing or a real right or a credit against the third
person;
2.) There must be some difference between the prestation due and that which is given in
substitution (aliud pro alio)
3.) There must be an agreement between the creditor and debtor that the obligation is
immediately extinguished by reason of the performance of a prestation different from that
due.
The undertaking really partakes in one sense of the nature of sale, that is, the creditor is
really buying the thing or property of the debtor, payment for which is to be charged against

SALES CASE DIGEST COMPILATION 39


the debtor’s debt. As such, the vendor in good faith shall be responsible, for the existence
and legality of the credit at the time of the sale but not for the solvency of the debtor, in
specified circumstances.
Hence, it may well be that the assignment of credit, which is in the nature of a sale of
personal property, produced the effects of a dation in payment which may extinguish the
obligation. However, as in any other contract of sale, the vendor or assignor is bound by
certain warranties. More specifically, the first paragraph of Article 1628 of the Civil Code
provides:
The vendor in good faith shall be responsible for the existence and legality of the credit at
the time of the sale, unless it should have been sold as doubtful; but not for the solvency of
the debtor, unless it has been so expressly stipulated or unless the insolvency was prior to
the sale and of common knowledge.
From the above provision, petitioner, as vendor or assignor, is bound to warrant the
existence and legality of the credit at the time of the sale or assignment. When Jomero
Realty claimed that it was no longer indebted to Lo since the latter also had an unpaid
obligation to it, it essentially meant that its obligation to Lo has been extinguished by
compensation. In other words, KJS alleged the non-existence of the credit and asserted its
claim to Lo’s warranty under the assignment. Therefore, it behooved on Lo to make good its
warranty and paid the obligation.
Furthermore, we find that Lo breached his obligation under the Deed of Assignment. Indeed,
by warranting the existence of the credit, petitioner should be deemed to have ensured the
performance thereof in case the same is later found to be inexistent. He should be held
liable to pay to respondent the amount of his indebtedness.
Hence, we affirm the decision of the Court of Appeals ordering petitioner to pay respondent
the sum of P335,462.14 with legal interest thereon.

2.)PNB vs. PINEDA


G.R. No. L-46658, May 13, 1991

FACTS:
In 1963, Ignacio and Lourdes Arroyo (spouses Arroyo) obtained a loan of
P580,000.00 from PNB to purchase 60% of the subscribed capital stock, and thereby
acquire the controlling interest of private respondent Tayabas Cement Company, Inc. (TCC).
Spouses Arroyo executed a real estate mortgage over a parcel of land (La Vista property) as
security for the said loan.
Thereafter, TCC filed with PNB an application and agreement for the establishment of an 8
year deferred letter of credit (L/C) for $7,000,000.00 in favor of Toyo Menka Kaisha, Ltd. of
Tokyo, Japan, to cover the importation of a cement plant machinery and equipment.
Upon approval of said application and opening of an L/C by PNB in favor of Toyo Menka
Kaisha, Ltd. for the account of TCC, the Arroyo spouses executed documents (Surety
Agreement and Covenant) to secure the loan accommodation.
The imported cement plant machinery and equipment arrived from Japan and were released
to TCC under a trust receipt agreement. Toyo Menka Kaisha made the corresponding
drawings against the L/C as scheduled. TCC, however, failed to remit and/or pay the amount
covered by the drawings. Thus, PNB notified TCC of its intention to repossess the imported
machinery and equipment for failure of TCC to settle its obligations under the L/C.
In the meantime, the personal accounts of the spouses Arroyo, which included another loan
of P160,000.00 secured by a real estate mortgage over Hacienda Bacon located in Isabela,
Negros Occidental, had likewise become due. The spouses Arroyo failed to satisfy their
obligations with PNB and the latter decided to foreclose the real estate mortgages.
At the auction sale of the La Vista Property, PNB was the highest bidder with a bid price of
P1,000,001.00. However, when said property was about to be awarded to PNB, the
representative of the mortgagor-spouses objected and demanded from the PNB the

SALES CASE DIGEST COMPILATION 40


difference between the bid price of P1,000,001.00 and the indebtedness of P499,060.25 of
the Arroyo spouses on their personal account. It was the contention of the spouses Arroyo's
representative that the foreclosure proceedings referred only to the personal account of the
mortgagor spouses without referenceTo remedy the situation, PNB filed a supplemental
petition requesting the Sheriff's Office to proceed with the sale of thesubject real properties
to satisfy not only the amount of P499,060.25 owed by the spouses Arroyos on their
personal account but also the amount of P35,019,901.49 owed by said spouses as sureties
of TCC. Said petition was opposed by the spouses Arroyo and the other bidder Araneta.
PNB filed a petition for mandamus to compel the sheriff to proceed with the foreclosure sale
of the mortgaged properties and the petition was granted. However, TCC filed a complaint
against PNB to restrain the foreclosure of the mortgages over the said properties as well as
a declaration that its obligation with PNB had been fully paid by reason of the latter's
repossession of the imported machinery and equipment.

ISSUE: Whether or not TCC's liability has been extinguished by the repossession of PNB of
the imported cement plant machinery and equipment and whether or not the repossession
amounts to dacion en pago.

HELD: No. It must be remembered that PNB took possession of the imported cement plant
machinery and equipment pursuant to the trust receipt agreement executed by PNB and
TCC giving the former the unqualified right to the possession and disposal of all property
shipped under the Letter of Credit until such time as all the liabilities and obligations under
said letter had been discharged.
PNB's possession of the subject machinery and equipment being precisely as a form of
security for the advances given to TCC under the Letter of Credit, said possession by itself
cannot be considered payment of the loan secured thereby. Payment would legally result
only after PNB had foreclosed on said securities, sold the same and applied the proceeds
thereof to TCC's loan obligation. Mere possession does not amount to foreclosure for
foreclosure denotes the procedure adopted by the mortgagee to terminate the rights of the
mortgagor on the property and includes the sale itself.
Neither can said repossession amount to dacion en pago. Dation in payment takes
place when property is alienated to the creditor in satisfaction of a debt in money and
the same is governed by sales. Dation in payment is the delivery and transmission of
ownership of a thing by the debtor to the creditor as an accepted equivalent of the
performance of the obligation. As aforesaid, the repossession of the machinery and
equipment in question was merely to secure the payment of TCC's loan obligation
and not for the purpose of transferring ownership thereof to PNB in satisfaction of
said loan. Thus, no dacion en pago was ever accomplished.
Proceeding from this finding, PNB has the right to foreclose the mortgages executed by the
spouses Arroyo as sureties of TCC. A surety is considered in law as being the same party
as the debtor in relation to whatever is adjudged touching the obligation of the latter, and
their liabilities are interwoven as to be inseparable. As sureties, the Arroyo spouses are
primarily liable as original promissors and are bound immediately to pay the creditor amount
outstanding.

3.)SSS V CA, 553 SCRA 677 (2008)

FACTS:
AG&P and Semirara Coal Company proposed to pay its arrears of premiums and
loan amortization delinquencies through dacion en pago which was subsequently accepted
by SSS. Thereafter, SSS directed herein defendant to submit certain documents necessary
for the agreement which AG&P immediately complied with. SSS finally approved the dacion
en pago which as of March 2001 amounted to P29, 261,902.45. To effect said transfer, a
Deed of Assignment had to be executed between the two parties which SSS failed to come
up. On the other hand, defendant continuously submitted drafts to SSS of the needed Deed

SALES CASE DIGEST COMPILATION 41


of Assignment. ON 2003, SSS sent to AG&P a revised copy of the Deed of Assignment,
however, the amount went from P29, 261,902.45 to P40, 846,610.64 allegedly because of
the additional interest and penalties. AG&P requested for the deduction of these interests
and penalties for the delay of the Deed of Assignment was the fault of SSS.
Thus, AG&P filed a complaint for the specific performance and damages against SSS. SSS
contended that the court has no jurisdiction over the case in accordance with R.A. 8282
which provides that any dispute should be filed in the Commission. RTC ruled in favor of
AG&P. upon appeal, the CA held that the court has jurisdiction and that the case be reverted
back to the Trial Court for actual proceedings. Thus, SSS appealed to the

ISSUE:
A. WON dacion en pago should be implemented
B. Which body has jurisdiction over non-implementation of a dacion en pago agreed by the
parties?

HELD:
1. Yes. The Supreme Court absolutely adopted the CA decision. Dacion en pago is the
delivery and transmission of ownership of a thing by the debtor to the creditor as an
accepted equivalent of the performance of the obligation. It is a special mode of payment
where the debtor offers another thing to the creditor who accepts it as equivalent of payment
of an outstanding debt. The undertaking really partakes in one sense of the nature of sale,
that is the creditor is really buying the thing or property of the debtor, payment for which is to
be charged against the debtor’s debt. As such, the essential elements of a contract of sale,
namely, consent, object certain, and cause or consideration must be present. In its modern
concept, what actually takes place in dacion en pago is an objective novation of the
obligation where the thing offered as an accepted equivalent of the performance of an
obligation is considered as the object of the contract of sale, while the debt is considered as
the purchase price. In any case, common consent is an essential prerequisite, be it sale or
novation, to have the effect of totally extinguishing the debt or obligation.
From the averments in their complaint, the appellate court observed that private respondents
are seeking to implement the Deed of Assignment which they had drafted and submitted to
SSS pursuant to the approval by SSS. The appellate court thus held that the subject of the
complaint is no longer the payment of the premium and loan amortization delinquencies, as
well as the penalties appurtenant thereto, but the enforcement of the dacion en pago. Thus,
the trial court was ordered to settle the controversy.
From the allegations of respondents’ complaint, it readily appears that there is no longer any
dispute with respect to respondents’ accountability to the SSS. Respondents had, in fact
admitted their delinquency and offered to settle them by way of dacion en pago
subsequently approved by the SSS in Resolution No. 270-s. 2001. SSS stated in said
resolution that “the dacion en pago proposal of AG&P Co. of Manila and Semirara Coals
Corporation to pay their liabilities in the total amount ofP30,652,710.71 as of 31 March 2001
by offering their 5.8 ha. property located in San Pascual, Batangas, be, as it is hereby,
approved..” This statement unequivocally evinces its consent to the dacion en pago.
2. The controversy, instead, lies in the non-implementation of the approved and agreed
dacion en pago on the part of the SSS. As such, respondents filed a suit to obtain its
enforcement which is, doubtless, a suit for specific performance and one incapable of
pecuniary estimation beyond the competence of the Commission. Pertinently, the Court
ruled in Singson v. Isabela Sawmill, as follows:
In determining whether an action is one the subject matter of which is not capable of
pecuniary estimation this Court has adopted the criterion of first ascertaining the nature of
the principal action or remedy sought. If it is primarily for the recovery of a sum of money,
the claim is considered capable of pecuniary estimation, and whether jurisdiction in the
municipal courts or in the courts of first instancewould depend on the amount of the claim.
However, where the basic issue is something other than the right to recover a sum of
money, where the money claim is purely incidental to, or a consequence of, the principal

SALES CASE DIGEST COMPILATION 42


relief sought, this Court has considered such actions as cases where the subject of the
litigation may not be estimated in terms of money, and are cognizable exclusively by courts
of first instance (now Regional Trial Courts).

4.)MAR YUSON vs ATTY. JEREMIAS R. VITAN


A.C. No. 6955, July 27, 2006

FACTS:
In October 2002, Mar Yuson who was a taxi driver and had 8 children, received a
sum of money by way of inheritance. He and his wife intended to use the money for several
purposes.
When they were able to purchase a secondhand taxi, and Atty. Vitan helped him with legal
matters regarding the purchase. Unfortunately, Yuson’s other plans were put on hold when
Atty. Vitan borrowed P100, 000 from them in December 2002. To guarantee payment, Atty.
Vitan executed in favor of Yuson several postdated checks to over the loaned amount, but
however, these turned out to be worthless.
Yuson maintained that he had repeatedly tried to recover the debt, but was unsuccessful
every time. When no payment was still made pursuant to the administrative case against
Atty. Vitan, Yuson demanded a collateral to secure the loan. Thus, in his favor, Atty. Vitan
executed a document denominated as a Deed of Absolute Sale, covering Atty. Vitan’s parcel
of land located in Sta. Maria, Bulacan. According to Yuson, their intention was to transfer the
title of the property to him temporarily, so that he could either sell or mortgage the said land.
Further, if it was mortgaged, Atty. Vitan would redeem it as partial or full payment of the
loan. Allegedly, the parties executed another Deed of Absolute Sale in favor of Atty. Vitan
wherein Yuson was vendor. The purpose for this was not explained by either party.
Yuson was able to mortgage the property for P30,000 but contrary to their earlier
agreement, Atty. Vita did not redeem it from the mortgage, sent a letter instead, promising
Yuson to pay on or before July 12, 2004.
In the IBP-NCLA, Atty. Vitan averred that he had settled his obligation through a Deed of
Absolute Sale over his residential property. The purpose of such was for Yuson to use,
mortgage, or sell the property and return to him the excess of the proceeds after obtaining
his money. Additionally, he called the second document as a Counter Deed of Sale,
executed to be sort of a collateral/security for the account of his liaison officer Estur, whom
he alleged that she was the one who incurred said debts.

ISSUE: Whether or Not Atty. Vitan’s obligation was extinguished by virtue of the first
Deed of Absolute Sale.

HELD: NO. Atty. Vitan contends that his obligation was already extinguished, because he
had allegedly sold his Bulacan property to complainant. Basically, he is asserting that what
had transpired was a dation in payment. Governed by the law on sales, it is a transaction
that takes place when a piece of property is alienated to the creditor in satisfaction of a debt
in money. It involves delivery and transmission of ownership of a thing -- by the debtor to the
creditor -- as an accepted equivalent of the performance of the obligation. However, the
records reveal that he did not really intend to sell and relinquish ownership over his property
in Sta. Maria, Bulacan, notwithstanding the execution of a Deed of Absolute Sale in favor of
Yuson. The second Deed of Absolute Sale, which reconveyed the property to respondent, is
proof that he had no such intention. This second Deed, which he referred to as his "safety
net," betrays his intention to counteract the effects of the first one.
Ergo, Atty. Vitan was taking back with his right hand what he had given with his left. The
second Deed of Absolute Sale returned the parties right back where they started, as if there
were no sale in favor of complainant to begin with. In effect, on the basis of the second Deed
of Sale, respondent took back and asserted his ownership over the property despite having
allegedly sold it. Thus, he fails to convince us that there was a bona fide dation in payment

SALES CASE DIGEST COMPILATION 43


or sale that took place between the parties; that is, that there was an extinguishment of
obligation.
It appears that the true intention of the parties was to use the Bulacan property to facilitate
payment. They only made it appear that the title had been transferred to complainant to
authorize him to sell or mortgage the property.Atty. Vitan himself admitted in his letter dated
July 30, 2004, that their intention was to convert the property into cash, so that payment
could be obtained by complainant and the excess returned to respondent. The records,
however, do not show that the proceeds derived were sufficient to discharge the obligation
of the lawyer fully; thus, he is still liable to the extent of the deficiency.

5.) AGRIFINA AQUINTEY VS SPOUSES FELECIDAD AND RICO TIBONG


G.R. NO. 166704,DECEMBER 20, 2006

FACTS: On May 6, 1999, petitioner Aquintey filed before RTC Baguio, a complaint for sum
of money and damages against respondents. Agrifina alleged that Felicidad secured loans
from her on several occasions at monthly interest rates of 6% to 7%. Despite demands,
spouses Tibong failed to pay their outstanding loans of P773,000,00 exclusive of interests.
However, spouses Tiong alleged that they had executed deeds of assignment in favor of
Agrifina amounting to P546,459 and that their debtors had executed promissory notes in
favor of Agrifina. Spouses insisted that by virtue of these documents, Agrifina became the
new collector of their debts. Agrifina was able to collect the total amount of P301,000 from
Felicdad’s debtors. She tried to collect the balance of Felicidad and when the latter reneged
on her promise, Agrifina filed a complaint in the office of the barangay for the collection of
P773,000.00. There was no settlement. RTC favored Agrifina. Court of Appeals affirmed the
decision with modification ordering defendant to pay the balance of total indebtedness in the
amount of P51,341,00 plus 6% per month.

ISSUE: Whether or not the deeds of assignment in favor of petitioner has the effect of
payment of the original obligation that would partially extinguish the same
RULING: YES. Substitution of the person of the debtor may be affected by delegacion.
Meaning, the debtor offers, the creditor accepts a third person who consent of the
substitution and assumes the obligation. It is necessary that the old debtor be released fro
the obligation and the third person or new debtor takes his place in the relation . Without
such release, there is no novation. Court of Appeals correctly found that the
respondent’s obligation to pay the balance of their account with petitioner was
extinguished pro tanto by the deeds of credit. CA decision is affirmed with the
modification that the principal amount of the respondents is P33,841.
In its modern concept, what actually takes place in dacion en pago is an objective novation
of the obligation where the thing offered as an accepted equivalent of the performance of an
obligation is considered as the object of the contract of sale, while the debt is considered as
the purchase price.

6.)FILINVEST CREDIT CORPORATION VS. PHILIPPINE ACETYLENE, CO., INC.


G.R. No. L-50449, January 30, 1982

FACTS: On October 30, 1971, the Philippine Acetylene Co., Inc., purchased from Alexander
Lim, as evidenced by a Deed of Sale, a Chevrolet 1969 model motor vehicle payable under
the terms and conditions of the promissory note provided by PhilAcetylene. As security for
the payment, PhilAcetylene executed a chattel mortgage over the same vehicle in favor of
Lim. Subsequently, Lim assigned to Filinvest Finance all his rights, title, and interests in the
promissory note and the chattel mortgage which subsequently assigned it to Filinvest Credit.
PhilAcetylene failed to comply with the terms in the promissory note and chattel mortgage.
With the choice of paying the full amount plus interest and charges or returning the
mortgaged property, PhilAcetylene informed Filinvest Credit that it was returning the
mortgaged property in full satisfaction of its indebtedness pursuant to Art. 1484 of the New

SALES CASE DIGEST COMPILATION 44


Civil Code. When it was returned to Filinvest, it had with it a document denominated as
“Voluntary Surrender with Special Power of Attorney to Sell.”
Filinvest could not however sell the vehicle since there were unpaid taxes on said vehicle.
Upon Filinvest’s offer to return the vehicle to PhilAcetylene, the latter refused to accept it.
PhilAectylene contends that Filinvest has no cause of action since its obligation was
extinguished when it returned the mortgaged property to Filinvest and assuming however
that the return of the property did not extinguish its obligation, it was nonetheless justified in
refusing payment since Filinvest is not entitled to recover the same due to the breach of
warranty committed by the original vendor-assignor Alexander Lim. Additionally, it argues
that by virtue of the return, it extinguished their obligation through dation in payment.

ISSUE: Whether or Not the return of the mortgaged motor vehicle to Filinvest by
virtue of its voluntary surrender by Philippine Acetylene totally extinguished and/or
cancelled its obligation to Filinvest .

HELD: NO. The mere return of the mortgaged motor vehicle by the mortgagor,
PhilAcetylene, to the mortgagee, Filinvest, does not constitute dation in payment or dacion
en pago in the absence, express or implied of the true intention of the parties. Dacion en
pago, according to Manresa, is the transmission of the ownership of a thing by the debtor to
the creditor as an accepted equivalent of the performance of obligation.
The evidence on the record fails to show that the mortgagee, consented, or at least
intended, that the mere delivery to, and acceptance by him, of the mortgaged motor vehicle
be construed as actual payment, more specifically dation in payment or dacion en pago. The
fact that the mortgaged motor vehicle was delivered to him does not necessarily mean that
ownership thereof, as juridically contemplated by dacion en pago, was transferred from
PhilAcetylene to Filinvest. In the absence of clear consent of Filinvest, there can be no
transfer of ownership of the mortgaged motor vehicle from appellant to appellee. Only
transfer of possession of the mortgaged motor vehicle took place, for it is quite possible that
Filinvest merely wanted to secure possession to forestall the loss, destruction, fraudulent
transfer of the vehicle to third persons, or its being rendered valueless if left in the hands of
PhilAcetylene.
The true intention of the parties is furnished by the document executed by appellant
captioned "Voluntary Surrender with Special Power of Attorney To Sell". The document
reveals that the possession of the mortgaged motor vehicle was voluntarily surrendered by
PhilAcetylene to Filinvest authorizing the latter to look for a buyer and sell the vehicle in
behalf of PhilAcetylene who retains ownership thereof, and to apply the proceeds of the sale
to the mortgage indebtedness, with the undertaking of the appellant to pay the difference, if
any, between the selling price and the mortgage obligation. With the stipulated conditions as
stated, Filinvest in essence was constituted as a mere agent to sell the motor vehicle which
was delivered to it, not as its property, for if it were, he would have full power of disposition
of the property, not only to sell it as is the limited authority given him in the special power of
attorney. Had Filinvest intended to completely release PhilAcetylene of its mortgage
obligation, there would be no necessity of executing the document. Nowhere in the said
document where it states, that the mere surrender of the mortgaged motor vehicle to the
appellee extinguished appellant’s obligation for the unpaid price.
On PhilAcetylene’s argument that by accepting the delivery of the mortgaged motor vehicle,
Filinvest is estopped from demanding payment of the unpaid obligation, the same is without
merit. As clearly set forth above, Filinvest never accepted the mortgaged motor vehicle in full
satisfaction of the mortgaged debt.
Under the law, the delivery of possession of the mortgagedproperty to the mortgagee, can
only operate to extinguish PhilAcetylene liability if Filinvest had actually caused the
foreclosure sale of the mortgaged property when it recovered possession thereof.

SALES CASE DIGEST COMPILATION 45


VII. DISTINGUISHED FROM LEASE
1.)FILINVEST CREDIT COROPORATION VS CA, JOSE SY BANG and ILUMINADA TAN
SY BANG
G.R. No. 82508, September 29, 1989

FACTS: Spouses Jose and Iluminada Sy Bang were engaged in the sale of gravel produced
from crushed rocks and used for construction purposes. They engaged the serviced of Mr.
Ruben Mercurio of Gemini Motor Sales, to look for a rock crusher. Mr. Mercurio then
referred them to Rizal Consolidated who had said machinery for sale.
They applied for financial assistance with Filinvest Credit regarding their purchase of the
machine. Fiinvest agreed to extend to the Spouses Sy Bang financial aid on the following
conditions: that the machinery be purchased in the Filinvest's name; that it be leased (with
option to purchase upon the termination of the lease period) to the Spouses Sy Bang; and
that Spouses Sy Bang execute a real estate mortgage as security for the amount advanced
by Filinvest. Accordingly, on May 18,1981, a contract of lease of machinery (with option to
purchase) was entered into by the parties whereby the spouses agreed to lease from
Filinvest the rock crusher for two years starting from July 5, 1981 payable as follows:
P10,000.00 – first 3 months, P23,000.00 – next 6 months, P24,800.00 – next 15 months. It
was likewise stipulated that at the end of the two-year period, the machine would be owned
by the spouses.
The spouses then issued a check for P150,550 as initial rental, and 24 postdated checks
corresponding to 24 monthly rentals in favor of Filinvest. They likewise executed a real
estate mortgage over two parcels of land to guarantee their compliance with the lease
contract. The rock crusher was then delivered to the spouses. However, 3 months later, the
souses stopped payment when petitioner had not acted on the complaints of the spouses
about the machine. As a consequence, petitioner extra-judicially foreclosed the real estate
mortgage. To thwart the impending auction, the spouses filed a complaint for rescission of
the contract of lease and annulment of the real estate mortgage.

ISSUE:
1) Whether or Not the nature of the contract is one of a contract of sale.
2) Whether or Not the remedies of the seller provided for in Article 1484 are
cumulative.

HELD:
1.) YES.

It is apparent here that the intent of the parties to the subject contract is for the so-called
rentals to be the installment payments. Upon the completion of the payments, then the rock
crusher, subject matter of the contract, would become the property of the private
respondents. This form of agreement has been criticized as a lease only in name.
Sellers desirous of making conditional sales of their goods, but who do not wish openly to
make a bargain in that form, for one reason or another, have frequently resorted to the
device of making contracts in the form of leases either with options to the buyer to purchase
for a small consideration at the end of term, provided the so-called rent has been duly paid,
or with stipulations that if the rent throughout the term is paid, title shall thereupon vest in the
lessee. It is obvious that such transactions are leases only in name. The so-called rent must
necessarily be regarded as payment of the price in installments since the due payment of
the agreed amount results, by the terms of bargain, in the transfer of title to the lessee.

2.) NO.

SALES CASE DIGEST COMPILATION 46


They are alternative. The seller of movable in installments, in case the buyer fails to pay 2 or
more installments, may elect to pursue either of the following remedies: (1) exact fulfillment
by the purchaser of the obligation; (2) cancel the sale; or (3) foreclose the mortgage on the
purchased property if one was constituted thereon. It is now settled that the said remedies
are alternative and not cumulative, and therefore, the exercise of one bars the exercise of
the others. Indubitably, the device – contract of lease with option to buy – is at times
resorted to as a means to circumvent Article 1484, particularly paragraph (3) thereof.
Through the set-up, the vendor, by retaining ownership over the property in the guise of
being the lessor, retains, likewise the right to repossess the same, without going through the
process of foreclosure, in the event the vendee-lessee defaults in the payment of the
installments. There arises therefore no need to constitute a chattel mortgage over the
movable sold. More important, the vendor, after repossessing the property and, in effect,
canceling the contract of sale, gets to keep all the installments-cum-rentals already paid.

VIII. DISTINGUISHED FROM CONTRACT TO SELL


1.)SPOUSES SERRANO VS. CAGUIAT
517 SCRA 57
G.R. NO. 139173 ,FEBRUARY 28, 2007

FACTS:
Spouses Serrano are registered owners of a lot located in Las Pinas. On March 23,
1990, Caguiat offered to buy the lot and the Serranos agreed to sell it at 1,500.00/sqm.
Caguiat then paid them a partial payment of 100,000.00 as evidenced by a receipt indicating
therein Caguiat’s promise to pay the remaining balance.
Respondent, after making known his readiness to pay the balance, requested from
petitioners the preparation of the necessary Deed of Sale.
Petitioners informed respondent in a letter that Amparo Herrera would be leaving for abroad
on or before April 15, 1990 and they are canceling the transaction and that respondent may
recover the earnest money (100,000) anytime. Petitioners also wrote him stating that they
already delivered a manager’s check to his counsel in said amount.
Respondent thus filed a complaint for specific performance and damages with the RTC of
Makati.
The trial court relying on Article 1482 of the Civil Code ruled that the payment of 100,000.00
being an earnest money signified perfection of the contract of sale and ordered the
petitioners toexecute a final deed of sale in favor of respondent.
The Court of Appeals denied petitioners’ motion for reconsideration in affirmation of the
lower court’s decision.

ISSUE: Whether or Not there was a contract of sale.

HELD:
No. The transaction was a contract to sell.
When petitioners declared in the Receipt of Partial Payment” that they –
“Received from Mr. Godofredo Caguiat the amount of one hundred thousand pesos as
Partial payment of our lot situated in Las Pinas…Mr. Caguiat promised to pay the balance of
the purchase price on or before March 23, 1990… And that we will execute and sign the final
deed of sale on this date,”
-- there can be no other interpretation than that they agreed to a conditional contract of
sale, consummation of which is subject only to the full payment of the purchase price.
A contract to sell is akin to a conditional sale where the efficacy or obligatory force of the
vendor’s obligation to transfer title is subordinated to the happening of a future and uncertain
event, so that if the suspensive condition does not take place, the parties would stand as if

SALES CASE DIGEST COMPILATION 47


the conditional obligation had never existed. The suspensive condition is commonly full
payment of the purchase price.
In this case, the “Receipt of Partial Payment” shows that the true agreement between the
parties is a contract to sell.
First, ownership of the parcel of land was retained by petitioners and was not to pass to
respondent until full payment of the purchase price. Second, the agreement between the
parties was not embodied in a deed of sale. The absence of a formal deed of conveyance
is a strong indication that the parties did not intend immediate transfer of ownership, but only
a transfer after full payment of the purchase price. Third, petitioners retained possession of
the certificate of the lot.
It is true that Article 1482 provides that whenever earnest money is given in a contract of
sale, it shall be considered as part of the price and proof of the perfection of the contract.
However, this article speaks of earnest money given in a contract of sale. In this case, the
earnest money forms part of the consideration only if the sale is consummated upon full
payment of the purchase price.
Clearly, respondent cannot compel petitioners to transfer ownership of the property to him.

ERWIN F.
2) MILA A. REYES vs VICTORIA T. TUPARAN
G.R. No. 188064 , June 1, 2011

FACTS:

Petitioner was the registered owner of a 1,274 square meter residential and
commercial lot located in Karuhatan, Valenzuela City. In December 1989, respondent leased
from petitioner a space on the ground floor of the RBJ Building for her pawnshop business.

On June 20, 1988, petitioner mortgaged the subject real properties to the Farmers
Savings Bank and Loan Bank, Inc. (FSL Bank) to secure a loan of ₱2,000,000.00 payable in
installments. On November 15, 1990, petitioners outstanding account on the mortgage
reached ₱2,278,078.13. Petitioner then decided to sell her real properties for at least
₱6,500,000.00 so she could liquidate her bank loan and finance her businesses.
Respondent verbally offered to conditionally buy petitioners real properties for
₱4,200,000.00 on the following terms: (a) payable in three (3) fixed installments; (b) the
conditional sale will be cancelled if the plaintiff (petitioner) can find a buyer of said properties
for the amount of ₱6,500,000.00

After, both parties worked together to obtain FSL Banks approval for respondent to
assume her (petitioners) outstanding bank account. The assumption would be part of
respondents purchase price for petitioners mortgaged real properties. On November 26,
1990, the parties and FSL Bank executed the corresponding Deed of Conditional Sale of
Real Properties with Assumption of Mortgage. Such contract contains stipulations:

8. That the title and ownership of the subject real properties shall remain with
the First Party until the full payment of the Second Party of the balance of the
purchase price and liquidation of the mortgage obligation of ₱2,000,000.00. Pending
payment of the balance and liquidation of the mortgage obligation, the Second Party
shall not sell, transfer and convey and otherwise encumber the subject real
properties without the written consent of the First and Third Party.

9. That upon full payment by the Second Party of the full balance of the
purchase price and the assumed mortgage obligation herein mentioned the Third
Party shall issue the corresponding Deed of Cancellation of Mortgage and the First

SALES CASE DIGEST COMPILATION 48


Party shall execute the corresponding Deed of Absolute Sale in favor of the Second
Party.

Since December 1990, respondent had taken possession of the subject real
properties and had been continuously collecting and receiving monthly rental income from
the tenants of the buildings and vendors of the sidewalk fronting the RBJ building without
sharing it with petitioner.

Respondent, however, defaulted in the payment of her obligations on their due dates.
To compensate for her delayed payments, respondent agreed to pay petitioner an interest of
6% a month. As of August 31, 1992, respondent had only paid ₱395,000.00, leaving a
balance of ₱805,000.00 as principal on the unpaid installments and ₱466,893.25 as unpaid
accumulated interest.

On September 2, 1992, respondent offered the amount of ₱751,000.00 only payable


on September 7, 1992, as full payment of the purchase price of the subject real properties
and demanded the simultaneous execution of the corresponding deed of absolute sale. On
September 10, 1992, Mila A. Reyes (petitioner) filed a complaint for Rescission of Contract
with Damages against Victoria T. Tuparan (respondent).

Respondent countered, among others, that the tripartite agreement erroneously


designated by the petitioner as a Deed of Conditional Sale of Real Property with
Assumption of Mortgage was actually a pure and absolute contract of sale with a term
period. It could not be considered a conditional sale because the acquisition of contractual
rights and the performance of the obligation therein did not depend upon a future and
uncertain event.

ISSUE:

Whether or no the Deed of Conditional Sale of Real Property with Assumption of


Mortgage is a contract to sell or contract of sale?

RULING:

The Court agrees with the ruling of the courts below that the subject Deed of
Conditional Sale with Assumption of Mortgage entered into by and among the two parties
and FSL Bank on November 26, 1990 is a contract to sell and not a contract of sale. The
title and ownership of the subject properties remains with the petitioner until the respondent
fully pays the balance of the purchase price and the assumed mortgage obligation.

Consistently, the Court handed down a similar ruling in the 2010 case of Heirs of
Atienza v. Espidol, where it was written:
Regarding the right to cancel the contract for non-payment of an installment,
there is need to initially determine if what the parties had was a contract of
sale or a contract to sell. In a contract of sale, the title to the property passes
to the buyer upon the delivery of the thing sold. In a contract to sell, on the
other hand, the ownership is, by agreement, retained by the seller and is not
to pass to the vendee until full payment of the purchase price. In the contract
of sale, the buyers non-payment of the price is a negative resolutory
condition; in the contract to sell, the buyers full payment of the price is a

SALES CASE DIGEST COMPILATION 49


positive suspensive condition to the coming into effect of the agreement. In
the first case, the seller has lost and cannot recover the ownership of the
property unless he takes action to set aside the contract of sale. In the
second case, the title simply remains in the seller if the buyer does not
comply with the condition precedent of making payment at the time specified
in the contract. Here, it is quite evident that the contract involved was one of a
contract to sell since the Atienzas, as sellers, were to retain title of ownership
to the land until respondent Espidol, the buyer, has paid the agreed price.
Indeed, there seems no question that the parties understood this to be the
case.

IX. ELEMENTS OF A CONTRACT OF SALE


1) IDA C. LABAGALA vs. NICOLASA T. SANTIAGO, AMANDA T. SANTIAGO
G.R. No. 132305. December 4, 2001
e.a.funa

FACTS

Jose T. Santiago is the registered owner of a parcel of land. Nicolasa and Amanda,
sisters of Jose (now respondents herein), claims to be co-owners of the subject property and
sued Jose for recovery of 2/3 share of the property.

Petitioner herein claimed to be the daughter of Jose and claims right over the subject
property through the a sale made by Jose in favor of the petitioner sometime in March 1979
when the latter was still a minor. Jose died intestate on February 6, 1984. Petitioner
registered the deed of sale only on January 26, 1987 to enable her to secure the
corresponding transfer certificate of title (TCT No. 172334) in petitioners name alone.
Petitioner admittedly did not pay any centavo for the property.

On August 5, 1987, respondents filed a complaint for recovery of title, ownership,


and possession against herein petitioner, Ida C. Labagala, before the Regional Trial Court of
Manila, to recover from her the 1/3 portion of said property pertaining to Jose but which
came into petitioners sole possession upon Joses death. Respondents insisted that the
deed of sale was a forgery. The deed showed that Jose affixed his thumbmark thereon but
respondents averred that, having been able to graduate from college, Jose never put his
thumbmark on documents he executed but always signed his name in full. They claimed that
Jose could not have sold the property belonging to his poor and unschooled sisters who
sacrificed for his studies and personal welfare. Respondents also pointed out that it is highly
improbable for petitioner to have paid the supposed consideration of P150,000 for the sale
of the subject property because petitioner was unemployed and without any visible means of
livelihood at the time of the alleged sale. They also stressed that it was quite unusual and
questionable that petitioner registered the deed of sale only on January 26, 1987, or almost
eight years after the execution of the sale.

ISSUE

Whether or not petitioner is entitled to Jose's 1/3 portion of the property he co-owned
with respondents, through sale.

SALES CASE DIGEST COMPILATION 50


RULING

Petitioner could not have given her consent to the contract, being a minor at the time.
Consent of the contracting parties is among the essential requisites of a contract, including
one of sale, absent which there can be no valid contract. Moreover, petitioner admittedly did
not pay any centavo for the property, which makes the sale void. Moreover, petitioner
admittedly did not pay any centavo for the property, which makes the sale void. Article 1471
of the Civil Code provides:

Art. 1471. If the price is simulated, the sale is void, but the act may be
shown to have been in reality a donation, or some other act or contract.

Neither may the purported deed of sale be a valid deed of donation. Again, as explained by
the Court of Appeals:

Even assuming that the deed is genuine, it cannot be a valid donation. It lacks the
acceptance of the donee required by Art. 725 of the Civil Code. Being a minor in 1979, the
acceptance of the donation should have been made by her father, Leon Labagala or [her]
mother Cornelia Cabrigas or her legal representative pursuant to Art. 741 of the same Code.
No one of those mentioned in the law - in fact no one at all - accepted the donation for Ida.

2) EUGENIO DOMINGO, et. al. Vs CA


G.R. No. 127540. October 17, 2001
e.a.funa

FACTS:

Paulina Rigonan owned three (3) parcels of land, located at Batac and Espiritu,
Ilocos Norte, including the house and warehouse on one parcel. She allegedly sold them to
private respondents, the spouses Felipe and Concepcion Rigonan, who claim to be her
relatives, which sale purportedly involved nine (9) parcels of land, inclusive of the three (3)
parcels in dispute, sold at the price of P850 on January 28, 1965; that since then, they had
been in continuous possession of the subject properties and had introduced permanent
improvements thereon;
In 1966, herein petitioners Eugenio Domingo, Crispin Mangabat and Samuel
Capalungan, who claim to be her closest surviving relatives, allegedly took possession of the
properties by means of stealth, force and intimidation, and refused to vacate the same.
Consequently, on February 2, 1976, herein respondent Felipe Rigonan filed a
complaint for reinvindicacion against petitioners in the Regional Trial Court of Batac, Ilocos
Norte, alleging that defendants (now petitioners) entered the properties illegally, and they
refused to leave them when asked to do so.
According to defendants, the alleged deed of absolute sale was void for being
spurious as well as lacking consideration. They said that Paulina Rigonan did not sell her
properties to anyone. As her nearest surviving kin within the fifth degree of consanguinity,
they inherited the three lots and the permanent improvements thereon when Paulina died in
1966. They said they had been in possession of the contested properties for more than 10
years.
ISSUE:
Whether or not respondents was able establish the existence and due execution of
the deed of sale?

SALES CASE DIGEST COMPILATION 51


RULING

We have to take into account the element of consideration for the sale. The price
allegedly paid by private respondents for nine (9) parcels, including the three parcels in
dispute, a house and a warehouse, raises further questions. Consideration is the why of a
contract, the essential reason which moves the contracting parties to enter into the contract.
On record, there is unrebutted testimony that Paulina as landowner was financially well off.
She loaned money to several people. We see no apparent and compelling reason for her to
sell the subject parcels of land with a house and warehouse at a meager price of P850 only.

The whole evidence on record does not show clearly that the fictitious P850.00
consideration was ever delivered to the vendor. Undisputably, the P850.00 consideration for
the nine (9) parcels of land including the house and bodega is grossly and shockingly
inadequate, and the sale is null and void ab initio.

NOTA BENE:
I submit that the AUTONOMY OF CONTRACTS is the rule. However,
it was only on the existence of the factual antecedents that the
Court considered the grossly and shockingly inadequate consideration,
to wit:

(a) That private respondents presented a carbon copy of this


deed, albeit a duplicate original, none of the witnesses directly
testified to prove positively and convincingly Paulinas execution
of the original deed of sale. The carbon copy did not bear her
signature, but only her alleged thumbprint. Juan Franco testified
during the direct examination that he was an instrumental
witness to the deed. However, when cross-examined and shown
a copy of the subject deed, he retracted and said that said deed
of sale was not the document he signed as witness. He declared
categorically he knew nothing about it;

(b) The ostensible irregularities abound regarding the execution


and registration of the alleged deed of sale.

3) SPS. RUDY PARAGAS and CORAZON B. PARAGAS vs. HRS. OF DOMINADOR


BALACANO
G.R. No. 168220. August 31, 2005
e.a.funa

FACTS

Prior to his death, Gregorio Balacano, an octogenarian, was admitted at the Veterans
General Hospital in Bayombong, Nueva Vizcaya on June 28, 1996 and stayed there until
July 19, 1996. He was transferred in the afternoon of July 19, 1996 to the Veterans Memorial
Hospital in Quezon City where he was confined until his death.

Gregorio, diagnosed to have liver cirrhosis, purportedly sold on July 22, 1996, or
barely a week prior to his death, to the Spouses Rudy (Rudy) and Corazon Paragas
(collectively, the Spouses Paragas). This sale appeared in a deed of absolute sale is
notarized. The Spouses Paragas then sold to Catalino.

SALES CASE DIGEST COMPILATION 52


The heirs of Domingo filed a complaint for annulment of sale and partition against
Catalino and the Spouses Paragas alleging that Gregorio was seriously ill, in fact dying at
that time, which vitiated his consent to the disposal of the property. On the other hand, for
the plaintiff, Atty. De Guzman explained that the execution of the deed was merely a
confirmation of a previous agreement between the Spouses Paragas and Gregorio that was
concluded at least a month prior to Gregorios death; that, in fact, Gregorio had previously
asked him to prepare a deed that Gregorio eventually signed on July 18, 1996. He also
explained that the deed, which appeared to have been executed on July 22, 1996, was
actually executed on July 18, 1996; he notarized the deed and entered it in his register only
on July 22, 1996. He claimed that he did not find it necessary to state the precise date and
place of execution (Bayombong, Nueva Vizcaya, instead of Santiago City) of the deed of
sale because the deed is merely a confirmation of a previously agreed contract between
Gregorio and the Spouses Paragas.

ISSUE

Whether or not Deed of Sale is valid.

RULING

We do not consider Atty. de Guzmans testimony sufficient evidence to establish the


fact that there was a prior agreement between Gregorio and the Spouses Paragas on the
sale. This testimony does not conclusively establish the meeting of the minds between
Gregorio and the Spouses Paragas on the price or consideration for the sale.

In Domingo v. Court of Appeals, the Court declared as null and void the deed of
sale therein inasmuch as the seller, at the time of the execution of the alleged contract, was
already of advanced age and senile. . We held

She died an octogenarian on March 20, 1966, barely over a year


when the deed was allegedly executed on January 28, 1965, but
before copies of the deed were entered in the registry allegedly on
May 16 and June 10, 1966. The general rule is that a person is not
incompetent to contract merely because of advanced years or by
reason of physical infirmities. However, when such age or infirmities
have impaired the mental faculties so as to prevent the person from
properly, intelligently, and firmly protecting her property rights then
she is undeniably incapacitated.

In the case at bar, the Deed of Sale was allegedly signed by Gregorio on his death
bed in the hospital. Gregorio was suffering from liver cirrhosis at that circumstances which
raise grave doubts on his physical and mental capacity to freely consent to the
contract.

SALES CASE DIGEST COMPILATION 53


X. SALES BY AND BETWEEN SPOUSES
1) HEIRS OF IGNACIA AGUILAR-REYES vs. SPOUSES CIPRIANO MIJARES AND
FLORENTINA MIJARES
G.R. No. 143826. August 28, 2003
e.a.funa

FACTS

The controversy stemmed from a dispute over Lot No. 4349-B-2, registered in the
name of Spouses Vicente Reyes and Ignacia Aguilar-Reyes. Said lot and the apartments
built thereon were part of the spouses conjugal properties having been purchased using
conjugal funds from their garments business.

Vicente and Ignacia were married in 1960, but had been separated de facto since
1974. Sometime in 1984, Ignacia learned that on March 1, 1983, Vicente sold Lot No. 4349-
B-2 to respondent spouses Cipriano and Florentina Mijares. On August 9, 1984, Ignacia,
through her counsel, sent a letter to respondent spouses demanding the return of her share
in the lot. Pending the appeal, Ignacia died and she was substituted by her compulsory
heirs. Respondent spouses claimed that they are buyers in good faith.

ISSUE

Whether the sale is void, valid, or voidable.

RULING

The sale is voidable [in its entirety.]

In the case of Heirs of Christina Ayuste v. Court of Appeals, it was categorically


held that

There is no ambiguity in the wording of the law. A sale of real


property of the conjugal partnership made by the husband without
the consent of his wife is voidable. The action for annulment must be
brought during the marriage and within ten years from the
questioned transaction by the wife. Where the law speaks in clear
and categorical language, there is no room for interpretation there is
room only for application.

Likewise, in Spouses Guiang v. Court of Appeals, the Court quoted with approval
the ruling of the trial court that under the Civil Code, the encumbrance or alienation of a
conjugal real property by the husband absent the wifes consent, is voidable and not void.
Thus

Under Article 166 of the Civil Code, the husband cannot generally alienate or
encumber any real property of the conjugal partnership without the wifes consent. The
alienation or encumbrance if so made however is not null and void. It is merely voidable. The
offended wife may bring an action to annul the said alienation or encumbrance. Thus, the
provision of Article 173 of the Civil Code of the Philippines, to wit:

Art. 173. The wife may, during the marriage and within ten years from
the transaction questioned, ask the courts for the annulment of any

SALES CASE DIGEST COMPILATION 54


contract of the husband entered into without her consent, when such
consent is required, or any act or contract of the husband which tends
to defraud her or impair her interest in the conjugal partnership
property. Should the wife fail to exercise this right, she or her heirs
after the dissolution of the marriage, may demand the value of property
fraudulently alienated by the husband.

In the case at bar, there is no dispute that the property is a conjugal property
having been purchased using the conjugal funds of the spouses during the
subsistence of their marriage. It is beyond cavil therefore that the sale of said lot to
respondent spouses without the knowledge and consent of Ignacia is voidable.

Even assuming that respondent spouses believed in good faith that Ignacia really
died when they executed the second deed of sale the fact remains that the sale of property
prior to Ignacias alleged demise was without her consent and therefore subject to
annulment.

2) Spouses ANTONIO and LUZVIMINDA GUIANG, petitioners, vs. COURT OF


APPEALS and GILDA CORPUZ, respondents.
G.R. No. 125172. June 26, 1998
e.a.funa

FACTS

Plaintiff Gilda Corpuz and defendant Judie Corpuz are legally married spouses. They
were married on December 24, 1968 in Bacolod City. Sometime on February 14, 1983, the
couple Gilda and Judie Corpuz, with plaintiff-wife Gilda Corpuz as vendee bought a parcel of
land (the property) from Manuel Callejo who signed as vendor through a conditional deed of
sale. Sometime on April 22, 1988, the couple Gilda and Judie Corpuz sold one-half portion
of the property to the defendants-spouses Antonio and Luzviminda Guiang. Plaintiff Gilda
Corpuz left for Manila sometime in June 1989. In the absence of his wife Gilda Corpuz,
defendant Judie Corpuz pushed through the sale of remaining half the property to defendant
Luzviminda Guiang thru a document known as Deed of Transfer of Rights.

Sometime on March 11, 1990, plaintiff returned home. For staying in their house sold
by her husband, plaintiff was complained against by defendant Luzviminda Guiang and her
husband Antonio Guiang before the Barangay authorities. On March 16, 1990, the parties
thereat signed a document known as amicable settlement. In full, the settlement provides
for, to wit:

That respondent, Mrs. Gilda Corpuz and her three children, namely:
Junie, Hariet and Judie to leave voluntarily the house of Mr. and Mrs.
Antonio Guiang, where they are presently boarding without any charge,
on or before April 7, 1990.

ISSUE

Whether or not the absence of respondents consent merely rendered the Deed
voidable under Article 1390 of the Civil Code.

RULING:

Article 1390, par. 2, refers to contracts visited by vices of consent, i.e., contracts
which were entered into by a person whose consent was obtained and vitiated through

SALES CASE DIGEST COMPILATION 55


mistake, violence, intimidation, undue influence or fraud. In this instance, private
respondents consent to the contract of sale of their conjugal property was totally inexistent
or absent.

Article. 124. The administration and enjoyment of the conjugal


partnership property shall belong to both spouses jointly. In case of
disagreement, the husbands decision shall prevail, subject to recourse
to the court by the wife for proper remedy, which must be availed of
within five years from the date of the contract implementing such
decision.

In the event that one spouse is incapacitated or otherwise unable to


participate in the administration of the conjugal properties, the other
spouse may assume sole powers of administration. These powers do
not include the powers of disposition or encumbrance which must have
the authority of the court or the written consent of the other spouse. In
the absence of such authority or consent, the disposition or
encumbrance shall be void. However, the transaction shall be
construed as a continuing offer on the part of the consenting spouse
and the third person, and may be perfected as a binding contract upon
the acceptance by the other spouse or authorization by the court before
the offer is withdrawn by either or both offerors.(165a) (Italics supplied)

In sum, the nullity of the contract of sale is premised on the absence [and not merely
vitiated] of private respondents consent. To constitute a valid contract, the Civil Code
requires the concurrence of the following elements: (1) cause, (2) object, and (3) consent,
the last element being indubitably absent in the case at bar.

Doctrinally and clearly, a void contract cannot be ratified.

3) ARTURO R. ABALOS, petitioner, vs. DR. GALICANO S. MACATANGAY, JR.,


respondent.
G.R. No. 155043. September 30, 2004
e.a.funa

FACTS

Spouses Arturo and Esther Abalos are the registered owners of a parcel of land.
Armed with a Special Power of Attorney dated June 2, 1988, purportedly issued by his
wife, Arturo executed a Receipt and Memorandum of Agreement (RMOA) dated October 17,
1989, in favor of respondent, binding himself to sell to respondent the subject property and
not to offer the same to any other party within thirty (30) days from date. Ostensibly, a
marital squabble was brewing between Arturo and Esther at the time and to protect his
interest, respondent caused the annotation of his adverse claim on the title of the spouses to
the property on November 14, 1989.

Respondent reiterated his demand upon them to comply with their obligation to turn
over possession of the property. Arturo and Esther failed to deliver the property which
prompted respondent to cause the annotation of another adverse claim. On January 12,
1990, respondent filed a complaint for specific performance with damages against
petitioners. Arturo filed his answer to the complaint while his wife was declared in default.

ISSUE

SALES CASE DIGEST COMPILATION 56


Whether or not the complaint for specific performance shall prosper.

RULING

The nullity of the RMOA as a contract of sale emanates not only from lack of Esthers
consent thereto but also from want of consideration and absence of respondents signature
thereon. Such nullity cannot be obliterated by Esthers subsequent confirmation of the
putative transaction as expressed in the Contract to Sell. Under the law, a void contract
cannot be ratified and the action or defense for the declaration of the inexistence of a
contract does not prescribe. A void contract produces no effect either against or in favor of
anyone it cannot create, modify or extinguish the juridical relation to which it refers.

The congruence of the wills of the spouses is essential for the valid disposition of
conjugal property. Where the conveyance is contained in the same document which bears
the conformity of both husband and wife, there could be no question on the validity of the
transaction. But when there are two documents on which the signatures of the spouses
separately appear, textual concordance of the documents is indispensable. Hence, in this
case where the wifes putative consent to the sale of conjugal property appears in a
separate document which does not, however, contain the same terms and conditions
as in the first document signed by the husband, a valid transaction could not have
arisen.

In all instances, the present law specifically requires the written consent of the other
spouse, or authority of the court for the disposition or encumbrance of conjugal partnership
property without which, the disposition or encumbrance shall be void.

4) CONCEPCION R. AINZA, substituted by her legal heirs, DR. NATIVIDAD A. TULIAO,


CORAZON A. JALECO and LILIA A. OLAYON, petitioners, vs. SPOUSES ANTONIO
PADUA and EUGENIA PADUA, respondents.
G.R. No. 165420. June 30, 2005
e.a.funa

FACTS

In April 1987, Ainza and her daughter Eugenia, married to Antonio, orally agreed that
Ainza pay P100k in exchange for half of the portion of Eugenia’s undivided conjugal property
(a lot located in QC). No Deed of Absolute Sale was executed. There was physical delivery
of the land through Concepcion’s other daughter (Natividad) acting as atty-in-fact.
Concepcion thereafter allowed Natividad and her husband occupy the purchased portion of
the land.

In 1994, Antonio caused the division of the lot into three (two were occupied by the
spouses), necessarily displacing Natividad. He also had each subdivision titled. Antonio
requested Natividad to vacate the premises. Antonio averred that his wife only admitted of
selling 1/3 of the property to Concepcion for which a receipt was issued signed by
Concepcion. The RTC ruled in favor of Concepcion. The CA reversed the RTC ruling.
Applying Article 124 of the Family Code, the Court of Appeals ruled that since the subject
property is conjugal, the written consent of Antonio must be obtained for the sale to be
valid.

ISSUE

SALES CASE DIGEST COMPILATION 57


Whether or not the contract of sale between Ainza and Eugenia is valid.

RULING

It is undisputed that the subject property was conjugal and sold by Eugenia in April
1987 or prior to the effectivity of the Family Code on August 3, 1988, Article 254 of which
repealed Title V, Book I of the Civil Code provisions on the property relations between
husband and wife. However, Article 256 thereof limited its retroactive effect only to cases
where it would not prejudice or impair vested or acquired rights in accordance with the Civil
Code or other laws. In the case at bar, vested rights of Concepcion will be impaired or
prejudiced by the application of the Family Code; hence, the provisions of the Civil Code
should be applied.

In Felipe v. Heirs of Aldon, et al. the legal effect of a sale of conjugal properties by the wife
without the consent of the husband was clarified, to wit:

In the instant case, Gimena, the wife, sold lands belonging to the
conjugal partnership without the consent of the husband and the sale is
not covered by the phrase except in cases provided by law.

The view that the contract made by Gimena is a voidable contract is


supported by the legal provision that contracts entered by the husband
without the consent of the wife when such consent is required, are
annullable at her instance during the marriage and within ten years
from the transaction questioned. (Art. 173, Civil Code).

The consent of both Eugenia and Antonio is necessary for the sale of the conjugal
property to be valid. Antonios consent cannot be presumed. Except for the self-serving
testimony of petitioner Natividad, there is no evidence that Antonio participated or consented
to the sale of the conjugal property. Eugenia alone is incapable of giving consent to the
contract. Therefore, in the absence of Antonios consent, the disposition made by
Eugenia is voidable.

5) MANUEL O. FUENTES and LETICIA L. FUENTES, vs CONRADO G. ROCA,


ANNABELLE R. JOSON, ROSE MARIE R. CRISTOBAL and PILAR MALCAMPO
G.R. No. 178902, April 21, 2010
e.a.funa

FACTS

On, Oct 11, 1982, Tarciano Roca bought a 358-square meter lot in Zambales from
his mother. Six years later in 1988, Tarciano offered to sell the lot to the petitioners Fuentes
spouses through the help of Atty. Plagata who would prepare the documents and
requirements to complete the sale. In the agreement between Tarciano and Fuentes
spouses there will be a Php 60,000 down payment and Php 140,000 will be paid upon the
removal of Tarciano of certain structures on the land and after the consent of the estranged
wife of Tarciano, Rosario, would be attained. Atty. Plagata went to Manila to get the
signature of Rosario but notarized the document at Zamboanga . The deed of sale was
executed January 11, 1989.

Tarciano and Rosario died while the Fuentes spouses and possession and control
over the lot. Eight years later in 1997, the children of Tarciano and Rosario filed a case to

SALES CASE DIGEST COMPILATION 58


annul the sale and reconvey the property on the ground that the sale was void since the
consent of Rosario was not attained.

Since Tarciano and Rosario were married in 1950, the CA concluded that their
property relations were governed by the Civil Code under which an action for annulment of
sale on the ground of lack of spousal consent may be brought by the wife during the
marriage within 10 years from the transaction. Consequently, the action that the Rocas, her
heirs, brought in 1997 fell within 10 years of the January 11, 1989 sale.

ISSUES

Whether or not the action for the declaration of nullity of that sale to the spouses
already prescribed;

HELD

Contrary to the ruling of the Court of Appeals, the law that applies to this case is the
Family Code, not the Civil Code. Although Tarciano and Rosario got married in 1950,
Tarciano sold the conjugal property to the Fuentes spouses on January 11, 1989, a few
months after the Family Code took effect on August 3, 1988.

The Family Code took effect on August 3, 1988. Its Chapter 4 on Conjugal
Partnership of Gains expressly superseded Title VI, Book I of the Civil Code on Property
Relations Between Husband and Wife. Further, the Family Code provisions were also made
to apply to already existing conjugal partnerships without prejudice to vested rights. Thus:

Art. 105. x x x The provisions of this Chapter shall also apply to


conjugal partnerships of gains already established between spouses
before the effectivity of this Code, without prejudice to vested rights
already acquired in accordance with the Civil Code or other laws, as
provided in Article 256. (n)

Consequently, when Tarciano sold the conjugal lot to the Fuentes spouses on
January 11, 1989, the law that governed the disposal of that lot was already the Family
Code.

In contrast to Article 173 of the Civil Code, Article 124 of the Family Code does not
provide a period within which the wife who gave no consent may assail her husbands sale of
the real property. It simply provides that without the other spouses written consent or a
court order allowing the sale, the same would be void. Article 124 thus provides:

Art. 124. x x x In the event that one spouse is incapacitated or


otherwise unable to participate in the administration of the conjugal
properties, the other spouse may assume sole powers of
administration. These powers do not include the powers of disposition
or encumbrance which must have the authority of the court or the
written consent of the other spouse. In the absence of such authority or
consent, the disposition or encumbrance shall be void. x x x

Under the provisions of the Civil Code governing contracts, a void or inexistent
contract has no force and effect from the very beginning. And this rule applies to contracts
that are declared void by positive provision of law, as in the case of a sale of conjugal
property without the other spouses written consent. A void contract is equivalent to nothing

SALES CASE DIGEST COMPILATION 59


and is absolutely wanting in civil effects. It cannot be validated either by ratification or
prescription.

But, although a void contract has no legal effects even if no action is taken to set it
aside, when any of its terms have been performed, an action to declare its inexistence is
necessary to allow restitution of what has been given under it. This action, according to
Article 1410 of the Civil Code does not prescribe. Thus:

Art. 1410. The action or defense for the declaration of the


inexistence of a contract does not prescribe.

Here, the Rocas filed an action against the Fuentes spouses in 1997 for annulment
of sale and reconveyance of the real property that Tarciano sold without their mothers (his
wifes) written consent. The passage of time did not erode the right to bring such an action.

6) DAVID V. PELAYO and LORENZA* B. PELAYO, Petitioners, vs MELKI E. PEREZ,


Respondent.
G.R. No. 141323, June 8, 2005
e.a.funa

FACTS

David Pelayo (Pelayo),by a Deed of Absolute Sale executed on January 11, 1988,
conveyed to Melki Perez (Perez) two parcels of agricultural land (the lots) situated in
Panabo, Davao which are portions of Lot 4192, Cad. 276 covered by OCT P-16873. Loreza
Pelayo (Loreza), wife of Pelayo, and another one whose signature is illegible
witnessed the execution of the deed. Loreza, however, signed only on the third page in
the space provided for witnesses on account of which Perez application for registration of
the deed with the Office of the Register of Deeds in Tagum, Davao was denied.

Perez thereupon asked Loreza to sign on the first and second pages of the deed but
she refused, hence, he instituted on August 8, 1991 the instant complaint for specific
performance against her and her husband Pelayo (defendants).

The CA reversed and set aside the RTC Decision, declaring as valid and enforceable
the questioned deed of sale and ordering herein petitioner Lorenza Pelayo to affix her
signature on all pages of said document.

ISSUE

Whether or not the sale is valid.

RULING

We agree with the CA ruling that petitioner Lorenza, by affixing her signature to the
Deed of Sale on the space provided for witnesses, is deemed to have given her implied
consent to the contract of sale.

Sale is a consensual contract that is perfected by mere consent, which may either be
express or implied. A wifes consent to the husbands disposition of conjugal property does
not always have to be explicit or set forth in any particular document, so long as it is shown
by acts of the wife that such consent or approval was indeed given. In the present case,
although it appears on the face of the deed of sale that Lorenza signed only as an

SALES CASE DIGEST COMPILATION 60


instrumental witness, circumstances leading to the execution of said document point to the
fact that Lorenza was fully aware of the sale of their conjugal property and consented to the
sale.

The foregoing circumstances lead the Court to believe that Lorenza knew of the full
import of the transaction between respondent and her husband; and, by affixing her
signature on the deed of sale, she, in effect, signified her consent to the disposition of their
conjugal property.

7) MERCEDES CALIMLIM- CANULLAS, petitioner, vs. HON. WILLELMO FORTUN,


Judge, Court of First instance of Pangasinan, Branch I, and CORAZON DAGUINES,
respondents.
G.R. No. L-57499, June 22, 1984
e.a.funa

FACTS

Petitioner MERCEDES Calimlim-Canullas and FERNANDO Canullas were married


on December 19, 1962. In 1978, FERNANDO abandoned his family and was living with
private respondent Corazon DAGUINES. During the pendency of this appeal, they were
convicted of concubinage in a judgment rendered on October 27, 1981 by the then Court of
First Instance of Pangasinan, Branch II, which judgment has become final.

On April 15, 1980, FERNANDO sold the subject property with the house thereon to
DAGUINES for the sum of P2,000.00. In the document of sale, FERNANDO described the
house as "also inherited by me from my deceased parents."

Unable to take possession of the lot and house, DAGUINES initiated a complaint on
June 19, 1980 for quieting of title and damages against MERCEDES. The latter resisted and
claimed that the house in dispute where she and her children were residing, including the
coconut trees on the land, were built and planted with conjugal funds and through her
industry; that the sale of the land together with the house and improvements to DAGUINES
was null and void because they are conjugal properties and she had not given her consent
to the sale.

ISSUE

Whether or not the sale was valid.

RULING

XXX, the law emphatically prohibits the spouses from selling property to each other
subject to certain exceptions.6 Similarly, donations between spouses during marriage are
prohibited. 7 And this is so because if transfers or con conveyances between spouses were
allowed during marriage, that would destroy the system of conjugal partnership, a basic
policy in civil law. It was also designed to prevent the exercise of undue influence by one
spouse over the other,8 as well as to protect the institution of marriage, which is the
cornerstone of family law. The prohibitions apply to a couple living as husband and wife
without benefit of marriage, otherwise, "the condition of those who incurred guilt would turn
out to be better than those in legal union." Those provisions are dictated by public interest
and their criterion must be imposed upon the wig of the parties. That was the ruling in
Buenaventura vs. Bautista, also penned by Justice JBL Reyes (CA) 50 O.G. 3679, and

SALES CASE DIGEST COMPILATION 61


cited in Matabuena vs. Cervantes. We quote hereunder the pertinent dissertation on this
point:

We reach a different conclusion. While Art. 133 of the Civil Code


considers as void a donation between the spouses during the marriage,
policy considerations of the most exigent character as wen as the
dictates of morality require that the same prohibition should apply to
a common-law relationship.

XI. OTHER RELATIVELY DISQUALIFIED (1491)


GUARDIANS, AGENTS AND ADMINISTRATORS

1) THE PHILIPPINE TRUST COMPANY, as Guardian of the Property of the minor,


MARIANO L. BERNARDO, Petitioner, vs. SOCORRO ROLDAN, FRANCISCO
HERMOSO, FIDEL C. RAMOS and EMILIO CRUZ, Respondents.
G.R. No. L-8477. May 31, 1956.
e.a.funa

FACTS

Mariano Bernardo, a minor, inherited 17 parcels of land from his deceased father.
Respondent, Mariano’s step-mother, was appointed his guardian. As guardian, she sold the
17 parcels to Dr. Ramos, her brother-in-law, for P14,700. After a week, Dr. Ramos sold the
lands to her for P15,000. Subsequently, she sold 4 out of 17 parcels to Emilio Cruz.
Petitioner replaced Roldan as guardian, and two months thereafter, this litigation sought to
declare as null and void the sale to Dr. Ramos, and the sale to Emilio Cruz.

ISSUE

Whether the sale of the land is null and void.

RULING

Remembering the general doctrine that guardianship is a trust of the highest order,
and the trustee cannot be allowed to have any inducement to neglect his ward’s interest,
and in line with the court’s suspicion whenever the guardian acquires ward’s property we
have no hesitation to declare that in this case, in the eyes of the law, Socorro Roldan took by
purchase her ward’s parcels thru Dr. Ramos, and that Article 1459 of the Civil Code applies
(Article 1491 New Civil Code).

We are aware of course that in Rodriguez vs. Mactal, 60 Phil. p. 13 wherein the
guardian Mactal sold in January 1926 the property of her ward to Silverio Chioco, and in
March 1928 she bought it from Chioco, this Court said:

“In order to bring the sale in this case within the part of Article 1459,
quoted above, it is essential that the proof submitted establish some
agreement between Silverio Chioco and Trinidad Mactal to the effect
that Chioco should buy the property for the benefit of Mactal. If there

SALES CASE DIGEST COMPILATION 62


was no such agreement, either express or implied, then the sale cannot
be set aside (Page 16)”

However, the underlined portion was not intended to establish a general principle of
law applicable to all subsequent litigations. It merely meant that the subsequent purchase by
Mactal could not be annulled in that particular case because there was no proof of a
previous agreement between Chioco and her.

2) RICARDO DISTAJO, ERNESTO DISTAJO, RAUL DISTAJO, FEDERICO DISTAJO,


ZACARIAS A. DISTAJO, EDUARDO DISTAJO, and PILAR DISTAJO TAPAR,
petitioners, vs. COURT OF APPEALS and LAGRIMAS SORIANO DISTAJO,
respondents.
G.R. No. 112954. August 25, 2000
e.a.funa

FACTS

During the lifetime of Iluminada Abiertas, she designated one of her sons, Rufo
Distajo, to be the administrator of her parcels of land. On May 29, 1963, Iluminada Abiertas
certified to the sale of Lot Nos. 1046 and 1047 in favor of Rufo Distajo. On June 4, 1969,
Iluminada Abiertas sold Lot No. 1057 to Rhodora Distajo, the daughter of Rufo Distajo. On
July 12, 1969, Iluminada Abiertas sold Lot No. 1018 to Rufo Distajo.

When Iluminada Abiertas died in 1971, Zacarias Distajo, Pilar Distajo-Tapar, and
Rizaldo Distajo, demanded possession of the seven parcels of land from Lagrimas S.
Distajo, and her husband, Rufo Distajo contending that Rufo Distajo cannot acquire the
subject parcels of land owned by Iluminada Abiertas because the Civil Code prohibits
the administrator from acquiring properties under his administration.

ISSUE

Whether or not the sale transactions are void for having been entered into by the
administrator of the properties.

RULING

[The sale is valid]

We disagree. The pertinent Civil Code provision provides:

Art. 1491. The following persons cannot acquire by purchase, even at a


public or judicial auction, either in person or through the mediation of
another:

(1) The guardian, the property of the person or persons who may be
under guardianship;
(2) Agents, the property whose administration or sale may have been
entrusted to them, unless the consent of the principal has been given;
(3) Executors and administrators, the property of the estate under
administration; x x x

SALES CASE DIGEST COMPILATION 63


Under paragraph (2) of the above article, the prohibition against agents purchasing
property in their hands for sale or management is not absolute. It does not apply if the
principal consents to the sale of the property in the hands of the agent or administrator. In
this case, the deeds of sale signed by Iluminada Abiertas shows that she gave consent to
the sale of the properties in favor of her son, Rufo, who was the administrator of the
properties.

Thus, the consent of the principal Iluminada Abiertas removes the transaction out of
the prohibition contained in Article 1491(2).

SALES CASE DIGEST COMPILATION 64


.
3) G.R. No. L-18727 August 31, 1964
JESUS MA. CUI vs. ANTONIO MA. CUI

Facts:

The Hospicio is a charitable institution established by the spouses Don Pedro Cui and Doña
Benigna Cui, now deceased, "for the care and support, free of charge, of indigent invalids,
and incapacitated and helpless persons." It acquired corporate existence by and endowed
with extensive properties by the said spouses through a series of donations, principally the
deed of donation executed on 2 January 1926.
Section 2 of Act No. 3239 gave the initial management to the founders jointly and, in case of
their incapacity or death, to "such persons as they may nominate or designate, in the order
prescribed to them."

After the death of the spouses, the administration passed to Mauricio Cui and Dionisio
Jakosalem. Which was then followed by Dr. Teodoro Cui, only son of Mauricio Cui.
Thereafter, beginning in 1932, a series of controversies and court litigations ensued
concerning the position of administrator.

Plaintiff Jesus Ma. Cui and defendant Antonio Ma. Cui are brothers, being the sons of
Mariano Cui, one of the nephews of the spouses Don Pedro Cui and Doña Benigna Cui. In
1960 the then incumbent administrator, Dr. Teodoro Cui, resigned in favor of Antonio Ma.
Cui pursuant to a "convenio" entered into between them and embodied in a notarial
document. Jesus Ma. Cui, however, had no prior notice of either the "convenio" or of his
brother's assumption of the position.

Upon the death of Dr. Teodoro Cui, Jesus wrote a letter to Antonio demanding that the office
be turned over to him. The demand not having been complied with Jesus filed the complaint
in this case. Romulo Cui later on intervened, claiming a right to the same office, being a
grandson of Vicente Cui, another one of the nephews mentioned by the founders of the
Hospicio in their deed of donation.

Issue:
WON Jesus Cui is qualified and therefore the rightful administrator of the Hospicio.
Held:

NO. Jesus is not qualified and Antonio should be the administrator of the Hospicio.

Jesus is the older of the two and therefore under equal circumstances would be preferred
pursuant to section 2 of the deed of donation. However, before the test of age may be,
applied the deed gives preference to the one, among the legitimate descendants of the
nephews therein named, should be “titulo de abogado” (lawyer), a doctor or a civil engineer
or a pharmacist, in that order. If failing these qualifications, should be the one who pays the
highest taxes among the other qualified.

The specific point in dispute is the meaning of the term "titulo de abogado." Jesus Ma. Cui
holds the degree of Bachelor of Laws from the University of Santo Tomas but is not a
member of the Bar, not having passed the examinations to qualify him as one. Antonio Ma.
Cui, on the other hand, is a member of the Bar and although disbarred in 1957, was
reinstated by resolution promulgated in 1960, about two weeks before he assumed the
position of administrator of the Hospicio de Barili.

SALES CASE DIGEST COMPILATION 65


The term "titulo de abogado" means not mere possession of the academic degree of
Bachelor of Laws but membership in the Bar after due admission thereto, qualifying one for
the practice of law. A Bachelor's degree alone, conferred by a law school upon completion
of certain academic requirements, does not entitle its holder to exercise the legal profession.
The English equivalent of "abogado" is lawyer or attorney-at-law. This term has a fixed and
general signification, and has reference to that class of persons who are by license officers
of the courts, empowered to appear, prosecute and defend, and upon whom peculiar duties,
responsibilities and liabilities are devolved by law as a consequence.

The founders of the Hospicio de San Jose de Barili established that it must be a lawyer, first
of all, because for all of the work of the administrator stipulated under Act No. 3239, it is to
be presumed, that a working knowledge of the law and a license to practice the profession
would be a distinct asset.

Under this particular criterion we hold that the plaintiff is not entitled, as against the
defendant, as the administrator. As far as moral character is concerned, the standard
required of one seeking reinstatement to the office of attorney cannot be less exacting than
that implied in paragraph 3 of the deed of donation as a requisite for the office which is
disputed in this case. When the defendant was restored to the roll of lawyers the restrictions
and disabilities resulting from his previous disbarment were wiped out.

4) A.M. Nos. 1302, 1391 and 1543 April 26, 1991


PAULINO VALENCIA vs. ATTY. ARSENIO FER CABANTING

Facts:

Paulino Valencia and his wife Romana allegedly bought a parcel of land, where they built
their residential house, from a certain Serapia Raymundo, an heir of Pedro Raymundo the
original owner. However, they failed to register the sale or secure a transfer certificate of title
in their names.

There was a land dispute between the Valencia spouses and Serapia Raymundo , another
heir of Pedro Raymundo. Since both were relatives and distant kin of Atty. Eduardo
Jovellanos, a conference was held in his to settle the land dispute. However, the parties
were not able to settle their differences.

Thereby, Serapia, assisted by Atty. Arsenio Fer. Cabanting, filed a complaint against Paulino
for the recovery of possession with damages, while the Valencias engaged the services of
Atty. Dionisio Antiniw. The latter advised them to present a notarized deed of sale in lieu of
the private document written in Ilocano. For this purpose, Paulino gave Atty. Antiniw an
amount of P200.00 to pay the person who would falsify the signature of the alleged vendor
was executed purporting to be a sale of the questioned lot.

The Trial Court rendered a decision in favor of plaintiff, Serapia Raymundo.

Paulino, thereafter, filed a Petition for Certiorari with Preliminary Injunction before the Court
of Appeals alleging that the trial court failed to provide a workable solution concerning his
house. While the petition was pending, the trial court, issued an order of execution stating
that "the decision in this case has already become final and executory. Hence, a writ of
execution was issued.

Four years after, Serapia sold 40 square meters of the litigated lot to Atty. Jovellanos and
the remaining portion she sold to her counsel, Atty. Arsenio Fer. Cabanting.

SALES CASE DIGEST COMPILATION 66


Thereafter, Paulino filed a disbarment proceeding against Atty. Cabanting on the ground that
said counsel allegedly violated Article 1491 of the New Civil Code as well as Article II of the
Canons of Professional Ethics, prohibiting the purchase of property under litigation by a
counsel.

The appellate court dismissed the petition of Paulino.

Constancia Valencia, daughter of Paulino, filed a disbarment proceeding against Attys.


Eduardo Jovellanos and Arsenio Cabanting for purchasing a litigated property allegedly in
violation of Article 1491 of the New Civil Code.

Issue:

WON Attys. Eduardo Jovellanos and Arsenio Cabanting purchased the subject property in
violation to the provitions of Article 1491 of the New Civil Code.

Held:

Yes, the purchase of the property by Atty. Cabanting in this case constitutes malpractice in
violation of Art. 1491. While it is true that Atty. Arsenio Fer. Cabanting purchased the lot after
finality of judgment, there was still a pending certiorari proceeding. A thing is said to be in
litigation not only if there is some contest or litigation over it in court, but also from the
moment that it becomes subject to the judicial action of the judge. Logic indicates, in
certiorari proceedings, that the appellate court may either grant or dismiss the petition.
Hence, it is not safe to conclude, for purposes under Art. 1491 that the litigation has
terminated when the judgment of the trial court become final while a certiorari connected
therewith is still in progress.

However, the sale in favor of Atty. Jovellanos does not constitute malpractice. There was no
attorney-client relationship between Serapia and Atty. Jovellanos, considering that the latter
did not take part as counsel. The transaction is not covered by Art. 1491 nor by the Canons
adverted to.

Art. 1491, prohibiting the sale to the counsel concerned, applies only while the litigation is
pending. Public policy prohibits the transactions in view of the fiduciary relationship involved.
It is intended to curtail any undue influence of the lawyer upon his client. Greed may get the
better of the sentiments of loyalty and disinterestedness. Any violation of this prohibition
would constitute and is a ground for suspension.

Wherefore, judgment is hereby rendered declaring: Arsenio Fer. Cabanting suspended from
the practice of law for six months and administrative cases against Attorney Eduardo
Jovellanos and additional charges therein dismissed.

5) A.C. No. 3046. October 26, 1998


REGALADO DAROY vs. ATTY. ESTEBAN ABECIA

Facts:

Atty. Esteban Abecia was counsel of complainant Daroy in a forcible entry case. They won
and to satisfy the judgment, the sheriff sold at public auction on a parcel of land belonging to
one of the defendants to complainant Daroy as highest bidder for P1,250.00. Upon failure of
the defendants to redeem the land, its ownership was consolidated in complainant Daroy.

SALES CASE DIGEST COMPILATION 67


A complaint for falsification of public document was filed against respondent Atty. Abecia by
Daroy. The latter claimed that respondent Abecia forged his signature in a deed of absolute
sale, transferring the subject parcel of land to Jose Gangay purportedly for the sum of
P1,250.00 and that in a fictitious deed of absolute sale, it was made to appear that Gangay
in turn conveyed the land to Nena Abecia, wife of respondent Abecia, for the sum of
P1,350.00. Complainant alleged that he entrusted the title to the land to Abecia as his
counsel and allowed him to take possession of the land upon the latter’s request. By means
of the forged deed of sale, Abecia was able to obtain new transfer certificates of title, first in
the name of Gangay and then in that of Mrs. Abecia, from the Registry of Deeds.

However, Atty. Abecia maintained it was Daroy who sold the land in question to Jose
Gangay, and the latter in turn sold the land to Nena Abecia.

The RTC rendered a report finding respondent Abecia guilty of malpractice and
recommending his disbarment.

Issue:

WON Atty. Abecia can validly acquire the land in question.

Held:

The parties were mistaken in thinking that respondent could not validly acquire the land.

The prohibition in Art. 1491 does not apply to the sale of a parcel of land, acquired by a
client to satisfy a judgment in his favor, to his attorney as long as the property was not the
subject of the litigation. For indeed, while judges, prosecuting attorneys, and others
connected with the administration of justice are prohibited from acquiring property or rights in
litigation or levied upon in execution, the prohibition with respect to attorneys in the case
extends only to property and rights which may be the object of any litigation in which they
may take part by virtue of their profession.

The parties in this case thought the transfer of the land to respondent Abecia was prohibited
and so they contrived a way whereby the land would be sold to Jose Gangay, whose wife
Anita is the sister of Mrs. Nena Abecia, and then Gangay would sell the land to Mrs. Abecia.

Wherefore, the complaint against respondent Atty. Esteban Abecia is dismissed.

6) A.C. No. 6210 December 9, 2004


Federico N. Ramos vs. Atty. Patricio A. Ngaseo

Facts:

Federico N. Ramos filed a complaint before the IBP charging his former counsel, respondent
Atty. Ngaseo, of violation of the Code of Professional Responsibility for demanding the
delivery of 1,000 sq. m. parcel of land which was the subject of litigation.

Federico Ramos alleged that he went to respondent Atty. Patricio Ngaseos to engage his
services as counsel in a case involving a piece of land. Respondent agreed to handle the
case for an acceptance fee of P20,000.00, appearance fee of P1,000.00 per hearing and the
cost of meals, transportation and other incidental expenses. Complainant alleges that he did
not promise to pay the respondent 1,000 sq. m. of land as appearance fees.

SALES CASE DIGEST COMPILATION 68


Complainant went to the respondent’s office to inquire about the status of the case.
Respondent informed him that the decision was adverse to them however respondent
assured him that they could still appeal the adverse judgment and asked for the additional
amount of P3,850.00 and another P2,000.00 as allowance for research made.

Respondent claims that after the trial court dismissed Civil Case No. SCC 2128, he filed a
timely notice of appeal and thereafter moved to be discharged as counsel because he had
colon cancer. Complainant, now assisted by one Johnny Ramos, implored respondent to
continue handling the case, with an offer to double the 1,000 sq. m. piece of land earlier
promised and the remaining balance of P20,000.00 acceptance fee. Johnny Ramos made a
written commitment and gave respondents secretary P2,000.00 of the P3,850.00 expenses
for the preparation of the appellants brief.

The Court of Appeals rendered a favorable decision ordering the return of the disputed 2-
hectare land to the complainant and his siblings. The said decision became final and
executory on January 18, 2002. Since then complainant allegedly failed to contact
respondent, which compelled him to send a demand letter on January 29, 2003.

Respondent argues that he did not violate Article 1491 of the Civil Code because when he
demanded the delivery of the 1,000 sq. m. of land which was offered and promised to him in
lieu of the appearance fees, the case has been terminated, when the appellate court ordered
the return of the 2-hectare parcel of land to the family of the complainant.

Issue:

WON the demands made by Atty. Patricio A. Ngaseo constitute a violation of Art. 1491 of
the New Civil Code.

Held:

In all cases where Article 1491 was violated, the illegal transaction was consummated with
the actual transfer of the litigated property either by purchase or assignment in favor of the
prohibited individual.

In the instant case, there was no actual acquisition of the property in litigation since the
respondent only made a written demand for its delivery which the complainant refused to
comply. Mere demand for delivery of the litigated property does not cause the transfer of
ownership, hence, not a prohibited transaction within the contemplation of Article 1491. Even
assuming arguendo that such demand for delivery is unethical; respondents act does not fall
within the purview of Article 1491. The letter of demand was made long after the judgment in
Civil Case became final and executory.

7) A.M. No. 133-J May 31, 1982


Bernardita R. Macariola Vs. Honorable Elias B. Asuncion

Facts:

Judge Asuncion rendered a decision on a complaint for partition on Civil Case No. 3010
where Bernardita R. Macariola is the defendant. It became final on June 8, 1863 for lack of
an appeal. And on October 16, 1963, a project of partition was submitted to Judge Asuncion.

SALES CASE DIGEST COMPILATION 69


Notwithstanding the fact that the project of partition was not signed by the parties
themselves but only by the respective counsel of plaintiffs and defendant, Judge Asuncion
approved it in his Order dated October 23, 1963.
One of the properties mentioned in the project of partition was Lot 1184. This lot according
to the decision was adjudicated to the plaintiffs in the Civil Case in equal shares subdividing
Lot 1184 into five lots denominated as Lot 1184-A to 1184-E.

Lot 1184-D was conveyed to Enriqueta D. Anota, a stenographer in Judge Asuncion's court ,
while Lot 1184-E which had an area of 2,172.5556 sq. meters was sold on July 31, 1964 to
Dr. Arcadio Galapon. The latter and his wife Sold a portion of Lot 1184-E with an area of
around 1,306 sq. meters to Judge Asuncion and his wife, Victoria S. Asuncion, which
particular portion was declared by the latter for taxation purposes.

On August 31, 1966, spouses Asuncion and spouses Galapon conveyed their respective
shares and interest in Lot 1184-E to "The Traders Manufacturing and Fishing Industries
Inc.". At the time of said sale Judge Asuncion was the President and Mrs. Asuncion is the
secretary the stockholders of the corporation.

Macariola then filed an instant complaint against Judge Asuncion with "acts unbecoming a
judge". He alleged that Judge Asuncion in acquiring by purchase a portion of Lot 1184-E
violated Article 1491 par. 5 of the New Civil Code with the other related laws.

Issue:

WON Judge Asuncion violated Article 1491 par. 5 of the New Civil Code in acquiring by
purchase a portion of Lot 1184-E.

Held:

NO. There is no merit in the contention of complainant that respondent Judge Elias B.
Asuncion violated Article 1491, paragraph 5, of the New Civil Code in acquiring by purchase
a portion of Lot No. 1184-E which was one of those properties involved in Civil Case No.
3010.

The prohibition in the aforesaid Article 1491 applies only to the sale or assignment of the
property which is the subject of litigation to the persons disqualified therein. For the
prohibition to operate, the sale or assignment of the property must take place during the
pendency of the litigation involving the property.

In the case at bar, the property was no longer subject of litigation. When the respondent
Judge purchased on March 6, 1965 a portion of Lot 1184-E, the decision in Civil Case No.
3010 which he rendered on June 8, 1963 was already final because none of the parties
therein filed an appeal within the reglementary period; hence, the lot in question was no
longer subject of the litigation.

Furthermore, respondent Judge did not buy the lot in question directly from the plaintiffs but
from Dr. Arcadio Galapon who earlier purchased from three of the plaintiffs, after the finality
of the decision in Civil Case No. 3010. The subsequent sale by spouses Asuncion and
spouses Galapon of their respective shares and interest in said lot to the Traders
Manufacturing and Fishing Industries, Inc., took place long after the finality of the decision in
Civil Case No. 3010 and of the subsequent two aforesaid orders therein approving the
project of partition.

Consequently, the sale of a portion of Lot 1184-E to respondent Judge having taken place
over one year after the finality of the decision in Civil Case No. 3010 as well as the two

SALES CASE DIGEST COMPILATION 70


orders approving the project of partition, and not during the pendency of the litigation, there
was no violation of paragraph 5, Article 1491 of the New Civil Code.

8) G.R. No. L-10439 October 17, 1916


Gan Tiangco vs. Silvino Pabinguit

Facts:

Candida Acabo was the owner of six parcels of land which she sold on June 12, 1911 to
Gan Tingco for P500. But the latter was unable to take possession of the six parcels of land
sold to him by Acabo, for they were in the possession of Silvino Pabinguit, who alleges
certain rights therein. He claims to have purchased them for P375 from Faustino Abad, a
minor who is only 19 years old, son of Candida Acabo; that Abad had become their owner
through purchase from Henry Gardner; that the latter, in turn, had owned them by reason of
having purchased them for P555 at a public auction on March 20, 1907.

Henry Gardner, stated that the deputy sheriff had executed in his favor a certificate of his
purchase at auction sale, but witness did not know where the document was and did not
need it because he, in turn, has sold everything he had purchased at that sale; that he was
formerly justice of the peace of the municipality of Guijulngan, of Tayasan, and knew of a
complaint by Silvestre Basaltos against Candida Acabo; that afterwards when the auction
was held, he took part therein, but that as he subsequently learned that he was forbidden to
do so, he sold what he had purchased to Faustino Abad, Candida Acabo's son, who was but
a boy at the time; that the writ of execution was returned to him and he made a record of that
matter; that he had it in the justice of the peace court and left it there when he ceased to
hold office, in 1909.

Candida Acabo testified that Alejandro Sanchez, while sheriff of Tayasan, did not take
possession of her lands by reason of the levy; that the only property which he levied upon
was four carabaos, and she did not know whether they had been sold at auction; and that
Sanchez had not told her that the lands had been levied upon, or that they had been sold at
auction.

Issues:
1. WON Henry Gardner had validly acquired the subject property through the
public auction and validly alienated the same to Faustino Abad.

2. WON Faustino Abad, who was a minor at the time of the sale, had validly
acquired the subject property and whether the subsequent sale to Silvino
Pabinguit was valid.

Held:

1.Gardner was prohibited from acquiring the ownership of Acabo's lands for being the justice
of the peace at the time of the public auction.

The appellant alleges that the property purchased by justice of the peace Gardner was not
the subject of litigation in the justice court; that the action was to recover a certain sum of
money, and that he had ordered the property sold on execution.

SALES CASE DIGEST COMPILATION 71


Judging from the legal precedents on which the Civil Code is based, it would not seem too
much to conclude that the said article of the Civil Code does not make any distinction
between properties in litigation. In effect, it appears to be as delicate a matter for a judge to
take part in the sale of property that had been the subject of litigation in his court, as to
intervene in auction of property which, though not directly litigated in his court, is
nevertheless levied upon and sold as the result of a writ of execution issued by him. What
the law intends to avoid is the improper interference with an interest of a judge in a thing
levied upon and sold by his order.

If under the law Gardner was prohibited from acquiring the ownership of Acabo's lands, then
he could not have transmitted to Faustino Abad the right of ownership that he did not
possess. What Gardner should have done in view of the fact that the sale, as he finally
acknowledged, was void, was to claim the price that had been deposited in court, and the
justice of the peace of Guijulngan should have declared the auction void and have ordered a
new sale to be held, besides correcting the errors that had been committed in the
proceedings

2. With respect to the sale made by Faustino Abad to Silvino Pabinguit, that Abad was
a minor at the time — a circumstance that deprived him of capacity to sell (Civil Code, art.
1263). Abad had no ownership to transmit to anyone and, besides, he had no personality to
enable him to contract by himself, on account of his lack of legal age.

XII. SUBJECT MATTER


1) G.R. No. L-26278 August 4, 1927
Leon Sibal Vs. Emiliano J. Valdez et al

Facts:

On May 11, 1923, the deputy sheriff by virtue of writ of execution in civil case No. 20203
(Macondray & Co., Inc. vs. Leon Sibal),levied an attachment on eight parcels of land
belonging to said Leon Sibal, designated in the second of attachment as parcels 1, 2, 3, 4, 5,
6, 7 and 8. On July 30, 1923, Macondray & Co., Inc., bought said eight parcels of land, at
the auction held by the sheriff of the Province of Tarlac, for the sum to P4,273.93. Within one
year from the sale of said parcel of land, and on the 24th day of September, 1923, Leon
Sibal, paid P2,000 to Macondray & Co., Inc., for the account of the redemption price of said
parcels of land, without specifying the particular parcels to which said amount was to
applied. The redemption price said eight parcels was reduced, by virtue of said transaction,
to P2,579.97 including interest.

On May 9 and 10, 1924, the deputy sheriff by virtue of a writ of execution in civil case No.
1301 (Emiliano J. Valdez vs. Leon Sibal) sold at public auction the personal properties of
Sibal, including the sugar cane in question to Valdez, who paid therefor the sum of P1,550,
of which P600 was for the sugar cane. Subsequently, the real property of Sibal consisted of
eleven parcels of land and a house and camarin situated in one of said parcels, was also
attached including all of his rights, interest and participation therein. At the auction, eight of
said eleven parcels, including the house and the camarin, were bought by Valdez and the
remaining three parcels were released from the attachment by virtue of claims presented by
third persons.

Macondray & Co. then sold and conveyed to Valdez for P2,579.97 all of its rights and
interest in the eight parcels of land acquired by it at public auction in connection with civil

SALES CASE DIGEST COMPILATION 72


case No. 20203. Said amount represented the unpaid balance of the redemption price of
said eight parcels, after payment by Leon Sibal of P2,000 on September 24, 1923, from the
account of the redemption price. Hence, Emilio J. Valdez became the absolute owner of said
eight parcels of land.

Issue:

1.WON the sugar cane in question is a personal property.


2.WON future crops to be harvested can be considered valid object of sale.

Held:
1. Growing crops raised by yearly labor and cultivation are considered personal property.

A crop raised on leased premises in no sense forms part of the immovable. It belongs to the
lessee, and may be sold by him, whether it be gathered or not, and it may be sold by his
judgment creditors. If it necessarily forms part of the leased premises the result would be
that it could not be sold under execution separate and apart from the land. If a lessee
obtains supplies to make his crop, the factor's lien would not attach to the crop as a separate
thing belonging to his debtor, but the land belonging to the lessor would be affected with the
recorded privilege. The law cannot be construed so as to result in such absurd
consequences.

For the purpose of attachment and execution, and for the purposes of the Chattel Mortgage
Law, "ungathered products" have the nature of personal property. Paragraph 2 of article 334
of the Civil Code has been modified by section 450 of Act No. 190 and by Act No. 1508 in
the sense that "ungathered products" as mentioned in said article of the Civil Code have the
nature of personal property. In other words, the phrase "personal property" should be
understood to include "ungathered products."

2. A valid sale may be made of a thing, which though not yet actually in existence, is
reasonably certain to come into existence as the natural increment or usual incident of
something already in existence, and then belonging to the vendor, and then title will vest in
the buyer the moment the thing comes into existence. (Emerson vs. European Railway Co.,
67 Me., 387; Cutting vs. Packers Exchange, 21 Am. St. Rep., 63.)

Things of this nature are said to have a potential existence. A man may sell property of
which he is potentially and not actually possessed. He may make a valid sale of the wine
that a vineyard is expected to produce; or the gain a field may grow in a given time; or the
milk a cow may yield during the coming year; or the wool that shall thereafter grow upon
sheep; or what may be taken at the next cast of a fisherman's net; or fruits to grow; or young
animals not yet in existence; or the good will of a trade and the like. The thing sold, however,
must be specific and identified. They must be also owned at the time by the vendor. (Hull vs.
Hull, 48 Conn., 250 [40 Am. Rep., 165].)

2) G.R. No. L-36902 January 30, 1982


LUIS PICHEL vs. PRUDENCIO ALONZO

Facts:
Prudencio Alonzo was awarded by the Government a parcel of land in accordance with
Republic Act No. 477. The award was cancelled by the Board of Liquidators on January 27,
1965 on the ground that, previous thereto, plaintiff was proved to have alienated the land to
another, in violation of law. In 1972, plaintiff's rights to the land were reinstated.

SALES CASE DIGEST COMPILATION 73


On August 14, 1968, plaintiff and his wife sold to defendant the fruits of the coconut trees
which may be harvested in the land in question for the period, September 15, 1968 to
January 1, 1976, in consideration of P4,200.00. Even as of the date of sale, however, the
land was still under lease to one, Ramon Sua, and it was the agreement that part of the
consideration of the sale, in the sum of P3,650.00, was to be paid by defendant directly to
Ramon Sua so as to release the land from the clutches of the latter. Pending said payment
plaintiff refused to snow the defendant to make any harvest.

In July 1972, defendant for the first time since the execution of the deed of sale in his favor,
caused the harvest of the fruit of the coconut trees in the land.

Issue:
WON the contract entered into between the parties is a deed of sale of fruits of the coconut
trees found in the vendor's land or a contract of lease of the land itself.

Held:

It is a contract of sale.

The "Deed of Sale dated August 14, 1968 is precisely what it purports to be. It is a document
evidencing the agreement of herein parties for the sale of coconut fruits of the land, and not
for the lease of the land itself. The document defines the object of the contract thus: "the
herein sale of the coconut fruits are for an the fruits on the aforementioned parcel of land
during the years ...(from) SEPTEMBER 15, 1968; up to JANUARY 1, 1976." Moreover, the
document in question expresses a valid contract of sale. It has the essential elements of a
contract of sale as defined under Article 1485 of the New Civil Code which provides thus:

Art. 1458. By the contract of sale one of the contracting parties obligates himself to transfer
the ownership of and to deliver a determinate thing, and the other to pay therefor a price
certain in money or its equivalent.

A contract of sale may be absolute or conditional.

The subject matter of the contract of sale in question are the fruits of the coconut trees on
the land during the years from September 15, 1968 up to January 1, 1976, which subject
matter is a determinate thing. Under Article 1461 of the New Civil Code, things having a
potential existence may be the object of the contract of sale. And in Sibal vs. Valdez, 50 Phil.
512, pending crops which have potential existence may be the subject matter of the sale.
Here, the Supreme Court, citing Mechem on Sales and American cases said which have
potential existence may be the subject matter of sale. Here, the Supreme Court, citing
Mechem on Sales and American cases said:

Mr. Mechem says that a valid sale may be made of a thing, which though not yet actually in
existence, is reasonably certain to come into existence as the natural increment or usual
incident of something already in existence, and then belonging to the vendor, and the title
will vest in the buyer the moment the thing comes into existence. (Emerson vs. European
Railway Co., 67 Me., 387; Cutting vs. Packers Exchange, 21 Am. St. Rep. 63) Things of this
nature are said to have a potential existence. A man may sell property of which he is
potentially and not actually possess. He may make a valid sale of the wine that a vineyard is
expected to produce; or the grain a field may grow in a given time; or the milk a cow may
yield during the coming year; or the wool that shall thereafter grow upon sheep; or what may
be taken at the next case of a fisherman's net; or fruits to grow; or young animals not yet in
existence; or the goodwill of a trade and the like. The thing sold, however, must be specific
and Identified. They must be also owned at the time by the vendor. (Hull vs. Hull 48 Conn.
250 (40 Am. Rep., 165) (pp. 522-523).

SALES CASE DIGEST COMPILATION 74


3) G.R. No. L-31271 April 29, 1974
Romeo Martinez and Leonor Suarez, Spouses vs. Hon. Court of Appeals,
Secretary and Undersecretary of Public Works & Communications

Facts:

The spouses Romeo Martinez and Leonor Suarez are the registered owners of two (2)
parcels of land. Both parcels of land are fishponds. The property involved in the instant case
is the second parcel. The disputed property was originally owned by one Paulino
Montemayor, who secured a "titulo real" over it way back in 1883. After the death of Paulino
Montemayor the said property passed to his successors-in-interest, Maria Montemayor and
Donata Montemayor, who in turn, sold it, as well as the first parcel, to a certain Potenciano
Garcia.

Because Potenciano Garcia was prevented by the then municipal president of Lubao, Pedro
Beltran, from restoring the dikes constructed on the contested property, Garcia filed a civil
case against Pedro Beltran to restrain the latter in his official capacity from molesting him in
the possession of said second parcel, and on even date, applied for a writ of preliminary
injunction, which was issued against said municipal president. The Court declared
permanent the preliminary injunction, which, decision, on appeal, was affirmed by the
Supreme Court.
On April 17, 1925 Garcia applied for the registration of both parcels of land in his name and
it was granted by the court. Thereafter, the ownership of these properties changed hands
until eventually they were acquired by the spouses.

To avoid any untoward incident, the disputants agreed to refer the matter to the Committee
on Rivers and Streams. This committee thereafter appointed a Sub-Committee which after
conducting an investigation and an ocular inspection of the contested property it submitted
its report that Parcel No. 2 was not a public river but a private fishpond owned by the herein
spouses. Hence, the Committee on Rivers and Streams rendered its decision restoring the
spouses to the exclusive possession, use and enjoyment of the creek in question which
forms part of their registered property.
However, the municipal officials refused to recognize the above decision. Thus, the spouses
instituted a Civil Case for a writ of preliminary injunction against said Mayor wherein said
municipal Mayor immediately elevated the injunction suit for review to the Supreme Court.
The writ of preliminary injunction was granted. Hence, the spouses proceeded to construct
the dikes in the disputed parcel of land.

However, some four (4) years later, the Secretary of Public Works and Communications
ordered another investigation of the said parcel of land and directed the spouses to remove
the dikes they had constructed, on the strength of the authority vested in him by Republic
Act No. 2056. The said order which gave rise to the instant proceedings, embodied a threat
that the dikes would be demolished should the spouses fail to comply therewith within thirty
(30) days.

The spouses Martinez replied to the order by commencing the present case, which was
decided in their favor by the lower Court. The Court of Appeals reversed the judgment of the
latter court.

Issue:

SALES CASE DIGEST COMPILATION 75


WON the spouses are innocent purchasers in good faith.

Held:

The spouses cannot be deemed as purchasers for value and in good faith.

Before purchasing a parcel of land, it cannot be contended that the spouses who were the
vendees did not know exactly the condition of the land that they were buying and the
obstacles or restrictions thereon that may be put up by the government in connection with
their project of converting Lot No. 2 in question into a fishpond. Nevertheless, they willfully
and voluntarily assumed the risks attendant to the sale of said lot. One who buys something
with knowledge of defect or lack of title in his vendor cannot claim that he acquired it in good
faith (Leung Lee v. Strong Machinery Co., et al., 37 Phil. 664).

The ruling that a purchaser of a registered property cannot go beyond the record to make
inquiries as to the legality of the title of the registered owner, but may rely on the registry to
determine if there is no lien or encumbrances over the same, cannot be availed of as against
the law and the accepted principle that rivers are parts of the public domain for public use
and not capable of private appropriation or acquisition by prescription.

4) MG.R. No. 176474 November 27, 2008


Heirs of Arturo Reyes, represented by Evelyn R. San Buenaventura vs Elena
Socco-Beltran

Facts:

The subject property in this case is a parcel of land identified as Lot No. 6-B. It was originally
part of a larger parcel of land, allocated to the Spouses Marcelo Laquian and Constancia
Socco. Upon their death the land was left to the siblings of Constancia, namely: Filomena
Eliza Socco, Isabel Socco de Hipolito, Miguel R. Socco, and Elena Socco-Beltran. The
subject property was adjudicated to respondent, but no title had been issued in her name.

On 25 June 1998, respondent Elena Socco-Beltran filed an application for the purchase of
Lot No. 6-B before the Department of Agrarian Reform (DAR), alleging that it was
adjudicated in her favor in the extra-judicial settlement of Constancia Soccos estate.

Petitioners filed their protest to respondents petition on the ground that the subject property
was sold by respondents brother, Miguel R. Socco, in favor of their father, Arturo Reyes, as
evidenced by the Contract to Sell, dated 5 September 1954. The Contract to Sell has the
following stipulation:

That I am one of the co-heirs of the Estate of the deceased Constancia Socco; and that I am
to inherit as such a portion of her lot consisting of Four Hundred Square Meters (400) more
or less located on the (sic) Zamora St., Municipality of Dinalupihan, Province of Bataan,
bounded as follows:
xxxx
That for or in consideration of the sum of FIVE PESOS (P5.00) per square meter, hereby
sell, convey and transfer by way of this conditional sale the said 400 sq.m. more or less unto
Atty. Arturo C. Reyes, his heirs, administrator and assigns x x x.

SALES CASE DIGEST COMPILATION 76


Petitioners averred that they took physical possession of the subject property in 1954 and
had been uninterrupted in their possession of the said property since then.

The parties entered into judicial and administrative trials in claiming a better right on the
disputed property.

Issue:
WON the Heirs of Reyes has validly acquired title to the disputed property by virtue of the
Contract to Sell instituted by Miguel Socco.

Held:

No. Petitioners cannot derive title to the subject property by virtue of the Contract to Sell.

It was unmistakably stated in the Contract and made clear to both parties thereto that the
vendor, Miguel R. Socco, was not yet the owner of the subject property and was merely
expecting to inherit the same as his share as a co-heir of Constancias estate. It was also
declared in the Contract itself that Miguel R. Socco’s conveyance of the subject to the buyer,
Arturo Reyes, was a conditional sale. It is, therefore, apparent that the sale of the subject
property in favor of Arturo Reyes was conditioned upon the event that Miguel Socco would
actually inherit and become the owner of the said property. Absent such occurrence, Miguel
R. Socco never acquired ownership of the subject property which he could validly transfer to
Arturo Reyes.

Under Article 1459 of the Civil Code on contracts of sale, the thing must be licit and the
vendor must have a right to transfer ownership thereof at the time it is delivered. The law
specifically requires that the vendor must have ownership of the property at the time it is
delivered. Petitioners claim that the property was constructively delivered to them in 1954 by
virtue of the Contract to Sell. However, as already pointed out by this Court, it was explicit in
the Contract itself that, at the time it was executed, Miguel R. Socco was not yet the owner
of the property and was only expecting to inherit it. Hence, there was no valid sale from
which ownership of the subject property could have transferred from Miguel Socco to Arturo
Reyes. Without acquiring ownership of the subject property, Arturo Reyes also could not
have conveyed the same to his heirs, herein petitioners.

Marcelo Laquian and Constancia Socco, having been identified as the original allocatee,
have fully paid for the subject property as provided under an agreement to sell. By the
nature of a contract or agreement to sell, the title over the subject property is transferred to
the vendee upon the full payment of the stipulated consideration. Upon the full payment of
the purchase price, and absent any showing that the allocatee violated the conditions of the
agreement, ownership of the subject land should be conferred upon the allocatee. Since the
extrajudicial partition transferring Constancia Soccos interest in the subject land to the
respondent is valid, there is clearly no need for the respondent to purchase the subject
property, despite the application for the purchase of the property erroneously filed by
respondent. The only act which remains to be performed is the issuance of a title in the
name of her legal heirs, now that she is deceased.

5) G.R. No. 104482. January 22, 1996

Belinda Tañedo for herself and in representation of her brothers and sisters, and
TEOFILA CORPUZ Tañedo, representing her minor daughter VERNA Tañedo vs. The
Court of Appeals, Spouses Ricardo M. Tañedo and Teresita Barera Tañedo

Facts:

SALES CASE DIGEST COMPILATION 77


On October 20, 1962, Lazaro Tañedo executed a notarized deed of absolute sale in favor of
his eldest brother, Ricardo Tañedo, and the latter’s wife, Teresita Barera whereby he
conveyed to the latter in consideration of P1,500.00, one hectare of whatever share he shall
have over his future inheritance from their parents. Upon the death of his father Matias,
Lazaro executed an Affidavit of Conformity dated February 28, 1980 to re-affirm, respect,
acknowledge and validate the sale he made in 1962.

On January 13, 1981, Lazaro executed another notarized deed of sale in favor of private
respondents covering his undivided ONE TWELVE (1/12) of a parcel of land known as Lot
191. In February 1981, Ricardo learned that Lazaro sold the same property to his children,
petitioners herein, through a deed of sale dated December 29, 1980. On June 7, 1982,
private respondents recorded the Deed of Sale in their favor in the Registry of Deeds.

Petitioners on July 16, 1982 filed a complaint for rescission (plus damages) of the deeds of
sale executed by Lazaro in favor of Spouses Ricardo and Teresita Tañedo covering the
property inherited by Lazaro from his father.

Issues:
1.WON Contract of Sale of a future inheritance executed between Lazaro and Ricardo on
October 20, 1962 is valid. And WON the subsequent execution of an Affidavit of Conformity
on February 28, 1980 ratified the sale.

2.WON the Spouses Ricardo Tañedo have better right to the disputed property.

Held:

1.The contract made in 1962 is not valid and cannot be the source of any right nor the
creator of any obligation between the parties.

Pursuant to Article 1347 of the Civil Code, no contract may be entered into upon a future
inheritance except in cases expressly authorized by law. Hence, the affidavit of conformity
dated February 28, 1980, insofar as it sought to validate or ratify the 1962 sale, is also
useless.

2. Yes, the spouses Ricardo have better right on the disputed property.

Article 1544 of the Civil Code governs the preferential rights of vendees in cases of multiple
sales.
It provides that:
xxxx
Should it be immovable property, the ownership shall belong to the person acquiring it who
in good faith first recorded it in the Registry of Property.
xxxx

The property in question is land, an immovable, and following the above-quoted law,
ownership shall belong to the buyer who in good faith registers it first in the registry of
property. Thus, although the deed of sale in favor of private respondents was later than the
one in favor of petitioners, ownership would vest in the former because of the undisputed
fact of registration. On the other hand, petitioners have not registered the sale to them at all.

Petitioners contend that they were in possession of the property and that private
respondents never took possession thereof. As between two purchasers, the one who
registered the sale in his favor has a preferred right over the other who has not registered
his title, even if the latter is in actual possession of the immovable property.

SALES CASE DIGEST COMPILATION 78


6) G.R. No. 143958. July 11, 2003
Alfred Fritz Frenzel vs. Ederlina P. Catito

Facts:

Petitioner Alfred Fritz Frenzel is an Australian citizen of German descent. He married


Teresita Santos, a Filipino citizen, in 1976. In 1981, Alfred and Teresita separated from bed
and board without obtaining a divorce.

Sometime in February 1983, Alfred met Ederlina Catito in Australia, a Filipina. Unknown to
Alfred, she was married to Klaus Muller, a German national. Alfred told Ederlina that he was
married but that he was eager to divorce his wife in Australia. Subsequently, Alfred proposed
marriage to Ederlina, but she refused.

During the period of their common-law relationship, Alfred aleggedly has purchased real and
personal properties and registered the same in the name of Ederlina. The properties include
house and lots in Manila and Davao, a parlor and beach resort, a car, guitar amplifier,
furniture, among others. The two also opened joint saving accounts where the funds of the
former were deposited.

When the relationship of Alfred and Ederlina deteriorated, the former filed suits against the
latter in the court. The first case was for recovery of real and personal properties located in
Quezon City and Manila. The second case was for specific performance, declaration of
ownership of real and personal properties, sum of money, and damages.

Issue:
WON Alfred, an alien, can validly recover the properties or the money used in purchasing
the properties.

Held:
The sales in question are in violation of the Constitution; hence, are null and void ab initio.

A contract that violates the Constitution and the law is null and void and vests no rights and
creates no obligations. It produces no legal effect at all.

The petitioner, being a party to an illegal contract, cannot come into a court of law and ask to
have his illegal objective carried out. One who loses his money or property by knowingly
engaging in a contract or transaction which involves his own moral turpitude may not
maintain an action for his losses. To him who moves in deliberation and premeditation, the
law is unyielding. The law will not aid either party to an illegal contract or agreement; it
leaves the parties where it finds them.

Under Article 1412 of the New Civil Code, the petitioner cannot have the subject properties
deeded to him or allow him to recover the money he had spent for the purchase thereof.
Equity as a rule will follow the law and will not permit that to be done indirectly which,
because of public policy, cannot be done directly. Where the wrong of one party equals that
of the other, the defendant is in the stronger position ... it signifies that in such a situation,
neither a court of equity nor a court of law will administer a remedy. The rule is expressed in
the maxims: ex dolo malo non oritur actio and in pari delicto potior est conditio defendentis.

The petitioner was fully aware that he was disqualified from acquiring and owning lands
under Philippine law even before he purchased the properties in question; and, to skirt the
constitutional prohibition, the petitioner had the deed of sale placed under the respondents
name as the sole vendee thereof. Such being the case, the plaintiff is subject to the

SALES CASE DIGEST COMPILATION 79


constitutional restrictions governing the acquisition of real properties in the Philippines by
aliens.

The petitioner cannot find solace in Article 1416 of the New Civil Code which reads:
Art. 1416. When the agreement is not illegal per se but is merely prohibited, and the
prohibition by the law is designed for the protection of the plaintiff, he may, if public policy is
thereby enhanced, recover what he has paid or delivered.[64]

The provision applies only to those contracts which are merely prohibited, in order to benefit
private interests. It does not apply to contracts void ab initio. The sale of the three parcels of
land in favor of the petitioner who is a foreigner is illegal per se. The transactions are void ab
initio because they were entered into in violation of the Constitution. Thus, to allow the
petitioner to recover the properties or the money used in the purchase of the parcels of land
would be subversive of public policy.

7) G.R. No. 135634. May 31, 2000


Heirs of Juan San Andres (Victor S. Ziga) and Salvacion S. Tria vs. Vicente
Rodriguez

Facts:

Juan San Andres was the registered owner of a lot where he sold a portion thereof,
consisting of 345 square meters, to respondent Vicente S. Rodriguez for P2,415.00. The
sale is evidenced by a Deed of Sale.

Upon the death of Juan San Andres on May 5, 1965, Ramon San Andres was appointed
judicial administrator of the decedent’s estate. Ramon San Andres engaged the services of
a geodetic engineer, Jose Peero, to prepare a consolidated plan of the estate. Engineer
Peero also prepared a sketch plan of the 345-square meter lot sold to respondent. From the
result of the survey, it was found that respondent had enlarged the area which he purchased
from the late Juan San Andres by 509 square meters.

Accordingly, the judicial administrator sent a letter to respondent demanding that the latter
vacate the portion allegedly encroached by him. However, respondent refused to do so,
claiming he had purchased the same from the late Juan San Andres. Thereafter, the judicial
administrator brought an action, in behalf of the estate of Juan San Andres, for recovery of
possession of the 509-square meter lot.

Respondent alleged that apart from the 345-square meter lot which had been sold to him by
Juan San Andres on September 28, 1964, the latter likewise sold to him the following day
the remaining portion of the lot consisting of 509 square meters, with both parties treating
the two lots as one whole parcel with a total area of 854 square meters. Respondent alleged
that the full payment of the 509-square meter lot would be effected within five (5) years from
the execution of a formal deed of sale after a survey is conducted over said property.

As proof of the sale to him of 509 square meters, respondent attached to his answer a
receipt signed by the late Juan San Andres representing an advance payment for a
residential lot adjoining his previously paid lot with the agreed price of Fifteen (15.00) Pesos
per square meter and the payment of the full consideration based on a survey shall be due
and payable in five (5) years period from the execution of the formal deed of sale.
Respondent also attached to his answer a letter of judicial administrator Ramon San Andres
asking payment of the balance of the purchase price where the respondent was only able to

SALES CASE DIGEST COMPILATION 80


pay P100 and only later on deposited in court the balance of the purchase price amounting
to P7,035.00 for the aforesaid 509-square meter lot.

While the proceedings were pending, judicial administrator Ramon San Andres died and
was substituted by his son Ricardo San Andres. On the other hand, respondent Vicente
Rodriguez died on August 15, 1989 and was substituted by his heirs.

The trial court rendered judgment in favor of petitioner. It ruled that there was no contract of
sale to speak of for lack of a valid object because there was no sufficient indication to
identify the property subject of the sale, hence, the need to execute a new contract.
Respondent appealed to the Court of Appeals, which rendered a decision reversing the
decision of the trial court.

Issue:

1.WON the second sale entered between Juan San Andres and Vicente S. Rodriguez of the
lot consisting of 509 square meters is valid.

2.WON the contract of sale entered the parties is absolute or conditional.


Held:

1.Yes, the sale is valid because all the essential elements of a contract of sale are present.

Art. 1458 of the Civil Code provides: By the contract of sale one of the contracting parties
obligates himself to transfer the ownership of and to deliver a determinate thing, and the
other to pay therefor a price certain in money or its equivalent.

A contract of sale may be absolute or conditional.

As thus defined, the essential elements of sale are the following: a) Consent or meeting of
the minds, that is, consent to transfer ownership in exchange for the price; b) Determinate
subject matter; and, c) Price certain in money or its equivalent.

Without any doubt, the receipt profoundly speaks of a meeting of the mind between San
Andres and Rodriguez for the sale of the property adjoining the 345 square meter portion
previously sold to Rodriguez on its three (3) sides excepting the frontage. The price is
certain, which is P15.00 per square meter. Evidently, this is a perfected contract of sale on a
deferred payment of the purchase price. All the pre-requisite elements for a valid purchase
transaction are present. Sale does not require any formal document for its existence and
validity. And delivery of possession of land sold is a consummation of the sale (Galar vs.
Husain, 20 SCRA 186 [1967]). A private deed of sale is a valid contract between the parties
(Carbonell v. CA, 69 SCRA 99 [1976]). Xsc

The subject lot is capable of being determined without the need of any new contract. The
fact that the exact area of these adjoining residential lots is subject to the result of a survey
does not detract from the fact that they are determinate or determinable. Under Article 1460
of the New Civil Code, a thing sold is determinate if at the time the contract is entered into,
the thing is capable of being determinate without necessity of a new or further agreement
between the parties.

2. The contract of sale between the parties is absolute.

A deed of sale is considered absolute in nature where there is neither a stipulation in the
deed that title to the property sold is reserved in the seller until full payment of the price, nor

SALES CASE DIGEST COMPILATION 81


one giving the vendor the right to unilaterally resolve the contract the moment the buyer fails
to pay within a fixed period.

In this case, there is no reservation of ownership nor a stipulation providing for a unilateral
rescission by either party. In fact, the sale was consummated upon the delivery of the lot to
respondent. Thus, Art. 1477 provides that the ownership of the thing sold shall be
transferred to the vendee upon the actual or constructive delivery thereof.

The stipulation that the "payment of the full consideration based on a survey shall be due
and payable in five (5) years from the execution of a formal deed of sale" is not a condition
which affects the efficacy of the contract of sale. It merely provides the manner by which the
full consideration is to be computed and the time within which the same is to be paid. But it
does not affect in any manner the effectivity of the contract. Consequently, the contention
that the absence of a formal deed of sale stipulated in the receipt prevents the happening of
a sale has no merit.

JULIUS

8) SUNCION ATILANO, CRISTINA ATILANO, ROSARIO ATILANO, assisted by their


respective husbands, HILARIO ROMANO, FELIPE BERNARDO, and MAXIMO
LACANDALO, ISABEL ATILANO and GREGORIO ATILANO, plaintiffs-appellees,
vs. LADISLAO ATILANO and GREGORIO M. ATILANO, defendants-appellants.

Facts: In 1916 Eulogio Atilano I purchased from Gerardo Villanueva a lot, No. 535 of the
then municipality of Zamboanga cadastre. Eulogio I thereafter obtained transfer certificate of
title No. 1134 in his name. In 1920 he had the land subdivided into five parts, identified as
lots Nos. 535-A, 535-B, 535-C, 535-D and 535-E. On May 18 of the same year, after the
subdivision had been effected, Eulogio Atilano I, for the sum of P150.00, executed a deed of
sale covering lot No. 535-E in favor of his brother Eulogio Atilano II, and had it registered
under his name. Three other portions, (lots Nos. 535-B, 535-C and 535-D) were likewise
sold to other persons, and Eulogio 1 retained 535-A.

Eulogio Atilano I died and the title to this lot passed to Ladislao Atilano (defendant) in whose
name the corresponding certificate (No. T-5056) was issued. Eulogio Atilano II became a
widower. He and his children obtained transfer certificate of title No. 4889 over lot No. 535-E
in their names as co-owners. He and his children had the land resurveyed so that it could
properly be subdivided. Thereafter, it was discovered that the land Eulogio Atilano II and his
children were actually occupying on the strength of the deed of sale executed in 1920 was
lot No. 535-A and not lot 535-E, as referred to in the deed.

When Eulogio Atilano II died, his heirs (plaintiff) filed the present action in the CFI of
Zamboanga, alleging that they had offered to surrender to the defendants the possession of
lot No. 535-A and demanded in return the possession of lot No. 535-E, but that the
defendants had refused to accept the exchange. [Why? lot No. 535-E has an area of 2,612
square meters as compared to the 1,808 square-meter area of lot No. 535-A] Defendants
alleged that the reference to lot No. 535-E in the deed of sale of May 18, 1920 was an
involuntary error; that the intention of the parties to that sale was to convey the lot correctly
identified as lot No. 535-A; that since 1916, when he acquired the entirety of lot No. 535, and
up to the time of his death, Eulogio Atilano I had been possessing and had his house on the
portion designated as lot No. 535-E and that Eulogio Atilano I even increased the area under

SALES CASE DIGEST COMPILATION 82


his possession when on June 11, 1920 he bought a portion of an adjoining lot, No. 536, from
its owner Fruto del Carpio. So defendants prayed that the plaintiffs be ordered to execute in
their favor the corresponding deed of transfer with respect to Lot No. 535-E.

Issue: WON the sale is valid considering that there is no determinate subject matter
referred to therein [or that there was mistake as to the subject matter referred to
therein].

Yes, it is valid.

Held: When one sells or buys real property—a piece of land, for example—one sells or buys
the property as he sees it, in its actual setting and by its physical metes and bounds, and
NOT by the mere lot number assigned to it in the certificate of title.

In the case at bar, the portion correctly referred to as lot No. 535-A was already in the
possession of the vendee, Eulogio Atilano II, who had constructed his residence therein. In
like manner the latter had his house on the portion correctly identified, after the subdivision,
as lot No. 535-E, even adding to the area thereof by purchasing a portion of an adjoining
property belonging to a different owner. The two brothers continued in possession of the
respective portions for the rest of their lives, obviously ignorant of the initial mistake in the
designation of the lot subject of the 1920 sale until 1959.

From all the facts and circumstances we are convinced that the object thereof, as intended
and understood by the parties, was that specific portion where the vendee was then already
residing, where he reconstructed his house, and where his heirs (plaintiffs) continued to
reside thereafter: namely, lot No. 535-A; and that its designation as lot No. 535-E in the deed
of sale was a simple mistake in the drafting of the document. The mistake did not vitiate the
consent of the parties, or affect the validity and binding effect of the contract between them.

2. Assuming it is valid, what is the remedy available?

Remedy here: The new Civil Code provides a remedy for such a situation by means of
reformation of the instrument. This remedy is available when, there having been a meeting
of the minds of the parties to a contract, their true intention is not expressed in the
instrument purporting to embody the agreement by reason of mistake, fraud, inequitable
conduct or accident. In the case at bar, however, the deed of sale executed in 1920 need no
longer be reformed. The parties have retained possession of their respective properties
conformably to the real intention of the parties to that sale, and all they should do is to
execute mutual deed of conveyance.

9) Melliza vs. City of Iloilo

Facts: Juliana Melliza donated to the then Municipality of Iloilo, 9,000 square meters of Lot
1214, to serve as site for the municipal hall. 1 The donation was however revoked by the
parties for the reason that the area donated was found inadequate to meet the requirements
of the development plan of the municipality, the so-called "Arellano Plan". 2

SALES CASE DIGEST COMPILATION 83


Subsequently, Lot No. 1214 was divided by Certeza Surveying Co., Inc. into Lots 1214-A
and 1214-B.

On November 15, 1932 Juliana Melliza executed an instrument conveying to city of ilolo:
Lots 1214-C and 1214-D but also such other portions of lots as were necessary for the
municipal hall site, such as Lot 1214-B.
On January 14, 1938 Juliana Melliza sold her remaining interest in Lot 1214 to Remedios
Sian Villanueva who thereafter obtained her own registered title thereto. Remedios in turn on
November 4, 1946 transferred her rights to said portion of land to Pio Sian Melliza, who
obtained Transfer Certificate of Title in his name.

On August 24, 1949 the City of Iloilo, donated the city hall site to the University of the
Philippines (Iloilo branch). The site donated consisted of Lots Nos. 1214-B, 1214-C and
1214-D, with a total area of 15,350 square meters, more or less.

Sometime in 1952, the University of the Philippines enclosed the site donated with a wire
fence. Pio Sian Melliza thereupon made representations, thru his lawyer, with the city
authorities for payment of the value of the lot (Lot 1214-B). No recovery was obtained,
because as alleged by plaintiff, the City did not have funds. On December 10, 1955 Pio Sian
Melliza filed an action in the Court of First Instance of Iloilo against Iloilo City and the
University of the Philippines for recovery of Lot 1214-B or of its value.

The Court of First Instance rendered its decision on August 15, 1957, dismissing the
complaint. Said court ruled that the instrument executed by Juliana Melliza in favor of Iloilo
municipality included in the conveyance Lot 1214-B.

Issue: WON disputed lot 1214-B was included as a subject matter in the public
instrument made by Mellia in favor of City.

Held: Petitioner's contention is without merit. There is no dispute that respondent purchased
a portion of Lot 1914-B-2 consisting of 345 square meters. This portion is located in the
middle of Lot 1914-B-2, which has a total area of 854 square meters, and is clearly what was
referred to in the receipt as the "previously paid lot." Since the lot subsequently sold to
respondent is said to adjoin the "previously paid lot" on three sides thereof, the subject lot is
capable of being determined without the need of any new contract. The fact that the exact
area of these adjoining residential lots is subject to the result of a survey does not detract
from the fact that they are determinate or determinable.

As the Court of Appeals explained:


Concomitantly, the object of the sale is certain and determinate. Under Article 1460 of the
New Civil Code, a thing sold is determinate if at the time the contract is entered into, the
thing is capable of being determinate without necessity of a new or further agreement
between the parties. Here, this definition finds realization.

Thus, all of the essential elements of a contract of sale are present, i.e., that there was a
meeting of the minds between the parties, by virtue of which the late Juan San Andres
undertook to transfer ownership of and to deliver a determinate thing for a price certain in
money.

SALES CASE DIGEST COMPILATION 84


10) Yu Tek vs. Gonzales

Facts: A written contract was executed between Basilio Gonzalez and Yu Tek and Co.,
where Gonzales was obligated to deliver 600 piculs of sugar of the 1st and 2nd grade to Yu
Tek, within the period of 3 months (1 January-31 March, 1912) at any place within the
municipality of Sta. Rosa, which Yu Tek & Co. or its representative may designate; and in
case, Gonzales does not deliver, the contract will be rescinded and Gonzales shall be
obligated to return the P3,000 received and also the sum of P1,200 by way of indemnity for
loss and damages.

No sugar had been delivered to Yu Tek & Co. under this contract nor had it been able to
recover the P 3,000. Yu Tek & Co. filed a complaint against Gonzales, and prayed for
judgment for the P 3,000 and the additional P1,200. Judgment was rendered for P 3,000
only, and from this judgment both parties appealed.

Issue: Whether or not there was a perfected contract of sale.

Held: No. This court has consistently held that there is a perfected sale with regard to the
“thing” whenever the article of sale has been physically segregated from all other articles.
Here, it is sought to show that the sugar was to be obtained exclusively from the crop raised
by the defendant. There is no clause in the written contract which even remotely suggests
such a condition.

The defendant undertook to deliver a specified quantity of sugar within a specified time. The
contract placed no restriction upon the defendant in the matter of obtaining the sugar. He
was equally at liberty to purchase it on the market or raise it himself. It may be true that
defendant owned a plantation and expected to raise the sugar himself, but he did not limit
his obligation to his own crop of sugar. In the case at bar, the undertaking of the defendant
was to sell to the plaintiff 600 piculs of sugar of the first and second classes. Was this an
agreement upon the “thing” which was the object of the contract? For the purpose of sale its
bulk is weighed, the customary unit of weight being denominated a “picul.” Now, if called
upon to designate the article sold, it is clear that the defendant could only say that it was
“sugar.” He could only use this generic name for the thing sold.
There was no “appropriation” of any particular lot of sugar. Neither party could point to any
specific quantity of sugar and say: “This is the article which was the subject of our contract.”

We conclude that the contract in the case at bar was merely an executory agreement; a
promise of sale and not a sale. There was no perfected sale. At there was no perfected sale,
it is clear that articles 1452, 1096, and 1182 of the New Civil Code are not applicable. The
defendant having defaulted in his engagement, the plaintiff is entitled to recover the P 3,000
which it advanced to the defendant, and this portion of the judgment appealed from must
therefore be affirmed.

11) NATIONAL GRAINS AUTHORITY and WILLLAM CABAL, petitioners vs.


THE INTERMEDIATE APPELLATE COURT and LEON SORIANO, respondents.

SALES CASE DIGEST COMPILATION 85


Facts: Petitioner National Grains Authority (now National Food Authority, NFA for short) is a
government agency created under Presidential Decree No. 4. One of its incidental functions
is the buying of palay grains from qualified farmers. On August 23, 1979, private respondent
Leon Soriano offered to sell palay grains to the NFA, through William Cabal, the Provincial
Manager of NFA stationed at Tuguegarao, Cagayan. He submitted the documents required
by the NFA for pre-qualifying as a seller, namely: (1) Farmer's Information Sheet
accomplished by Soriano and certified by a Bureau of Agricultural Extension (BAEX)
technician, Napoleon Callangan, (2) Xerox copies of four (4) tax declarations of the riceland
leased to him and copies of the lease contract between him and Judge Concepcion Salud,
and (3) his Residence Tax Certificate. Private respondent Soriano's documents were
processed and accordingly, he was given a quota of 2,640 cavans of palay. The quota noted
in the Farmer's Information Sheet represented the maximum number of cavans of palay that
Soriano may sell to the NFA.

In the afternoon of August 23, 1979 and on the following day, August 24, 1979, Soriano
delivered 630 cavans of palay. The palay delivered during these two days were not
rebagged, classified and weighed. when Soriano demanded payment of the 630 cavans of
palay, he was informed that its payment will be held in abeyance since Mr. Cabal was still
investigating on an information he received that Soriano was not a bona tide farmer and the
palay delivered by him was not produced from his farmland but was taken from the
warehouse of a rice trader, Ben de Guzman. On August 28, 1979, Cabal wrote Soriano
advising him to withdraw from the NFA warehouse the 630 cavans Soriano delivered stating
that NFA cannot legally accept the said delivery on the basis of the subsequent certification
of the BAEX technician, Napoleon Callangan that Soriano is not a bona fide farmer.

Instead of withdrawing the 630 cavans of palay, private respondent Soriano insisted that the
palay grains delivered be paid. He then filed a complaint for specific performance and/or
collection of money with damages on November 2, 1979, against the National Food
Authority and Mr. William Cabal, Provincial Manager of NFA with the Court of First Instance
of Tuguegarao, and docketed as Civil Case No. 2754.

Meanwhile, by agreement of the parties and upon order of the trial court, the 630 cavans of
palay in question were withdrawn from the warehouse of NFA. An inventory was made by
the sheriff as representative of the Court, a representative of Soriano and a representative of
NFA. RTC rendered judgment ordering petitioner National Food Authority, its officers and
agents to pay respondent Soriano (as plaintiff in Civil Case No. 2754) the amount of P
47,250.00 representing the unpaid price of the 630 cavans of palay plus legal interest
thereof. CA affirmed RTC decision.

Issue: WON there was a contract of sale.

Held: Yes. In the case at bar, Soriano initially offered to sell palay grains produced in his
farmland to NFA. When the latter accepted the offer by noting in Soriano's Farmer's
Information Sheet a quota of 2,640 cavans, there was already a meeting of the minds
between the parties.

The object of the contract, being the palay grains produced in Soriano's farmland and the
NFA was to pay the same depending upon its quality. The fact that the exact number of
cavans of palay to be delivered has not been determined does not affect the perfection of
the contract. Article 1349 of the New Civil Code provides: ". . .. The fact that the quantity is

SALES CASE DIGEST COMPILATION 86


not determinate shall not be an obstacle to the existence of the contract, provided it is
possible to determine the same, without the need of a new contract between the parties." In
this case, there was no need for NFA and Soriano to enter into a new contract to determine
the exact number of cavans of palay to be sold. Soriano can deliver so much of his produce
as long as it does not exceed 2,640 cavans.

Petitioners contend that the 630 cavans of palay delivered by Soriano on August 23, 1979
was made only for purposes of having
it offered for sale. Further, petitioners stated that the procedure then prevailing in matters of
palay procurement from qualified farmers were: firstly, there is a rebagging wherein the
palay is transferred from a private sack of a farmer to the NFA sack; secondly, after the
rebagging has been undertaken, classification of the palay is made to determine its variety;
thirdly, after the determination of its variety and convinced that it passed the quality
standard, the same will be weighed to determine the number of kilos; and finally, it will be
piled inside the warehouse after the preparation of the Warehouse Stock Receipt (WSP)
indicating therein the number of kilos, the variety and the number of bags.

Under this procedure, rebagging is the initial operative act signifying acceptance, and
acceptance will be considered complete only after the preparation of the Warehouse Stock
Receipt (WSR). When the 630 cavans of palay were brought by Soriano to the Carig
warehouse of NFA they were only offered for sale. Since the same were not rebagged,
classified and weighed in accordance with the palay procurement program of NFA, there
was no acceptance of the offer which, to petitioners' mind is a clear case of solicitation or an
unaccepted offer to sell. The petition is not impressed with merit.

12) Schuback and Sons vs. Court of Appeals

Facts: Sometime in 1981, defendant established contact with plaintiff through the Philippine
Consulate General in Hamburg, West Germany, because he wanted to purchase MAN bus
spare parts from Germany. Plaintiff communicated with its trading partner, Johannes
Schuback and Sohne Handelsgesellschaft M.B.N. & Co. (Schuback Hamburg), regarding
the spare parts defendant wanted to order.

On October 16, 1981, defendant submitted to plaintiff a list of the parts he wanted to
purchase with specific part numbers and description. Plaintiff referred the list to Schuback
Hamburg for quotations. Upon receipt of the quotations, plaintiff sent to defendant a letter
dated 25 November, 1981 enclosing its offer on the items listed by defendant.

On December 4, 1981, defendant informed plaintiff that he preferred genuine to replacement


parts, and requested that he be given 15% on all items. On December 17, 1981, plaintiff
submitted its formal offer containing the item number, quantity, part number, description, unit
price and total to defendant. On December, 24, 1981, defendant informed plaintiff of his
desire to avail of the prices of the parts at that time and enclosed Purchase Order No. 0101
dated 14 December 1981. Said Purchase Order contained the item number, part number
and description. Defendant promised to submit the quantity per unit he wanted to order on
December 28 or 29.

On December 29, 1981, defendant personally submitted the quantities he wanted to Mr.
Dieter Reichert, General Manager of plaintiff, at the latter's residence (t.s.n., 13 December,

SALES CASE DIGEST COMPILATION 87


1984, p. 36). The quantities were written in ink by defendant in the same Purchase Order
previously submitted. At the bottom of said Purchase Order, defendant wrote in ink above
his signature: "NOTE: Above P.O. will include a 3% discount. The above will serve as our
initial P.O."

Plaintiff immediately ordered the items needed by defendant from Schuback Hamburg to
enable defendant to avail of the old prices. Schuback Hamburg in turn ordered (Order No.
12204) the items from NDK, a supplier of MAN spare parts in West Germany. On January 4,
1982, Schuback Hamburg sent plaintiff a proforma invoice to be used by defendant in
applying for a letter of credit. Said invoice required that the letter of credit be opened in favor
of Schuback Hamburg. Defendant acknowledged receipt of the invoice. An order
confirmation was later sent by Schuback Hamburg to plaintiff which was forwarded to and
received by defendant on February 3, 1981.

On February 16, 1982, plaintiff reminded defendant to open the letter of credit to avoid delay
in shipment and payment of interest. Defendant replied, mentioning, among others, the
difficulty he was encountering in securing: the required dollar allocations and applying for the
letter of credit, procuring a loan and looking for a partner-financier, and of finding ways 'to
proceed with our orders".
In the meantime, Schuback Hamburg received invoices from, NDK for partial deliveries on
Order No.12204. Schuback Hamburg paid NDK. The latter confirmed receipt of payments
made on February 16, 1984.

On October 18, 1982, Plaintiff again reminded defendant of his order and advised that the
case may be endorsed to its lawyers. Defendant replied that he did not make any valid
Purchase Order and that there was no definite contract between him and plaintiff. Plaintiff
sent a rejoinder explaining that there is a valid Purchase Order and suggesting that
defendant either proceed with the order and open a letter of credit or cancel the order and
pay the cancellation fee of 30% of F.O.B. value, or plaintiff will endorse the case to its
lawyers.

Schuback Hamburg issued a Statement of Account to plaintiff enclosing therewith Debit


Note, charging plaintiff 30% cancellation fee, storage and interest charges in the total
amount of DM 51,917.81. Said amount was deducted from plaintiff's account with Schuback
Hamburg. Demand letters were sent to defendant by plaintiff's counsel dated March 22,
1983 and June 9, 1983 to no avail. Consequently, petitioner filed a complaint for recovery of
actual or compensatory damages, unearned profits, interest, attorney's fees and costs
against private respondent.
In its decision dated June 13, 1988, the trial court ruled in favor of petitioner by ordering
private respondent to pay petitioner, among others, actual compensatory damages in the
amount of DM 51,917.81, unearned profits in the amount of DM 14,061.07, or their peso
equivalent.

Thereafter, private respondent elevated his case before the Court of Appeals. On February
18, 1992, the appellate court reversed the decision of the trial court and dismissed the
complaint of petitioner. It ruled that there was no perfection of contract since there was no
meeting of the minds as to the price between the last week of December 1981 and the first
week of January 1982.

SALES CASE DIGEST COMPILATION 88


Issue: Whether or not a contract of sale has been perfected between the parties.

Held: Yes. The Supreme Court reversed the decision of the Court of Appeals and
reinstate the decision of the trial court. It bears emphasizing that a "contract of sale is
perfected at the moment there is a meeting of minds upon the thing which is the object of the
contract and upon the price. . .”

Article 1319 of the Civil Code states: "Consent is manifested by the meeting of the offer and
acceptance upon the thing and the cause which are to constitute the contract. The offer
must be certain and the acceptance absolute. A qualified acceptance constitutes a counter
offer." The facts presented to us indicate that consent on both sides has been manifested.
The offer by petitioner was manifested on December 17, 1981 when petitioner submitted its
proposal containing the item number, quantity, part number, description, the unit price and
total to private respondent. On December 24, 1981, private respondent informed petitioner
of his desire to avail of the prices of the parts at that time and simultaneously enclosed its
Purchase Order No. 0l01 dated December 14, 1981. At this stage, a meeting of the minds
between vendor and vendee has occurred, the object of the contract: being the spare parts
and the consideration, the price stated in petitioner's offer dated December 17, 1981 and
accepted by the respondent on December 24,1981.

Although said purchase order did not contain the quantity he wanted to order, private
respondent made good, his promise to communicate the same on December 29, 1981. At
this juncture, it should be pointed out that private respondent was already in the process of
executing the agreement previously reached between the parties.

There appears this statement made by private respondent: "Note. above P.O. will include a
3% discount. The above will serve as our initial P.O." This notation on the purchase order
was another indication of acceptance on the part of the vendee, for by requesting a 3%
discount, he implicitly accepted the price as first offered by the vendor. The immediate
acceptance by the vendee of the offer was impelled by the fact that on January 1, 1982,
prices would go up, as in fact, the petitioner informed him that there would be a 7% increase,
effective January 1982. On the other hand, concurrence by the vendor with the said discount
requested by the vendee was manifested when petitioner immediately ordered the items
needed by private respondent from Schuback Hamburg which in turn ordered from NDK, a
supplier of MAN spare parts in West Germany.

On the part of the buyer, the situation reveals that private respondent failed to open an
irrevocable letter of credit without recourse in favor of Johannes Schuback of Hamburg,
Germany. This omission, however, does not prevent the perfection of the contract between
the parties, for the opening of the letter of credit is not to be deemed a suspensive condition.
The facts herein do not show that petitioner reserved title to the goods until private
respondent had opened a letter of credit. Petitioner, in the course of its dealings with private
respondent, did not incorporate any provision declaring their contract of sale without effect
until after the fulfillment of the act of opening a letter of credit.

The opening of a letter of credit in favor of a vendor is only a mode of payment. It is not
among the essential requirements of a contract of sale enumerated in Article 1305 and 1474
of the Civil Code, the absence of any of which will prevent the perfection of the contract from
taking place.

SALES CASE DIGEST COMPILATION 89


13) Aurora Alcantara-Daus vs. Sps. Hermoso and Soccoro de Leon June 16, 2003

Facts: Hermoso De Leon inherited a parcel of land from his father by virtue of a Deed of
Extra-judicial Partition. In 1960s, he engaged the services of the late Atty. Juan to take care
of the documents of the properties of his parents. Atty. Juan let them sign voluminous
documents. After the death of Atty. Juan, some documents surfaced and most revealed that
their properties had been conveyed by sale or quitclaim to Hermoso’s brothers and sisters,
to Atty. Juan and his sisters, when in truth and in fact, no such conveyances were ever
intended by them. His sign in the quitclaim in favor of brother RODOLFO DE LEON was
forged. They just discovered that land was sold to herein petitioner AURORA, thus they
demanded for annulment of contracts and reconveyance.

AURORA averred that she bought it in good faith and for value on DECEMBER 1975 and
has been in continuous, public, peaceful, open possession over the same and has been
appropriating the produce thereof without objection from anyone.

RTC – In favor of AURORA and said claim was barred by laches because 18 years has
passed and that the deed was a notarial document hence presumptively authentic

CA – Reversed

Issue: WON the Deed of Absolute Sale dated December 6, 1975 executed by
RODOLFO (deceased) over the land in question in favor of AURORA was perfected
and binding upon the parties therein?

Ruling: A contract of sale is consensual. It is perfected by mere consent, upon a meeting of


the minds on the offer and the acceptance thereof based on subject matter, price and terms
of payment. At this stage, the seller’s ownership of the thing sold is not an element in the
perfection of the contract of sale.

The contract, however, creates an obligation on the part of the seller to transfer ownership
and to deliver the subject matter of the contract. It is during the delivery that the law requires
the seller to have the right to transfer ownership of the thing sold. In general, a perfected
contract of sale cannot be challenged on the ground of the seller’s non-ownership of the
thing sold at the time of the perfection of the contract.

Further, even after the contract of sale has been perfected between the parties, its
consummation by delivery is yet another matter. It is through tradition or delivery that the
buyer acquires the real right of ownership over the thing sold.

Undisputed is the fact that at the time of the sale, RODOLFO WAS NOT THE OWNER of
the land he delivered to petitioner AURORA. Thus, the consummation of the contract and
the consequent transfer of ownership would depend on whether he subsequently acquired
ownership of the land in accordance with Article 1434 of the Civil Code. Thus there’s a need
to resolve the issue of the authenticity and the due execution of the Extrajudicial Partition

SALES CASE DIGEST COMPILATION 90


and Quitclaim in his favor. However, after poring over the records, we find no reason to
reverse the factual finding of the appellate court. A comparison of the genuine signatures of
HERMOSO with his purported signature on the Deed of Extrajudicial Partition with Quitclaim
will readily reveal that the latter is a forgery.

Without the corroborative testimony of the attesting witnesses, the lone account of the notary
regarding the due execution of the Deed is insufficient to sustain the authenticity of this
document.

Issue on possession in good faith: It is well-settled that no title to registered land in


derogation of that of the registered owner shall be acquired by prescription or adverse
possession. Neither can prescription be allowed against the hereditary successors of the
registered owner, because they merely step into the shoes of the decedent and are merely
the continuation of the personality of their predecessor in interest.

Issue on prescription and laches: Article 1141 of the New Civil Code provides that real
actions over immovable properties prescribe after thirty years. This period for filing an action
is interrupted when a complaint is filed in court. RODOLFO alleged that the land had been
allocated to him by his brother HERMOSO in March 1963, but that the Deed of Extrajudicial
Partition assigning the contested land to the latter was executed only on September 16,
1963. In any case, the Complaint to recover the land from petitioner was filed on February
24, 1993, which was within the 30-year prescriptive period.

Also, laches cannot be used to defeat justice or to perpetuate fraud and injustice. Thus, the
assertion of laches to thwart the claim of HERMOSO is foreclosed, because the Deed upon
which petitioner bases her claim is a forgery. THUS, assailed decision is affirmed.

14) NOEL V CA

Facts: Gregorio Nanaman and Hilaria Tabuclin (Nanaman spouses) were a childless,
legally-married couple. Gregorio, however, had a child named Virgilio Nanaman by another
woman. Since he was two years old, Virgilio was reared by Gregorio and Hilaria. He was
sent to school by the couple until he reached third year of the law course.

During their marriage, Gregorio and Hilaria acquired certain property including a 34.7-
hectare land in Tambo, Iligan City on which they planted sugarcane, corn and bananas.
They also lived there with Virgilio and fifteen tenants.

On October 2, 1945, Gregorio died. Hilaria then administered the property with the help of
Virgilio enjoyed the procedure of the land to the exclusion of Juan Nanaman, the brother of
Gregorio, and Esperanza and Caridad Nanaman, Gregorio's daughters by still another
woman.

In 1953, Virgilio declared the property in his name for taxation purposes.

SALES CASE DIGEST COMPILATION 91


On November 1, 1952, Hilaria and Virgilio, mortgaged the 34.7-hectare land in favor of
private respondent, in consideration of the amount of P4,800.00

On February 16, 1954, Hilaria and Virgilio executed a deed of sale over the same tract of
land also in favor of private respondent in consideration of the sum of P16,000.00. On May
15, 1954, Hilaria died. On October 27, 1954, Esperanza and Caridad Nanaman filed
intestate estate proceedings concerning the estate of their father, Gregorio. Included in the
list of property of the estate was the 34.7-hectare land.

On June 16, 1956, when Edilberto Noel took over as regular administrator of the estate, he
was not able to take possession of the land in question because it was in the possession of
private respondent and some heirs of Hilaria.

Later, Private respondent and the heirs of the Nanaman spouses executed an amicable
settlement of the Nanaman estate. In the document, private respondent agreed "to relinquish
his rights to one-half (1/2) of the entire parcel of land in Tambo, Iligan City, indicated in item
1 under the Estate, sold to him by Hilaria Tabuclin, infavor of all the heirs of the
abovementioned intestate [estate] for the reason that not all of the heirs of Gregorio
Nanaman have signed and agreed.”

The court approved the amicable settlement but when it was questioned by some heirs, the
court set aside its approval and declared it null and void. Noel, as regular administrator, and
as ordered by the court filed an action against private respondent for the reversion of title
over the 34.7-hectare land to the Nanaman estate and to order private respondent to pay the
rentals and attorney's fees to the estate.

Issues:
1. Whether or not Hilaria and Virgilio could dispose of the entire property sold to
private respondent and assuming that they did not have full ownership thereof;
2. Whether the right of action to recover the share of the collateral heirs of Gregorio
had prescribed or been lost through laches.

Ruling:
1. Gregorio died in 1945 long before the effectivity of the Civil Code of the Philippines on
August 30, 1950. Under Article 2263 of the said Code, "rights to the inheritance of a person
who died, with or without a will, before the effectivity of this Code, shall be governed by the
Civil Code of 1889, by other previous laws, and by the rules of Court."

Thus, succession to the estate of Gregorio was governed primarily by the provisions of the
Spanish Civil Code of 1889. Under Article 953 thereof, a spouse like Hilaria, who is survived
by brothers or sisters or children of brothers or sisters of the decedent, as is obtaining in this
case, was entitled to receive in usufruct the part of the inheritance pertaining to said heirs.
Hilaria, however, had full ownership, not merely usufruct, over the undivided half of the
estate (Spanish Civil Code of 1889, Art. 493). It is only this undivided half-interest that she
could validly alienate.

SALES CASE DIGEST COMPILATION 92


On the other hand, Virgilio was not an heir of Gregorio under the Spanish Civil Code of
1889. Although he was treated as a child by the Nanaman spouses, illegitimate children who
were not natural were disqualified to inherit under the said Code. Article 998 of the Civil
Code, which gave an illegitimate child certain hereditary rights, could not benefit Virgilio
because the right of ownership of the collateral heirs of Gregorio had become vested upon
his. Therefore, Virgilio had no right at all to transfer ownership over which he did not own.

In a contract of sale, it is essential that the seller is the owner of the property he is selling.
The principal obligation of a seller is "to transfer the ownership of" the property sold (Civil
Code of the Philippines, Art. 1458). This law stems from the principle that nobody can
dispose of that which does not belong to him (Azcona v. Reyes, 59 Phil. 446 [1934]; Coronel
v. Ona, 33 Phil. 456 [1916). NEMO DAT QUAD NON HABET.

2. The action to recover the undivided half-interest of the collateral heirs of Gregorio
prescribes in ten years. The cause of action is based on Article 1456 of the Civil Code of the
Philippines, which made private respondent a trustee of an implied trust in favor of the said
heirs. Under Article 1144 of the Civil Code, actions based upon an obligation created by law,
can be brought within ten years from the time the right of action accrues.

The ten-year prescriptive period within which the collateral heirs of Gregorio could file an
action to recover their share in the property sold to private respondent accrued only on
march 2, 1954, when the deed of sale was registered with the Register of Deeds. From
March 2, 1954 to April 30, 1963, when the complaint for the recovery of the property was
filed, less than ten years had elapsed. Therefore, the action had not been barred by
prescription.

15) CONCHITA NOOL vs. COURT OF APPEALS

Facts: In their complaint, plaintiff-appellants alleged inter alia that they are the owners of
subject parcels of land, and they bought the same from Conchita's other brothers, Victorino
Nool and Francisco Nool; that as plaintiffs were in dire need of money, they obtained a loan
from the Ilagan Branch of the Development Bank of the Philippines, in Ilagan, Isabela,
secured by a real estate mortgage on said parcels of land, which were still registered in the
names of Victorino Nool and Francisco Nool, at the time, and for the failure of plaintiffs to
pay the said loan, including interest and surcharges, totaling P56,000.00, the mortgage was
foreclosed; that within the period of redemption, plaintiffs contacted defendant Anacleto Nool
for the latter to redeem the foreclosed properties from DBP, which the latter did; and as a
result, the titles of the two (2) parcels of land in question were transferred to Anacleto Nool;
that as part of their arrangement or understanding, Anacleto Nool agreed to buy from plaintiff
Conchita Nool the two (2) parcels of land under controversy, for a total price of P100,000.00,
P30,000.00 of which price was paid to Conchita, and upon payment of the balance of
P14,000.00, plaintiffs were to regain possession of the two (2) hectares of land, which
amounts defendants failed to pay, and the same day the said arrangement was made;
another covenant was entered into by the parties, whereby defendants agreed to return to
plaintiffs the lands in question, at any time the latter have the necessary amount; that
plaintiffs asked the defendants to return the same but despite the intervention of the
Barangay Captain of their place, defendants refused to return the said parcels of land to
plaintiffs; thereby impelling them (plaintiffs) to come to court for relief.

SALES CASE DIGEST COMPILATION 93


In their Answer, defendants-appellees theorized that they acquired the lands in question
from the Development Bank of the Philippines, through negotiated sale, and were misled by
plaintiffs when defendant Anacleto Nool signed the private writing, agreeing to return subject
lands when plaintiffs have the money to redeem the same; defendant Anacleto having been
made to believe, then, that his sister, Conchita, still had the right to redeem the said
properties.

The pivot of inquiry here, as aptly observed below, is the nature and significance of the
privatedocument, marked Exhibit "D" for plaintiffs, which document has not been denied by
the defendants, as defendants even averred in their Answer that they gave an advance
payment of P30,000.00 therefor, and acknowledged that they had a balance of P14,000.00
to complete their payment. On this crucial issue, the lower court adjudged the said private
writing (Exhibit "D") as an option to sell not binding upon and considered the same validly
withdrawn by defendants for want of consideration; and decided the case in the manner
above-mentioned.

There is no quibble over the fact that the two (2) parcels of land in dispute were mortgaged
to the Development Bank of the Philippines, to secure a loan obtained by plaintiffs from DBP
(Ilagan Branch), Ilagan, Isabela. For the non-payment of said loan, the mortgage was
foreclosed and in the process, ownership of the mortgaged lands was consolidated in DBP
(Exhibits 3 and 4 for defendants). After DBP became the absolute owner of the two parcels
of land, defendants negotiated with DBP and succeeded in buying the same. By virtue of
such sale by DBP in favor of defendants, the titles of DBP were cancelled and the
corresponding Transfer Certificates of Title (Annexes "C" and "D" to the Complaint) issued to
the defendants.

It should be stressed that Manuel S. Mallorca, authorized officer of DBP, certified that the
one-year redemption period was from March 16, 1982 up to March 15, 1983 and that the
mortgagors' right of redemption was not exercised within this period. Hence, DBP became
the absolute owner of said parcels of land for which it was issued new certificates of title,
both entered on May 23, 1983 by the Registry of Deeds for the Province of Isabela. About
two years thereafter, on April 1, 1985, DBP entered into a Deed of Conditional Sale involving
the same parcels of land with Private Respondent Anacleto Nool as vendee. Subsequently,
the latter was issued new certificates of title on February 8, 1988.

Issue: Whether or not Conchita can still reacquire the property subject of the contract
of repurchase arising from the contract of sale.

Held: No. Conchita cannot reacquire the property as stipulated in contract of repurchase.

In the present case however, it is likewise clear that the sellers can no longer deliver the
object of the sale to the buyers, as the buyers themselves have already acquired title and
delivery thereof from the rightful owner, the DBP. Thus, such contract may be deemed to be
inoperative and may thus fall, by analogy, under item no. 5 of Article 1409 of the Civil Code:
"Those which contemplate an impossible service." Article 1459 of the Civil Code provides
that "the vendor must have a right to transfer the ownership thereof [object of the sale] at the
time it is delivered." Here, delivery of ownership is no longer possible. It has become
impossible.

SALES CASE DIGEST COMPILATION 94


Furthermore, Article 1505 of the Civil Code provides that "where goods are sold by a person
who is not the owner thereof, and who does not sell them under authority or with consent of
the owner, the buyer acquires no better title to the goods than the seller had, unless the
owner of the goods is by his conduct precluded from denying the seller's authority to sell."
Here, there is no allegation at all that petitioners were authorized by DBP to sell the property
to the private respondents. Jurisprudence, on the other hand, teaches us that "a person can
sell only what he owns or is authorized to sell; the buyer can as a consequence acquire no
more than what the seller can legally transfer." No one can give what he does not have —
nono dat quod non habet. On the other hand, Exhibit D presupposes that petitioners could
repurchase the property that they "sold" to private respondents. As petitioners "sold" nothing,
it follows that they can also "repurchase" nothing. Nothing sold, nothing to repurchase. In
this light, the contract of repurchase is also inoperative — and by the same analogy, void.

16) HEIRS OF SEVERINA SAN MIGUEL, namely: MAGNO LAPINA, PACENCIA


LAPINA, MARCELO LAPINA, SEVERINO LAPINA, ROSARIO LAPINA,
FRANCISCO LAPINA, CELIA LAPINA assisted by husband RODOLFO TOLEDO,
petitioners, vs. THE HONORABLE COURT OF APPEALS, DOMINADOR SAN
MIGUEL, GUILLERMO F. SAN MIGUEL, PACIENCIA F. SAN MIGUEL,
CELESTINO, assisted by husband, ANTERO CELESTINO, represented by their
Attorney-in-Fact ENRICO CELESTINO, AUGUSTO SAN MIGUEL, ANTONIO SAN
MIGUEL, RODOLFO SAN MIGUEL, CONRADO SAN MIGUEL and LUCITA SAN
MIGUEL, respondents.

Facts: Severina San Miguel claims to own a parcel of land situated in Panapan, Bacoor,
Cavite. Without Severina’s knowledge, Dominador managed to cause the subdivision of the
land into three (3) lots:

“LRC PSU –1312 - with an area of 108 square meters;


“LRC PSU -1313 - Lot 1, with an area of 299 square meters;
“LRC PSU -1313 - Lot 2, with an area of 225 square meters.”

On September 25, 1974, Dominador, et al. filed a petition with the CFI Cavite, as a land
registration court, to issue title over Lots 1 and 2 of LRC Psu-1313, in their names which
resulted into the issuance of Original Certificate of Title No. 0-1816 in the names of
Dominador, et al. Subsequently, Severina filed with the CFI of Cavite a petition for review of
the decision alleging that the land registration proceedings were fraudulently concealed by
Dominador from her. As a result, the court declared Original Certificate of Title No. 0-1816
as null and void, and the Register of Deeds of Cavite issued TCT No. T-223511 in the
names of Severina and her heirs.

There were writs of possession and demolition issued by the court in favor of Severina, but
both were left unheaded. So Severina’s heirs, decided not to pursue the writs, and entered
into a compromise with Dominador, et al. The compromise provides that Severina’s heirs
were to sell the subject lots to Dominador, et al. for one and a half million pesos (P1.5 M)
with the delivery of Transfer Certificate of Title No. T-223511 (hereafter, “the certificate of
title”) conditioned upon the purchase of another lot[11] which was not yet titled at an
additional sum of three hundred thousand pesos (P300,000.00)
Pursuant to the kasunduan, Severina’s heirs and Dominador, et al. executed a deed of sale
designated as “kasulatan sa bilihan ng lupa.”

SALES CASE DIGEST COMPILATION 95


Dominador, et al. filed with the trial court a motion praying that Severina’s heirs deliver the
owner’s copy of the certificate of title to them. Severina’s heirs opposed the motion stressing
that under the kasunduan, the certificate of title would only be surrendered upon Dominador,
et al.’s payment of the amount of three hundred thousand pesos (P300,000.00) within two
months from August 6, 1993, which was not complied with.

Dominador, et al. admitted non-payment of three hundred thousand pesos (P300,000.00) for
the reason that Severina’s heirs have not presented any proof of ownership over the untitled
parcel of land covered by LRC- Psu-1312. Apparently, the parcel of land is declared in the
name of a third party, a certain Emiliano Eugenio.

Severina’s heirs countered that in light of the provision in the kasunduan where Dominador,
et al. admitted their ownership over the parcel of land, this admission dispensed with the
requirement that they produce actual proof of title over it. Specifically, they called the trial
court’s attention to the following statement in the kasunduan:

“7. Na kinikilala ni SAN MIGUEL na ang tunay na may-ari ng nasabing lote na sakop ng
plano LRC Psu-1312 ay sina LAPINA at sila na ang magpapatitulo nito at sina LAPINA ay
walang pananagutan sa pagpapatitulo nito at sa paghahabol ng sino mang tao;”

According to Severina’s heirs, since Dominador, et al. have not paid the amount of three
hundred thousand pesos (P300,000.00), then they were justified in withholding release of
the certificate of title.

RTC: "Heirs of Severina San Miguel are hereby ordered to surrender to Heirs of Dominador
San Miguel the TCT No. 223511."

CA: Affirmed.

Issue: Whether Dominador, et al. may be compelled to pay the three hundred
thousand pesos (P300,000.00) as agreed upon in the kasunduan (as a pre-requisite for
the release of the certificate of title), despite Severina’s heirs’ lack of evidence of
ownership over the parcel of land covered by LRC Psu-1312.

Ruling: No, they cannot be compelled.

Article 1458. By the contract of sale one of the contracting parties obligates himself to
transfer the ownership of and to deliver a determinate thing, and the other to pay a price
certain in money or its equivalent xxx.

Article 1459. The thing must be licit and the vendor must have a right to transfer the
ownership thereof at the time it is delivered.

Article 1495. The vendor is bound to transfer the ownership of and deliver, as well as
warrant the thing which is the object of sale.

SALES CASE DIGEST COMPILATION 96


True, in contracts of sale, the vendor need not possess title to the thing sold at the perfection
of the contract. However, the vendor must possess title and must be able to transfer title at
the time of delivery. In a contract of sale, title only passes to the vendee upon full payment of
the stipulated consideration, or upon delivery of the thing sold.

Under the facts of the case, Severina’s heirs are not in a position to transfer title. SC notes
that there is no proof of ownership in favor of Severina’s heirs. In fact, it is a certain Emiliano
Eugenio, who holds a tax declaration over the said land in his name. Though tax
declarations do not prove ownership of the property of the declarant, tax declarations and
receipts can be strong evidence of ownership of land when accompanied by possession for
a period sufficient for prescription. Severina’s heirs have nothing to counter this document.
Therefore, to insist that Dominador, et al. pay the price under such circumstances would
result in Severina’s heirs’ unjust enrichment. The essence of a sale is the transfer of title or
an agreement to transfer it for a price actually paid or promised.

In Nool v. Court of Appeals, we held that if the sellers cannot deliver the object of the sale to
the buyers, such contract may be deemed to be inoperative. By analogy, such a contract
may fall under Article 1405, No. 5 of the Civil Code, to wit:

Article 1405. The following contracts are inexistent and void from the beginning: xxx

(5) Those which contemplate an impossible service.

Severina’s heirs insist that delivery of the certificate of title is predicated on a condition -
payment of three hundred thousand pesos (P300,000.00) to cover the sale of Lot 3 of LRC
PSU 1312.

We find this argument not meritorious.

The condition cannot be honored for reasons afore-discussed. Article 1183 of the Civil Code
provides that:

“Impossible conditions, those contrary to good customs or public policy and those prohibited
by law shall annul the obligation which depends upon them. If the obligation is divisible, that
part thereof which is not affected by the impossible or unlawful condition shall be valid xxx.”

Hence, the non-payment of the three hundred thousand pesos (P300,000.00) is not a valid
justification for refusal to deliver the certificate of title. Besides, we note that the certificate of
title covers Lots 1 and 2 of LRC Psu-1313, which were fully paid for by Dominador, et al.
Therefore, Severina’s heirs are bound to deliver the certificate of title covering the lots.

XIII. PRICE

SALES CASE DIGEST COMPILATION 97


1) Mapalo v. Mapalo

Facts: Miguel Mapalo and Candida Quiba, simple illiterate farmers, were registered owners
of a residential land in Manaoag, Pangasinan. Out of love and affection for Maximo Mapalo,
Miguel’s brother who was about to get married, they decided to donate the eastern half of
the land. However, they were deceived into signing a deed of absolute sale of the entire land
on October 15, 1936. The document showed a consideration of P500, but the spouses
actually did not receive anything.

The spouses built a fence segregating the donated land. They continued to possess the
western part up to the present. Not known to them, on March 15, 1938, Maximo registered
the deed of sale in his favor and was able to obtain a TCT. On October 20, 1951, Maximo
sold the entire land to the Narcisos, and a TCT was issued. The Narcisos took possession of
the eastern part and filed a suit against Miguel and Candida, as well as Floro Guieb and
Rosalia Mapalo Guieb who had a house on the western portion consented by the spouses.
The spouses filed an answer with counterclaim, seeking cancellation of the TCT of the
Narcisos on the ground that their consent to the deed of sale in favor of Maximo was
obtained through fraud.

The spouses also instituted a complaint to nullify the deeds of sale in 1936 and 1951. The
trial court tried the case jointly. It ruled in favor of Miguel and Candida. The appellate court,
however, reversed the judgment and rendered the sale valid on the ground of prescription.
According to the appellate court, the sale is voidable and subject to annulment only within 4
years after discovery of fraud. It reckoned March 15, 1938, the date of registration, to be the
reckoning period.

Issues: (1) Whether, under the old civil code which was in effect during the execution of the
sale, the sale to Maximo is void or merely voidable (2) Whether the Narcisos were
purchasers in good faith

Held:

I. For a contract to exist at all, three essential requisites must concur: (1) consent, (2) object,
and (3) cause or consideration. The Court of Appeals is right in that the element of consent
is present as to the deed of sale of October 15, 1936. For consent was admittedly given,
albeit obtained by fraud.
Accordingly, said consent, although defective, did exist. In such case, the defect in the
consent would provide a ground for annulment of a voidable contract, not a reason for nullity
ab initio. The parties are agreed that the second element of object is likewise present in the
deed of October 15, 1936, namely, the parcel of land subject matter of the same.
Not so, however, as to the third element of cause or consideration. As regards the eastern
portion of the land, the Mapalo spouses are not claiming the same, it being their stand that
they have donated and freely given said half of their land to Maximo Mapalo, and since they
did not appeal from the decision of the trial court finding that there was a valid and effective
donation of the eastern portion of their land in favor of Maximo Mapalo, the same
pronouncement has become final as to them, rendering it no longer proper herein to
examine the existence, validity efficacy of said donation as to said eastern portion.

SALES CASE DIGEST COMPILATION 98


Now, as to the western portion, however, the fact not disputed herein is that no donation by
the Mapalo spouses obtained as to the said portion. Accordingly, we start with the fact that
liberality as a cause or consideration does not exist as regards the western portion of the
land in relation to the deed of 1936; that there was no donation with respect to the same.
Was there a cause or consideration to support the existence of a contrary of sale?
Since the deed of sale of 1936 is governed by the Old Civil Code, it should be asked
whether its case is one wherein there is no consideration, or one with a statement of a false
consideration. If the former, it is void and inexistent; if the latter, only voidable, under the Old
Civil Code. As observed earlier, the deed of sale of 1936 stated that it had for its
consideration Five Hundred (P500.00) Pesos. In fact, however, said consideration was
totally absent. According to Manresa, what is meant by a contract that states a false
consideration is one that has in fact a real consideration but the same is not the one stated
in the document.
A contract of purchase and sale is null and void and produces no effect whatsoever where
the same is without cause or consideration in that the purchase price which appears thereon
as paid has in fact never been paid by the purchaser to the vendor.
II. What was the necessity, purpose and reason of Pacifico Narciso in still going to the
spouses Mapalo and asked them to permit their brother Maximo to dispose of the above-
described land? To this question it is safe to state that this act of Pacifico Narciso is a
conclusive manifestation that they (the Narcisos) did not only have prior knowledge of the
ownership of said spouses over the western half portion in question but that they also have
recognized said ownership. It also conclusively shows their prior knowledge of the want of
dominion on the part of their vendor Maximo Mapalo over the whole land and also of the flaw
of his title thereto. Under this situation, the Narcisos may be considered purchasers in value
but certainly not as purchasers in good faith.

2) MODINA vs CA
Facts: The parcels of land in question are those under the name of Ramon Chiang. He
theorized that these were sold to him by his wife, Merlinda Plana, as evidenced by a Deed of
Absolute Sale dated December 17, 1975, and were subsequently sold to Serafin Modina on
August 3, 1979 and August 24, 1979, respectively.
MODINA brought a Complaint for Recovery of Possession with Damages against the private
respondents, Ernesto Hontarciego, Paul Figueroa and Teodoro Hipalla, before the RTC.
Upon learning the institution of the said case, MERLINDA presented a Complaint-in-
intervention, seeking the declaration of nullity of the Deed of Sale between her husband and
MODINA because titles were never legally transferred.
Fraudulent acts were allegedly employed to obtain a Torrens Title in his favor. However, she
confirmed the validity of the lease contracts with the other private respondents. MERLINDA
also admitted that the land were those ordered sold by the CFI in the case “Intestate Estate
of Nelson Plana” where she was appointed as the administratix, as the widow. An Authority
to Sell was issued by the said Probate Court for the sale of the same properties.
RTC: Ruled in favor or Merlinda (1) declaring as void and inexistent the sale of Lots in
question and the Certificates of Title in the name of Ramon Chiang; (2) declaring as void
and inexistent the sale of the same properties by Ramon Chiang in favor of Serafin Modina.
CA affirmed in toto.
Issues: (1) WON the sale of subject lots should be nullified? (2) WON the principle of
in pari delicto is applicable in this case? (3) WON Modina is a purchaser in good
faith?
HELD:

SALES CASE DIGEST COMPILATION 99


I. Petitioner theorizes that the sale in question is null and void for being violative of Article
1490 prohibiting sales between spouses. Consequently, what is applicable is Article 1412
supra on the principle of in pari delicto, which leaves both guilty parties where they are, and
keeps undisturbed the rights of third persons to whom the lots involved were sold; petitioner
stressed. Art. 1490, husband and wife are prohibited to sell properties to each other. And
where, as in this case, the sale is inexistent for lack of consideration, the principle of in pari
delicto non oritur actio does not apply except (1) when a separation of property was agreed
upon in the marriage settlements; or (2) when there has been a judicial separation of
property under Art. 191.
In the case at bar, the exception not having, the sale by the latter in favor of the former of the
properties in question is invalid for being prohibited by law. Not being the owner of subject
properties, Ramon Chiang could not have validly sold the same to plaintiff Serafin Modina.
The sale by Ramon Chiang in favor of Serafin Modina is, likewise, void and inexistent.
II. The principle of in pari delicto non oritur actio denies all recovery to the guilty parties inter
se. It applies to cases where the nullity arises from the illegality of the consideration or the
purpose of the contract. When two persons are equally at fault, the law does not relieve
them. The exception to this general rule is when the principle is invoked with respect to
inexistent contracts.
In the case at bar, the Trial Court found that subject Deed of Sale was a nullity for lack of
any consideration. Under Article 1409 a contract without consideration is one such void
contract. One of the characteristics of a void or inexistent contract is that it produces no
effect. So also, inexistent contracts can be invoked by any person whenever juridical effects
founded thereon are asserted against him. A transferor can recover the object of such
contract by accion reivindicatoria and any possessor may refuse to deliver it to the
transferee, who cannot enforce the transfer.
It bears emphasizing that as the contracts under controversy are inexistent contracts within
legal contemplation, Articles 1411 and 1412 of the New Civil Code are inapplicable. In pari
delicto doctrine applies only to contracts with illegal consideration or subject matter, whether
the attendant facts constitute an offense or misdemeanor or whether the consideration
involved is merely rendered illegal.
The statement below that it is likewise null and void for being violative of Article 1490 should
just be treated as a surplusage or an obiter dictum on the part of the Trial Court as the issue
of whether the parcels of land in dispute are conjugal in nature or they fall under the
exceptions provided for by law, was neither raised nor litigated upon before the lower Court.
Whether the said lots were ganancial properties was never brought to the fore by the parties
and it is too late to do so now. Since one of the characteristics of a void or inexistent contract
is that it does not produce any effect, MERLINDA can recover the property from petitioner
who never acquired title thereover.
III. Modina is not a purchaser in good faith. It was found that there were circumstances
known to MODINA which rendered their transaction fraudulent under the attendant
circumstances. As a general rule, in a sale under the Torrens system, a void title cannot give
rise to a valid title. The exception is when the sale of a person with a void title is to a third
person who purchased it for value and in good faith. A purchaser in good faith is one who
buys the property of another without notice that some other person has a right to or interest
in such property and pays a full and fair price at the time of the purchase or before he has
notice of the claim or interest of some other person in the property.

3) ASUNCION MENESES VDA. DE CATINDIG, petitioner-appellant, vs. The Heirs of


CATALINA ROQUE
Facts: The said fishpond, known as Lot No. 4626 of the Malolos Cadastre, has an area of
more than thirteen hectares. The co-owners of the fishpond leased it to Mrs. Catindig for a

SALES CASE DIGEST COMPILATION 100


term of ten years counted from October 1, 1941 for a total rental of six thousand. After the
termination of the lease on September 30, 1951, Mrs. Catindig remained in possession of
the fishpond because she was negotiating with the co-owners for the purchase thereof. She
wanted to buy it for P52,000.
On October 18, 1960 German Ramirez, one of the co-owners, executed a deed wherein he
sold his 2/16 share to Mrs. Catindig for P6,500. The sale was annotated on the title on
October 19, 1960. Two weeks later, Pedro Villanueva, one of the co-owners, learned of the
sale executed by German Ramirez.
On November 18, 1960 the respondents filed this action against Mrs. Catindig to compel her
to allow them to redeem the portion sold by German Ramirez. In April, 1962 the respondents
amended their complaint by including, inter alia, a prayer for the recovery of the possession
of the fishpond.
The Court of Appeals found that:
1. The consideration of P52,000 was not paid by Mrs. Catindig to the co-owners because
she was not able to obtain a loan, the proceeds of which would have been used to pay the
co-owners who had executed simulated sales of their shares, as shown in the private
documents, Exhibits 6 to 26. (The originals of those documents were allegedly lost. Only
photostatic copies thereof were presented in evidence).
2. Because Mrs. Catindig did not pay the price of P52,000, the projected sale, "which was in
truth a simulated one so as to enable her just to mortgage the property in order to secure the
necessary amount with which to pay the consideration" was void ab initio. There was no
notarized deed of sale because Mrs. Catindig did not pay the price to the co-owners except
German Ramirez.
3. Ines Roque and the heirs of Roberto Roque did not barter their shares for the two parcels
of land owned by Mrs. Catindig. What the said co-owners did was to possess the lands of
Mrs. Catindig in exchange for the latter's possession of their shares in the fishpond.
Issue: WON the sale of the fishpond to her is void for nonpayment of the price.
Held:
Yes, the sale is void because the price is simulated.
The conclusive factual finding of the Appellate Court that the alleged sales on April 13 or 14,
1950 of respondents' shares are simulated and void ab initio (See Onglengco vs. Ozaeta, 70
Phil. 43) renders untenable appellant Catindig's contentions that the remedies available to
the respondents, such as an action for annulment, rescission or reformation, are barred by
prescription or laches.
The alleged sales were absolutely simulated, fictitious or inexistent contracts (Arts. 1346 and
1409[2], Civil Code). "The action or defense for the declaration of the inexistence of a
contract does not prescribe" (Art. 1410, Ibid; Eugenio vs. Perdido, 97 Phil. 41).
Mere lapse of time cannot give efficacy to a void contract (Tipton vs. Velasco, 6 Phil. 67).
The Appellate Court's finding that the price was not paid or that the statement in the
supposed contracts of sale (Exh. 6 to 26) as to the payment of the price was simulated
fortifies the view that the alleged sales were void. "If the price is simulated, the sale is void
..." (Art. 1471, Civil Code).
A contract of sale is void and produces no effect whatsoever where the price, which appears
thereon as paid, has in fact never been paid by the purchaser to the vendor (Ocejo, Perez &
Co. vs. Flores and Bas, 40 Phil. 921; Mapalo vs. Mapalo, L-21489, May 19, 1966, 64 O. G.
331, 17 SCRA 114, 122). Such a sale is non-existent (Borromeo vs. Borromeo, 98 Phil. 432)
or cannot be considered consummated (Cruzado vs. Bustos and Escaler, 34 Phil. 17;
Garanciang vs. Garanciang, L-22351, May 21, 1969, 28 SCRA 229).

SALES CASE DIGEST COMPILATION 101


The foregoing discussion disposes of whatever legal issues were raised by appellant
Catindig which are interwoven with her factual contentions, including the issue as to whether
she is entitled to demand the execution of a notarized deed of sale for the 14/16 pro indiviso
portion of the fishpond. She is not entitled because, as already held, the alleged sales in her
favor are void.

4) Sps. Lequin vs. Sps. Vizconde


Facts:
Petitioner Ramon Lequin, husband of petitioner Virginia Lequin, is the brother of respondent
Salome L. Vizconde and brother-in-law of respondent Raymundo Vizconde. With this
consanguine and affinity relation, the instant case developed as follows:
In 1995, petitioners, residents of Diamond Court, Brixton Ville Subdivision, Camarin,
Caloocan City, bought the subject lot consisting of 10,115 square meters from one Carlito de
Leon (de Leon). The sale was negotiated by respondent Raymundo Vizconde. The subject
lot is located near the Sto. Rosario to Magsaysay road in Aliaga, Nueva Ecija. Adjacent
thereto and located in between the subject lot and the road is a dried up canal (or sapang
patay in the native language).
In 1997, respondents represented to petitioners that they had also bought from Carlito de
Leon a 1,012-square meter lot adjacent to petitioners’ property and built a house thereon. As
later confirmed by de Leon, however, the 1,012-square meter lot claimed by respondents is
part of the 10,115-square meter lot petitioners bought from him. Petitioners believed the
story of respondents, since it was Raymundo who negotiated the sale of their lot with de
Leon. With the consent of respondents, petitioners then constructed their house on the 500-
square meter half-portion of the 1,012 square-meter lot claimed by respondents, as this was
near the road. Respondents’ residence is on the remaining 512 square meters of the lot.
Given this situation where petitioners’ house stood on a portion of the lot allegedly owned by
respondents, petitioners consulted a lawyer, who advised them that the 1,012-square meter
lot be segregated from the subject lot whose title they own and to make it appear that they
are selling to respondents 512 square meters thereof. This sale was embodied in the
February 12, 2000 Kasulatanwhere it was made to appear that respondents paid PhP
15,000 for the purchase of the 512-square meter portion of the subject lot. In reality, the
consideration of PhP 15,000 was not paid to petitioners. Actually, it was petitioners who paid
respondents PhP 50,000 for the 500-square meter portion where petitioners built their house
on, believing respondents’ representation that the latter own the 1,012-square meter lot.
In July 2000, petitioners tried to develop the dried up canal located between their 500-
square meter lot and the public road. Respondents objected, claiming ownership of said
dried up canal or sapang patay.
This prompted petitioners to look into the ownership of the dried up canal and the 1,012
square-meter lot claimed by respondents. Carlito de Leon told petitioners that what he had
sold to respondents was the dried up canal or sapang patay and that the 1,012-square
meter lot claimed by respondents really belongs to petitioners.
Thus, on July 13, 2001, petitioners filed a Complaint4 for Declaration of Nullity of Contract,
Sum of Money and Damages against respondents with the Regional Trial Court (RTC),
Branch 28 in Cabanatuan City, praying, among others, for the declaration of the February
12, 2000 Kasulatan as null and void ab initio, the return of PhP 50,000 they paid to
respondents, and various damages. The case was docketed as Civil Case No. 4063.
The RTC found the Kasulatan allegedly conveying 512 square meters to respondents to be
null and void due to: (1) the vitiated consent of petitioners in the execution of the simulated
contract of sale; and (2) lack of consideration, since it was shown that while petitioners were
ostensibly conveying to respondents 512 square meters of their property, yet the

SALES CASE DIGEST COMPILATION 102


consideration of PhP 15,000 was not paid to them and, in fact, they were the ones who paid
respondents PhP 50,000. The RTC held that respondents were guilty of fraudulent
misrepresentation.
Aggrieved, respondents appealed the above RTC Decision to the CA.
In reversing and vacating the RTC Decision, the CA found no simulation in the contract of
sale, i.e., Kasulatan. Relying on Manila Banking Corporation v. Silverio, the appellate court
pointed out that an absolutely simulated contract takes place when the parties do not intend
at all to be bound by it, and that it is characterized by the fact that the apparent contract is
not really desired or intended to produce legal effects or in any way alter the juridical
situation of the parties. It read the sale contract (Kasulatan) as clear and unambiguous, for
respondents (spouses Vizconde) were the buyers and petitioners (spouses Lequin) were the
sellers. Such being the case, petitioners are, to the CA, the owners of the 1,012-square
meter lot, and as owners they conveyed the 512-square meter portion to respondents.
The CA viewed petitioners’ claim that they executed the sale contract to make it appear that
respondents bought the property as mere gratuitous allegation. Besides, the sale contract
was duly notarized with respondents claiming the 512-square meter portion they bought
from petitioners and not the whole 1,012-square meter lot as alleged by petitioners.
Moreover, the CA dismissed allegations of fraud and machinations against respondents to
induce petitioners to execute the sale contract, there being no evidence to show how
petitioners were defrauded and much less the machinations used by respondents. It
ratiocinated that the allegation of respondents telling petitioners that they own the 1,012-
square meter lot and for which petitioners sold them 512 square meters thereof does not fall
in the concept of fraud. Anent the PhP 50,000 petitioners paid to respondents for the 500-
square meter portion of the 1,012-square meter lot claimed by respondents, the CA ruled
that the receipt spoke for itself and, thus, required respondents to return the amount to
petitioners
Issues:
The issues boil down to two core questions: (1) Whether or not the Kasulatan
covering the 512 square-meter lot is a valid contract of sale; and (2) who is the legal
owner of the other 500 square-meter lot.
RULING:
I. Yes. A contract, as defined in the Civil Code, is a meeting of minds, with respect to the
other, to give something or to render some service.12 For a contract to be valid, it must have
three essential elements: (1) consent of the contracting parties; (2) object certain which is
the subject matter of the contract; and (3) cause of the obligation which is established.
The requisites of consent are (1) it should be intelligent or with an exact notion of the matter
to which it refers; (2) it should be free; and (3) it should be spontaneous. In De Jesus v.
Intermediate Appellate Court, it was explained that intelligence in consent is vitiated by error,
freedom by violence, intimidation or undue influence, and spontaneity by fraud.
Article (Art.) 1330 of the Civil Code provides that when consent is given through fraud, the
contract is voidable.
Tolentino defines fraud as "every kind of deception whether in the form of insidious
machinations, manipulations, concealments or misrepresentations, for the purpose of
leading another party into error and thus execute a particular act." Fraud has a "determining
influence" on the consent of the prejudiced party, as he is misled by a false appearance of
facts, thereby producing error on his part in deciding whether or not to agree to the offer.
One form of fraud is misrepresentation through insidious words or machinations. Under Art.
1338 of the Civil Code, there is fraud when, through insidious words or machinations of one
of the contracting parties, the other is induced to enter into a contract which without them he
would not have agreed to. Insidious words or machinations constituting deceit are those that

SALES CASE DIGEST COMPILATION 103


ensnare, entrap, trick, or mislead the other party who was induced to give consent which he
or she would not otherwise have given.
Deceit is also present when one party, by means of concealing or omitting to state material
facts, with intent to deceive, obtains consent of the other party without which, consent could
not have been given. Art. 1339 of the Civil Code is explicit that failure to disclose facts when
there is a duty to reveal them, as when the parties are bound by confidential relations,
constitutes fraud.
From the factual milieu, it is clear that actual fraud is present in this case. The sale between
petitioners and de Leon over the 10,115 square-meter lot was negotiated by respondent
Raymundo Vizconde. As such, Raymundo was fully aware that what petitioners bought was
the entire 10,115 square meters and that the 1,012-square meter lot which he claims he also
bought from de Leon actually forms part of petitioners’ lot. It cannot be denied by
respondents that the lot which they actually bought, based on the unrebutted testimony and
statement of de Leon, is the dried up canal which is adjacent to petitioners’ 10,115-square
meter lot. Considering these factors, it is clear as day that there was deception on the part of
Raymundo when he misrepresented to petitioners that the 1,012-square meter lot he bought
from de Leon is a separate and distinct lot from the 10,115-square meter lot the petitioners
bought from de Leon. Raymundo concealed such material fact from petitioners, who were
convinced to sign the sale instrument in question and, worse, even pay PhP 50,000 for the
500 square-meter lot which petitioners actually own in the first place.
There was vitiated consent on the part of petitioners. There was fraud in the execution of the
contract used on petitioners which affected their consent. Petitioners’ reliance and belief on
the wrongful claim by respondents operated as a concealment of a material fact in their
agreeing to and in readily executing the contract of sale, as advised and proposed by a
notary public. Believing that Carlito de Leon indeed sold a 1,012-square meter portion of the
subject property to respondents, petitioners signed the contract of sale based on
respondents’ representations.
Had petitioners known, as they eventually would sometime in late 2000 or early 2001 when
they made the necessary inquiry from Carlito de Leon, they would not have entered or
signed the contract of sale, much less pay PhP 50,000 for a portion of the subject lot which
they fully own. Thus, petitioners’ consent was vitiated by fraud or fraudulent machinations of
Raymundo. In the eyes of the law, petitioners are the rightful and legal owners of the subject
512 square-meter lot anchored on their purchase thereof from de Leon. This right must be
upheld and protected.
On the issue of lack of consideration, the contract of sale or Kasulatan states that
respondents paid petitioners PhP 15,000 for the 512-square meter portion, thus:
Na kaming magasawang Ramon Lequin at Virginia R. Lequin, nawang may sapat na gulang,
pilipino at nakatira sa 9 Diamond Court, Brixton Ville Subdivision, Camarin, Kalookan City,
alang-alang sa halagang LABINGLIMANG LIBONG PISO (P 15,000.00) salaping pilipino na
binayaran sa amin ng buong kasiyahang loob namin ng magasawang Raymundo Vizconde
at Salome Lequin, nawang may sapat na gulang, pilipino at nakatira sa Sto. Rosario, Aliaga,
Nueva Ecija, ay amin naman ngayon inilipat, ibinigay at ipinagbili ng bilihang tuluyan sa
naulit na magasawang Raymundo Vizconde at Salome Lequin, at sa kanilang mga
tagapagmana ang x x x.
On its face, the above contract of sale appears to be supported by a valuable consideration.
We, however, agree with the trial court’s finding that this is a simulated sale and
unsupported by any consideration, for respondents never paid the PhP 15,000 purported
purchase price.
Section 9 of Rule 130 of the Revised Rules on Evidence gives both the general rule and
exception as regards written agreements, thus:

SALES CASE DIGEST COMPILATION 104


SEC. 9. Evidence of written agreements.—When the terms of an agreement have been
reduced to writing, it is considered as containing all the terms agreed upon and there can be,
between the parties and their successors in interest, no evidence of such terms other than
the contents of the written agreement.
However, a party may present evidence to modify, explain or add to the terms of the written
agreement if he puts in issue in his pleading:
(a) An intrinsic ambiguity, mistake or imperfection in written agreement;
(b) The failure of the written agreement to express the true intent and agreement of the
parties thereto;
(c) The validity of the written agreement; or
(d) The existence of other terms agreed to by the parties or their successors in interest after
the execution of the written agreement.
The term "agreement" includes wills.
The second exception provided for the acceptance of parol evidence applies to the instant
case. Lack of consideration was proved by petitioners’ evidence aliunde showing that the
Kasulatan did not express the true intent and agreement of the parties. As explained above,
said sale contract was fraudulently entered into through the misrepresentations of
respondents causing petitioners’ vitiated consent.
Moreover, the evidence of petitioners was uncontroverted as respondents failed to adduce
any proof that they indeed paid PhP 15,000 to petitioners. Indeed, having asserted their
purchase of the 512-square meter portion of petitioners based on the Kasulatan, it behooves
upon respondents to prove such affirmative defense of purchase. Unless the party asserting
the affirmative defense of an issue sustains the burden of proof, his or her cause will not
succeed. If he or she fails to establish the facts of which the matter asserted is predicated,
the complainant is entitled to a verdict or decision in his or her favor.
In the instant case, the record is bereft of any proof of payment by respondents and, thus,
their affirmative defense of the purported purchase of the 512-square meter portion fails.
There can be no doubt that the contract of sale or Kasulatan lacked the essential element of
consideration. It is a well-entrenched rule that where the deed of sale states that the
purchase price has been paid but in fact has never been paid, the deed of sale is null and
void ab initio for lack of consideration. Moreover, Art. 1471 of the Civil Code, which provides
that "if the price is simulated, the sale is void," also applies to the instant case, since the
price purportedly paid as indicated in the contract of sale was simulated for no payment was
actually made.
Consideration and consent are essential elements in a contract of sale. Where a party’s
consent to a contract of sale is vitiated or where there is lack of consideration due to a
simulated price, the contract is null and void ab initio.
II. The payment of PhP 50,000 by petitioners to respondents as consideration for the
transfer of the 500-square meter lot to petitioners must be restored to the latter. Otherwise,
an unjust enrichment situation ensues. The facts clearly show that the 500-square meter lot
is legally owned by petitioners as shown by the testimony of de Leon; therefore, they have
no legal obligation to pay PhP 50,000 therefor. Art. 22 of the Civil Code provides that "every
person who through an act or performance by another, or any other means, acquires or
comes into possession of something at the expense of the latter without just or legal ground,
shall return the same to him." Considering that the 512 square-meter lot on which
respondents’ house is located is clearly owned by petitioners, then the Court declares
petitioners’ legal ownership over said 512 square-meter lot. The amount of PhP 50,000
should only earn interest at the legal rate of 6% per annum from the date of filing of
complaint up to finality of judgment and not 12% since such payment is neither a loan nor a

SALES CASE DIGEST COMPILATION 105


forbearance of credit. After finality of decision, the amount of PhP 50,000 shall earn interest
of 12% per annum until fully paid.
The award of moral and exemplary damages must be reinstated in view of the fraud or
fraudulent machinations employed by respondents on petitioners. The grant of damages in
the concept of attorney’s fees in the amount of PhP 10,000 must be maintained considering
that petitioners have to incur litigation expenses to protect their interest in conformity to Art.
2208(2) of the Civil Code.

5) BUENAVENTURA VS CA (Nov. 20, 2003)


FACTS:
Defendant spouses Leonardo Joaquin and Feliciana Landrito are the parents of plaintiffs
Consolacion, Nora, Emma and Natividad as well as of defendants Fidel, Tomas, Artemio,
Clarita, Felicitas, Fe, and Gavino, all surnamed JOAQUIN. The married Joaquin children are
joined in this action by their respective spouses. Sought to be declared null and void ab initio
are certain deeds of sale of real property executed by defendant parents Leonardo Joaquin
and Feliciana Landrito in favor of their co-defendant children.
The petitioners argue that the deeds of sale are simulated as they are null and void ab initio
because:
1. There was no actual valid consideration for the deeds of sale over the properties in litis;
2. Assuming that there was consideration in the sums reflected in the questioned deeds, the
properties are more than three-fold times more valuable than the measly sums appearing
therein;
3. The deeds of sale do not reflect and express the true intent of the parties (vendors and
vendees); and
4. The purported sale of the properties in litis was the result of a deliberate conspiracy
designed to unjustly deprive the rest of the compulsory heirs of their legitime.

The trial court ruled in favor of the defendants. The Court of Appeals affirmed the trial court’s
decision.
ISSUE:
(I) WON petitioners have a legal interest over the properties subject of the Deeds of Sale
(II) WON the Deeds of Sale are void for lack of consideration
(III) WON the Deeds of Sale are void for gross inadequacy of price
RULING:
I. No, petitioners do not have a legal interest over the properties subject of the Deeds of
Sale. Petitioners failed to show any legal right to the properties. In actions for the annulment
of contracts, such as this action, the real parties are those who are parties to the
agreement or are bound either principally or subsidiarily or are prejudiced in their
rights with respect to one of the contracting parties and can show the detriment which
would positively result to them from the contract even though they did not intervene in it.
Petitioners do not have any legal interest over the properties subject of the Deeds of Sale.
As the appellate court stated, petitioners’ right to their parents properties is merely

SALES CASE DIGEST COMPILATION 106


inchoate and vests only upon their parents death. While still living, the parents of
petitioners are free to dispose of their properties.
In their overzealousness to safeguard their future legitime, petitioners forget that
theoretically, the sale of the lots to their siblings does not affect the value of their parents’
estate. While the sale of the lots reduced the estate, cash of equivalent value replaced the
lots taken from the estate.
II. It is not the act of payment of price that determines the validity of a contract of sale.
Payment of the price has nothing to do with the perfection of the contract. Payment of the
price goes into the performance of the contract. Failure to pay the consideration is different
from lack of consideration. The former results in a right to demand the fulfillment or
cancellation of the obligation under an existing valid contract while the latter prevents the
existence of a valid contract.
A contract of sale is not a real contract, but a consensual contract. As a consensual contract,
a contract of sale becomes a binding and valid contract upon the meeting of the minds as to
price. If there is a meeting of the minds of the parties as to the price, the contract of sale is
valid, despite the manner of payment, or even the breach of that manner of payment. If the
real price is not stated in the contract, then the contract of sale is valid but subject to reform.
Petitioners’ failure to prove absolute simulation of price is magnified by their lack of
knowledge of their respondent siblings’ financial capacity to buy the questioned lots. On the
other hand, the Deeds of Sale which petitioners presented as evidence plainly showed the
cost of each lot sold. Not only did respondents’ minds meet as to the purchase price, but the
real price was also stated in the Deeds of Sale. As of the filing of the complaint, respondent
siblings have also fully paid the price to their respondent father.
III. Article 1355: Except in cases specified by law, lesion or inadequacy of cause shall
not invalidate a contract, unless there has been fraud, mistake or undue influence.
Art. 1470. Gross inadequacy of price does not affect a contract of sale, except as may
indicate a defect in the consent, or that the parties really intended a donation or some
other act or contract.
Petitioners failed to prove any of the instances mentioned in Articles 1355 and 1470 of the
Civil Code which would invalidate, or even affect, the Deeds of Sale. Indeed, there is no
requirement that the price be equal to the exact value of the subject matter of sale. All the
respondents believed that they received the commutative value of what they gave.
Courts cannot follow one every step of his life and extricate him from bad bargains, protect
him from unwise investments, relieve him from one-sided contracts, or annul the effects of
foolish acts. There must be, in addition, a violation of the law, the commission of what the
law knows as an actionable wrong, before the courts are authorized to lay hold of the
situation and remedy it.

6) MACAPAGAL vs. CATALINA O. REMORIN, CORAZON CALUZA BAMRUNGCHEEP,


and LAURELIA CALUZA-VALENCIANO
FACTS: Lots 24 and 25 were registered in the name of Candido Caluza under Transfer
Certificate of Title (TCT) No. 160544. Purificacion Arce-Caluza (Purificacion) is his second
wife. Corazon Caluza-Bamrungcheep (Corazon) is his legally adopted daughter during his
first marriage. After Candido died in 1981, Corazon and Purificacion executed a Deed of
Extrajudicial Settlement adjudicating between themselves the properties of Candido, as the
latter's surviving heirs.

SALES CASE DIGEST COMPILATION 107


Lots 24 and 25, together with Lot 23, which was registered in Candido's name, were
adjudicated to Corazon. Purificacion got Candido's land in Bulacan. However, administration
of Lots 23, 24 and 25 were entrusted to Purificacion by Corazon as she had to leave for
Thailand after her marriage to a Thai.
Unknown to Corazon, Purificacion executed an Affidavit of Loss alleging that the TCTs of
Lots No 23, 24 and 25 were lost and could no longer be found. She filed a petition for the
issuance of new owner's duplicates of title alleging that she was her deceased husband's
sole heir. The petition was granted and new TCTs were issued in Purificacion's name.
Purificacion then sold the lots to Catalina Remorin (Catalina) and Catalina mortgaged Lots
24 and 25 to L & R Lending Corp.
Corazon filed a complaint for reconveyance and damages against Purificacion and Catalina
upon discovery of sale. Plaintiff alleged that the two defendants connived with each other in
transferring the three lots in their names through simulated sales. Corazon likewise filed a
criminal complaint for falsification and perjury against the two.
Catalina executed a Deed of Transfer, signed by Purificacion as witness, admitting the
wrong they did in illegally transferring the lots in their names and acknowledging Corazon to
be the rightful owner under the Deed of Extrajudicial Settlement. Corazon presented the
Deed of Transfer before the Register of Deeds of Quezon City and Catalina's TCT over Lots
24 and 25 was cancelled and a TCT was issued in Corazon's name.
Prior thereto, however, Catalina mortgaged Lots 24 and 25 to respondent Laurelia Caluza-
Valenciano (Laurelia) to pay off her mortgage indebtedness to L & R Lending Corporation.
The inscription of the mortgage in favor of Laurelia was carried over to Corazon's TCT.
-Corazon, Purificacion, Catalina, and Laurelia executed a Memorandum of Agreement to
settle Civil Case. It stipulated that Corazon cedes and grants unto and in favor of
Purificacion full ownership and other real rights over the southernmost apartment as well as
the portion of the lot occupied thereby subject to the condition that Purificacion shall assume
satisfaction of the mortgage debt contracted by Catalina in favor of Laurelia and shall cause
transfer of said annotation to the title to be issued in her (Purificacion's) name; and
furthermore that any and all expenses for segregation survey, re-titling and annotation of
said mortgage shall be shouldered by said Purificacion Arce-Caluza;
Before the agreement could be implemented, Purificacion died. Consequently, another
compromise agreement was executed stating that Corazon and Catalina agreed that title to
the southernmost apartment as well as the portion of the lot occupied thereby shall be
transferred direct to its interested buyer with defendant Catalina assuming and paying (from
the proceeds of the sale) her mortgage obligation with Laurelia; any and all expenses for
segregation survey, re-titling, capital gains taxes and those connected with the annotation
and/or release of said mortgage should now be shouldered by defendant Catalina O.
Remorin. Corazon then sold the subject Lot to Laurelia by virtue of a deed entitled "Sale of
Unsegregated Portion of Land." However, Catalina also sold the same lot to Macapagal
claiming to be authorized under the Compromise Agreement. Macapagal sought to nullify
the sale executed by Corazon in favor of Laurelia and to declare valid the one executed by
Catalina in her favor.
RTC rendered judgment in favor of petitioner. Corazon and Laurelia appealed to the Court of
Appeals which reversed the decision of the trial court.
Macapagal’s contention: the sale executed by Catalina in her favor should prevail over the
one executed by Corazon in favor of Laurelia, as Catalina was the one authorized to sell the
disputed property under the Compromise Agreement
Respondent’s contention: Corazon, the registered owner of the disputed property, did not
give Catalina authority to sell the lot. It was provided in the Agreement that Catalina shall

SALES CASE DIGEST COMPILATION 108


pay off her mortgage obligation and incidental expenses from the proceeds of the sale only
to reassure Catalina that her obligation would be paid in the event that Corazon sells the
property.
ISSUE: WON Catalina was authorized to sell the land as provided by their Compromise
Agreement.
RULING: The Compromise Agreement dated September 9, 1988 cannot be taken as a
waiver of Corazon's authority to sell and grant thereof to Catalina considering that the
Agreement merely provided that Catalina pay off her mortgage obligation and incidental
expenses from the proceeds of the sale. Although it was imperative, as part of the
compromise, that the money come from the proceeds of the sale, it was not expressly
stated, nor did it necessarily mean, that Catalina herself be the one to directly sell the
property. Authority to sell must be couched in clear and unmistakable language.
Moreover, intent to give Catalina authority to sell may not be easily attributed to Corazon
considering that the latter had to file the reconveyance case as a result of Purificacion's and
Catalina's acts of transferring the disputed lot in their names.
In contract interpretation, analysis is not to be limited to the words used in the contract, as
they may not accurately reflect the parties' true intent. If the words of the contract appear to
be contrary to the evident intention as revealed by the circumstances, the latter shall prevail
over the former.
The fact that the deed of sale between respondents Corazon and Laurelia did not accurately
reflect the true consideration thereof is not cause for declaration of its nullity. When the
parties intended to be bound by the contract except that it did not reflect the actual purchase
price of the property, there is only a relative simulation of the contract which remains valid
and enforceable. It cannot be declared null and void since it does not fall under the category
of an absolutely simulated or fictitious contract. The contract of sale is valid but subject to
reformation.
Petition denied.

7) CLARA M. BALATBAT vs CA, SPS. REPUYAN


G.R. No. 109410, August 28, 1996
FACTS: A parcel of land was acquired by plaintiff Aurelio Roque and Maria Mesina during
their conjugal union. Maria died on August 28, 1966.
On June 15, 1977, Aurelio filed a case for partition. The trial court in that case for partition,
held that Aurelio is entitled to the ½ portion of his share in the conjugal property, and the
other half which formed part of the estate of Maria Mesina, will be divided equally between
him and their 4 children receiving 1/5 each. The decision having become final and
executory, the Register of Deeds of Manila issued a transfer certificate of title on October 5,
1979 according to the ruling of the court.
On April 1, 1980, Aurelio sold his 6/10 shares to spouses Aurora Tuazon-Repuyan and Jose
Repuyan, as evidenced by a deed of absolute sale. On June 21, 1980, Aurora caused the
annotation of her affidavit of adverse claim. However, on August 20, 1980, Aurelio filed a
complaint for rescission of contract grounded on the buyers’ failure to pay the balance of the
purchase price.
Subsequently, on February 4, 1982, another deed of absolute sale was executed between
Aurelio and his children, and herein petitioner Clara Balatbat, involving the entire lot.
Balatbat filed a motion for the issuance of writ of possession, which was granted by the court

SALES CASE DIGEST COMPILATION 109


on September 20, 1982, subject to valid rights and interests of third persons. Balatbat filed a
motion to intervene in the rescission case, but did not file her complaint in intervention. The
court ruled that the sale between Aurelio and Aurora is valid. On March 3, 1987 however,
Balatbat filed a notice of lis pendens before the Register of Deeds regarding the subject
property.
ISSUES:
W/N the alleged sale to Spouses Repuyan was merely executor
HELD:
No. The sale was consummated, hence, valid and enforceable.
Contrary to petitioner's contention that the sale dated April 1, 1980 in favor of Spouses
Repuyan was merely executory for the reason that there was no delivery of the subject
property and that consideration/price was not fully paid, the Court finds the sale as
consummated, hence, valid and enforceable. The Court dismissed vendor's Aurelio Roque
complaint for rescission of the deed of sale and declared that the Sale dated April 1, 1980,
as valid and enforceable. No appeal having been made, the decision became final and
executory.
Examining the terms and conditions of the "Deed of Sale" dated April 1, 1980, the
P45,000.00 balance is payable only "after the property covered by T.C.T. No. 135671 has
been partitioned and subdivided, and title issued in the name of the BUYER" hence, vendor
Roque cannot demand payment of the balance unless and until the property has been
subdivided and titled in the name of private respondents. Devoid of any stipulation that
“ownership in the thing shall not pass to the purchaser until he has fully paid the price,”
ownership of the thing shall pass from the vendor to the vendee upon actual or constructive
delivery of the thing sold even if the purchase price has not yet been fully paid. The failure of
the buyer to make good the price does not, in law, cause ownership to evert to the seller
unless the bilateral contract of sale is first rescinded or resolved pursuant to Article 1191 of
the New Civil Code.
Non-payment only creates a right to demand the fulfillment of the obligation or to rescind the
contract.
With respect to the non-delivery of the possession of the subject property to the private
respondent, suffice it to say that ownership of the thing sold is acquired only from the time of
delivery thereof, actual or constructive.
A contract of sale being consensual, it is perfected by the mere consent of the parties.
Delivery of the thing bought or payment of the price is not necessary for the perfection of the
contract; and failure of the vendee to pay to price after the execution of the contract does not
make the sale null and void for lack of consideration but results at most in default on the part
of the vendee, for which the vendor may exercise his legal remedies.

8) IMELDA ONG, ET AL. vs ALFREDO ON ET AL.


G.R. No. L-67888, October 8, 1985
FACTS: On February 25, 1976, Imelda Ong for and in consideration of One (1.00) Peso and
other valuable considerations, executed in favor of Sandra Maruzzo, then a minor, a
Quitclaim Deed whereby she transferred, released, and assigned all her rights and title over
a parcel of land in Makati.
However, on November 19, 1980, Imelda Ong revoked the aforesaid Quitclaim and donated
the property to her son Rex.

SALES CASE DIGEST COMPILATION 110


Subsequently, Sandra Maruzzo on June 20, 1983, through here guardian ad litem Alfredo
Ong, filed with the RTC an action for the recovery of ownership/possession and nullification
of the Deed of Donation in favor of Rex.
Petitioners claimed that the Quitclaim Deed is null and void inasmuch as it is equivalent to a
Deed of Donation, acceptance of which by the donee is necessary to give it validity. Further,
it is averred that the donee, Sandra Maruzzo, being a minor, had no legal personality and
therefore incapable of accepting the donation.
The trial court ruled in favor of Maruzzo and held that the Quitclaim Deed is equivalent to a
Deed of Sale and, hence, there was a valid conveyance in favor of the latter.
Appealing to the IAC, petitioners additionally contends that the One (1.00) Peso
consideration is not a consideration at all to sustain the ruling that the Quitclaim Deed is
equivalent to a sale. The IAC however affirmed the TC.
ISSUE: W/N the quitclaim is equivalent to a deed of sale or to a deed of donation
HELD: The Quitclaim Deed is equivalent to a deed of sale. A careful perusal of the subject
deed reveals that the conveyance of the one- half (½) undivided portion of the above-
described property was for and in consideration of the One (P 1.00) Peso and the other
valuable considerations (emphasis supplied) paid by private respondent Sandra Maruzzo
through her representative, Alfredo Ong, to petitioner Imelda Ong. Stated differently, the
cause or consideration is not the One (P1.00) Peso alone but also the other valuable
considerations.
Although the cause is not stated in the contract it is presumed that it is existing unless the
debtor proves the contrary (Article 1354 of the Civil Code). One of the disputable
presumptions is that there is a sufficient cause of the contract. It is a legal presumption of
sufficient cause or consideration supporting a contract even if such cause is not stated
therein (Article 1354, New Civil Code of the Philippines.) This presumption cannot be
overcome by a simple assertion of lack of consideration especially when the contract itself
states that consideration was given, and the same has been reduced into a public
instrument with all due formalities and solemnities. To overcome the presumption of
consideration the alleged lack of consideration must be shown by preponderance of
evidence in a proper action.
The execution of a deed purporting to convey ownership of a realty is in itself prima facie
evidence of the existence of a valuable consideration, the party alleging lack of
consideration has the burden of proving such allegation.
Even granting that the Quitclaim deed in question is a donation, Article 741 of the Civil Code
provides that the requirement of the acceptance of the donation in favor of minor by parents
of legal representatives applies only to onerous and conditional donations where the
donation may have to assume certain charges or burdens. Donation to an incapacitated
donee does not need the acceptance by the lawful representative if said donation does not
contain any condition. In simple and pure donation, the formal acceptance is not important
for the donor requires no right to be protected and the donee neither undertakes to do
anything nor assumes any obligation. The Quitclaim now in question does not impose any
condition.

9) ISAAC BAGNAS ET AL. vs. CA


G.R. No. L-38498, August 10, 1989

SALES CASE DIGEST COMPILATION 111


FACTS: Hilario Mateum died without a will and was survived only by collateral relatives.
Bagnas et al., the petitioners, were his nearest kin. The respondents Retonil et al. on the
other hand were relatives to a farther extent.
Retonil et al. claims ownership of 10 parcels of land from the estate of Hilarion which they
contend were sold by Hilario through two deeds of sale where the consideration for the lands
was one (1.00) Peso and services rendered, being rendered and to be rendered.
Bagnas et al. filed a case against respondents seeking annulment of the deeds of sale as
fictitious, fraudulent or falsified, or alternatively, as donations void for want of acceptance
embodied in a public instrument. In answer to the complaint, the respondents denied the
alleged fictitious or fraudulent character of the sales in their favor, asserting that the said
sales were made for good and valuable consideration.
ISSUE: W/N the said consideration is valid.
HELD: The Court ruled that the deeds of sale are void and are of no force and effect.
Upon the consideration alone that the apparent gross, not to say enormous, disproportion
between the stipulated price (in each deed) of P l.00 plus unspecified and unquantified
services and the undisputably valuable real estate allegedly sold worth at least P10,500.00
going only by assessments for tax purposes which, it is well-known, are notoriously low
indicators of actual value plainly and unquestionably demonstrates that they state a false
and fictitious consideration, and no other true and lawful cause having been shown, the
Court finds both said deeds, insofar as they purport to be sales, not merely voidable, but
void ab initio.
The transfers in question being void, it follows as a necessary consequence that the
properties purportedly conveyed remained part of the estate of Hilario Mateum, said
transfers notwithstanding, recoverable by his intestate heirs, the petitioners herein, whose
status as such is not challenged.
Even if the “contract of sale” would be shown as a donation (apparently, this was the intent
of the donor), failure to conform to the requirements would not make it a valid donation.
There is no Contract of Sale for lack of consideration. Likewise, there is also no valid deed of
donation for failure to conform to the requirements of donation.

10) REPUBLIC vs PRDC and CA


G.R. No. L-10141, January 31, 1958
FACTS: The Republic brought an action against Apostol for the collection of sums owing to
it for his purchase of Palawan Almaciga and other logs. His total debt amounted to some
P34,000. PRDC intervened claiming that Apostol, as President of the company, without prior
authority, took goods (steel sheets, pipes, bars, etc) from PRDC warehouse and
appropriated them to settle his personal debts in favor of the government. The Republic
opposed the intervention of PRDC, arguing that price is always paid in money and that
payment in kind is no payment at all; hence, money and not the goods of PRDC are under
dispute.
ISSUE: W/N payment in kind is equivalent to price paid in money.
HELD: Yes. The Government argues that "Price . . . is always paid in terms of money and
the supposed payment being in kind, it is no payment at all, "citing Article 1458 of the new
Civil Code.

SALES CASE DIGEST COMPILATION 112


However, the same Article provides that the purchaser may pay "a price certain in money or
its equivalent," which means that they meant of the price need not be in money. Whether the
G.I. sheets, black sheets, M. S. Plates, round bars and G. I. pipes claimed by the
respondent corporation to belong to it and delivered to the Bureau of Prison by Macario
Apostol in payment of his account is sufficient payment therefore, is for the court to pass
upon and decide after hearing all the parties in the case. Should the trial court hold that it is
as to credit Apostol with the value or price of the materials delivered by him, certainly the
herein respondent corporation would be affected adversely if its claim of ownership of such
sheets, plates, bars and pipes is true.

11) VILLANUEVA VS. CA


267 SCRA 89
G.R. NO. 107624
JANUARY 28, 1997
FACTS: Petitioner Gamaliel Villanueva has been a tenant-occupant of a unit in an
apartment building erected on a parcel of land owned by private respondents dela Cruz. In
1986, Jose dela Cruz offered said land with the apartment building for sale and petitioners
(Gamaliel and Irene) showed interest in the property.
As initial step, Jose gave Irene a letter of authority for her to inspect the property. Since the
property was in arrears for payment of realty taxes, Jose approached Irene and asked for a
certain amount to pay for the taxes so that the property would be cleared of any
encumbrance. Irene gave 10k (5k on 2 occasions). It was agreed by them that the 10k would
form part of the sale price of 550k.
Thereafter, Jose went to Irene, bringing with him Mr. Sabio, requesting her to allow Sabio to
purchase ½ of the property, to which they consented, so they would just purchase the other
half (265k, having paid the 10k). Dela Cruz executed in favor of their co-defendants (Guido
and Felicitas Pile) a Deed of Assignment of the other ½ portion of the land, wherein
Gamaliel’s apartment unit is situated. This was purportedly as full payment and satisfaction
of an indebtedness obtained from the Piles. TCT was later issued in the name of the Piles.
Soon, Gamaliel learned about the assignment and issuance of new TCT. Petitioners
elevated their complaint to the Court (specific performance). They contend that a contract of
sale has been perfected and that the 10k formed part of the purchase price (necessarily
then, there must have been an agreement as to the price). They cite Art 1482: Whenever
earnest money is given in a contract of sale, it shall be considered as part of the price and
proof of perfection of the contract. On the other hand, private respondents claim that what
was agreed upon was that the 10k be primarily intended as payment for realty tax, and was
going to for part of the consideration of the sale if the transaction would finally be
consummated. They insist that there was no clear agreement as to the true amount of
consideration.
ISSUE: Was there a perfected contract of sale? NO
HELD: After a review of the evidence, SC found that there was no agreement as to the price
(based on the testimonies). To settle the conflicting claims, petitioners could have presented
the contract of sale. However, it was not presented in evidence. Petitioners aver that even if
the (unsigned) deed was not produced, Jose “admitted preparing said deed in accordance
with their agreement”.
We do not agree with petitioners. Assuming arguendo that such draft deed existed, it does
not necessarily follow that there was already a definite agreement as to the price. If there

SALES CASE DIGEST COMPILATION 113


was, why then did private respondent Jose de la Cruz not sign it? If indeed the draft deed of
sale was that important to petitioners' cause, they should have shown some effort to procure
it. They could have secured it through a subpoena ducestecum or thru the use of one of the
modes of discovery. But petitioners made no such effort. And even if produced, it would not
have commanded any probative value as it was not signed.
The price of the leased land not having been fixed, the essential elements which give life to
the contract were lacking. It follows that the lessee cannot compel the lessor to sell the
leased land to him.
The price must be certain; it must be real, not fictitious. It is not necessary that the certainty
of the price be actual or determined at the time of executing the contract. The fact that the
exact amount to be paid therefor is not precisely fixed, is no bar to an action to recover such
compensation, provided the contract, by its terms, furnishes a basis or measure for
ascertaining the amount agreed upon. The price could be made certain by the application of
known factors. A contract of sale is not void for uncertainty when the price, though not
directly stated in terms of pesos and centavos, can be made certain by reference to existing
invoices identified in the agreement.
In the instant case, however, what is dramatically clear from the evidence is that there was
no meeting of mind as to the price, expressly or impliedly, directly or indirectly.

12) MORENO, JR. VS. PRIVATE MANAGEMENT OFFICE


507 SCRA 63
G.R. NO. 159373
NOVEMBER 16, 2006
FACTS: The subject-matter in the civil case is the J. Moreno Building (formerly known as the
North Davao Mining Building) – or more specifically, the 2nd, 3rd, 4th, 5th and 6th floors of
the building.
Moreno is the owner of the Ground Floor, the 7th Floor and the Penthouse of the J. Moreno
Building and the lot on which it stands.
Private Management Office (formerly, Asset Privatization Trust or APT) on the other hand, is
the owner of the 2nd, 3rd, 4th, 5th and 6th floors of the building, the subject-matter of this
suit.
On February 13, 1993, APT called for a conference for the purpose of discussing Moreno’s
right of first refusal over the floors of the building owned by APT. At said meeting, APT
informed Moreno that the proposed purchase price for said floors was P21 Million.
In a letter dated February 22, 1993, APT, informed Moreno that the Board of Trustees (BOT)
of APT "is in agreement that Mr. Jose Moreno, Jr. has the right of first refusal" and
requested Moreno to deposit 10% of the "suggested indicative price" of P21 million on or
before February 26, 1993.
Moreno paid the P21 million on February 26, 1993. APT issued an OR for the said payment.
But later, APT wrote Moreno that its Legal Department has questioned the basis for the
computation of the indicative price for the said floors. Thus, on April 2, 1993, APT wrote
Moreno that the APT BOT has "tentatively agreed on a settlement price of P42,274,702.17"
for the said floors.

SALES CASE DIGEST COMPILATION 114


RTC ruled in favor of Moreno, declared that there was a perfected contract of sale and
ordered APT to sell the subject floors at P21M.
CA reversed, hence the petition.
ISSUE: WON there was a perfected contract of sale over the subject floors at the price of 21
Million. NO
HELD: A contract of sale is perfected at the moment there is a meeting of minds upon the
thing which is the object of the contract and upon the price. Consent is manifested by the
meeting of the offer and the acceptance upon the thing and the cause which are to
constitute the contract. The offer must be certain and the acceptance absolute.
To reach that moment of perfection, the parties must agree on the same thing in the same
sense, so that their minds meet as to all the terms. They must have a distinct intention
common to both and without doubt or difference; until all understand alike, there can be no
assent, and therefore no contract. The minds of parties must meet at every point; nothing
can be left open for further arrangement
So long as there is any uncertainty or indefiniteness, or future negotiations or considerations
to be had between the parties, there is not a completed contract, and in fact, there is no
contract at all.
Once there is concurrence of the offer and acceptance of the object and cause, the stage of
negotiation is finished. This situation does not obtain in the case at bar. The letter of
February 22, 1993 and the surrounding circumstances clearly show that the parties are not
past the stage of negotiation, hence there could not have been a perfected contract of sale.
The letter is clear evidence that APT did not intend to sell the subject floors at the price
certainofP21M, viz.:
(This letter was addressed to Moreno’s Atty.)
xxx We are pleased to inform you that the Board is in agreement that Mr. Jose Moreno, Jr.
has the right of first refusal. This will be confirmed by our Board during the next board
meeting on February 26, 1993. In the meantime, please advise Mr. Moreno that the
suggested indicative price for APT’s five (5) floors of the building in question is P21 Million.
If Mr. Moreno is in agreement, he should deposit with APT the amount of P2.1 Million
equivalent to 10% of the price on or before February 26, 1993. The balance will be due
within fifteen (15) days after Mr. Moreno receives the formal notice of approval of the
indicative price. xxx
The letter clearly states that P21M is merely a "suggested indicative price" of the subject
floors as it was yet to be approved by the BOT.
Before the Board could confirm the suggested indicative price, the Committee on
Privatization must first approve the terms of the sale or disposition. The imposition of this
suspensive condition finds basis under Proclamation No. 5022 which vests in the Committee
the power to approve the sale of government assets, including the price of the asset to be
sold (apparently government pala itong APT, and may procedure na sinusunod sa law).
Other discussions that may be relevant:
On Moreno’s argument that the "suggested indicative price" of P21M is not a proposed
price, but the selling price indicative of the value at which APT was willing to sell.
The trial court relied upon the definition of the word "indicative" under the Webster
Comprehensive Dictionary, International Edition. According to Webster, "to indicate" is to

SALES CASE DIGEST COMPILATION 115


point out; direct attention; to indicate the correct page. "Indicative" is merely the adjective of
the verb to indicate. xxx
Under the Rules of Court, the terms are presumed to have been used in their primary and
general acceptation, but evidence is admissible to show that they have a local, technical, or
otherwise peculiar signification, and were so used and understood in the particular instance,
in which case the agreement must be construed accordingly.
The reliance of the trial court in the Webster definition of the term "indicative," as also
adopted by Moreno, is misplaced. The transaction at bar involves the sale of an asset under
a privatization scheme which attaches a peculiar meaning or signification to the term
"indicative price." Under No. 6.1 of the General Bidding Procedures and Rules of
respondent, "an indicative price is a ballpark figure and [respondent] supplies such a figure
purely to define the ball-park." The plain contention of Moreno that the transaction involves
an "ordinary arms-length sale of property" is unsubstantiated and leaves much to be desired.
This case sprung from a case of specific performance initiated by Moreno who has the
burden to prove that the case should be spared from the application of the technical terms in
the sale and disposition of assets under privatization.
He failed to discharge the burden.
It appears in the case at bar that Moreno’s construction of the letter of February 22, 1993 –
that his assent to the "suggested indicative price" of P21M converted it as the price certain,
thus giving rise to a perfected contract of sale– is his own subjective understanding. As
such, it is not shared by APT. Under American jurisprudence, mutual assent is judged by an
objective standard, looking to the express words the parties used in the contract. Under the
objective theory of contract, understandings and beliefs are effective only if shared. Based
on the objective manifestations of the parties in the case at bar, there was no meeting of the
minds.

13) NAVARRA VS. PLANTERS


527 SCRA 561
G.R. NO. 172674
JULY 12, 2007
FACTS: The Navarras obtained a loan of P1,200,000.00 from Planters Bank and, by way of
security therefor, executed a deed of mortgage over their five (5) parcels of land.
Unfortunately, the couple failed to pay their loan obligation. Hence, Planters Bank foreclosed
on the mortgage. The one year redemption period expired without the Navarras having
redeemed the foreclosed properties.
On the other hand, co-petitioner RRRC Development Corporation (RRRC) is a real estate
company owned by the parents of Carmelita Bernardo Navarra. RRRC itself obtained a loan
from Planters Bank secured by a mortgage over another set of properties owned by RRRC.
The loan having been likewise unpaid, Planters Bank similarly foreclosed the mortgaged
assets of RRRC.
Unlike the Navarras, however, RRRC was able to negotiate with the Bank for the redemption
of its foreclosed properties by way of a concession whereby the Bank allowed RRRC to refer
to it would-be buyers of the foreclosed RRRC properties who would remit their payments
directly to the Bank, which payments would then be considered as redemption price for
RRRC. Eventually, the foreclosed properties of RRRC were sold to third persons whose
payments therefor, directly made to the Bank, were in excess by P300,000.00 for the
redemption price.

SALES CASE DIGEST COMPILATION 116


In the meantime, Jorge Navarra sent a letter {*contents of the letters are found in the ruling*}
to Planters Bank, proposing to repurchase the five (5) lots earlier auctioned to the Bank, with
a request that he be given until August 31, 1985 to pay the down payment of P300,000.00.
In response, Planters Bank, thru its Vice-President Ma. Flordeliza Aguenza, wrote back
Navarra via a letter dated August 16, 1985. Then, on January 21, 1987, Planters Bank sent
a letter to Jorge Navarra informing him that it could not proceed with the documentation of
the proposed repurchase of the foreclosed properties on account of his non-compliance with
the Bank’s request for the submission of the needed board resolution of RRRC, thus,
demanding that they surrender and vacate the properties in question for their failure to
exercise their right of redemption.
The Navarras filed their complaint for Specific Performance with Injunction against Planters
Bank, alleging that a perfected contract of sale was made between them and Planters Bank
whereby they would repurchase the subject properties for P1,800,000.00 with a down
payment of P300,000.00. In its Answer, Planters Bank asserted that there was no perfected
contract of sale because the terms and conditions for the repurchase have not yet been
agreed upon.
The RTC ruled that there was a perfected contract of sale between the Navarras and
Planters Bank. The CA reversed the decision citing Article 1319 as basis, declaring that the
acceptance of the offer was not absolute ISSUES:
WON there was a perfected contract to repurchase the foreclosed properties between the
petitioners and the private respondent Planters Development Bank. NO
WON the parties never got past the negotiation stage. YES
HELD:
In general, contracts undergo three distinct stages: negotiation, perfection or birth, and
consummation. Negotiation begins from the time the prospective contracting parties
manifest their interest in the contract and ends at the moment of their agreement. Perfection
or birth of the contract takes place when the parties agree upon the essential elements of
the contract, i.e., consent, object and price. Consummation occurs when the parties fulfill or
perform the terms agreed upon in the contract, culminating in the extinguishment thereof.
A negotiation is formally initiated by an offer which should be certain with respect to both the
object and the cause or consideration of the envisioned contract. In order to produce a
contract, here must be acceptance, which may be express or implied, but it must not qualify
the terms of the offer. In other words, it must be identical in all respects with that of the offer
so as to produce consent or meeting of the minds.
Here, the Navarras assert that the following exchange of correspondence between them and
Planters Bank constitutes the offer and acceptance, thus:
Letter dated July 18, 1985 of Jorge Navarra:
This will formalize my request for your kind consideration in allowing my brother and me to
buy back my house and lot and my restaurant building and lot together with the adjacent
road lot. Since my brother, who is working in Saudi Arabia, has accepted this arrangement
only recently as a result of my urgent offer to him, perhaps it will be safe for us to set August
31, 1985 as the last day for the payment of a P300,000.00 down payment. I hope you will
grant us the opportunity to raise the funds within this period, which includes an allowance for
delays.
The purchase price, I understand, will be based on the redemption value plus accrued
interest at the prevailing rate up to the date of our sales contract.

SALES CASE DIGEST COMPILATION 117


Maybe you can give us a long term payment scheme on the basis of my brother’s annual
savings of roughly US$30,000.00 everytime he comes home for his home leave. I realize
that this is not a regular transaction but I am seeking your favor to give me a chance to
reserve whatever values I can still recover from the properties and to avoid any legal
complications that may arise as a consequence of the total loss of the Balangay lot. I hope
that you will extend to me your favorable action on this grave matter.
Letter dated August 16, 1985 of Planters Bank:
Regarding your letter dated July 18, 1985, requesting that we give up to August 31, 1985 to
buy back your house and lot and restaurant and building subject to a P300,000.00
downpayment on the purchase price, please be advised that the Collection Committee has
agreed to your request. Please see Mr. Rene Castillo, Head, Acquired Assets Unit, as soon
as possible for the details of the transaction so that they may work on the necessary
documentation.
Given the above, the basic question that comes to mind is: Was the offer certain and the
acceptance absolute enough so as to engender a meeting of the minds between the
parties? Definitely not.
While the foregoing letters indicate the amount of P300,000.00 as down payment, they are,
however, completely silent as to how the succeeding instalment payments shall be made.
At most, the letters merely acknowledge that the down payment of P300,000.00 was agreed
upon by the parties. However, this fact cannot lead to the conclusion that a contract of sale
had been perfected. Quite recently, this Court held that before a valid and binding contract of
sale can exist, the manner of payment of the purchase price must first be established since
the agreement on the manner of payment goes into the price such that a disagreement on
the manner of payment is tantamount to a failure to agree on the price.
The Navarras’ letter/offer failed to specify a definite amount of the purchase price for the
sale/repurchase of the subject properties. It merely stated that the "purchase price will be
based on the redemption value plus accrued interest at the prevailing rate up to the date of
the sales contract."
The ambiguity of this statement only bolsters the uncertainty of the Navarras’ so-called
"offer" for it leaves much rooms for such questions, as: what is the redemption value? What
prevailing rate of interest shall be followed: is it the rate stipulated in the loan agreement or
the legal rate? When will the date of the contract of sale be based, shall it be upon the time
of the execution of the deed of sale or upon the time when the last instalment payment shall
have been made? To our mind, these questions need first to be addressed, discussed and
negotiated upon by the parties before a definite purchase price can be arrived at.
Significantly, the Navarras wrote in the same letter the following:
Maybe you can give us a long-term payment scheme on the basis of my brother’s annual
savings of roughly US$30,000.00 every time he comes home for his home leave.
Again, the offer was not clear insofar as concerned the exact number of years that will
comprise the long-term payment scheme. As we see it, the absence of a stipulated period
within which the repurchase price shall be paid all the more adds to the indefiniteness of the
Navarras’ offer. Clearly, then, the lack of a definite offer on the part of the spouses could not
possibly serve as the basis of their claim that the sale/repurchase of their foreclosed
properties was perfected. The reason is obvious: one essential element of a contract of sale
is wanting: the price certain. Here, what is dramatically clear is that there was no meeting of
minds vis-a-vis the price, expressly or impliedly, directly or indirectly.

SALES CASE DIGEST COMPILATION 118


Further, the tenor of Planters Bank’s letter reply negates the contention of the Navarras that
the Bank fully accepted their offer. The letter specifically stated that there is a need to
negotiate on the other details of the transaction before the sale may be formalized.
Such statement in the Bank’s letter clearly manifests lack of agreement between the parties
as to the terms of the purported contract of sale/repurchase, particularly the mode of
payment of the purchase price and the period for its payment. The law requires acceptance
to be absolute and unqualified.
As it is, the Bank’s letter is not the kind which would constitute acceptance as contemplated
by law for it does not evince any categorical and unequivocal undertaking on the part of the
Bank to sell the subject properties to the Navarras.
The Navarras’ attempt to prove the existence of a perfected contract of sale all the more
becomes futile in the light of the evidence that there was in the first place no acceptance of
their offer. It should be noted that aside from their first letter dated July 18, 1985, the
Navarras wrote another letter dated August 20, 1985, this time requesting the Bank that the
down payment of P300,000.00 be instead taken from the excess payment made by the
RRRC in redeeming its own foreclosed properties.
The very circumstance that the Navarras had to make this new request is a clear indication
that no definite agreement has yet been reached at that point. As we see it, this request
constitutes a new offer on the part of the Navarras, which offer was again conditionally
accepted by the Bank as in fact it even required the Navarras to submit a board resolution of
RRRC before it could proceed with the proposed sale/repurchase.
The eventual failure of the spouses to submit the required board resolution precludes the
perfection of a contract of sale/repurchase between the parties.
Evidently, what transpired between the parties was only a prolonged negotiation to buy and
to sell, and, at the most, an offer and a counter offer with no definite agreement having been
reached by them. With the hard reality that no perfected contract of sale/repurchase exists in
this case, any independent transaction between the Planters Bank and a third-party, like the
one involving the Gatchalian Realty, cannot be affected.

14) AMADO VS. SALVADOR


G.R. NO. 171401 DECEMBER 13, 2007
FACTS: Judge Amado is the owner of a lot, a portion of which is the subject of the present
litigation. It was alleged that sometime in 1979, Judge Amado and Salvador agreed that the
latter would sell the lot in favor of Salvador at P60/sqm. The payment was to be made in
cash or construction material, whichever the Judge preferred and to whomever the latter
wished during his lifetime. The terms of payment, though, were not stipulated.
Thereafter, Salvador undertook and the location of the squatters in said land and eventually
built several structures thereon for his business. Salvador claims that by October 1980, he
had already given Judge Amado total cash advances of P30,310.93 and delivered
construction materials amounting to P36,904.45, the total of which exceeded the agreed
price for the subject property.
Petitioner heirs averred that Judge Amado and Salvador were co-borrowers from a bank. A
loan agreement was executed by them with Capitol City Dev’t bank as lender and the Lot of
Judge Amado was used as collateral. The loaned amount was released to Salvador and
Judge Amado’s share was paid to him in several instalments. Salvador failed to pay his
share in the amortization of the lot so that Judge Amado had to pay the loan to avoid
foreclosure.

SALES CASE DIGEST COMPILATION 119


Thereafter, Judge Amado demanded Salvador to leave the premises and an ejectment case
was filed to that effect. Salvador filed a case for specific performance contending that a
balance of P4,040.62 was not paid to Judge Amado because of the latter’s failure to execute
the deed of sale. Salvador presented several documentary evidence.
RTC dismissed the complaint because Salvador’s evidence does not show that the money
and construction materials were intended as payment for the subject property. CA reversed
the decision on the finding that the construction materials delivered were not paid for.
ISSUE: WON there was a perfected contract of sale. NO
HELD: No Convincing Proof as to Manner of Payment
In the present case, Salvador fails to allege the manner of payment of the purchase price on
which the parties should have agreed. No period was set within which the payment must be
made. Of the purchase price of P66,360.00, which the parties purportedly agreed upon, the
amount which should be paid in cash and the amount for construction materials was not
determined. This means that the parties had no exact notion of the consideration for the
contract to which they supposedly gave their consent. Thus, such failure is fatal to
Salvador’s claim that a sale had been agreed upon by the parties.
Furthermore, after carefully examining the records, serious doubts became apparent as to
whether cash advances and deliveries of construction materials evidenced by numerous
statements of accounts and delivery receipts were actually intended as payment for the land.
First of all, the statements of accounts and the delivery receipts do not indicate that the
construction materials or the cash advances were made in connection with the sale of the
subject property. Any doubt as to the real meaning of the contract must be resolved against
the person who drafted the instrument and is responsible for the ambiguity thereof. Since
Salvador prepared these statements of accounts and therefore caused the ambiguity, he
cannot benefit from the resulting ambiguity. Salvador is hardly an ignorant and illiterate
person; rather, he is a businessman engaged in manufacturing and distributing construction
materials and operates no less than two branches. It should have been noted in the
statement of accounts, or even in another document, that the cash advances and deliveries
of construction materials were made in connection with a transaction as important as a sale
of land. As they are, the statements of accounts and especially the straightforward delivery
receipts are insufficient proof that Judge Amado sold his property to Salvador.
Secondly, one of the delivery receipts presented by Salvador was partially paid. If Judge
Amado had already agreed that the construction materials were payment for the subject
property, the act of partially paying for construction materials would be incongruous to such
intention.
Thirdly, Salvador himself gave conflicting statements on whether he has completed
payment. Other proofs presented gave no weight to respondent’s allegations. The testimony
of the witness presented by Salvador was not given credence. Finally, the act of Salvador in
relocating the squatters is not substantial proof of ownership.

15) BANK OF COMMERCE VS MANALO


FACTS: The Xavierville Estate, Inc. was the owner of parcels of land in Quezon City, known
as the Xavierville Estate Subdivision, with an area of 42 hectares. XEI caused the
subdivision of the property into residential lots, which was then offered for sale to individual
lot buyers.
Sometime in 1972, then XEI president Emerito Ramos, Jr. contracted the services of Engr.
Carlos Manalo, Jr. who was in business of drilling deep water wells and installing pumps

SALES CASE DIGEST COMPILATION 120


under the business name Hurricane Commercial, Inc. For P34,887.66, Manalo, Jr. installed
a water pump at Ramos residence at the corner of Aurora Boulevard and Katipunan Avenue,
Quezon City. Manalo, Jr. then proposed to XEI, through Ramos, to purchase a lot in the
Xavierville subdivision, and offered as part of the downpayment the P34,887.66 Ramos
owed him. XEI, through Ramos, agreed. In a letter dated February 8, 1972, Ramos
requested Manalo, Jr. to choose which lots he wanted to buy so that the price of the lots and
the terms of payment could be fixed and incorporated in the conditional sale. Manalo, Jr. met
with Ramos and informed him that he and his wife Perla had chosen Lots 1 and 2 of Block 2
with a total area of 1,740.3 square meters.
In a letter dated August 22, 1972 to Perla Manalo, Ramos confirmed the reservation of the
lots. He also pegged the price of the lots at P200.00 per square meter, or a total of
P348,060.00, with a 20% down payment of the purchase price amounting to P69,612.00
less the P34,887.66 owing from Ramos, payable on or before December 31, 1972; the
corresponding Contract of Conditional Sale would then be signed on or before the same
date, but if the selling operations of XEI resumed after December 31, 1972, the balance of
the downpayment would fall due then, and the spouses would sign the aforesaid contract
within 5 days from receipt of the notice of resumption of such selling operations. It was also
stated in the letter that, in the meantime, the spouses may introduce improvements thereon
subject to the rules and regulations imposed by XEI in the subdivision. Perla Manalo
conformed to the letter agreement.
The spouses Manalo took possession of the property on September 2, 1972, constructed a
house thereon, and installed a fence around the perimeter of the lots.
The spouses Manalo were notified of the resumption of the selling operations of XEI.
However, they did not pay the balance of the downpayment on the lots because Ramos
failed to prepare a contract of conditional sale and transmit the same to Manalo for their
signature. On August 14, 1973, Perla Manalo went to the XEI office and requested that the
payment of the amount representing the balance of the downpayment be deferred, which,
however, XEI rejected. On August 10, 1973, XEI furnished her with a statement of their
account as of July 31, 1973, showing that they had a balance of P34,724.34 on the
downpayment of the two lots after deducting the account of Ramos, plus P3,819.68 interest
thereon from September 1, 1972 to July 31, 1973, and that the interests on the unpaid
balance of the purchase price of P278,448.00 from September 1, 1972 to July 31, 1973
amounted to P30,629.28. The spouses were informed that they were being billed for said
unpaid interests.
Subsequently, XEI turned over its selling operations to OBM, including the receivables for
lots already contracted and those yet to be sold. Subsequently, the Commercial Bank of
Manila (CBM) acquired the Xavierville Estate from OBM.
In a letter dated August 5, 1986, the CBM requested Perla Manalo to stop any on-going
construction on the property since it (CBM) was the owner of the lot and she had no
permission for such construction. She agreed to have a conference meeting with CBM
officers where she informed them that her husband had a contract with OBM, through XEI,
to purchase the property. When asked to prove her claim, she promised to send the
documents to CBM. However, she failed to do so. On September 5, 1986, CBM reiterated its
demand that it be furnished with the documents promised, but Perla Manalo did not respond.
On July 27, 1987, CBM filed a complaint for unlawful detainer against the spouses with the
MTC Court of Quezon City.
In the meantime, the CBM was renamed the Boston Bank of the Philippines. After CBM filed
its complaint against the spouses Manalo, the latter filed a complaint for specific
performance and damages against the bank before the RTC of Quezon City.

SALES CASE DIGEST COMPILATION 121


Boston Bank, now petitioner, maintains that, as held by the CA, the records do not reflect
any schedule of payment of the 80% balance of the purchase price, or P278,448.00.
Petitioner insists that unless the parties had agreed on the manner of payment of the
principal amount, including the other terms and conditions of the contract, there would be no
existing contract of sale or contract to sell.
ISSUE: WON the manner of payment has been agreed upon and WON it is essential for
there to be an existing contract of sale or contract to sell
RULING: No, it was not agreed upon thus, there was no contract to sell. The Court agrees
with petitioners contention that, for a perfected contract of sale or contract to sell to exist in
law, there must be an agreement of the parties, not only on the price of the property sold,
but also on the manner the price is to be paid by the vendee.
Under Article 1458 of the New Civil Code, in a contract of sale, whether absolute or
conditional, one of the contracting parties obliges himself to transfer the ownership of and
deliver a determinate thing, and the other to pay therefor a price certain in money or its
equivalent. A contract of sale is perfected at the moment there is a meeting of the minds
upon the thing which is the object of the contract and the price. From the averment of
perfection, the parties are bound, not only to the fulfillment of what has been expressly
stipulated, but also to all the consequences which, according to their nature, may be in
keeping with good faith, usage and law. On the other hand, when the contract of sale or to
sell is not perfected, it cannot, as an independent source of obligation, serve as a binding
juridical relation between the parties.
A definite agreement as to the price is an essential element of a binding agreement to sell
personal or real property because it seriously affects the rights and obligations of the parties.
Price is an essential element in the formation of a binding and enforceable contract of sale.
The fixing of the price can never be left to the decision of one of the contracting parties. But
a price fixed by one of the contracting parties, if accepted by the other, gives rise to a
perfected sale.
It is not enough for the parties to agree on the price of the property. The parties must also
agree on the manner of payment of the price of the property to give rise to a binding and
enforceable contract of sale or contract to sell. This is so because the agreement as to the
manner of payment goes into the price, such that a disagreement on the manner of payment
is tantamount to a failure to agree on the price.
In a contract to sell property by installments, it is not enough that the parties agree on the
price as well as the amount of downpayment. The parties must, likewise, agree on the
manner of payment of the balance of the purchase price and on the other terms and
conditions relative to the sale. Even if the buyer makes a downpayment or portion thereof,
such payment cannot be considered as sufficient proof of the perfection of any purchase and
sale between the parties.
There is no showing, in the records, of the schedule of payment of the balance of the
purchase price on the property amounting to P278,448.00. The said parties confined
themselves to agreeing on the price of the property (P348,060.00), the 20% downpayment
of the purchase price (P69,612.00), and credited respondents for the P34,887.00 owing from
Ramos as part of the 20% downpayment. The determination of the terms of payment of the
P278,448.00 had yet to be agreed upon on or before December 31, 1972, or even
afterwards, when the parties sign the corresponding contract of conditional sale.
Jurisprudence is that if a material element of a contemplated contract is left for future
negotiations, the same is too indefinite to be enforceable. And when an essential element of
a contract is reserved for future agreement of the parties, no legal obligation arises until
such future agreement is concluded.

SALES CASE DIGEST COMPILATION 122


Indeed, the parties are in agreement that there had been no contract of conditional sale ever
executed by XEI, OBM or petitioner, as vendor, and the respondents, as vendees.
Respondents failed to allege and prove, in the trial court, that, as a matter of business
usage, habit or pattern of conduct, XEI granted all lot buyers the right to pay the balance of
the purchase price in installments of 120 months of fixed amounts with pre-computed
interests, and that XEI and the respondents had intended to adopt such terms of payment
relative to the sale of the two lots in question. Habit, custom, usage or pattern of conduct
must be proved like any other facts.
As a consequence, respondents and XEI (or OBM for that matter) failed to forge a perfected
contract to sell the two lots; hence, respondents have no cause of action for specific
performance against petitioner.

16) LAURA and ERIBERTO BAUTISTA vs. CA and FERNANDO MORELOS


Facts: The dispute involves a parcel of land situated along Maceda Street,
Sampaloc, Manila, containing an area of approximately 105 square meters. This parcel of
land was previously owned and registered in the name of the late Cesar Morelos under
Transfer Certificate of Title No. 27604. Cesar is the uncle of petitioner Laura Morelos
Bautista, being the brother of her mother, Rosario Morelos.
Cesar, who was married to Rosario Duran, did not have any children. Rosario died in
1972. Cesar died of cardiac arrest on April 15, 1982. During his lifetime, Cesar sold and
conveyed the above-mentioned parcel of land in favor of petitioner Laura Morelos Bautista,
as evidenced by a Deed of Absolute Sale notarized by Luis M. de Guzman. Accordingly,
Transfer Certificate of Title No. 254843 was issued in the name of petitioner Laura Bautista.
Respondent Fernando Morelos, claiming to be the illegitimate child of Cesar Morelos with
Angelina Lim-Gue, instituted a complaint for the declaration of nullity of sale and title with
damages. At the trial, he presented testimonies of expert witnesses who claimed that the
signature of Cesar Morelos on the Deed of Absolute Sale and the fingerprint appearing on
his Residence Certificate were not his.
Petitioners countered that the Deed of Absolute Sale was valid. The witness to the Deed,
Carmelita Marcelino, testified that she saw Cesar Morelos and petitioner Laura Bautista sign
the same.
After hearing, the court a quo rendered judgment declaring the Deed of Sale dated April 5,
1982 executed between the late Cesar Morelos in favor of Laura Bautista valid, and
dismissed for insufficient evidence the claims and counterclaims for damages of the parties.
Respondent appealed to the Court of Appeals, which reversed and set aside the judgment of
the trial court.
Issues:
I.
WHETHER OR NOT THE TESTIMONIES OF EXPERT WITNESSES ARE CONCLUSIVE
TO BE A STRONG BASIS TO NULLIFY A DULY EXECUTED AND NOTARIZED DEED OF
ABSOLUTE SALE.
II.
WHETHER OR NOT THE DEED OF ABSOLUTE SALE IS VALID.
III.

SALES CASE DIGEST COMPILATION 123


WHETHER OR NOT PRIVATE RESPONDENT HAS THE LEGAL PERSONALITY TO
SEEK THE ANNULMENT OF THE DEED OF ABSOLUTE SALE.
RULING: Petitioner asserts the validity of the Deed of Absolute Sale and invoke the
testimony of Carmelita Marcelino, the instrumental witness to the signing of the document,
who confirmed that it was the decedent Cesar Morelos who affixed his signature to the
document.
On the other hand, respondent contends that the decedents signature on the Deed was
forged. He presented the testimony of Francisco Cruz, Jr., Chief Examiner of the PC-INP
Crime Laboratory Service, that the signature of decedent on the questioned instrument,
when compared to other documents bearing the authentic signature of Cesar Morelos, did
not match and appeared to have been authored by a different person. Cruz, Jr. declared that
the latest document bearing the genuine signature of the decedent is dated March 31, 1982,
while the alleged forged signature was made on April 5, 1982, or a mere lapse of five
days. According to him, it is not possible to have significant variation between the two
signatures, considering the proximity of time when the signatures where affixed.
Another witness, Major Braulio Monge, Chief of the Fingerprint Division of the PC-INP,
testified that the thumbmark of Cesar Morelos appearing on the residence certificate
indicated in the Deed of Absolute Sale, when compared to those affixed on previous
residence certificates issued to the decedent, did not match and appears to be the
thumbmark of another person.
In the case at bar, the presumption of validity and regularity prevails over allegations of
forgery and fraud. As against direct evidence consisting of the testimony of a witness who
was physically present at the signing of the contract and who had personal knowledge
thereof, the testimony of an expert witness constitutes indirect or circumstantial evidence at
best. Carmelita Marcelino, the witness to the Deed of Absolute Sale, confirmed the
genuineness, authenticity and due execution thereof. Having been physically present to see
the decedent Cesar Morelos and petitioner Laura Bautista affix their signatures on the
document, the weight of evidence preponderates in favor of petitioners.
Witness Francisco Cruz, Jr. failed to establish the fact that the signature on the Deed of
Absolute Sale was not that of Cesar Morelos. He merely concluded that the document was a
forgery without citing any factual basis for arriving at that conclusion. Cruz did not point out
distinguishing marks, characteristics and discrepancies in and between genuine and false
specimens of writing, which would ordinarily escape detection by an ordinary lay person.
When the trial court and the appellate court arrived at divergent factual assessments in their
respective decisions and the bases therefor refer to documents made available to the
scrutiny of both courts, the well-settled rule that factual findings of trial courts deserve
respect and even finality will not apply. In the case at bar, the varying factual assessments
pertained to the authenticity of the signature of the late Cesar Morelos on the questioned
Deed of Absolute Sale conveying the 105-square meter property in favor of his niece, Laura
Bautista.
In Jimenez v. Commission on Ecumenical Mission and Relations of the United Presbyterian
Church in the United States of America, we held that the authenticity of a questioned
signature cannot be determined solely upon its general characteristics, similarities or
dissimilarities with the genuine signature. Dissimilarities as regards spontaneity, rhythm,
pressure of the pen, loops in the strokes, signs of stops, shades, that may be found between
the questioned signature and the genuine one are not decisive on the question of the
formers authenticity. The result of examinations of questioned handwriting, even with the
benefit of aid of experts and scientific instruments, is, at best, inconclusive. There are other
factors that must be taken into consideration, such as the position of the writer, the condition
of the surface on which the paper where the questioned signature is written, his state of

SALES CASE DIGEST COMPILATION 124


mind, feelings and nerves, and the kind of pen and paper used. These play an important role
on the general appearance of the signature. Unless, therefore, there is, in a given case,
absolute absence, or manifest dearth, of direct or circumstantial competent evidence on the
character of a questioned handwriting, much weight should not be given to characteristic
similarities, or dissimilarities, between a questioned handwriting and an authentic one.
Besides, a notarial document is evidence of the facts in the clear unequivocal manner
therein expressed and has in its favor the presumption of regularity. The authenticity and
due execution of the Deed of Absolute Sale must therefore be upheld.
As to the alleged insufficient consideration of the sale of the property, the mere inadequacy
of the price does not affect its validity when both parties are in a position to form an
independent judgment concerning the transaction, unless fraud, mistake or undue influence
indicative of a defect in consent is present. A contract may consequently be annulled on the
ground of vitiated consent and not due to the inadequacy of the price. In the case at bar,
however, no evidence to prove fraud, mistake or undue influence indicative of vitiated
consent was presented other than the respondents self-serving allegations.

17) Cometa vs. CA


Facts:
CFI awarded to Cometa the sum of P57, 396.85 of which the sheriff levied on 3 commercial
lots of Cometa located in Makati. 2 of the lots were sold to Franco at public auction.
Later, Herco Realty filed a civil case to annul the levy on execution and sale of the real
properties alleging that the ownership of the lots had been transferred by Cometa to Herco
before the execution of the sale. It also assailed the legality of the levy contending that the
personal properties of Cometa must be exhausted first.
Meanwhile, the RTC Branch 60 issued an order directing the Register of Deeds to cancel
the certificates of title of Cometa and to issue new ones in favor of Franco.

Issue: Whether the levy and sale is valid and proper. For if the respondent acquired no
interest in the property by virtue of the levy and sale, then, he is not entitled to its
possession.

Ruling:
There is no question that petitioners were remiss in attending with dispatch to the protection
of their interests as regards the subject lots, and for that reason the case in the lower court
was dismissed on a technicality and no definitive pronouncement on the inadequacy of the
price paid for the levied properties was ever made. In this regard, it bears stressing that
procedural rules are not to be belittled or dismissed simply because their non-observance
may have resulted in prejudice to a party’s substantive rights as in this case. Like all rules,
they are required to be followed except only when for the most persuasive of reasons they
may be relaxed to relieve a litigant of an injustice not commensurate with the degree of his
thoughtlessness in not complying with the procedure prescribed.
While there is no dispute that mere inadequacy of the price per se will not set aside a judicial
sale of real property, nevertheless, where the inadequacy of the price is purely shocking to
the conscience, such that the mind revolts at it and such that a reasonable man would
neither directly or indirectly be likely to consent to it, the same will be set aside.

SALES CASE DIGEST COMPILATION 125


The subject lots were sold en masse, not separately as above provided. The unusually low
price for which they were sold to the vendee, not to mention his vehement unwillingness to
allow redemption therein, only serves to heighten the dubiousness of the transfer.
With regard to the applicability of prescription and laches, there can be no question that they
operate as a bar in equity. However, it must be pointed out that the question of prescription
or laches cannot work to defeat justice or to perpetrate fraud and injustice.
The rule on redemption is liberally construed in favor of the original owner of the property
and the policy of the law is to aid rather than defeat him in the exercise of his right of
redemption. Thus, we allowed parties in several cases to perfect their right of redemption
even beyond the period prescribed therefore.
WHEREFORE, in view of all the foregoing, the challenged Decision of the Court of Appeals
dated January 25, 1999, which affirmed the trial court’s denial of petitioners right of
redemption, as well as the subsequent Resolution dated January 27, 2000, in CA-G.R. SP
No. 48227 entitled Zacarias Cometa, et al. v. Hon. Pedro Laggui, et al., are REVERSED and
SET ASIDE; and another one hereby rendered ordering respondent Jose Franco to accept
the tender of redemption made by petitioners and to deliver the proper certificate of
redemption to the latter.

18) BRAVO-GUERRERO vs. EDWARD P. BRAVO


FACTS: Spouses Mauricio and Simona Bravo owned 2 parcels of land measuring 287 and
291 square meters and located in Makati City, Metro Manila. The Properties are registered
under TCT Nos. 58999 and 59000 issued by the Register of Deeds of Rizal on 23 May 1958.
The Properties contain a large residential dwelling, a smaller house and other
improvements.
Mauricio and Simona had three children - Roland, Cesar and Lily, all surnamed Bravo.
Cesar died without issue. Lily Bravo married David Diaz, and had a son, David B. Diaz, Jr.
("David Jr."). Roland had six children, namely, Elizabeth Bravo-Guerrero, Edward, Roland,
Senia, Benjamin, and their half-sister, Ofelia.
Simona executed a General Power of Attorney ("GPA") on 17 June 1966 appointing
Mauricio as her attorney-in-fact. In the GPA, Simona authorized Mauricio to "mortgage or
otherwise hypothecate, sell, assign and dispose of any and all of my property, real, personal
or mixed, of any kind whatsoever and wheresoever situated, or any interest therein."
Mauricio subsequently mortgaged the Properties to the PNB and DBP for P10,000 and
P5,000, respectively.
On 25 October 1970, Mauricio executed a Deed of Sale with Assumption of Real Estate
Mortgage conveying the Properties to vendees Roland A. Bravo, Ofelia A. Bravo and
Elizabeth Bravo-Guerrero.” The sale was conditioned on the payment of P1,000 and on the
assumption by the vendees of the PNB and DBP mortgages over the Properties.
As certified by the Clerk of Court of the Regional Trial Court of Manila, the Deed of Sale was
notarized by Atty. Victorio Q. Guzman on 28 October 1970 and entered in his Notarial
Register. However, the Deed of Sale was not annotated on TCT Nos. 58999 and 59000.
Neither was it presented to PNB and DBP. The mortage loans and the receipts for loan
payments issued by PNB and DBP continued to be in Mauricio’s name even after his death
on 20 November 1973. Simona died in 1977.
On 23 June 1997, Edward, represented by his wife, Fatima Bravo, filed an action for the
judicial partition of the Properties. Edward claimed that he and the other grandchildren of
Mauricio and Simona are co-owners of the Properties by succession. Despite this,
petitioners refused to share with him the possession

SALES CASE DIGEST COMPILATION 126


and rental income of the Properties. Edward later amended his complaint to include a prayer
to annul the Deed of Sale, which he claimed was merely simulated to prejudice the other
heirs.
The trial court upheld Mauricio’s sale of the Properties to the vendees. The trial court ruled
that the sale did not prejudice the compulsory heirs, as the Properties were conveyed for
valuable consideration.
Citing Article 166 of the Civil Code, the Court of Appeals reversed trial court’s decision and
declared the Deed of Sale void for lack of Simona’s consent. It also found that there was
insufficient proof that the vendees made the mortgage payments on the Properties, since the
PNB and DBP receipts were issued in Mauricio’s name. The appellate court opined that the
rental income of the Properties, which the vendees never shared with respondents, was
sufficient to cover the mortgage payments to PNB and DBP.
ISSUE: WON the sale of the properties was simulated or void for gross inadequacy of price
RULING: No, the sale of the properties is not void either for being simulated or for
inadequacy of price.
Respondents, however, contend that the sale of the Properties was merely simulated. As
proof, respondents point to the consideration of P1,000 in the Deed of Sale, which
respondents claim is grossly inadequate compared to the actual value of the Properties.
Simulation of contract and gross inadequacy of price are distinct legal concepts, with
different effects. When the parties to an alleged contract do not really intend to be bound by
it, the contract is simulated and void. A simulated or fictitious contract has no legal effect
whatsoever because there is no real agreement between the parties.
In contrast, a contract with inadequate consideration may nevertheless embody a true
agreement between the parties. A contract of sale is a consensual contract, which becomes
valid and binding upon the meeting of minds of the parties on the price and the object of the
sale. The concept of a simulated sale is thus incompatible with inadequacy of price. When
the parties agree on a price as the actual consideration, the sale is not simulated despite the
inadequacy of the price.
Gross inadequacy of price by itself will not result in a void contract. Gross inadequacy of
price does not even affect the validity of a contract of sale, unless it signifies a defect in the
consent or that the parties actually intended a donation or some other contract. Inadequacy
of cause will not invalidate a contract unless there has been fraud, mistake or undue
influence. In this case, respondents have not proved any of the instances that would
invalidate the Deed of Sale.
Respondents even failed to establish that the consideration paid by the vendees for the
Properties was grossly inadequate. As the trial court pointed out, the Deed of Sale stipulates
that, in addition to the payment of P1,000, the vendees should assume the mortgage loans
from PNB and DBP. The consideration for the sale of the Properties was thus P1,000 in
cash and the assumption of the P15,000 mortgage.
Respondents argue that P16,000 is still far below the actual value of the Properties The tax
declarations placed the assessed value of both Properties at P16,160. Compared to this, the
price of P16,000 cannot be considered grossly inadequate, much less so shocking to the
conscience as to justify the setting aside of the Deed of Sale.

LADY ANN

SALES CASE DIGEST COMPILATION 127


19) THE DIRECTOR OF LANDS, applicant, vs. TIMOTEO ABARCA, ET AL.,claimant.
DATU BUALAN, ET AL. (Bagobos), JUAN A. SARENAS and DOMINGO
BRAGANZA, appellants.

G.R. No. L-38581 December 18, 1934

Cornelio Reta and Romualdo C. Quimpo for appellants Datu Bualan et al.
Celestino Chaves for appellants Sarenas and Braganza.
ABAD SANTOS, J.:
This appeal concerns lot No. 700 of cadastral No. 1 of the Court of First Instance of Davao.
This lot was claimed by Datu Bualan and a number of other Bagobos, on the one side, and,
on the other, by Juan A. Sarenas and Domingo Braganza. The facts and circumstances
which gave rise to the conflicting claims of the parties may be stated briefly as follows:
FACTS:
About fourteen years ago in civil case No. 346 of the court below, the lot now in
question was the subject of litigation between Datu Bualan and his co-claimants, on the one
hand, and Ciriaco Lizada, on the other. Juan A. Sarenas and Domingo Braganza were the
attorneys for Datu Bualan and his co-claimants in that suit, wherein a judgment was
rendered declaring Datu Bualan and his co-claimants the owners of the land involved in the
litigation.
Subsequently, a controversy arose between the Bagobos and their attorneys as to the
amount of fees due the latter, whereupon the attorneys took possession of the property now
in question. Action was brought by the Bagobos against their former attorneys for the
recovery of the land.
In this action (civil case No. 607) judgment was rendered ordering the attorneys to return
the property seized by them, and requiring the Bagobos to pay their former attorneys the
sum of P6,000 as fees. As a result of this judgment Datu Bualan and his co-claimants paid
Sarenas and Braganza the sum of P5,126.13. They also paid to the municipal treasurer of
Davao in the name of Sarenas and Braganza, for taxes and penalties due on the property in
the year 1926, while the same was in the possession of the latter, the sum of P1,035.87.
The Bagobos assumed that, by these payments which amounted in all to P6,162, the
judgment rendered against them for P6,000 together with interests due thereon, was fully
satisfied.chanroblesvirtualawlibrary chanrobles virtual law library
Claiming that the sum paid to the municipal treasurer of Davao should not be credited on the
amount of the judgment obtained by them, Sarenas and Braganza caused the clerk of the
court to issue a writ of execution on the said judgment. By reason of the writ of execution so
issued, the sheriff levied on the property here in question and sold it to Sarenas and
Braganza for the sum of P877.25. Upon the failure of the Bagobos to redeem the property,
they filed their claim in the present cadastral case, alleging that they were the absolute
owners of lot No. 700.chanroblesvirtualawlibrary chanrobles virtual law library

ISSUE :
Whether or not the declaration of sheriff’s sale to the lawyers due to inadequacy of
price paid were valid?
RULING:

The court decided that the lower court was right in declaring the sheriff's sale null
and void on the ground of the inadequacy of the price paid. It appears that in 1927 the
assessed value of the contested property was more than P60,000. A judicial sale of real

SALES CASE DIGEST COMPILATION 128


property will be set aside when the price is so inadequate as to shock the conscience of the
court. (National Bank vs. Gonzalez, 45 Phil., 693.)chanrobles virtual law library
In the instant case there is another important consideration. In fairness and equity, which
after all are the true aims of the law, the amount paid by Datu Bualan and his co-claimants
for taxes and penalties due on the contested property should be credited on the judgment
obtained by Sarenas and Braganza in civil case No. 607. Such taxes and penalties accrued
while the property was in that possession under a claim of ownership. It follows that the error
assigned by Datu Bualan and his co-claimants against the judgment below, to the effect that
the lower court erred in subjecting the property sought to be registered to a lien in favor of
Sarenas and Braganza for P877.25 with interests, must be
sustained.chanroblesvirtualawlibrarychanrobles virtual law library
The order appealed from is affirmed in so far as it decrees the registration of the property in
question in the names of Datu Bualan and his co-claimants, and reversed in so far as it
requires the notation of a lien in favor of Braganza and Sarenas. Costs will be taxed against
Sarenas and Braganza.

XIV. POLICITACION

1. MANILA WATER CONTAINER VS PHILIPPINE NATIONAL BANK GR.


No.166862 ; December20, 2006

THE FACTS

Petitioner was the owner of a 8,015 square meter parcel of land located in
Mandaluyong, Metro Manila. The property was covered by Transfer Certificate of Title (TCT)
No. 332098 of the Registry of Deeds of Rizal. To secure a P900,000.00 loan it had obtained
from respondent Philippine National Bank (PNB), petitioner executed a real estate mortgage
over the lot. Respondent PNB later granted petitioner a new credit accommodation of P
1,000,000.00; and, on November 16, 1973, petitioner executed an Amendment of Real
Estate Mortgage over its property. On March 3, 1981, petitioner secured another loan of
P653,000 from respondent PNB, payable in quarterly installments of P 32,650. plus interests
and other charges.
On August 5, 1982, respondent PNB filed a petition for extrajudicial
foreclosure of the real estate mortgage and sought to have the property sold at public
auction for P911,532.21, petitioners outstanding obligation to respondent PNB as of June
30, 1982, plus interests and attorneys fees.
After due notice and publication, the property was sold at public auction on
September 28, 1982 where respondent PNB was declared the winning bidder for P
1,000,000.00 The Certificate of Sale issued in its favor was registered with the Office of the
Register of Deeds of Rizal, and was annotated at the dorsal portion of the title on February
17, 1983. Thus, the period to redeem the property was to expire on February 17, 1984.
On February 10, 1984 petitioner reiterated its request for a one year
extension from February 17, 1984 within which to redeem/repurchase the property on
installment basis. PNB personnel informed petitioner that the bank does not accept partial
redemption.
Meanwhile, the Special Asset Management Department (SAMD) had
prepared a statement of account of petitioner’s obligation. It also recommended the
management of PNB to allow petitioner to repurchase the property for P 1574,560.00. PNB
rejected the offer and recommendation of SAMD. It instead suggested to petitioner to
purchase the property for P 2,660,000.00, in its minimum market value. Petitioner declared

SALES CASE DIGEST COMPILATION 129


that it had already agreed to SAMD’s offer to purchase for P 1,574,560.47 and deposited a
P725,000.00

ISSUE :

Whether or not petitioner and respondent PNB had entered into a perfected contract for
petitioner to repurchase the property for respondent.

RULING :

The SC affirmed the ruling of the appellate court that there was no perfected
contract of sale between the parties.
A contract is meeting of minds between two persons whereby one binds himself,
with respect to the other, to give something or to render some service. Under the New Civil
Code, there is no contract unless the following requisites concur:
1. Consent of the contracting parties;

2. Objection certain which is the subject matter of the contract;

3. Cause of the obligation which is established.

Contract is perfected by mere consent which is manifested by the meeting of the


offer and the acceptance upon the thing and causes which are to constitute the
contract. Once perfected, the bind between other contracting parties and the
obligations arising therefrom have the form of law between the parties and should be
complied in good faith. The absence of any essential element will negate the
existence of a perfected contract of sale.

The court ruled in Boston Bank of the Philippines vs. Manalo:

“ A definite agreement as to the price is an essential element of a binding


agreement to sell personal or real property because it seriously affects the rights and
obligations of the parties. Price is an essential element in the formation of a binding and
enforceable contract of sale. The fixing of the price can never be left to the decision of one of
the contracting parties, if accepted by the other, gives rise to a perfected sale.”

In the case at bar, the parties to the contract between Manila Metal Corporation
and PNB and not to Special Asset Management Department. Since the price offered by PNB
was not accepted, there is no contract. Hence it cannot serve as a binding juridical relation
between the parties.

2. HERMINIO TAYAG vs. AMANCIA LACSON, ROSENDO LACSON, ANTONIO


LACSON, JUAN LACSON, TEODISIA LACSON-ESPINOSA and the COURT OF
APPEALS
G.R. No. 134971 ; March 25, 2004

FACTS :

Respondents Angelica Tiotuyco Vda. de Lacson, and her children Amancia,


Antonio, Juan, and Teodosia, all surnamed Lacson, were the registered owners of three
parcels of land located in Mabalacat, Pampanga, registered in the Registered of Deeds of

SALES CASE DIGEST COMPILATION 130


San Fernando, Pampanga. The properties, which were tenanted agricultural lands, were
administered by Renato Espinosa for the owner.
On March 17, 1996, a group of original farmers individually executed in favor of
the petitioner their respective rights as tenants/tillers of the landholdings possessed and
tilled by them for and in consideration of P50.00 per square meter. The said amount was
made payable “when the legal impediments to the sale of the property to the petitioner no
longer existed.’’ The petitioner was also granted the exclusive right to buy the property if
and when the respondents, with the concurrence of the defendants-tenants, agreed to sell
the property. The petitioner gave varied sums of money to the tenants as partial payments,
and the latter issued receipts for the said amounts.
On July 24, 1996, the petitioner called a meeting of the defendants-tenants to
work out the implementation of the terms of their separate agreements. However, on August
8, 1996, the defendants-tenants, through Joven Mariano, wrote the petitioner stating that
they were not attending the meeting and instead gave notice of their collective decision to
sell all their rights and interests, as tenants/lesses, over the landholding to the respondents.
On August 19, 1996, the petitioner filed a complaint with the RTC of San
Fernando, Pampanga against the defendants-tenants, as well as the respondents, for the
court to fix a period within which to pay the agreed purchase price of P50.00 per square
meter to the defendants, as provided for in the Deeds of Assignment.

ISSUE :

Whether or not there was a valid option contract between Tayag and the
farmers by virtue of the deeds of Assignment.

RULING :

The SC defined Option Contract. “An option is a contract by which the owner
of the property agrees with another person that he shall have the right to buy his property at
a fixed price within a certain time. It is a condition offered or contract by which the owner
stipulates with another that the latter shall have the right to buy the property at a fixed price
within a certain time, or under, or in compliance with certain terms and conditions, or which
gives to the owner of the property the right to sell or demand a sale. It imposes no binding
obligation on the person holding the option, aside from the consideration for the offer. Until
accepted it is not treated as a contract. “ The second party gets in praesenti, not lands, not
an agreement that he shall have the lands if he elects. An option contract is a separate and
distinct contract from which the parties may enter into upon the conjunction of the option.
Farmers had no right to grant Tayag the option/right to buy the property as the
were merely tenants. In this case, the defendants-tenants-subtenants, under the deeds of
assignment, granted to the petitioner no only an option but the exclusive right to buy the
landholding. But the grantors were merely the defendants-tenants, and not the respondents,
the registered owners of the property. Not being the registered owners of the property, the
defendants-tenants could not legally grant to the petitioner the option, much less the
“exclusive right’ to buy the property. As the Latin saying goes, “NEMO DAT QUOD NON
HABET.”
Deeds of Assignment not valid; conditions stipulated did not arise. The full
payment of 50/sqm under Tayag and the farmers’ “option contracts’ were on the following
conditions:
1. That the Lacsons would agree to sell their property.
2. That the Deeds of Assignment were subject to the approval of DAR.
3. That there was a prohibited period within which the farmers were able to sell their
interest in the land.
There is no showing in Tayag’s complaint that the farmers had agreed to sell
their property, and that the legal impediments to the agreement no longer existed. They had

SALES CASE DIGEST COMPILATION 131


yet to submit the Deeds of Assignment to the Department of Agrarian Reform which, in turn,
had to act on and approve or disapprove the same. Unless the DAR approves the deeds,
Tayag has no right to enforce the same by asking the trial court to fix a period within which
to pay.

3. POLYTECHNIC UNIVERSITY OF THE PHILIPPINES vs GOLDEN HORIZON REALTY


CORPORATION
G.R. No. 183612 ; March 15, 2010

FACTS :

Petitioner National Development (NDC) is a government-owned and


controlled corporation, created under the Commonwealth Act No. 182, as amended by Com.
Act No. 311 and Presidential Decree No. 668. Petitioner Polytechnic University of the
Philippines (PUP) is a public, non-sectarian, non-profit educational institution created in
1978 by virtue of PD. No. 1341.
National Development Company (NDC) had in its disposal a 10 hectare
property, commonly called as NDC compound, located along Pureza St., Santa Mesa,
Manila. September 7, 1977: NDC entered into a Contract of Lease with Golden Horizon
Realty Corporations (GHRC) over a portion of the NDC Compound for a period of ten years,
renewable for another ten years with mutual consent of the parties.
On May 4, 1978, a second Contract of Lease was executed between NDC
and GHRC. In addition, GHRC as lessee was granted the “option to purchase the area
leased, the price to be negotiated and determined at the time the option to purchase is
exercised.” Sometime after September 1988, GHRC discovered that NDC had decided to
secretly dispose the property to a third party.
In the meantime, then President Corazon C. Aquino issued Memorandum
Order No. 214 dated January 6, 1989, ordering the transfer of the whole NDC Compound to
the National Government, which in turn would convey the said property in favor of PUP at
acquisition cost. The order of conveyance of the 10.31 hectare property would automatically
result in the cancellation of NDC’S total obligation in favor of the National Government in the
amount of P57,193,201.64.
PUP demanded that GHRC vacate the premises, insisting that the latter’s
lease contract had already expired. Its demand letter unheeded by GHRC, PUP filed an
ejectment case. GHRC argued that Memorandum Order No. 214 is a nullity.

ISSUE :
Whether or not the option to purchase the portion leased to GHRC was
violated by the sale of the NDC Compound in favor of PUP pursuant to Memorandum Order
No. 214.

RULING :
YES. The contract between NDC and GHRC contained an option to purchase
in favor to the lessee. The second lease contract contained the following provision:
III. It is mutually agreed by the parties that this Contract of Lease shall in full force and effect
for a period of ten (10)years counted from the effectivity of the payment of rental as provided
under sub-paragraph (b) of Article I, with option to renew for another ten (10) years with the
mutual consent of both parties. In no case should the rentals be increased by more than
100% of the original amount fixed. Lessee shall also have the option to purchase the area
leased, the price to be negotiated and determined at the time the option to purchase is
exercised.

Option Contract vs. Right of First Refusal. An option is a contract by which the
owner of the property agrees with another person that the latter shall have the right to buy
the former’s property at a fixed price within a certain time. It is a condition offered or contract

SALES CASE DIGEST COMPILATION 132


by which the owner stipulates with another that the latter shall have the right to buy the
property at a fixed price within a certain time, or under, or in compliance with certain terms
and conditions; or which gives to the owner of the property the right to sell or demand a sale.
It binds the party, who has given the option, not to enter into the principal contract with any
other person during the period designated, and, within that period, to enter into such contract
with the one to whom the option was granted, if the latter should decide to use the option.
Upon the other hand, a right of first refusal is a contractual grant, not of the
sale of a property, but of the first priority to buy the property in the event the owner sells the
same. As distinguished from an option contract, in a right of first refusal, while the object
might be made determinate, the exercise of the right of first refusal would be dependent not
only on the owner’s eventual intention to enter into a binding juridical relation with another
but also on terms, including the price, that are yet to be firmed up.
The contract between the parties involve a right of first refusal
As the option to purchase clause in the second lease contract has no definite period within
which the leased premises will be offered for sale to respondent lessee and the price is
made subject to negotiation and determined only at the time the option to buy is exercised, it
is obviously a mere right of refusal, usually inserted in lease contracts to give the lessee the
first crack to buy the property in case the lessor decides to sell the same.

When a lease contract contains a right of first refusal, the lessor has the legal duty to the
lessee not to sell the leased property to anyone at any price until after the lessor has made
an offer to sell the property to the lessee and the lessee has failed to accept it. Only after the
lessee has failed to exercise his right of first priority could the lessor sell the property to other
buyers under the same terms and conditions offered to the lessee, or under terms and
conditions more favorable to the lessor.

Respondent GHRC thus timely exercised its option to purchase on August 12, 1988
and NDC violated the right of first refusal
However, considering that NDC had been negotiating through the National Government for
the sale of the property in favor of PUP as early as July 15, 1988 without first offering to sell
it to respondent and even when respondent communicated its desire to exercise the option
to purchase granted to it under the lease contract, it is clear that NDC violated respondent’s
right of first refusal. Under the premises, the matter of the right of refusal not having been
carried over to the impliedly renewed month-to-month lease after the expiration of the
second lease contract on October 21, 1988 becomes irrelevant since at the time of the
negotiations of the sale to a third party, petitioner PUP, respondent’s right of first refusal was
still subsisting.

Indeed, basic is the rule that a party to a contract cannot unilaterally withdraw a right
of first refusal that stands upon valuable consideration
We have categorically ruled that it is not correct to say that there is no consideration for the
grant of the right of first refusal if such grant is embodied in the same contract of lease.
Since the stipulation forms part of the entire lease contract, the consideration for the lease
includes the consideration for the grant of the right of first refusal. In entering into the
contract, the lessee is in effect stating that it consents to lease the premises and to pay the
price agreed upon provided the lessor also consents that, should it sell the leased property,
then, the lessee shall be given the right to match the offered purchase price and to buy the
property at that price.

The true value of the land at the time of the sale to PUP was P1,500
GHRC, which did not offer any amount to petitioner NDC, and neither disputed the
P1,500.00 per square meter actual value of NDC’s property at that time it was sold to PUP
at P554.74 per square meter, as duly considered by this Court in the Firestone case, should
be bound by such determination. Accordingly, the price at which the leased premises should

SALES CASE DIGEST COMPILATION 133


be sold to respondent in the exercise of its right of first refusal under the lease contract with
petitioner NDC, which was pegged by the RTC at P554.74 per square meter, should be
adjusted to P1,500.00 per square meter, which more accurately reflects its true value at that
time of the sale in favor of petitioner PUP.

4. ADELFA PROPERTIES VS CA
G.R. No. 111238, January 25, 1995

Second Division, Regalado (J): 3 concur

Facts :

Rosario Jimenez- Castaneda, Salud Jimenez and their brothers, Jose and Dominador
Jimenez, were the registered co-owners of a parcel of land consisting of 17,710sq.ms (TCT
309773) situated in Barrio Culasi, Las Pinas, Metro Manila. On July 28, 1998, Jose and
Dominador Jimenez sold their share consisting of ½ of said parcel of land, specifically the
eastern portion thereof, to Adelfa Properties pursuant to “Kasulatan sa Bilihan ng Lupa”.
Subsequently, a “ Confirmatory Extrajudicial Partition Agreement” was executed by the
Jimenezes, wherein the eastern portion of the subject lot, with an area of 8,855 sq.ms. was
adjudicated to Jose and Dominador Jimenez, while the western portion was allocated to
Rosario and Salud Jimenez.

Thereafter, Adela Properties expressed interest in buying the western portion


of the property from Rosario and Salud. Accordingly, on November 25, 1989, an “ Exclusive
Option to Purchase” was executed between the parties, with the condition that the selling
price shall be P 2,856,150, that the option money of P50,000 shall be credited as partial
payment upon the consummation of sale, that the balance is to be paid on or before
November 30, 1989, and that in case of default by Adelfa Properties to pay the balance, the
option is cancelled and 50% of the option money shall be forfeited and the other 50%
refunded upon the sale of the property to a third party.

Before Adelfa Properties could make payment, it received summons on November


29, 1989, together with a copy of a complaint filed by the nephews and nieces of Rosario
and Salud against the latter, Jose and Dominador Jimenez, and Adelfa Properties in the
RTC Makati (Civil Case 89-5541), for annulment of the deed of sale in favor of Household
Corporation and recovery of ownership of the property covered by TCT 309773.

As a consequence, in a letter dated November 29, 1989, Adelfa Properties


informed Rosario and Salud that it would hold payment of the full purchase price and
suggested that the latter settle the case with their nephews and nieces. Salud Jimenez
refused to heed the suggestion of Adelfa Properties and attributed the suspension of
payment of the purchase price to lack of word of honor.

On December 14, 1989, Rosario and Salud sent Francisca Jimenez to see Atty.
Bernardo, in his capacity as Adelfa Properties’ counsel, and to inform the latter that they
were cancelling the transaction. In turn, Atty. Bernardo offered to pay the purchase price
provided that P500,000.00 be deducted therefrom for the settlement of the civil case. This
was rejected by Rosario and Salud. On December 22, 1989, Atty. Bernardo wrote Rosario
and Salud on the same matter but this time reducing the amoung from P500,000.00 to
P300,000.00 and this was also rejected by the latter. On February 23, 1990, the RTC
dismissed Civil Case 89-5541.
On April 16, 1990, Atty. Bernardo wrote Rosario and Salud informing the latter
that in view of the dismissal of the case against them, Adelfa Properties was willing to pay
the purchase price, and he requested that the corresponding deed of absolute sale be
executed. This was ignored by Rosario and Salud.

SALES CASE DIGEST COMPILATION 134


On July 27, 1990, Jimenez’ counsel sent a letter to Adelfa Properties enclosing
therein a check for P25,000.00 representing the refund of 50% of the option money paid
under the exclusive option to purchase. Rosario and Salud then requested Adelfa Properties
to return the owner’s duplicate copy of the certificate of title of Salud Jimenez. Adelfa
Properties failed to surrender the certificate of title.

Rosario and Salud Jimenez filed Civil Case 7532 in the RTC Pasay City
(Branch 113) for annulment of the contract with damages, praying, among others, that the
exclusive option to purchase be declared null and void; that Adelfa Properties be ordered to
return the owner’s duplicate certificate of title; and that the annotation of the option contract
on TCT 309773 be cancelled.

ISSUE : Whether or not the contract is a Contract of Sale, Option Contract or Contract to
Sell.

RULINGS :

.
Agreement between parties a contract to sell and not an option contract or a contract of sale

The alleged option contract is a contract to sell, rather than a contract of sale. The distinction
between the two is important for in contract of sale, the title passes to the vendee upon the
delivery of the thing sold; whereas in a contract to sell, by agreement the ownership is
reserved in the vendor and is not to pass until the full payment of the price. In a contract of
sale, the vendor has lost and cannot recover ownership until and unless the contract is
resolved or rescinded; whereas in a contract to sell, the title is retained by the vendor until
the full payment of the price. Thus, a deed of sale is considered absolute in nature where
there is neither a stipulation in the deed that title to the property sold is reserved in the seller
until the full payment of the price, nor one giving the vendor the right to unilaterally resolve
the contract the moment the buyer fails to pay within a fixed period.

That the parties really intended to execute a contract to sell is holstered by


the fact that the deed of absolute sale would have been issued only upon the payment of the
balance of the purchase price, as may be gleaned from Adelfa Properties’ letter dated April
16, 1990 wherein it informed the vendors that it “ is now ready and willing to pay you
simultaneously with the execution of the corresponding deed of absolute sale.”

Contract interpreted to ascertain intent of parties; Title not controlling if text


shows otherwise. The important task in contract interpretation is always the ascertainment of
the intention of the contracting parties and that task is to be discharged by looking to the
words they used to project that intention in their contract, all the words not just a particular
word or two, and words in context not words standing alone.

Moreover, judging from the subsequent acts of the parties which will
hereinafter be discussed, it is undeniable that the intention of the parties was to enter into a
contract to sell. In addition, the title of a contract does not necessarily determine its true
nature. Hence, the fact that the document under discussion is entitled “Exclusive Option to
Purchase” is not controlling where the text thereof shows that it is a contract to sell.

The test in determining whether a contract is a “contract of sale or


purchase” or a mere “option” is whether or not the agreement could be specifically enforced.
There is no doubt that Adelfa’s obligation to pay the purchase price is specific, definite and
certain, and consequently binding and enforceable. Had the Jimenezes chosen to enforce

SALES CASE DIGEST COMPILATION 135


the contract, they could have specifically compelled Adelfa to pay the balance of
P2,806,150.00. This is distinctly made manifest in the contract itself as an integral
stipulation, compliance with which could legally and definitely be demanded from petitioner
as a consequence. Adelfa Properties justified in suspending payment of balance by reason
of vindicatory action filed against it in Civil Case 89-5541, it is easily discernible that,
although the complaint prayed for the annulment only of the contract of sale executed
between Adelfa Properties and the Jimenez brothers, the same likewise prayed for the
recovery of therein Jimenez’ share in that parcel of land specifically covered by TCT 309773.
In other words, the Jimenezes were claiming to be co-owners of the entire parcel of land
described in TCT 309773, and not only of a portion thereof nor did their claim pertain
exclusively to the eastern half adjudicated to the Jimenez brothers.

Therefore, Adelfa Properties was justified in suspending payment of the


balance of the purchase price by reason of the aforesaid vindicatory action filed against it.
The assurance made by the Jimenezes that Adelfa Properties did not have to worry about
the case because it was pure and simple harassment is not the kind of guaranty
contemplated under the exceptive clause in Article 1590 wherein the vendor is bound to
make payment even with the existence of a vindicatory action if the vendee should give a
security for the return of the price. Jimenezes may no longer be compelled to sell and deliver
subject property. Be that as it may, and the validity of the suspension of payment
notwithstanding, the Jimenezes may no longer be compelled to sell and deliver the subject
property to Adelfa Properties for two reasons, that is, Adelfa’s failure to duly effect the
consignation of the purchase price after the disturbance had ceased; and secondarily, the
fact that the contract to sell had been validly rescinded by the Jimenezes. Rescission in a
contract to sell Article 1592 of the Civil Code which requires rescission either by judicial
action or notarial act is not applicable to a contract to sell. Furthermore, judicial action for
rescission of a contract is not necessary where the contract provides for automatic
rescission in case of breach, as in the contract involved in the present controversy. By
Adelfa’s failure to comply with its obligation, the Jimenezes elected to resort to and did
announce the rescission of the contract through its letter to Adelfa dated 27 July 1990. That
written notice of rescission is deemed sufficient under the circumstances.

5. SPOUSES JULIO D. VILLAMOR AND MARINA VILLAMOR, petitioners, vs.


THE HON. COURT OF APPEALS AND SPOUSES MACARIA LABINGISA REYES
AND ROBERTO REYES, respondents.

GR. No. 977332 October 10, 1991

Facts:

In 1971, the Reyeses sold a portion of their lot in Caloocan City to the sps Villamor. Later
that year, they executed a Deed of Option wherein they stated that they have offered the
remaining portion of the lot for sale, and that the Villamor spouses agreed to buy the same.

The option to buy and sell was to be exercised ‘whenever the need of such sale arises,
either on our part or on the part of the spouses (Julio) Villamor and Marina V. Villamor, at the
same price of P70.00 per square meter. The Deed also mentioned that the cause or the
impelling reason on the part of Reyes executing the deed of option as appearing in the deed
itself is the Villamor's having agreed to buy the 300 square meter portion of private
respondents' land at P70.00 per square meter "which was greatly higher than the actual
reasonable prevailing price."

SALES CASE DIGEST COMPILATION 136


In 1984, the Villamors offered to repurchase the portion of the lot they sold to the Villamor
spouses in 1971. However, the Vilamor spouses decided they would rather buy the
remaining portion and wanted to exercise their option as provided for in the Deed of Option.
In 1984, they filed a case for specific performance against the Reyeses.
.

Issues: WON there was a valid and distinct consideration for the option contract? YES
WON the option may still be enforced? NO

Held:

The consideration in the contract was the price difference


The CA failed to give due consideration to the Villamors' evidence which shows that in 1969
the Villamor spouses bough an adjacent lot from the brother of Macaria Labing-isa for only
P18.00 per square meter which the Reyeses did not rebut. Thus, expressed in terms of
money, the consideration for the deed of option is the difference between the purchase price
of the 300 square meter portion of the lot in 1971 (P70.00 per sq.m.) and the prevailing
reasonable price of the same lot in 1971.

Whatever it is, (P25.00 or P18.00) though not specifically stated in the deed of option, was
ascertainable. The Villamor’s allegedly paying P52.00 per square meter for the option may,
as opined by the appellate court, be improbable but improbabilities do not invalidate a
contract freely entered into by the parties.

The Reyeses were also given the right to sell


The "deed of option" entered into by the parties in this case had unique features. Ordinarily,
an optional contract is a privilege existing in one person, for which he had paid a
consideration and which gives him the right to buy, for example, certain merchandise or
certain specified property, from another person, if he chooses, at any time within the agreed
period at a fixed price.

The "deed of option" went on and stated that the sale of the other half would be made
"whenever the need of such sale arises, either on our (Reyeses) part or on the part of the
Spouses Julio Villamor and Marina V. Villamor. It appears that while the option to buy was
granted to the Villamors, the Reyeses were likewise granted an option to sell. In other
words, it was not only the Villamors who were granted an option to buy for which they paid a
consideration. The Reyeses as well were granted an option to sell should the need for such
sale on their part arise.

Period to exercise the option not stipulated; deemed prescribed afetr 10 years
However, the Deed of Option did not provide for the period within which the parties may
demand the performance of their respective undertakings in the instrument. The parties
could not have contemplated that the delivery of the property and the payment thereof could
be made indefinitely and render uncertain the status of the land. The failure of either parties
to demand performance of the obligation of the other for an unreasonable length of time
renders the contract ineffective.

Under Article 1144 (1) of the Civil Code, actions upon written contract must be brought
within ten (10) years. The Deed of Option was executed on November 11, 1971. The
acceptance, as already mentioned, was also accepted in the same instrument. The

SALES CASE DIGEST COMPILATION 137


complaint in this case was filed by the Villamors on July 13, 1987, seventeen (17) years from
the time of the execution of the contract. Hence, the right of action had prescribed.

To allow the Villamors to demand the delivery of the property subject of this case thirteen
(13) years or seventeen (17) years after the execution of the deed at the price of only
P70.00 per square meter is inequitous, considering the rise of prices of real estate in Manila.

6. SANCHEZ vs. RIGOS


G.R. No. L-25494; June 14, 1972

FACTS :

On April 3, 1961, plaintiff (Nicolas Sanchez) and defendant (Severina Rigos)


executed an instrument, entitled “ Option to Purchase,” whereby Mrs. Rigos “agreed,
promised and committed to sell’ to Sanchez, for the sum of P1,510.00, a parcel of land
situated in the barrios of Abar and Sibot, municipality of San Jose, province of Nueva Ecija,
within two (2) years from said date with the understanding that said option shall be deemed
“terminated and elapsed, ”if” Sanchez shall fail to exercise his right to buy the property”
within the stipulated period. Inasmuch as several tenders of payment of the sum of
P1,510.00, made by Sanchez within said period, were rejected by Mrs. Rigos, on March 12,
1963, the former deposited said amount with the Court of First Instance of Nueva Ecija and
commenced against the latter an action for specific performance and damages.
The lower court rendered judgment for Sanchez, ordering Mrs. Rigos to accept
the sum judicially consigned by him and to execute, in his favour, the requisite deed of
conveyance. Mrs . Rigos was, likewise, sentenced to pay P200.00, as attorney’s fes, and
the costs. Hence, this appeal by Mrs. Rigos.
The plaintiff alleges that, by virtue of the option under consideration, “defendant
agreed and committed to sell” and “the plaintiff agreed and committed to buy” the land
described in the option. Hence, plaintiff maintains that the promise contained in the contract
is “reciprocally demandable,” pursuant to the first paragraph of said Article 1479.
On the other hand, the defendant contends that the contract between the
parties “is a unilateral promise to sell, and the same being unsupported by any valuable
consideration, by force of the New Civil Code, is null and void.”

ISSUE :
Whether the contract entered into by the parties is nevertheless binding for lack
of a separate consideration.

RULING : YES.

This Court itself, in the case of Atkins, Kroll & Co., Inc. vs. Cua Hian Tek (102
Phil, 948), decided later than Southwestern Sugar & Molasses Co. vs. Atlantic & Pacific Co.,
97 Phil, 249, saw no distinction between Articles 1324 and 1479 of the Civil Code and
applied the former where a unilateral promise to sell similar to the one sued upon was
involved, treating such promise as an option which, although not binding as a contract in
itself for lack of a separate consideration, nevertheless generated a bilateral contract of
purchase and sale upon acceptance. In other words, since there may be no valid contract
without a cause or consideration promisor is not bound by his promise and may, accordingly
withdraw it. Pending notice of its withdrawal, his accepted promise partakes, however, of the
nature of an offer to sell which, if accepted, results in a perfected contract of sale.
The option did not impose upon plaintiff the obligation to purchase defendant’s
property. Where both partners indicated in the instrument in the caption, as an “Option to
Purchase,” and under the provisions thereof, the defendant “agreed, promised and

SALES CASE DIGEST COMPILATION 138


committed” herself to sell the land therein described to the plaintiff for P 1,510.00, but there
is nothing in the contract to indicate that her aforementioned agreement, promise and
undertaking is supported by a consideration “distinct from the price” stipulated for the sale of
the land, it is not a “contract to buy and sell.” It merely granted plaintiff an “option” to buy.
It should be noted that : Article 1354 applies to contracts in general, whereas
the second paragraph of Article 1479 refers to “sales” in particular, and, more specifically, to
“an accepted unilateral promise to buy or to sell.”
In order that a unilateral promise may be “binding” upon the promisor, Article
1479 requires the concurrence of a condition namely, that the promise be “ supported by a
consideration distinct from the price.” Accordingly, the promise cannot compel the promisor
to comply with the promise, unless the former establishes the existence of said distinct
consideration. In other words, the promise has the burden of proving such consideration.
The view that an option to sell can still be withdrawn, even if accepted, if the
same is not supported by any consideration, has the advantage of avoiding a conflict
between Article 1324 – on the general principles on contracts – and 1479 – on sales – of the
Civil Code, in line with the cardinal rule of statutory construction that, in construing different
provisions of one and the same law or code, such interpretation should be favoured as will
reconcile or harmonize said provisions and avoid a conflict between the same. Indeed, the
presumption is that, in the process of drafting the Code, its author has maintained a
consistent philosophy or position. Moreover, the decision in Southwestern Sugar & Molasses
Co. vs. Atlantic Gulf & Pacific Co., supra, holding that Article 1324 is modified by Article
1479 of the Civil Code, in effect, considers the latter as an exception to the former, and
exceptions are not favoured, unless the intention to the contrary is clear, and it is not so,
insofar as said two (2) articles are concerned. What is more, the reference, in both the
second paragraph of Articlee 1479 and Article 1324, to an option or promise supported by or
founded upon a consideration, strongly suggests that the two (2) provisions intended to
enforce or implement the same principle.

7. PERCELINO DIAMANTE, petitioner,


vs.
HON. COURT OF APPEALS and GERARDO DEYPALUBUS, respondents.

Hernandez, Velicaria, Vibar & Santiago for petitioner.


Amancio B. Sorongon for private respondent.

A fishery lot, encompassing an area of 9.4 hectares and designated as Lot No. 518-A of the
Cadastral Survey of Dumangas, Iloilo, was previously covered by Fishpond Permit No. F-
2021 issued in the name of Anecita Dionio. Upon Anecita's death, her heirs, petitioner
Diamante and Primitivo Dafeliz, inherited the property which they later divided between
themselves; petitioner got 4.4. hectares while Dafeliz got 5 hectares. It is the petitioner's
share that is the subject of the present controversy. Primitivo Dafeliz later sold his share to
private respondent.

On 21 May 1959, petitioner sold to private respondent his leasehold rights over the property
in question for P8,000.00 with the right to repurchase the same within three (3) years from
said date.

On 16 August 1960, private respondent filed an application with the Bureau of Fisheries,
dated 12 July 1960, for a fishpond permit and a fishpond lease agreement over the entire lot,
submitting therewith the deeds of sale executed by Dafeliz and the petitioner.

SALES CASE DIGEST COMPILATION 139


Pressed by urgent financial needs, petitioner, on 17 October 1960, sold all his remaining
rights over the property in question to the private respondent for P4,000.00.

On 25 October 1960, private respondent, with his wife's consent, executed in favor of the
petitioner an Option to Repurchase the property in question within ten (10) years from said
date, with a ten-year grace period.

Private respondent submitted to the Bureau of Fisheries the definite deed of sale; he did not,
however, submit the Option to Repurchase.

Thereafter, on 2 August 1961, the Bureau of Fisheries issued to private respondent


Fishpond Permit No. 4953-Q; on 17 December 1962, it approved FLA No. 1372 in the
latter's favor.

ISSUE :

Whether or not the encumbrance executed was an option contract or a right to


repurchase.

RULING :

The Secretary was of the opinion that the Option to Repurchase was an
encumbrance on the property which affected the absolute and exclusive character of private
respondent's ownership over the 4.4 hectares sold to him by petitioner. This is a clear case
of a misapplication of the law on conventional redemption and a misunderstanding of the
effects of a right to repurchase granted subsequently in an instrument different from the
original document of sale.

Article 1601 of the Civil Code provides:

Conventional redemption shall take place when the vendor reserves the right
to repurchase the thing sold, with the obligation to comply with the provisions
of article 1616 and other stipulations which may have been agreed upon.

In Villarica, et al. vs. Court of Appeals, et al., 4 decided on 29 November 1968, or barely
seven (7) days before the respondent Court promulgated its decision in this case, this Court,
interpreting the above Article, held:

The right of repurchase is not a right granted the vendor by the vendee in a
subsequent instrument, but is a right reserved by the vendor in the same
instrument of sale as one of the stipulations of the contract. Once the
instrument of absolute sale is executed, the vendor can no longer reserve the
right to repurchase, and any right thereafter granted the vendor by the
vendee in a separate instrument cannot be a right of repurchase but some
other right like the option to buy in the instant case. . . .

In the earlier case of Ramos, et al. vs. Icasiano, et al., 5 decided in 1927, this Court had
already ruled that "an agreement to repurchase becomes a promise to sell when made after
the sale, because when the sale is made without such an agreement, the purchaser

SALES CASE DIGEST COMPILATION 140


acquires the thing sold absolutely, and if he afterwards grants the vendor the right to
repurchase, it is a new contract entered into by the purchaser, as absolute owner already of
the object. In that case the vendor has not reserved to himself the right to repurchase."

In Vda. de Cruzo, et al. vs. Carriaga, et al., 6 this Court found another occasion to apply the
foregoing principle.

Hence, the Option to Repurchase executed by private respondent in the present case, was
merely a promise to sell, which must be governed by Article 1479 of the Civil Code which
reads as follows:

Art. 1479. — A promise to buy and sell a determinate thing for a price certain
is reciprocally demandable.

An accepted unilateral promise to buy or to sell a determinate thing for a price


certain is binding upon the promissor if the promise is supported by a
consideration distinct from the price.

A copy of the so-called Option to Repurchase is neither attached to the records nor quoted
in any of the pleadings of the parties. This Court cannot, therefore, properly rule on whether
the promise was accepted and a consideration distinct from the price, supports the option.
Undoubtedly, in the absence of either or both acceptance and separate consideration, the
promise to sell is not binding upon the promissor (private respondent).

A unilateral promise to buy or sell is a mere offer, which is not converted into
a contract except at the moment it is accepted. Acceptance is the act that
gives life to a juridical obligation, because, before the promise is accepted,
the promissor may withdraw it at any time. Upon acceptance, however, a
bilateral contract to sell and to buy is created, and the offeree ipso
facto assumes the obligations of a purchaser; the offeror, on the other hand,
would be liable for damages if he fails to deliver the thing he had offered for
sale.

xxx xxx xxx

. . . The contract of option is a separate and distinct contract from the contract
which the parties may enter into upon the consummation of the option, and a
consideration for an optional contract is just as important as the consideration
for any other kind of contract. Thus, a distinction should be drawn between
the consideration for the option to repurchase, and the consideration for the
contract of repurchase itself.7

Even if the promise was accepted, private respondent was not bound thereby in the absence
of a distinct consideration.

8. SPOUSES CIPRIANO VASQUEZ and VALERIANA GAYANELO, petitioners,


vs.
HONORABLE COURT OF APPEALS and SPOUSES MARTIN VALLEJERA and
APOLONIA OLEA,respondents.

July 12, 1991

FACTS:

SALES CASE DIGEST COMPILATION 141


Petitioner Vasquez and Gayanelo sold their land in Negros Occidental to
Respondents Martin Vallejera and Apolonia Olea. In a document separate from the deed of
sale, petitioner was granted the a right to repurchase signed by him.

Petitioner availed of the right to repurchase but respondents resisted


because of lack of consideration separate from the purchase price and on the ground that
the right to repurchase was made in a separate document.

ISSUE:

WON the petitioner has the right to repurchase under the contract.

HELD:

Petitioner cannot avail of the right to repurchase because it was not accepted
by the respondents and no consideration was given. The document for the right to
repurchase was not signed by the respondents and as such, shows no acceptance from the
promisee.

The right of repurchase is not a right granted the vendor by the vendee in a
subsequent instrument, but is a right reserved by the vendor in the same instrument of sale
as one of the stipulations of the contract. Once the instrument of absolute sale is executed,
the vendor can no longer reserve the right to repurchase, and any right thereafter granted
the vendor by the vendee in a separate instrument cannot be a right of repurchase but some
other right like the option to buy in the instant case.

XV. RIGHT OF FIRST REFUSAL

1. ANG YU ASUNCION, ARTHUR GO AND KEH TIONG, Petitioners, v. THE HON.


COURT OF APPEALS and BUEN REALTY DEVELOPMENT
CORPORATION, Respondents.
G.R. No. 109125 December 2, 1994

Facts:

On July 29, 1987: An amended Complaint for Specific Performance was filed
by petitioners Ang Yu Asuncion and others against Bobby Cu Unjieng, Rose Cu Unjieng and
Jose Tan before RTC.

Petitioners (Ang Yu) alleged that they are the tenants or lessees of residential
and commercial spaces owned by Bobby Unijeng and others located in Binondo, Manila
(since 1935). On several occasions before October 9, 1986, the lessors informed the
lessees (petitioners) that they are offering to sell the premises and are giving them priority to
acquire the same; that during the negotiations, Bobby Cu Unjieng offered a price of P6-
million while they made a counter offer of P5-million; they wrote them on October 24, 1986
asking that they specify the terms and conditions of the offer to sell; that when plaintiffs did
not receive any reply, they sent another letter dated January 28, 1987 with the same
request.

SALES CASE DIGEST COMPILATION 142


The RTC found that Cu Unjiengs’ offer to sell was never accepted by the
petitioners (Ang Yu) for the reason that they did not agree upon the terms and conditions of
the proposed sale, hence, there was no contract of sale at all. The Court of Appeals affirmed
the decision of the lower court. This decision was brought to the Supreme Court by petition
for review on certiorari which subsequently denied the appeal on May 6, 1991 “for
insufficiency in form and substance”. (Referring to the first case filed by Ang Yu)

On November 15, 1990, while the case was pending consideration by this
Court, the Cu Unjieng spouses executed a Deed of Sale transferring the subject petitioner to
petitioner Buen Realty and Development Corporation. Petitioner Buen Realty and
Development Corporation, as the new owner of the subject property, wrote a letter to the
lessees demanding that the latter vacate the premises.

August 30, 1991: the RTC ordered the Cu Unjiengs to execute the necessary
Deed of Sale of the property in litigation in favor of plaintiffs Ang Yu Asuncion, Keh Tiong
and Arthur Go for the consideration of P15 Million pesos in recognition of petitioners’ right of
first refusal and that a new Transfer Certificate of Title be issued in favor of the buyer. The
court also set aside the title issued to Buen Realty Corporation for having been executed in
bad faith. On September 22, 1991, the Judge issued a writ of execution.

Issue: WON Buen Realty can be bound by the writ of execution by virtue of the notice of lis
pendens, carried over on TCT No. 195816 issued in the name of Buen Realty, at the time of
the latter’s purchase of the property on 15 November 1991 from the Cu Unjiengs. NO

Held:

Right of first refusal is not a perfected contract of sale under Article 1458 of the Civil
Code
In the law on sales, the so-called “right of first refusal” is an innovative juridical relation.
Needless to point out, it cannot be deemed a perfected contract of sale under Article 1458 of
the Civil Code.

In a right of first refusal, while the object might be made determinate, the exercise of the
right, however, would be dependent not only on the grantor’s eventual intention to enter into
a binding juridical relation with another but also on terms, including the price, that obviously
are yet to be later firmed up. Prior thereto, it can at best be so described as merely
belonging to a class of preparatory juridical relations governed not by contracts (since the
essential elements to establish the vinculum juris would still be indefinite and inconclusive)
but by, among other laws of general application, the pertinent scattered provisions of the
Civil Code on human conduct.

The proper action for violation of the right of first refysal is to file an action for
damages and NOT writ of execution
The final judgment in Civil Case No. 87-41058, it must be stressed, has merely accorded a
“right of first refusal” in favor of petitioners (Ang Yu et. al). The consequence of such a
declaration entails no more than what has heretofore been said. In fine, if, as it is here so
conveyed to us, petitioners are aggrieved by the failure of private respondents to honor the
right of first refusal, the remedy is not a writ of execution on the judgment, since there is
none to execute, but an action for damages in a proper forum for the purpose.

Unconditional mutual promise to buy vs. Accepted unilateral promise

SALES CASE DIGEST COMPILATION 143


An unconditional mutual promise to buy and sell, as long as the object is made determinate
and the price is fixed, can be obligatory on the parties, and compliance therewith may
accordingly be exacted.

An accepted unilateral promise which specifies the thing to be sold and the price to be paid,
when coupled with a valuable consideration distinct and separate from the price, is what
may properly be termed a perfected contract of option. This contract is legally binding, and in
sales, it conforms with the second paragraph of Article 1479 of the Civil Code, viz:

Art. 1479. . . .
An accepted unilateral promise to buy or to sell a determinate thing for a price certain is
binding upon the promissor if the promise is supported by a consideration distinct from the
price. (1451a)

Observe, however, that the option is not the contract of sale itself. The optionee has the
right, but not the obligation, to buy. Once the option is exercised timely, i.e., the offer is
accepted before a breach of the option, a bilateral promise to sell and to buy ensues and
both parties are then reciprocally bound to comply with their respective undertakings.

Buen Realty cannot be ousted from the ownership and possession of the property
Furthermore, whether private respondent Buen Realty Development Corporation, the
alleged purchaser of the property, has acted in good faith or bad faith and whether or not it
should, in any case, be considered bound to respect the registration of the lis pendens in
Civil Case No. 87-41058 are matters that must be independently addressed in appropriate
proceedings. Buen Realty, not having been impleaded in Civil Case No. 87-41058, cannot
be held subject to the writ of execution issued by respondent Judge, let alone ousted from
the ownership and possession of the property, without first being duly afforded its day in
court

2. Paranaque Kings Vs. CA

Facts:

PR Catalina L. Santos is the owner of 8 parcels of land located at Parañaque,


Metro Manila. On November 28, 1977, a certain Frederick Chua leased the subject property
from defendant Catalina L. Santos, the said lease was registered in the Register of Deeds.

February 12, 1979: Frederick Chua assigned all his rights and interest and participation in
the leased property to Lee Ching Bing, by virtue of a deed of assignment and with the
conformity of defendant Santos, the said assignment was also registered.

On August 6, 1979: Lee Ching Bing also assigned all his rights and interest in the leased
property to Parañaque Kings Enterprises, Incorporated by virtue of a deed of assignment
and with the conformity of defendant Santos. Their contract provided that in case the
properties subject of the lease agreement are sold or encumbered, Lessors shall impose as
a condition that the buyer or mortgagee thereof shall recognize and be bound by all the
terms and conditions of this lease agreement and shall respect this Contract of Lease as if
they are the LESSORS thereof and in case of sale, LESSEE shall have the first option or
priority to buy the properties subject of the lease;"
On September 21, 1988: Catalina Santos sold the eight parcels of land subject of the
lease to defendant David Raymundo for a consideration of P5,000,000. Upon learning of this
fact, the representative of Paranaque King wrote a letter to defendant Santos, requesting her
to rectify the error and consequently realizing the error, she had it reconveyed to her for the
same consideration of P5M.

SALES CASE DIGEST COMPILATION 144


Only 2 days after Catalina Santos sold her properties did she reply to Paranaque
Kings’ letter saying period has lapsed.
July 6, 1989: counsel for defendant Santos informed the petitioners Paranaque
Kings that the new owner is RAYMUNDO.

Issue: Is such right of first refusal enforceable by an action for specific performance? YES
(WON the complaint filed by Paranaque Kings states a valid cause of action. YES)

Held:

Paranaque Kings was granted a first option or priority to purchase the subject
property (Based on the Par. 9 of the Lease Contract)
A careful examination of the complaint filed by Paranaque Kings reveals that it sufficiently
alleges an actionable contractual breach on the part of private respondents.

Under paragraph 9 of the contract of lease between respondent Santos and petitioner, the
latter was granted the "first option or priority" to purchase the leased properties in case
Santos decided to sell. If Santos never decided to sell at all, there can never be a breach,
much less an enforcement of such "right."

But on September 21, 1988, Santos sold said properties to Respondent Raymundo without
first offering these to petitioner. Santos indeed realized her error, since she repurchased the
properties after petitioner complained. Thereafter, she offered to sell the properties to
petitioner for P15 million, which petitioner, however, rejected because of the "ridiculous"
price. But Santos again appeared to have violated the same provision of the lease contract
when she finally resold the properties to respondent Raymundo for only P9 million without
first offering them to petitioner at such price. Whether there was actual breach which entitled
petitioner to damages and/or other just or equitable relief, is a question which can better be
resolved after trial on the merits where each party can present evidence to prove their
respective allegations and defenses.

The basis of the right of first refusal must be the current offer to sell of the seller or
offer to purchase of any prospective buyer.
Only after the optionee fails to exercise its right of first priority under the same terms and
within the period contemplated, could the owner validly offer to sell the property to a third
person, again, under the same terms as offered to the optionee.

The contention of Raymundo that he is not a privy to the contract is untenable


With respect to the contention of respondent Raymundo that he is not privy to the lease
contract, not being the lessor nor the lessee referred to therein, he could thus not have
violated its provisions, but he is nevertheless a proper party. Clearly, he stepped into the
shoes of the owner-lessor of the land as, by virtue of his purchase, he assumed all the
obligations of the lessor under the lease contract. Moreover, he received benefits in the form
of rental payments. Furthermore, the complaint, as well as the petition, prayed for the
annulment of the sale of the properties to him. Both pleadings also alleged collusion
between him and respondent Santos which defeated the exercise by petitioner of its right of
first refusal.

In order then to accord complete relief to petitioner, respondent Raymundo was a


necessary, if not indispensable, party to the case. A favorable judgment for the petitioner will
necessarily affect the rights of respondent Raymundo as the buyer of the property over
which petitioner would like to assert its right of first option to buy.

SALES CASE DIGEST COMPILATION 145


Deed of Assignment included the option to purchase
The provisions of the deeds of assignment with regard to matters assigned were very clear.
Under the first assignment between Frederick Chua as assignor and Lee Ching Bing as
assignee, it was expressly stated that:

. . . . the ASSIGNOR hereby CEDES, TRANSFERS and ASSIGNS to herein ASSIGNEE, all
his rights, interest and participation over said premises afore-described, . . . .

And under the subsequent assignment executed between Lee Ching Bing as assignor and
the petitioner, represented by its Vice President Vicenta Lo Chiong, as assignee, it was
likewise expressly stipulated that;

. . . . the ASSIGNOR hereby sells, transfers and assigns all his rights, interest and
participation over said leased premises, . . . .

One of such rights included in the contract of lease and, therefore, in the assignments of
rights was the lessee's right of first option or priority to buy the properties subject of the
lease, as provided in paragraph 9 of the assigned lease contract. The deed of assignment
need not be very specific as to which rights and obligations were passed on to the assignee.
It is understood in the general provision aforequoted that all specific rights and obligations
contained in the contract of lease are those referred to as being assigned. Needless to state,
respondent Santos gave her unqualified conformity to both assignments of rights.

WHEREFORE, the petition is GRANTED. The assailed decisions of the trial court and Court
of Appeals are hereby REVERSED and SET ASIDE. The case is REMANDED to the
Regional Trial Court of Makati for further proceedings.

3. ROSENCOR V. CA

March 08, 2001

FACTS:

Respondents are tenants of a two-storey residential apartment in Tomas Morato, QC. The
lease was not covered by any contract. Lessees were verbally given by the lessors the pre-
emptive right to purchase the property in case of sale.

The original lessors died and their heir also promised the lessees the same pre-emptive right
to purchase. The new lessors represented by Eufrocina de Leon demanded the lessees to
vacate the property because the building will allegedly be demolished but after the lessees
declined, she sent them a letter offering to sell the property for 2M. Lessees made a counter
offer of 1M but no reply was made by the lessors.

De Leon subsequently informed the lessees that the property was already sold to Rosencor.
Lessees claimed that they were deceived because the property was already sold to
Rosencor before it was offered to them. They offered to reimburse the payment to the
lessors but the offer was declined as hence, this petition.

ISSUE:
WON the lessors should recognize the pre-emptive right of the lessees even if it was only
given verbally.

SALES CASE DIGEST COMPILATION 146


HELD:
The right of first refusal is not covered by the Statute of Frauds. The application of such
statute presupposes the existence of a perfected contact which is no applicable in this case.
As such, a right of first refusal need not be written to be enforceable and can be proved by
oral evidence.

Lessees have proven that the lessors admit the right of first refusal given to them when the
property was offered to them by 2M.

The prevailing doctrine is that a contract of sale entered in violation of right of first refusal is
rescissible. However, this doctrine cannot be applied here because the vendees (Rosencor)
is in good faith. Under Art.1358, recission cannot take place when things which are the
object of sale is legally in possession of third person who did not act in bad faith.

Rosencor could not have acted in bad faith because they are not aware of the right of first
refusal given verbally. Respondents should instead file for damages.

4. DR. DANIEL VAZQUEZ and MA. LUIZA M. VAZQUEZ, petitioners vs. AYALA
CORPORATION, respondent.
G.R. No. 149734. November 19, 2004

Daniel Vasquez owns Conduit Development, Inc. In 1981, Vasquez enters into a
Memorandum of Agreement (MOA) with Ayala Corporation wherein Ayala bought Conduit
from Vasquez. Ayala committed to develop Conduit’s lands including 4 parcels of land
adjacent to Vasquez’ retained land. Be it noted that these parcels of land were in the
3rd phase of Ayala’s development plan. Paragraph 5.15 of the MOA provides:

“5.15 The Buyer (Ayala) agrees to give the sellers (Vasquez) a first option to purchase four
developed lots next to the “Retained Area” at the prevailing market price at the time of the
purchase.”

In 1990, Ayala was able to develop the said lots. (This was after some slump, and some
litigation between Conduit’s former contractor (GP construction) and GP’s subcontractor
(Lancer Builders).) Ayala then offered to sell the 4 parcels of land to Vasquez at P6.5k/sq.
m. which was the market price in 1990. Vasquez refused the offer. Vasquez contended that
the purchase price should be P460/sq. m. which was the market price in 1981 (time of
purchase). Ayala then lowered the purchase price to P5k/sq. m. but Vasquez refused again.
Instead he made a counter offer to buy the lots at P2k/sq. m. This time, Ayala refused.
ISSUE: Whether or not Paragraph 5.15 of the MOA is an option contract or right of first
refusal.
HELD: No. The said paragraph is a mere right of first refusal. Although the paragraph has a
definite object, i.e., the sale of the 4 lots, the period within which they will be offered for sale
to Vasquez and, necessarily, the price for which the subject lots will be sold are not
specified. The phrase “at the prevailing market price at the time of the purchase” connotes
that there is no definite period within which Ayala is bound to reserve the subject lots for
Vasquez to exercise his privilege to purchase. Neither is there a fixed or determinable price
at which the subject lots will be offered for sale. The price is considered certain if it may be

SALES CASE DIGEST COMPILATION 147


determined with reference to another thing certain or if the determination thereof is left to the
judgment of a specified person or persons.
Further, paragraph 5.15 was inserted into the MOA to give Vasquez the first crack to buy the
subject lots at the price which Ayala would be willing to accept when it offers the subject lots
for sale. It is not supported by an independent consideration.

ZOLA

5.TANAY RECREATION CENTER AND DEVELOPMENT CORP., petitioner, vs. CATALINA


MATIENZO FAUSTO+ and ANUNCIACION FAUSTO PACUNAYEN, respondents.
G.R. No. 140182. April 12, 2005

Facts:

Petitioner Tanay Recreation Center and Development Corp. (TRCDC) is the lessee of a
3,090-square meter property located in Sitio Gayas, Tanay, Rizal, owned by Catalina
Matienzo Fausto,[1] under a Contract of Lease executed on August 1, 1971. On this
property stands the Tanay Coliseum Cockpit operated by petitioner. The lease contract
provided for a 20-year term, subject to renewal within sixty days prior to its expiration. The
contract also provided that should Fausto decide to sell the property, petitioner shall have
the priority right to purchase the same.[2]

On June 17, 1991, petitioner wrote Fausto informing her of its intention to renew the
lease.[3] However, it was Faustos daughter, respondent Anunciacion F. Pacunayen, who
replied, asking that petitioner remove the improvements built thereon, as she is now the
absolute owner of the property.[4] It appears that Fausto had earlier sold the property to
Pacunayen on August 8, 1990, for the sum of P10,000.00 under a Kasulatan ng Bilihan
Patuluyan ng Lupa,[5] and title has already been transferred in her name under Transfer
Certificate of Title (TCT) No. M-35468.[6]

Despite efforts, the matter was not resolved. Hence, on September 4, 1991, petitioner
filed an Amended Complaint for Annulment of Deed of Sale, Specific Performance with
Damages, and Injunction, docketed as Civil Case No. 372-M.[7]

RTC- After trial on the merits, the Regional Trial Court of Morong, Rizal (Branch 78),
rendered judgment extending the period of the lease for another seven years from August 1,
1991 at a monthly rental of P10,000.00, and dismissed petitioners claim for damages.[9]

(It acknowledged the Petitioner’s Right of First Refusal.)

CA- Affirmed with modification- TRCDC ordered to vacate the premises on the ground
that ROFL only applies to sale of property to third persons and not between relatives.

ISSUE: WON TRCDC is entitled to the priority right of repurchase or otherwise known
as ROFL?

SALES CASE DIGEST COMPILATION 148


SC- WHEREFORE, the instant Petition for Review is PARTIALLY GRANTED. The
Court of Appeals Decision dated June 14, 1999 in CA-G.R. CV No. 43770 is MODIFIED:

Respondent ORDERED to recognize the obligation of Catalina Matienzo Fausto


under the Contract of Lease with respect to the priority right of petitioner Tanay Recreation
Center and Development Corp. to purchase the subject property under reasonable terms
and conditions.

RATIO:

• Petitioners right of first refusal in this case is expressly provided for in the notarized
Contract of Lease dated August 1, 1971, between Fausto and petitioner.

• When a lease contract contains a right of first refusal, the lessor is under a legal duty
to the lessee not to sell to anybody at any price until after he has made an offer to sell to
the latter at a certain price and the lessee has failed to accept it.

• Petitioners right of first refusal is an integral and indivisible part of the contract of lease
and is inseparable from the whole contract. The consideration for the lease includes the
consideration for the right of first refusal[19] and is built into the reciprocal obligations of
the parties.

• It was erroneous for the CA to rule that the right of first refusal does not apply when
the property is sold to Faustos relative.[20] When the terms of an agreement have been
reduced to writing, it is considered as containing all the terms agreed upon. As such, there
can be, between the parties and their successors in interest, no evidence of such terms
other than the contents of the written agreement, except when it fails to express the true
intent and agreement of the parties.[21] In this case, the wording of the stipulation giving
petitioner the right of first refusal is plain and unambiguous, and leaves no room for
interpretation. It simply means that should Fausto decide to sell the leased property during
the term of the lease, such sale should first be offered to petitioner. The stipulation does
not provide for the qualification that such right may be exercised only when the sale is
made to strangers or persons other than Faustos kin. Thus, under the terms of petitioners
right of first refusal, Fausto has the legal duty to petitioner not to sell the property to
anybody, even her relatives, at any price until after she has made an offer to sell to
petitioner at a certain price and said offer was rejected by petitioner. Pursuant to their
contract, it was essential that Fausto should have first offered the property to petitioner
before she sold it to respondent. It was only after petitioner failed to exercise its right of
first priority could Fausto then lawfully sell the property to respondent.

• A lease contract is not essentially personal in character.[26] Thus, the rights and
obligations therein are transmissible to the heirs. The general rule is that heirs are bound
by contracts entered into by their predecessors-in-interest except when the rights and
obligations arising therefrom are not transmissible by (1) their nature, (2) stipulation or (3)
provision of law.[27]

SALES CASE DIGEST COMPILATION 149


6) G.R. No. 183612POLYTECHNIC UNIVERSITY OF THE PHILIPPINES, Petitioner vs
GOLDEN HORIZON REALTY CORPORATION, Respondent.

G.R. No. 184260


NATIONAL DEVELOPMENT COMPANY, Petitioner, vs GOLDEN HORIZON REALTY
CORPORATION, Respondent.

FACTS:

• National Development Company (NDC)- is a government- owned and controlled


corporation, created under Commonwealth Act No. 182, as amended by Com. Act No.
311 and Presidential Decree (P.D.) No. 668

• Polytechnic University of the Philippines (PUP)-s a public, non-sectarian, non-profit


educational institution created in 1978 by virtue of P.D. No. 1341.

• In the early sixties, NDC had in its disposal a ten (10)-hectare property located
along Pureza St., Sta. Mesa, Manila.

• On September 7, 1977, NDC entered into a Contract of Lease (C-33-77) with Golden
Horizon Realty Corporation (GHRC) over a portion of the property, with an area of 2,407
square meters for a period of ten (10) years, renewable for another ten (10) years with
mutual consent of the parties.[3]

• Sometime after September 1988, GHRC discovered that NDC had decided to secretly
dispose the property to a third party. On October 21, 1988, GHRC filed in the RTC a
complaint for specific performance, damages with preliminary injunction and temporary
restraining order.[7]

• In the meantime, then President Corazon C. Aquino issued Memorandum Order No. 214
dated January 6, 1989, ordering the transfer of the whole NDC Compound to the National
Government, which in turn would convey the said property in favor of PUP at acquisition
cost. The memorandum order cited the serious need of PUP, considered the Poor Mans
University, to expand its campus, which adjoins the NDC Compound, to accommodate its
growing student population, and the willingness of PUP to buy and of NDC to sell its
property.

• On February 20, 1989, the RTC issued a writ of preliminary injunction enjoining NDC and
its attorneys, representatives, agents and any other persons assisting it from proceeding
with the sale and disposition of the leased premises

• On November 14, 2001, this Court rendered a decision in G.R. Nos. 143513 (Polytechnic
University of the Philippines v. Court of Appeals) and 143590 (National Development
Corporation v. Firestone Ceramics, Inc.),[15] which declared that the sale to PUP by NDC

SALES CASE DIGEST COMPILATION 150


of the portion leased by Firestone pursuant to Memorandum Order No. 214 violated the
right of first refusal granted to Firestone under its third lease contract with NDC.

• The RTC resumed the proceedings and when mediation and pre-trial failed to settle the
case amicably

RTC- On November 25, 2004, the RTC rendered its decision upholding the
right of first refusal granted to GHRC under its lease contract with NDC and
ordering PUP to reconvey the said portion of the property in favor of GHRC.

CA- affirmed in toto the decision of the RTC.

ISSUE: Whether or not our ruling in Polytechnic University of the Philippines v. Court of
Appeals applies in this case involving another lessee of NDC who claimed that the option to
purchase the portion leased to it was similarly violated by the sale of the NDC Compound in
favor of PUP pursuant to Memorandum Order No. 214.

SC- We rule in the affirmative.

RATIO:

• The second lease contract contained the following provision:Lessee shall also have the
option to purchase the area leased, the price to be negotiated and determined at the time
the option to purchase is exercised. [EMPHASIS SUPPLIED]

• When a lease contract contains a right of first refusal, the lessor has the legal duty to the
lessee not to sell the leased property to anyone at any price until after the lessor has
made an offer to sell the property to the lessee and the lessee has failed to accept it. Only
after the lessee has failed to exercise his right of first priority could the lessor sell the
property to other buyers under the same terms and conditions offered to the lessee, or
under terms and conditions more favorable to the lessor.[30]

• An option is a contract by which the owner of the property agrees with another person that
the latter shall have the right to buy the formers property at a fixed price within a certain
time. Upon the other hand, a right of first refusal is a contractual grant, not of the sale of
a property, but of the first priority to buy the property in the event the owner sells the
same.[28] As distinguished from an option contract, in a right of first refusal, while the
object might be made determinate, the exercise of the right of first refusal would be
dependent not only on the owners eventual intention to enter into a binding juridical
relation with another but also on terms, including the price, that are yet to be firmed up.[29]

• As the option to purchase clause in the second lease contract has no definite period
within which the leased premises will be offered for sale to respondent lessee and the
price is made subject to negotiation and determined only at the time the option to buy is
exercised, it is obviously a mere right of refusal, usually inserted in lease contracts to give
the lessee the first crack to buy the property in case the lessor decides to sell the

SALES CASE DIGEST COMPILATION 151


same. That respondent was granted a right of first refusal under the second lease contract
appears not to have been disputed by petitioners. What petitioners assail is the CAs
erroneous conclusion that such right of refusal subsisted even after the expiration of the
original lease period, when respondent was allowed to continue staying in the leased
premises under an implied renewal of the lease and without the right of refusal carried
over to such month-to-month lease. Petitioners thus maintain that no right of refusal was
violated by the sale of the property in favor of PUP pursuant to Memorandum Order No.
214. Petitioners position is untenable.

7) G.R. No. 106837 August 4, 1993


HENRY MACION and ANGELES MACION, petitioners,
vs.
HON. JAPAL M. GUIANI, in his capacity as Presiding Judge of the Regional Trial Court
Branch 14, Cotabato City and DELA VIDA INSTITUTE represented by MS. JOSEPHINE
LANZADERAS, respondents.

FACTS:
On April 26, 1991, the petitioners (MACION) and private respondent (DELA VIDA
INSTITUTE) entered into a contract to sell under which terms, private respondent, as
president of De la Vida Institute, assured petitioners that they would buy the said properties
on or before July 31, 1991 in the amount of P1,750,000.00. In the meantime, petitioners
surrendered the physical possession of the two lots to private respondent who promptly built
an edifice worth P800,000.00.1
But on July 31, 1991, the sale did not materialize. Consequently, petitioners filed a complaint
for unlawful detainer against private respondent (MTCC Civil Case No. 2739). In retaliation,
private respondent filed a complaint for reformation of the contract to sell executed on April
26, 1991 (Civil Case 592).2
On February 6, 1992, the parties entered into a compromise agreement which stipulated
among others that petitioners would give private respondent five (5) months to raise the
amount of P2,060,000.00;3 that in the event of failure to raise the said amount within the
designated period, private respondent would vacate the premises immediately.
On March 25, 1992, the trial court approved the compromise agreement dated February 6,
1992.
Two (2) months after,DELA VIDA INSTITUTE, alleging that they had negotiated a loan from
the Bank of the Philippine Islands, wrote letters dated May 19, 20 and 26 requesting
petitioners to execute with them a contract to sell in their favor. On May 28, 1992, private
respondent filed with the trial court an urgent motion for an order directing petitioners to
execute a contract to sell in private respondent's favor in accordance with paragraph 7 of the
compromise agreement.7
On July 8, 1992, MACIONS filed a motion for execution of judgement alleging that after a
lapse of five (5) months from February 6, 1992, private respondent have failed to settle their
obligations with petitioners.8

RTC-denied the motion for execution and directed MACIONs to execute the required
contract to sell in favor of private respondent. Respondent judge opined that the proximate
cause of private respondent's failure to comply with the compromise agreement was the
refusal of petitioners to execute a contract to sell as required under the agreement.
Respondent judge added that petitioners should have executed the contract to sell because
anyway they would not be prejudiced since there was no transfer of ownership involved in a
contract to sell.9

SALES CASE DIGEST COMPILATION 152


Hence this instant petition for certiorari, with prayer for a temporary restraining order
enjoining respondent judge from enforcing its August 6, 1992 order.

ISSUE: whether or not respondent judge committed grave abuse of discretion in ordering
petitioner to execute a contract to sell in favor of private respondent.

HELD:
in the interpretation of the compromise agreement, we must delve in the contemporaneous
and subsequent acts of the parties to fathom the real intention of the parties. 11 A review of
the facts reveal that even prior to the signing of the compromise agreement and the filing of
Civil Case No. 592 before the trial court, the parties had already entered into a contract to
sell. Thereafter, when the transaction failed to materialize, the parties filed suits against each
other; petitioners, their unlawful detainer case, and private respondent a complaint for
reformation of contract, alleging that petitioners in fact had caused the preparation of the
contract to sell dated April 26, 1991 with the understanding that the land would be used as a
collateral in obtaining a loan with DBP

Under these factual circumstances, we opine that the compromise agreement must be
interpreted as bestowing upon private respondent-buyer the power to demand a contract to
sell from petitioner-sellers. Where the seller promised to execute a deed of absolute sale
upon completing payment of the price, it is a contract to sell. 12 In the case at bar, the sale is
still in the executory stage since the passing of title is subject to a suspensive condition,
namely, that if private respondent is able to secure the needed funds to be used in the
purchased of the two (2) lots owned by petitioners. A mere executory sale, one where the
sellers merely promise to transfer the property at some future date, or where some
conditions have to be fulfilled before the contract is converted from an executory to an
executed one, does not pass ownership over the real estate being sold. 13
In our jurisdiction, it has been that an accepted bilateral promise to buy and sell is in a sense
similar to, but not exactly the same, as a perfected contract of sale because there is already
a meeting of minds upon the thing which is the object of the contract and upon the
price. 14 But a contract of sale is consummated only upon the delivery and payment. It
cannot be denied that the compromise agreement, having been signed by both parties, is
tantamount to a bilateral promise to buy and sell a certain thing for a price certain. Hence,
this gives the contracting parties rights in personam, such that each has the right to demand
from the other the fulfillment of their respective undertakings. 15 Demandability may be
exercised at any time after the execution of the Deed. 16
The order of respondent judge directing petitioners to issue a contract to sell does not place
petitioners in any danger of losing their property without consideration, for, to repeat, in a
contract to sell there is no immediate transfer of ownership. In contracts to sell, payment is a
positive suspensive condition, failure of which does not constitute a breach but an event that
prevents the obligation of the vendor to convey title from materializing, in accordance with
Article 1184 of the Civil Code. 17 Petitioners as promisors were never obliged to convey title
before the happening of the suspensive condition. In fact, nothing stood in the way of their
selling the property to another after a unsuccessful demand for said price upon the
expiration of the time agreed upon.
Since the period given by the petitioners under the compromise agreement has already
lapsed, we order the trial court to fix anew a period within which private respondents could
secure the needed funds for the purchase of the
land.

SALES CASE DIGEST COMPILATION 153


SALES CASE DIGEST COMPILATION 154
XVI. PERFECTION STAGE

1) G.R. No. L-26872 July 25, 1975


VILL2) NCO REALTY COMPANY, plaintiff-appellee and EDITH PEREZ DE
TAGLE, intervenor-appellee,
vs.
BORMAHECO, INC., FRANCISCO N. CERVANTES and ROSARIO N.
CERVANTES, defendants-appellants. Meer, Meer & Meer for plaintiff-appellee.

FACTS:
Francisco N. Cervantes and his wife, Rosario P. Navarra-Cervantes, are the owners
of lots 3, 15 and 16 located at 245 Buendia Avenue, Makati, Rizal with a total area of
three thousand five hundred square meters (TCT Nos. 43530, 43531 and 43532,
Exh. A, A-1 and A-2).
Cervantes is the president of Bormaheco, Inc., a dealer and importer of industrial and
agricultural machinery. The entire lots are occupied by the building, machinery and
equipment of Bormaheco, Inc. and are adjacent to the property of Villonco Realty
Company situated at 219 Buendia Avenue.
In the early part of February, 1964 there were negotiations for the sale of the said
lots and the improvements thereon between Romeo Villonco of Villonco Realty
Company "and Bormaheco, Inc., represented by its president, Francisco N.
Cervantes, through the intervention of Edith Perez de Tagle, a real estate broker.
Bormaheco, Inc. and Villonco Realty Company continued their negotiations for the
sale of the Buendia Avenue property. Cervantes and Teofilo Villonco had a final
conference on February 27, 1964. As a result of that conference Villonco Realty
Company, through Teofilo Villonco, in its letter of March 4, 1964 made a revised
counter- offer (Romeo Villonco's first counter-offer was dated February 24, 1964,
Exh. C) for the purchase of the property.
The check for P100,000 (Exh. E) mentioned in the foregoing letter-contract was
delivered by Edith Perez de Tagle to Bormaheco, Inc. on March 4, 1964 and was
received by Cervantes. In the voucher-receipt evidencing the delivery the broker
indicated in her handwriting that the earnest money was "subject to the terms and
conditions embodied in Bormaheco's letter"
Then, unexpectedly, in a letter dated March 30, 1964, or twenty-six days after the
signing of the contract of sale, Exhibit D, Cervantes returned the earnest money, with
interest amounting to P694.24 (at ten percent per annum). Cervantes cited as an
excuse the circumstance that "despite the lapse of 45 days from February 12, 1964
there is no certainty yet" for the acquisition of the Punta property
On that same date, April 7, 1964 Villonco Realty Company filed the complaint (dated
April 6) for specific performance against Bormaheco,
After trial, the lower court rendered a decision ordering the Cervantes spouses to
execute in favor of Bormaheco, Inc. a deed of conveyance for the three lots in
question and directing Bormaheco, Inc. (a) to convey the same lots to Villonco Realty
Company
Bormaheco, Inc. and the Cervantes spouses appealed.

Issue:

SALES CASE DIGEST COMPILATION 155


WON that no contract of sale was perfected because Cervantes made a supposedly
qualified acceptance of the revised offer contained in Exhibit D, which acceptance amounted
to a counter-offer, and because the condition that Bormaheco, inc. would acquire the Punta
land within the forty-five-day period was not fulfilled

HELD: Petition devoid of merit.

"By the contract of sale one of the contracting parties obligates himself to transfer the
ownership of and to deliver a determining thing, and the other to pay therefor a price certain
in money or its equivalent. A contract of sale may be absolute or conditional" (Art. 1458, Civil
Code).

"The contract of sale is perfected at the moment there is a meeting of minds upon the thing
which is the object of the contract and upon the price. From that moment, the parties may
reciprocally demand performance, subject to the provisions of the law governing the form of
contracts" (Art. 1475, Ibid.).

"Contracts are perfected by mere consent, and from that moment the parties are bound not
only to the fulfillment of what has been expressly stipulated but also to all the consequences
which, according to their nature, may be in keeping with good faith, usage and law" (Art.
1315, Civil Code).

"Consent is manifested by the meeting of the offer and the acceptance upon the thing and
the cause which are to constitute the contract. The offer must be certain and the acceptance
absolute. A qualified acceptance constitutes a counter-offer" (Art. 1319, Civil Code). "An
acceptance may be express or implied" (Art. 1320, Civil Code).

Bormaheco's acceptance of Villonco Realty Company's offer to purchase the


Buendia Avenue property, as shown in Teofilo Villonco's letter dated March 4, 1964
(Exh. D), indubitably proves that there was a meeting of minds upon the subject
matter and consideration of the sale.

Not only that Bormaheco's acceptance of the part payment of one hundred ,thousand
pesos shows that the sale was conditionally consummated or partly executed subject to the
purchase by Bormaheco, Inc. of the Punta property. The nonconsummation of that purchase
would be a negative resolutory condition

The controlling fact is that there was agreement between the parties on the subject matter,
the price and the mode of payment and that part of the price was paid. "Whenever earnest
money is given in a contract of sale, it shall be considered as part of the price and as proof
of the perfection of the contract" (Art. 1482, Civil Code).

"It is true that an acceptance may contain a request for certain changes in the terms of
the offer and yet be a binding acceptance. 'So long as it is clear that the meaning of the
acceptance is positively and unequivocally to accept the offer, whether such request is
granted or not, a contract is formed.' " (Stuart vs. Franklin Life Ins. Co., 165 Fed. 2nd 965,
citing Sec. 79, Williston on Contracts

SALES CASE DIGEST COMPILATION 156


2) RIZALINO, substituted by his heirs, JOSEFINA, ROLANDO and FERNANDO,
ERNESTO, LEONORA, BIBIANO, JR., LIBRADO and ENRIQUETA, all surnamed
OESMER,Petitioners,- versus -PARAISO DEVELOPMENT
CORPORATION,Respondent.

FACTS:
Petitioners Rizalino, Ernesto, Leonora, Bibiano, Jr., Librado, and Enriqueta, all surnamed
Oesmer, together with Adolfo Oesmer (Adolfo) and Jesus Oesmer (Jesus), are brothers and
sisters, and the co-owners of undivided shares of two parcels of agricultural and tenanted
land situated in Barangay Ulong Tubig, Carmona, Cavite, identified as Lot 720 with an area
of 40,507 square meters (sq. m.) and Lot 834 containing an area of 14,769 sq. m., or a total
land area of 55,276 sq. m. Both lots are unregistered and originally owned by their parents,
Bibiano Oesmer and Encarnacion Durumpili, who declared the lots for taxation purposes
under Tax Declaration No. 3438[3] (cancelled by I.D. No. 6064-A) for Lot 720 and Tax
Declaration No. 3437[4] (cancelled by I.D. No. 5629) for Lot 834. When the spouses Oesmer
died, petitioners, together with Adolfo and Jesus, acquired the lots as heirs of the former by
right of succession.

Respondent Paraiso Development Corporation is known to be engaged in the real estate


business.

Sometime in March 1989, Rogelio Paular, a resident and former Municipal Secretary of
Carmona, Cavite, brought along petitioner Ernesto to meet with a certain Sotero Lee,
President of respondent Paraiso Development Corporation, at Otani Hotel in Manila. The
said meeting was for the purpose of brokering the sale of petitioners properties to
respondent corporation.

Pursuant to the said meeting, a Contract to Sell[5] was drafted by the Executive Assistant of
Sotero Lee, Inocencia Almo. On 1 April 1989, petitioners Ernesto and Enriqueta signed the
aforesaid Contract to Sell. A check in the amount of P100,000.00, payable to Ernesto, was
given as option money. Sometime thereafter, Rizalino, Leonora, Bibiano, Jr.,
and Librado also signed the said Contract to Sell. However, two of the brothers, Adolfo and
Jesus, did not sign the document.

On 5 April 1989, a duplicate copy of the instrument was returned to respondent


corporation. On 21 April 1989, respondent brought the same to a notary public for
notarization.

In a letter[6] dated 1 November 1989, addressed to respondent corporation, petitioners


informed the former of their intention to rescind the Contract to Sell and to return the amount
of P100,000.00 given by respondent as option money.
Respondent did not respond to the aforesaid letter. On 30 May 1991, herein petitioners,
together with Adolfo and Jesus, filed a Complaint[7] for Declaration of Nullity or for Annulment
of Option Agreement or Contract to Sell with Damages before the Regional Trial Court
(RTC) of Bacoor, Cavite

RTC- After trial on the merits, the lower court rendered a Decision[9] dated 27 March 1996 in
favor Paraiso Development Corporation. It declared that The assailed Contract to Sell is
valid and binding only to the undivided proportionate share of the signatory of this document
and recipient of the check. co-owner Ernesto Durumpili Oesmer. The latter is hereby ordered
to execute the Contract of Absolute Sale concerning his 1/8 share over the subject two
parcels of land in favor of herein [respondent] corporation.

CA- modified. Declared that the Contract to Sell is valid and binding with respect to the
undivided proportionate shares of the six signatories of the said document, herein

SALES CASE DIGEST COMPILATION 157


petitioners, namely: Ernesto, Enriqueta, Librado, Rizalino, Bibiano, Jr., and Leonora (all
surnamed Oesmer.

Issues:
1) WON Contract to Sell (Exhibit D) is not binding upon petitioner Ernesto
Oesmers co-owners (herein petitioners Enriqueta, Librado, Rizalino, Bibiano,
Jr., and Leonora
2) Won the P100,000 given by respondent was option money or earnest money.

HELD:
1) CONTRACT OF SALE IS BINDING TO PEITIONERS,
Article 1874 of the Civil Code, which expressly provides that:

Art. 1874. When a sale of a piece of land or any interest therein is through an
agent, the authority of the latter shall be in writing; otherwise, the sale shall be
void.

The law itself explicitly requires a written authority before an agent can sell an
immovable. The conferment of such an authority should be in writing, in as clear and precise
terms as possible. It is worth noting that petitioners signatures are found in the Contract to
Sell. The Contract is absolutely silent on the establishment of any principal-agent
relationship between the five petitioners and their brother and co-petitioner Ernesto as to the
sale of the subject parcels of land. Thus, the Contract to Sell, although signed on the margin
by the five petitioners, is not sufficient to confer authority on petitioner Ernesto to act as their
agent in selling their shares in the properties in question.

However, despite petitioner Ernestos lack of written authority from the five petitioners to sell
their shares in the subject parcels of land, the supposed Contract to Sell remains valid and
binding upon the latter.

As can be clearly gleaned from the contract itself, it is not only petitioner Ernesto who signed
the said Contract to Sell; the other five petitioners also personally affixed their signatures
thereon. Therefore, a written authority is no longer necessary in order to sell their shares in
the subject parcels of land because, by affixing their signatures on the Contract to Sell, they
were not selling their shares through an agent but, rather, they were selling the same directly
and in their own right.

It is well-settled that contracts are perfected by mere consent, upon the acceptance by the
offeree of the offer made by the offeror. From that moment, the parties are bound not only to
the fulfillment of what has been expressly stipulated but also to all the consequences which,
according to their nature, may be in keeping with good faith, usage and law. To produce a
contract, the acceptance must not qualify the terms of the offer. However, the acceptance
may be express or implied. For a contract to arise, the acceptance must be made known to
the offeror. Accordingly, the acceptance can be withdrawn or revoked before it is made
known to the offeror.[13]

In the case at bar, the Contract to Sell was perfected when the petitioners consented to the
sale to the respondent of their shares in the subject parcels of land by affixing their
signatures on the said contract. Such signatures show their acceptance of what has been
stipulated in the Contract to Sell and such acceptance was made known to respondent
corporation when the duplicate copy of the Contract to Sell was returned to the latter bearing
petitioners signatures.

SALES CASE DIGEST COMPILATION 158


2) THE MONEY GIVEN BY RESPONDENT IS EARNEST MONEY.
In the instant case, the consideration of P100,000.00 paid by respondent to petitioners was
referred to as option money. However, a careful examination of the words used in the
contract indicates that the money is not option money but earnest money.Earnest money
and option money are not the same but distinguished thus: (a) earnest money is part of the
purchase price, while option money is the money given as a distinct consideration for an
option contract; (b) earnest money is given only where there is already a sale, while option
money applies to a sale not yet perfected; and, (c) when earnest money is given, the buyer
is bound to pay the balance, while when the would-be buyer gives option money, he is not
required to buy, but may even forfeit it depending on the terms of the option.[20]

The sum of P100,000.00 was part of the purchase price. Although the same was
denominated as option money, it is actually in the nature of earnest money or down
payment when considered with the other terms of the contract. Doubtless, the
agreement is not a mere unilateral promise to sell, but, indeed, it is a Contract to Sell
as both the trial court and the appellate court declared in their Decisions

3) SERAFIN, RAUL, NENITA, NAZARETO, NEOLANDA, all surnamed NARANJA,


AMELIA NARANJA-RUBINOS, NILDA NARANJA-LIMANA, and NAIDA
NARANJA-GICANO,- versus -COURT OF APPEALS, LUCILIA P. BELARDO,
represented by her Attorney-in-Fact, REBECCA CORDERO, and THE LOCAL
REGISTER OF DEEDS, BACOLOD CITY,Respondents.

FACTS:
Roque Naranja was the registered owner of a parcel of land, denominated as Lot No. 4 in
Consolidation-Subdivision Plan (LRC) Pcs-886, Bacolod Cadastre, with an area of 136
square meters and covered by Transfer Certificate of Title (TCT) No. T-18764. Roque was
also a co-owner of an adjacent lot, Lot No. 2, of the same subdivision plan, which he co-
owned with his brothers, Gabino and Placido Naranja. When Placido died, his one-third
share was inherited by his children, Nenita, Nazareto, Nilda, Naida and Neolanda, all
surnamed Naranja, herein petitioners.

In 1976, Roque, who was single and had no children, lived with his half sister, Lucilia P.
Belardo (Belardo), in Pontevedra, Negros Occidental. Roque had no other source of income
except for the P200.00 monthly rental of his two properties. To show his gratitude to
Belardo, Roque sold Lot No. 4 and his one-third share in Lot No. 2 to Belardo on August 21,
1981, through a Deed of Sale of Real Property which was duly notarized by Atty. Eugenio
Sanicas.

On December 2, 1983, Roque died of influenza. Unknown to Belardo, petitioners, the


children of Placido and Gabino Naranja, executed an Extrajudicial Settlement Among
Heirs[12] on October 11, 1985, adjudicating among themselves Lot No. 4.

In 1987, Belardo decided to register the Deed of Sale dated August 21, 1981. With no title in
hand, she was compelled to file a petition with the RTC to direct the Register of Deeds to
annotate the deed of sale even without a copy of the TCTs.

On December 11, 1989, Atty. Sanicas prepared a certificate of authorization, giving


Belardos daughter, Jennelyn P. Vargas, the authority to collect the payments from Esso
Standard Eastern, Inc. But it appeared from the companys Advice of Fixed Payment that
payment of the lease rental had already been transferred from Belardo to Amelia Naranja-
Rubinos because of the Extrajudicial Settlement Among Heirs.

SALES CASE DIGEST COMPILATION 159


On June 23, 1992, Belardo,[15] through her daughter and attorney-in-fact, Rebecca Cordero,
instituted a suit for reconveyance with damages

Subsequently, petitioners also filed a case against respondent for annulment of sale
and quieting of title with damages, praying, among others, that judgment be rendered
nullifying the Deed of Sale

RTC- THERE WAS NO VALID CONTRACT OF SALE (DEED OF SALE


DEFECTIVE)

On March 5, 1997, the RTC rendered a Decision in the consolidated cases in favor of
petitioners. The trial court noted that the Deed of Sale was defective in form since it did not
contain a technical description of the subject properties but merely indicated that they were
Lot No. 4, covered by TCT No. T-18764 consisting of 136 square meters, and one-third
portion of Lot No. 2 covered by TCT No. T-18762

CA- held that the unregisterability of a deed of sale will not undermine its validity and
efficacy in transferring ownership of the properties to private respondent.

ISSUE:

Whether or not the alleged deed of sale [of real properties] is valid considering that the
consent of the late roque naranja had been vitiated; x x x there [is] no conclusive showing
that there was consideration and there [are] serious irregularities in the notarization of the
said documents

HELD:

The Court does not agree with petitioners contention that a deed of sale must contain
a technical description of the subject property in order to be valid.

To be valid, a contract of sale need not contain a technical description of the subject
property. Contracts of sale of real property have no prescribed form for their validity; they
follow the general rule on contracts that they may be entered into in whatever form, provided
all the essential requisites for their validity are present.[22] The requisites of a valid contract
of sale under Article 1458 of the Civil Code are: (1) consent or meeting of the minds;
(2) determinate subject matter; and (3) price certain in money or its equivalent.

The failure of the parties to specify with absolute clarity the object of a contract by
including its technical description is of no moment. What is important is that there is, in fact,
an object that is determinate or at least determinable, as subject of the contract of sale. The
form of a deed of sale provided in Section 127 of Act No. 496 is only a suggested form. It is
not a mandatory form that must be strictly followed by the parties to a contract.

On the alleged nullity of the deed of sale, we hold that petitioners failed to submit
sufficient proof to show that Roque executed the deed of sale under the undue influence of
Belardo or that the deed of sale was simulated or without consideration.

A notarized document carries the evidentiary weight conferred upon it with respect to
its due execution, and documents acknowledged before a notary public have in their favor
the presumption of regularity. It must be sustained in full force and effect so long as he who
impugns it does not present strong, complete, and conclusive proof of its falsity or nullity on
account of some flaws or defects provided by law.[23]

SALES CASE DIGEST COMPILATION 160


4. Dalion vs Sabesaje

FACTS:
On May 28, 1973, Sabesaje sued to recover ownership of a parcel of land, based on
a private document of absolute sale, dated July 1, 1965 (Exhibit "A"), allegedly
executed by Dalion, who, however denied the fact of sale, contending that the
document sued upon is fictitious, his signature thereon, a forgery; and that subject
land is conjugal property, which he and his wife acquired in 1960 from Saturnina
Sabesaje as evidenced by the "Escritura de Venta Absoluta" (Exhibit "B"). The
spouses denied claims of Sabesaje that after executing a deed of sale over the
parcel of land, they had pleaded with Sabesaje, their relative, to be allowed to
administer the land because Dalion did not have any means of livelihood. They
admitted, however, administering since 1958, five (5) parcels of land in Sogod,
Southern Leyte, which belonged to Leonardo Sabesaje, grandfather of Sabesaje,
who died in 1956. They never received their agreed 10% and 15% commission on
the sales of copra and abaca, respectively. Sabesaje's suit, they countered, was
intended merely to harass, preempt and forestall Dalion's threat to sue for these
unpaid commissions.

RTC- SALE IS VALID: Ordering the defendants to deliver to the plaintiff (SABESAJE) the
parcel of land subject of this case,

CA- AFFIRMED RTC DECISION


Hence this recourse.
ISSUE:
a) WON f the contract of sale of a parcel of land is valid; (YES)
b) WON it a public document is necessary for transfer of ownership (NO)

HELD:

A) The appellate court upheld the validity of the sale on the basis of Secs. 21 and 23 of
Rule 132 of the Revised Rules of Court.
"SEC. 21. Private writing, its execution and authenticity, how proved. Before any private
writing may be received in evidence, its due execution and authenticity must be proved
either:
(a) By anyone who saw the writing executed;
(b) By evidence of the genuineness of the handwriting of the maker; or
(c) By a subscribing witness
Here, people who witnessed the execution of subject deed positively testified on the
authenticity thereof. Hence, it is valid.
Assuming authenticity of his signature and the genuineness of the document, Dalion
nonetheless still impugns the validity of the sale on the ground that the same is
embodied in a private document, and did not thus convey title or right to the lot in
question since "acts and contracts which have for their object the creation,

SALES CASE DIGEST COMPILATION 161


transmission, modification or extinction of real rights over immovable property must
appear in a public instrument" (Art. 1358, par 1, NCC).
This argument is misplaced. The provision of Art. 1358 on the necessity of a public
document is only for convenience, not for validity or enforceability. It is not a requirement for
the validity of a contract of sale of a parcel of land that this be embodied in a public
instrument.
A contract of sale is a consensual contract, which means that the sale is perfected by mere
consent. No particular form is required for its validity. Upon perfection of the contract, the
parties may reciprocally demand performance (Art. 1475, NCC), i.e., the vendee may
compel transfer of ownership of the object of the sale, and the vendor may require the
vendee to pay the thing sold (Art. 1458, NCC).

B) A sale of a real property may be in a private instrument, but that contract is valid
and binding between the parties upon its perfection. And a party may compel the
other party to execute a public instrument embodying their contract affecting real
rights once the contract appearing in a private instrument has been perfected
(See Art. 1357)

5) Limketai Sons Milling vs CA

FACTS:
Phil.Remnants Co. constituted BPI to manage, administer and sell its real property located
in Pasig, Metro Manila.
BPI gave authority to real estate broker Pedro Revilla Jr. to sell the lot for P1000 per square
meter.
Revilla contacted Alfonso Lim of petitioner company who agreed to buy the land and
thereafter was allowed to view the land.
Lim and Alfonso LImketkai went to BPI to confirm the sale and both finally agreed that the
land would be sold for P1000 per square meter. Notwithstanding the agreement, Alfonso
asked BPI if it was possible to pay in terms provided that in case the term is disapproved,
the price shall be paid in cash.
Two or three days later, petitioner learned that its offer to pay on terms had been frozen.
Alfonso Lim went to BPI on July 18, 1988 and tendered the full payment of P33,056,000.00
to Albano. The payment was refused because Albano stated that the authority to sell that
particular piece of property in Pasig had been withdrawn from his unit
An action for specific performance with damages was thereupon filed on August 25, 1988 by
petitioner against BPI. In the course of the trial, BPI informed the trial court that it had sold
the property under litigation to NBS

ISSUE:
WON there was a perfected contract of sale between Limketkai Co. and BPI.

HELD:
There was already a perfected contract of sale because both parties already agreed to the
sale of P1000/sq.m. Even if Lim tried to negotiate for a payment in terms, it is clear that if it
be disapproved, the payment will be made in cash.
The perfection of the contract took place when Aromin and Albano, acting for BPI, agreed to
sell and Alfonso Lim with Albino Limketkai, acting for petitioner Limketkai, agreed to buy the
disputed lot at P1,000.00 per square meter. Aside from this there was the earlier agreement
between petitioner and the authorized broker. There was a concurrence of offer and
acceptance, on the object, and on the cause thereof.

SALES CASE DIGEST COMPILATION 162


The phases that a contract goes through may be summarized as follows:
a. preparation, conception or generation, which is the period of negotiation and bargaining,
ending at the moment of agreement of the parties;
b. perfection or birth of the contract, which is the moment when the parties come to agree on
the terms of the contract; and
c. consummation or death, which is the fulfillment or performance of the terms agreed upon
in the contract

6) Heirs of Biano vs CA

Facts:

On October 23, 1953, the late Ernesto Biona, married to plaintiff-appellee Soledad Biona,
was awarded Homestead Patent No. V-840 over the property subject of this suit, a parcel of
agricultural land denominated as lot 177 of PLS-285-D, located in Bo. 3, Banga, Cotabato,
containing an area of ten (10) hectares, forty-three (43) acres and sixty-eight (68) centares,
Original Certificate of Title No. (V-2323) P-3831 was issued in his name by the Register of
Deeds of Cotabato (Exh. C). On June 3, 1954, Ernesto and Soledad Biona obtained a loan
from the then Rehabilitation Finance Corporation (now the Development Bank of the
Philippines) and put up as collateral the subject property (Exh. 4). On June 12, 1956,
Ernesto Biona died (Exh. B) leaving as his heirs herein plaintiffs-appellees, namely, his wife,
Soledad Estrobillo Vda. De Biona, and five daughters, Editha B. Blancaflor, Marianita B. de
Jesus, Vilma B. Blancaflor, Elsie B. Ramos and Perlita B. Carmen.

On March 1, 1960, plaintiff-appellee Soledad Biona obtained a loan from defendant-


appellant in the amount of P1,000 and as security therefore, the subject property was
mortgaged.

The two-year period elapsed but Soledad Biona was not able to pay her
indebtedness. Defendant-appellant continued occupying and cultivating the subject property
without protest from plaintiffs-appellees

On June 19, 1985, plaintiffs-appellees, filed a complaint for recovery of ownership,


possession, accounting and damages, with a prayer for a writ of preliminary mandatory
injunction and/ or restraining order against defendant-appellant alleging, among others, that
the latter had unlawfully been depriving them of the use, possession and enjoyment of the
subject property

On September 19, 1986, defendant-appellant filed his answer with counterclaim traversing
the material allegations in the complaint and alleging, by way of affirmative and special
defenses, that: on September 11, 1961, Soledad Biona, after obtaining the loan of
P1,000.00 from defendant-appellant, approached and begged the latter to buy the whole of
Lot No. 177 since it was then at the brink of foreclosure by the Development Bank of the
Philippines and she had no money to redeem the same nor the resources to support herself
and her five small children; that defendant-appellant agreed to buy the property for the
amount of P4,300.00, which consideration was to include the redemption price to be paid to
the Development Bank of the Philippines; that the purchase price paid by defendant far
exceeded the then current market value of the property and defendant had to sell his own
eight-hectare parcel of land in Surallah to help Soledad Biona; that to evidence the
transaction, a deed of sale was handwritten by Soledad Biona and signed by her and the
defendant; that at the time of the sale, half of the portion of the property was already

SALES CASE DIGEST COMPILATION 163


submerged in water and from the years 1969 to 1984, two and one-half hectares thereof
were eroded by the Allah River; that by virtue of his continuous and peaceful occupation of
the property from the time of its sale and for more than twenty- five years thereafter,
defendant possesses a better right thereto subject only to the rights of the tenants whom he
had allowed to cultivate the land under the Land Reform Program of the government;

RTC rendered a decision ordering the defendant to vacate possession of the lot in question
to the extent of six-tenths (6/10) of the total area thereof and to deliver the same to the
plaintiff Soledad Estrobillo Biona upon the latter's payment of the sum of P1,000.00 TO THE
FORMER IN REDEMPTION OF ITS MORTGAGE CONSTITUTED UNDER exh. "1" of
defendant

Dissatisfied, herein private respondent appealed to the Court of Appeals which reversed
the trial court's ruling. The dispositive portion reads as follows:

WHEREFORE, premises considered, the judgment appealed from is set aside and a new
one entered dismissing the complaint, and the plaintiffs-appellees are ordered to execute a
registrable deed of conveyance of the subject property in favor of the defendant-appellant
within ten (10) days from the finality of this decision. With costs against plaintiffs-appellees.[3]

Issue: whether or not the deed of sale is valid and if it effectively conveyed to the private
respondents the subject property

Held: Yes.

The contract of sale between the contracting parties was consummated by the delivery
of the subject land to private respondent who since then had occupied and cultivated the
same continuously and peacefully until the institution of this suit."[5]
We agree with the private respondent that all the requisites for a valid contract of sale
are present in the instant case. For a valuable consideration of P4,500.00, Soledad Biona
agreed to sell and actually conveyed the subject property to private respondent. The fact
that the deed of sale was not notarized does not render the agreement null and void and
without any effect. The provision of Article 1358 of the Civil Code[9] on the necessity of a
public document is only for convenience, and not for validity or enforceability.[10] The
observance of which is only necessary to insure its efficacy, so that after the existence of
said contract had been admitted, the party bound may be compelled to execute the proper
document.[11] Undeniably, a contract has been entered into by Soledad Biona and the private
respondent. Regardless of its form, it was valid, binding and enforceable between the
parties. We quote with favor the respondent court's ratiocination on the matter:

7) SECUYA v. VDA. DE SELMA

FACTS:
Before the grant of her application for private sale of Lot 5679, a friar land, the
beautiful Maxima Caballero executed a document entitled "Agreement of Partition," wherein
she stipulated to transfer one-third (1/3) of the lot to and accepted by Paciencia Sabellano,
her aunt. When the application was approved, Maxima failed to transfer the agreed portion
to Paciencia who took possession thereof. Paciencia thereafter sold the same to
Dalmacio Secuya. When Paciencia died, her only heir, Ramon Sabellano, executed a

SALES CASE DIGEST COMPILATION 164


private document, "Deed of Confirmation of Sale," confirming the sale between Paciencia
and Dalmacio. The document was, however, lost. Meanwhile, Maxima sold the entire lot to
Silverio Aro, husband of Cesaria Caballero. Upon Silverio's death, the lot was transferred to
Cesaria from whom respondent bought the lot. Respondent was assured that petitioners
who were occupying a portion of the land were tenants. A clean title to the whole lot was
transferred to respondent. Petitioners, heirs of Dalmacio Secuya, filed an action for quieting
of title on the ground that respondent's title is a cloud on their title as owners and possessors
of the property subject of litigation. They claimed that they had been occupying the property
for forty-seven years though they did not pay the land taxes. The trial court rendered
judgment against respondent. It was affirmed, on appeal, by the Court of Appeals.

ISSUES:
1. Whether or not the Agreement is one of partition
2. Whether or not there was a repudiation of the Express Trust

HELD:
(1) NO. It was an express trust. Trust is the right to the beneficial enjoyment of property,
the legal title to which is vested in another. It is a fiduciary relationship that obliges the
trustee to deal with the property for the benefit of the beneficiary. Trust relations between
parties may either be express or implied. An express trust is created by the intention
of the trustor or of the parties. An implied trust comes into being by operation of
law. The present Agreement of Partition involves an express trust. Under Article 1444 of the
Civil Code, "[n]o particular words are required for the creation of an express trust, it being
sufficient that a trust is clearly intended." That Maxima Caballero bound herself to give one
third of Lot No. 5629 to Paciencia Sabellona upon the approval of the former's application is
clear from the terms of the Agreement. Likewise, it is evident that Paciencia acquiesced to
the covenant and is thus bound to fulfill her obligation therein. As a result of the Agreement,
Maxima Caballero held the portion specified therein as belonging to Paciencia Sabellona
when the application was eventually approved and a sale certificate was issued in her name.
Thus, she should have transferred the same to the latter, but she never did so during her
lifetime. Instead, her heirs sold the entire Lot No. 5679 to Silvestre Aro in 1955.

(2)YES. While no time limit is imposed for the enforcement of rights under express trusts,
prescription may, however, bar a beneficiary's action for recovery, if a repudiation of the trust
is proven by clear and convincing evidence and made known to the beneficiary. There was
a repudiation of the express trust when the heirs of Maxima Caballero failed to deliver
or transfer the property to Paciencia Sabellona, and instead sold the same to a third
person not privy to the Agreement. In the memorandum of incumbrances of TCT No.
3087, issued in the name of Maxima, there was no notation of the Agreement between her
and Paciencia. Equally important, the Agreement was not registered; thus, it could not bind
third persons. Neither was there any allegation that Silvestre Aro, who purchased the
property from Maxima's heirs, knew of it. Consequently, the subsequent sales transactions
involving the land in dispute and the titles covering it must be upheld, in the absence of proof
that the said transactions were fraudulent and irregular.

8) YUVIENCO versus DACUYCUY

ART. 1319. Consent is manifested by the meeting of the offer and the acceptance upon the
thing and the cause which are constitute the contract. The offer must be certain the
acceptance absolute. A qualified acceptance constitute a counter-offer.

SALES CASE DIGEST COMPILATION 165


Acceptance made by letter or telegram does not bind offerer except from the time it came to
his knowledge. The contract, in a case, is presumed to have been entered into in the place
where the offer was made.
Facts:
In essence, the theory of petitioners is that while it is true that they did express willingness to
sell to private respondents the subject property (land and building) for P6,500,000.00
provided the latter made known their own decision to buy it not later than July 31, 1978, the
respondents' reply that they were agreeable was not absolute, so much so that when
ultimately petitioners' representative went to Cebu City with a prepared and duly signed
contract for the purpose of perfecting and consummating the transaction, respondents and
said representative found variance between the terms of payment stipulated in the prepared
document and what respondents had in mind, hence the bank draft which respondents were
delivering to the representative was returned and the document remained unsigned by
respondents.
The respondents, in their complaint, contended “That on August 1, 1978 Pedro Gamboa
arrived Tacloban City bringing with him the prepared contract to purchase and to sell
referred to in his telegram dated July 27, 1978 for the purpose of closing the transactions
referred to in paragraphs 8 and 9 hereof, however, to the complete surprise of plaintiffs, the
defendant without giving notice to plaintiffs, changed the mode of payment with respect to
the balance of P4,500,000.00 by imposing upon plaintiffs to pay same amount within thirty
(30) days from execution of the contract instead of the former term of ninety (90) days.”
Issues:

1. Whether or not the complaint sufficiently states a cause of action?


2. Whether or not the claim alleged therein is unenforceable under the Statute of Frauds?

Ruling:

1. The court held that although there was no perfected contract of sale in the light of the
letter of Atty. Gamboa of July 12, 1978 and the letter-reply thereto of Yao; it being
doubtful whether or not, under Article 1319 of the Civil Code, the said letter may be
deemed as an offer to sell that is "certain", and more, the Yao telegram is far from being
an "absolute" acceptance under said article, still there appears to be a cause of action
alleged in Paragraphs 8 to 12 of the respondents' complaint, considering it is alleged
therein that subsequent to the telegram of Yao, it was agreed that the petitioners would
sell the property to respondents for P6.5 M, by paving P2 M down and the balance in 90
days and which agreement was allegedly violated when in the deeds prepared by Atty.
Gamboa and taken to Tacloban, only 30 days were given to respondents.

2. Further, the court ruled that in any sale of real property on installments, the Statute of
Frauds read together with the perfection requirements of Article 1475 of the Civil Code
must be understood and applied in the sense that the idea of payment on installments
must be in the requisite of a note or memorandum therein contemplated.

9) Claudel vs CA
G.R. No. 85240 July 12, 1991
HEIRS OF CECILIO (also known as BASILIO) CLAUDEL, namely, MODESTA CLAUDEL,
LORETA HERRERA, JOSE CLAUDEL, BENJAMIN CLAUDEL, PACITA CLAUDEL,
CARMELITA CLAUDEL, MARIO CLAUDEL, ROBERTO CLAUDEL, LEONARDO

SALES CASE DIGEST COMPILATION 166


CLAUDEL, ARSENIA VILLALON, PERPETUA CLAUDEL and FELISA CLAUDEL,
petitioners, vs. HON. COURT OF APPEALS, HEIRS OF MACARIO, ESPERIDIONA,
RAYMUNDA and CELESTINA, all surnamed CLAUDEL, respondents.

FACTS:
As early as Dec 28, 1922, Cecilio Claudel acquired from the Bureau of Lands Lot No. 1230.
He dutifully paid real estate taxes until his death. His widow Basilia, and later her son Jose,
thereafter paid the taxes. However, this parcel of land would later become the subject of
protracted litigation 39 years after his death.

2 branches of Cecilio’s family contested the ownership over the land: on one hand the
children of Cecilio (Modesto, Loreta, Jose, Benjamin, Pacita, etc --- known as Heirs of
Cecilio), and on the other, the brother and sisters of Cecilio: Macario, etc (private
respondents Siblings of Cecilio).

In 1972, the Heirs of Cecilio partitioned the lot among themselves and obtained TCTs on
their shares.

4 years later, the Siblings of Cecilio filed a “complaint for cancellation of titles and
reconveyance with damages”. They alleged that 46 years earlier, their parents had
purchased from Cecilio several portions of Lot 1230 for P30. They admitted that the
transaction was verbal. However, as proof of sale, the Siblings of Cecilio presented a
subdivision plan of the said land indicating the portions allegedly sold to the Siblings of
Cecilio. The complaint was dismissed by CFI. CA reversed the decision (one of the grounds:
The Statute of Frauds applies only to executory contracts and not to consummated sales as
in the case at bar where oral evidence may be admitted).

ISSUE: W/N the contract of sale was sufficiently proved NO

HELD:
The rule of thumb is that a sale of land, once consummated, is valid regardless of the form it
may have been entered into. For nowhere does law or jurisprudence prescribe that the
contract of sale be put in writing before such contract can validly cede or transmit rights over
a certain real property between the parties themselves.

However, in the event that a third party, as in this case, disputes the ownership of the
property, the person against whom that claim is brought cannot present any proof of such
sale and hence has no means to enforce the contract. Thus the Statute of Frauds was
precisely devised to protect the parties in a contract of sale of real property so that no such
contract is enforceable unless certain requisites, for purposes of proof, are met.

The provisions of the Statute of Frauds pertinent to the present controversy, state:

Art. 1403 (Civil Code). The following contracts are unenforceable, unless they are
ratified:
xxx xxx xxx
2) Those that do not comply with the Statute of Frauds as set forth in this
number. In the following cases, an agreement hereafter made shall be unenforceable
by action unless the same, or some note or memorandum thereof, be in writing, and
subscribed by the party charged, or by his agent; evidence, therefore, of the
agreement cannot be received without the writing, or a secondary evidence of its
contents:
xxx xxx xxx
e) An agreement for the leasing for a longer period than one year, or for the sale of real
property or of an interest therein;

SALES CASE DIGEST COMPILATION 167


The purpose of the Statute of Frauds is to prevent fraud and perjury in the enforcement of
obligations depending for their evidence upon the unassisted memory of witnesses by
requiring certain enumerated contracts and transactions to be evidenced in Writing.

The provisions of the Statute of Frauds originally appeared under the old Rules of Evidence.
However when the Civil Code was re-written in 1949 (to take effect in 1950), the provisions
of the Statute of Frauds were taken out of the Rules of Evidence in order to be included
under the title on Unenforceable Contracts in the Civil Code. The transfer was not only a
matter of style but to show that the Statute of Frauds is also a substantive law.

Therefore, except under the conditions provided by the Statute of Frauds, the existence of
the contract of sale made by Cecilio with his siblings cannot be proved.

Additional issue (in case Maam asks)


On the second issue, the belated claim of the SIBLINGS OF CECILIO who filed a complaint
in court only in 1976 to enforce a light acquired allegedly as early as 1930, is difficult to
comprehend.

The Civil Code states:

Art. 1145. The following actions must be commenced within six years:
(1) Upon an oral contract . . . (Emphasis supplied).

If the parties SIBLINGS OF CECILIO had allegedly derived their right of action from the oral
purchase made by their parents in 1930, then the action filed in 1976 would have clearly
prescribed. More than six years had lapsed.

We do not agree with the parties SIBLINGS OF CECILIO when they reason that an implied
trust in favor of the SIBLINGS OF CECILIO was established in 1972, when the HEIRS OF
CECILIO executed a contract of partition over the said properties.

But as we had pointed out, the law recognizes the superiority of the torrens title.

Above all, the torrens title in the possession of the HEIRS OF CECILIO carries more weight
as proof of ownership than the survey or subdivision plan of a parcel of land in the name of
SIBLINGS OF CECILIO.

The Court has invariably upheld the indefeasibility of the torrens title. No possession by any
person of any portion of the land could defeat the title of the registered owners thereof.

MEKI

10. SPOUSES ALFREDO V. SPOUSES BORRAS

Facts:

Armando and Adelia alleged in their complaint that Godofredo and Carmen mortgaged the
Subject Land for P7,000.00 with the Development Bank of the Philippines. To pay the debt,
Carmen and Godofredo sold the Subject Land to Armando and Adelia for P15,000.00, the
buyers to pay the DBP loan and its accumulated interest, and the balance to be paid in cash
to the sellers. Armando and Adelia gave Godofredo and Carmen the money to pay the loan

SALES CASE DIGEST COMPILATION 168


to DBP which signed the release of mortgage and returned the owner’s duplicate copy of
OCT No. 284 to Godofredo and Carmen.
Armando and Adelia subsequently paid the balance of the purchase price of the Subject
Land for which Carmen issued a receipt dated 11 March 1970. Godofredo and Carmen then
delivered to Adelia the owner’s duplicate copy of OCT No. 284, with the document of
cancellation of mortgage, official receipts of realty tax payments, and tax declaration in the
name of Godofredo.

Subsequently, Armando and Adelia discovered that Godofredo and Carmen had re-sold
portions of the Subject Land to several persons. On 8 February 1994, Armando and Adelia
filed an adverse claim with the Register of Deeds of Bataan. Armando and Adelia discovered
that Godofredo and Carmen had secured an owner’s duplicate copy of OCT No. 284 after
filing a petition in court for the issuance of a new copy. Godofredo and Carmen claimed in
their petition that they lost their owner’s duplicate copy. Thus, Armando and Adelia filed a
complaint for specific performance. In their answer, Godofredo and Carmen and the
subsequent Buyers (collectively “petitioners”) argued that the action is unenforceable under
the Statute of Frauds. Petitioners pointed out that there is no written instrument evidencing
the alleged contract of sale over the Subject Land in favor of Armando and Adelia.

Issue:
1) WON there was a perfected contract of sale
2) WON the alleged sale of the Subject Land in favor of Armando and Adelia is valid
and enforceable

Ruling:

Based on the findings of the lower court, the contract of sale between the spouses
Godofredo and Carmen and the spouses Armando and Adelia was a perfected contract. The
contract of sale of the Subject Land has also been consummated because the sellers and
buyers have performed their respective obligations under the contract. The trial and
appellate courts correctly refused to apply the Statute of Frauds to this case. The Statute of
Frauds provides that a contract for the sale of real property shall be unenforceable unless
the contract or some note or memorandum of the sale is in writing and subscribed by the
party charged or his agent. The existence of the receipt dated 11 March 1970, which is a
memorandum of the sale, removes the transaction from the provisions of the Statute of
Frauds. The Statute of Frauds applies only to executory contracts and not to contracts either
partially or totally performed. Thus, where one party has performed one’s obligation, oral
evidence will be admitted to prove the agreement. In the instant case, the parties have
consummated the sale of the Subject Land, with both sellers and buyers performing their
respective obligations under the contract of sale. In addition, a contract that violates the
Statute of Frauds is ratified by the acceptance of benefits under the contract. Godofredo and
Carmen benefited from the contract because they paid their DBP loan and secured the
cancellation of their mortgage using the money given by Armando and Adelia. Clearly, both
the sellers and the buyers have consummated the verbal contract of sale of the Subject
Land. The Statute of Frauds was enacted to prevent fraud. This law cannot be used to
advance the very evil the law seeks to prevent.

11. GALANG VS. COURT OF APPEALS

Facts:

July 1976, the Buenaventura siblings sold two parcels of land to Marcelino and Guadalupe
Galang. The terms of the sale are as follows:
1. Galang shall pay 25% downpayment;

SALES CASE DIGEST COMPILATION 169


2. The next 25% shall be paid within the next three months or upon removal of the
encargado (tiller-farmer) of the land. After payment, the Buenaventuras shall deliver the title
to Galang;
3. The remaining 50% shall be payable within one year from the execution of the contract of
sale subject to 12% interest per annum in case of delay.
Galang made the downpayment. However, upon Galang’s offer of the second payment
within the agreed period, the Buenaventuras refused to accept the payment as they insisted
that the contract did not really contain the true intention of the parties.
Now under Article 1191 of the Civil Code, in situations like this, the adverse party (the
Galangs) has the option to either compel the other party to fulfill the contract of sale (specific
performance) or ask for its rescission. Galang sued the Buenaventuras in court and filed for
specific performance against them.
The trial court however ruled that specific performance is not an option because it appears
that the encargador is an agricultural tenant protected by PD 1038 (Strengthening the
Security of Tenure of Tenant-Tillers in Non-Rice/Corn Producing Private Agricultural Lands).
Said encargador cannot be simply excluded from the land. Hence, the condition of excluding
the encargador is a legally impossible condition imposed on the contract of sale and cannot
be complied with the by Buenaventuras without violating the law. The Court of Appeals
affirmed the ruling of the trial court.
Issue: Whether or not the removal of the “encargado” is a condition precedent to the
fulfillment of the contract of sale such that finding that it was a legally impossible condition
would entitle the buyers to the rescission of the contract?

Whether or not there is a perfected contract of sale

Ruling:

1) There was no basis for rescinding the contract because the removal of the
“encargado” was not a condition precedent to the contract of sale. Rather, it was one
of the alternative periods for the payment of the second installment given by the
seller himself to the buyers. Secondly, even granting that it was indeed a condition
precedent rendering necessary the determination of the legal status of the
“encargado,” the lower courts were rash in holding that the “encargado” was a tenant
of the land in question.

2) Yes, there is a perefected contract of sale. Reviewing the terms of the Deed of Sale,
it is clear that the parties had reached the stage of perfection of the contract of sale,
there being already “a meeting of the minds upon the thing which is the object of the
contract and upon the price,” and on the basis of which both parties had the personal
right to reciprocally demand from the other the fulfillment of their respective
obligations. But contracts of sale may either be absolute or conditional. 8 One form of
conditional sales, is what is now popularly termed as a “Contract to Sell,” where
ownership or title is retained until the fulfillment of a positive condition, normally the
payment of the purchase price in the manner agreed upon. The breach of that
condition can prevent the obligation to convey title from acquiring a binding
force. 9 Where the condition is imposed, instead, upon the perfection of the contract,
the failure of such condition would prevent such perfection. 10 What we have here is
a contract to sell for it is the transfer of ownership, not the perfection of the contract
that was subjected to a condition. Ownership was not to vest in the buyers until full
payment of the purchase price and the transfer of the title to the buyers. Apart from
full payment of the purchase price, we find no other condition which would affect the

SALES CASE DIGEST COMPILATION 170


obligations of the parties, i.e., to pay, on the part of the buyer and to convey
ownership, on the part of the seller.

12. ROQUE VS LAPUZ

Facts:
Plaintiff Roque and defendant Lapuz entered into an agreement of sale covering Lots, 1, 2
and 9, Block 1, of said property payable in 120 equal monthly installments at the rate of
P16.00, P15.00 per square meter , respectively. In accordance with said agreement,
defendant paid to plaintiff the sum of 150.00 as deposit and the further sum of P740.56 to
complete the payment of four monthly installments covering months of July, August,
September and October, 1954.
On January 24, 1955, defendant requested plaintiff that he be allowed to abandon and
substitute Lots 1, 2 and 9, the subject lots with Lots 4 and 12, Block 2 of the Rockville
Subdivision, which are corner lots, to which request plaintiff acceded. The evidence
discloses that defendant proposed to plaintiff modification of their previous contract to sell
because he found it quite difficult to pay the monthly installments on the three lots, and
besides the two lots he had chosen were better lots, being in the corner. In addition, it was
agreed that the purchase price of these two lots would be at a uniform rate of P17.00 per
square meter payable in 120 equal installments, with interest at 8% annually on the balance
unpaid. Pursuant to this new agreement, defendant occupied and possessed Lots 4 and 12,
and enclosed them. However, aside from the deposit of 150.00 and the amount of 740.56,
which were paid under their previous agreement, defendant failed to make any further
payment on account of the agreed monthly installments for the two lots in dispute, under the
new contract to sell. Plaintiff demanded upon defendant not only to pay the stipulated
monthly installments in arrears, but also to make up-to-date his payments, but defendant
refused to comply with plaintiff’s demands.
On or about November 3, 1957, plaintiff demanded upon defendant to vacate lots in
question and to pay the reasonable rentals thereon at the rate of 60.00 per month from
August 1955. On January 22, 1960, Petitioner Felipe C. Roque filed the complaint against
Nicanor Lapuz and cancellation of the agreement of sale between them involving the two
lots in question and prayed that judgment be rendered ordering the rescission and
cancellation of the agreement of sale, the defendant to vacate the two parcels of land and
remove his house therefrom and to pay the plaintiff the reasonable rental thereof.
The Court of Appeals rendered its decision that the defendant Nicanor Lapuz is granted a
period of ninety (90) days from entry hereof within which to pay the balance. Hence, this
appeal.
ISSUE: 1) Whether or not private respondent is entitled to the benefits of the third
paragraph of Article 1191, New Civil Code, for the fixing of period
2) Whether or not there is a perfected contract of sale
RULING:
No. Respondent as obligor is not entitled to the benefits of paragraph 3 of Art. 1191, New
Civil Code. Having been in default and acted in bad faith, he is not entitled to the new period
of 90 days from entry of judgment within which to pay petitioner the balance of P11, 434.44
with interest due on the purchase price of P12, 325.00 for the two lots. To allow and grant
respondent an additional period for him to pay the balance of the purchase price, which
balance is about 92% of the agreed price, would be tantamount to excusing his bad faith and
sanctioning the deliberate infringement of a contractual obligation that is repugnant and
contrary to the stability, security and obligatory force of contracts. Moreover, respondent's
failure to pay the succeeding 116 monthly installments after paying only 4 monthly
installments is a substantial and material breach on his part, not merely casual, which takes
the case out of the application of the benefits of pa paragraph 3, Art. 1191, New Civil Code.
Pursuant to Art. 1191 of the New Civil Code, petitioner is entitled to rescission with payment
of damages which the trial court and the appellate court, in the latter's original decision,
granted in the form of rental at the rate of P60.00 per month from August, 1955 until

SALES CASE DIGEST COMPILATION 171


respondent shall have actually vacated the premises, plus P2,000.00 as attorney's fees. The
Court affirmed the same to be fair and reasonable. The Court also sustained the right of the
petitioner to the possession of the land, ordering thereby respondent to vacate the same and
remove his house therefrom.

13. ROMERO VS CA and ENRIQUETA CHUA VDA. DE ONGSIONG


Facts:

Petitioner Virgilio R. Romero was engaged in the business of production, manufacture and
exportation of perlite filter aids, permalite insulation and processed perlite ore. Petitioner and
his foreign partners decided to put up a central warehouse in Metro Manila.

Alfonso Flores and his wife, accompanied by a broker, offered a parcel of land measuring
1,952 square meters. Located in Barangay San Dionisio, Parañaque, Metro Manila, in the
name of private respondent Enriqueta Chua vda. de Ongsiong. Petitioner visited the
property and, except for the presence of squatters in the area, he found the place suitable
for a central warehouse.

Flores spouses called on petitioner with a proposal that should he advance the amount of
P50,000.00 which could be used in taking up an ejectment case against the squatters,
private respondent would agree to sell the property for only P800.00 per square meter.
Petitioner expressed his concurrence. On 09 June 1988, a contract, denominated Deed of
Conditional Sale.

Pursuant to the agreement, private respondent filed a complaint for ejectment against
Melchor Musa and 29 other squatter families. Judgement was rendered order squatters to
vacate said area.

Private respondent sought to return the P50,000.00 she received from petitioner since, she
said, she could not "get rid of the squatters" on the lot.

Atty. Joaquin Yuseco, Jr., counsel for private respondent, advised Atty. Apostol that the
Deed of Conditional Sale had been rendered null and void by virtue of his client's failure to
evict the squatters from the premises within the agreed 60-day period. He added that private
respondent had "decided to retaPrivate respondent entered into a “Conditional Deed of Sale”
with petitioner over a parcel of land in Paranaque, the latter advancing P50,000 for the
eviction of squatters therein. An ejectment suit was then filed by the private respondent
against the squatters. Although successful, private respondent sought the return of the
downpayment she received because “she could not get rid of the squatters”.

Issue:

Wheter or not there is a perfected contract of sale

Ruling:

Yes. A perfected contract of sale may either be absolute or conditional depending on


whether the agreement is devoid of, or subject to, any condition imposed on the passing of
title of the thing to be conveyed or on the obligation of a party thereto. When ownership is
retained until the fulfillment of a positive condition the breach of the condition will simply
prevent the duty to convey title from acquiring an obligatory force. If the condition is imposed
on an obligation of a party which is not complied with, the other party may either refuse to
proceed or waive said condition. Where, of course, the condition is imposed upon the

SALES CASE DIGEST COMPILATION 172


perfection of the contract itself, the failure of such condition would prevent the juridical
relation itself from coming into existence.

In determining the real character of the contract, the title given to it by the parties is not as
much significant as its substance. For example, a deed of sale, although denominated as a
deed of conditional sale, may be treated as absolute in nature, if title to the property sold is
not reserved in the vendor or if the vendor is not granted the right to unilaterally rescind the
contract predicated on the fulfillment or non-fulfillment, as the case may be, of the prescribed
condition. The term "condition" in the context of a perfected contract of sale pertains, in
reality, to the compliance by one party of an undertaking the fulfillment of which would
beckon, in turn, the demandability of the reciprocal prestation of the other party. The
reciprocal obligations referred to would normally be, in the case of vendee, the payment of
the agreed purchase price and, in the case of the vendor, the fulfillment of certain express
warranties (which, in the case at bench is the timely eviction of the squatters on the
property).

It would be futile to challenge the agreement here in question as not being a duly perfected
contract. A sale is at once perfected when a person (the seller) obligates himself, for a price
certain, to deliver and to transfer ownership of a specified thing or right to another (the
buyer) over which the latter agrees. From the moment the contract is perfected, the parties
are bound not only to the fulfillment of what has been expressly stipulated but also to all the
consequences which, according to their nature, may be in keeping with good faith, usage
and law. Under the agreement, private respondent is obligated to evict the squatters on the
property. Private respondent's failure "to remove the squatters from the property" within the
stipulated period gives petitioner the right to either refuse to proceed with the agreement or
waive that condition in consonance with Article 1545 of the Civil Code. This option clearly
belongs to petitioner and not to private respondent. In contracts of sale particularly, Article
1545 of the Civil Code allows the obligee to choose between proceeding with the agreement
or waiving the performance of the condition. Here, evidently, petitioner has waived the
performance of the condition imposed on private respondent to free the property from
squatters.

The right of resolution of a party to an obligation is predicated on a breach of faith by the


other party that violates the reciprocity between them. It is private respondent who has failed
in her obligation under the contract. Petitioner did not breach the agreement. He has agreed,
in fact, to shoulder the expenses of the execution of the judgment in the ejectment case and
to make arrangements with the sheriff to effect such execution in the property.

14. HOMESITE AND HOUSING CORP VS CA


Facts:
The PHHC board of directors passed Resolution No. 513, subject to the approval of the
Quezon City Council of the above-mentioned Consolidation Subdivision Plan, Lot 4.
containing 4,182.2 square meters is awaded to Spouses Rizalino Mendoza and Adelaida
Mendoza, at a price of twenty-one pesos (P21.00) per square meter" and "that this award
shall be subject to the approval of the OEC (PHHC) Valuation Committee and higher
authorities".
The city council disapproved the proposed consolidation subdivision plan on August 20,
1961. The said spouses were advised by registered mail of the disapproval of the plan.
Another subdivision plan was prepared and submitted to the city council for approval. The
revised plan, which included Lot 4, with a reduced area of 2,608.7, was approved by the city
council.

SALES CASE DIGEST COMPILATION 173


On April 26, 1965 the PHHC board of directors passed a resolution recalling all awards of
lots to persons who failed to pay the deposit or down payment for the lots awarded to them.
The Mendozas never paid the price of the lot nor made the 20% initial deposit.
On October 18, 1965 the PHHC board of directors passed Resolution No. 218, withdrawing
the tentative award of Lot 4 to the Mendoza -spouses under Resolution No. 513 and re-
awarding said lot jointly and in equal shares to Miguela Sto. Domingo, Enrique Esteban,
Virgilio Pinzon, Leonardo Redublo and Jose Fernandez, subject to existing PHHC rules and
regulations.
The five awardees made the initial deposit. The corresponding deeds of sale were executed
in their favor. The subdivision of Lot 4 into five lots was approved by the city council and the
Bureau of Lands.
On March 16, 1966 the Mendoza spouses asked for reconsideration of the withdrawal of the
previous award to them of Lot 4 and for the cancellation of the re-award of said lot to Sto.
Domingo and four others. Before the request could be acted upon, the spouses filed the
instant action for specific performance and damages.
Issue:
WON there was a perfected sale of Lot 4, with the reduced area, to the Mendozas which
they can enforce against the PHHC by an action for specific performance.
Ruling:
No, there was no perfected sale of Lot 4. It was conditionally or contingently awarded to the
Mendozas subject to the approval by the city council of the proposed consolidation
subdivision plan and the approval of the award by the valuation committee and higher
authorities.
The city council did not approve the subdivision plan. The Mendozas were advised in 1961
of the disapproval. In 1964, when the plan with the area of Lot 4 reduced to 2,608.7 square
meters was approved, the Mendozas should have manifested in writing their acceptance of
the award for the purchase of Lot 4 just to show that they were still interested in its purchase
although the area was reduced and to obviate ally doubt on the matter. They did not do so.
The PHHC board of directors acted within its rights in withdrawing the tentative award.
"The contract of sale is perfected at the moment there is a meeting of minds upon the thing
which is the object of the contract and upon the price. From that moment, the parties may
reciprocally demand performance, subject to the law governing the form of contracts." (Art.
1475, Civil Code).
"In conditional obligations, the acquisition of rights, as well as the extinguishment or loss of
those already acquired, shall depend upon the happening of the event which constitutes the
condition. (Art. 1181, Civil Code).
Under the facts of this case, we cannot say there was a meeting of minds on the purchase of
Lot 4 with an area of 2,608.7 square meters at P21 a square meter.

15. MARTIN VS REYES

Facts:
Plaintiff Roque and defendant Lapuz entered into an agreement of sale covering Lots, 1, 2
and 9, Block 1, of said property payable in 120 equal monthly installments at the rate of
P16.00, P15.00 per square meter , respectively. In accordance with said agreement,
defendant paid to plaintiff the sum of 150.00 as deposit and the further sum of P740.56 to
complete the payment of four monthly installments covering months of July, August,
September and October, 1954.

SALES CASE DIGEST COMPILATION 174


On January 24, 1955, defendant requested plaintiff that he be allowed to abandon and
substitute Lots 1, 2 and 9, the subject lots with Lots 4 and 12, Block 2 of the Rockville
Subdivision, which are corner lots, to which request plaintiff acceded. The evidence
discloses that defendant proposed to plaintiff modification of their previous contract to sell
because he found it quite difficult to pay the monthly installments on the three lots, and
besides the two lots he had chosen were better lots, being in the corner. In addition, it was
agreed that the purchase price of these two lots would be at a uniform rate of P17.00 per
square meter payable in 120 equal installments, with interest at 8% annually on the balance
unpaid. Pursuant to this new agreement, defendant occupied and possessed Lots 4 and 12,
and enclosed them. However, aside from the deposit of 150.00 and the amount of 740.56,
which were paid under their previous agreement, defendant failed to make any further
payment on account of the agreed monthly installments for the two lots in dispute, under the
new contract to sell. Plaintiff demanded upon defendant not only to pay the stipulated
monthly installments in arrears, but also to make up-to-date his payments, but defendant
refused to comply with plaintiff’s demands.
On or about November 3, 1957, plaintiff demanded upon defendant to vacate lots in
question and to pay the reasonable rentals thereon at the rate of 60.00 per month from
August 1955. On January 22, 1960, Petitioner Felipe C. Roque filed the complaint against
Nicanor Lapuz and cancellation of the agreement of sale between them involving the two
lots in question and prayed that judgment be rendered ordering the rescission and
cancellation of the agreement of sale, the defendant to vacate the two parcels of land and
remove his house therefrom and to pay the plaintiff the reasonable rental thereof.
The Court of Appeals rendered its decision that the defendant Nicanor Lapuz is granted a
period of ninety (90) days from entry hereof within which to pay the balance. Hence, this
appeal.
ISSUE: 1) Whether or not private respondent is entitled to the benefits of the third
paragraph of Article 1191, New Civil Code, for the fixing of period
RULING:
No. Respondent as obligor is not entitled to the benefits of paragraph 3 of Art. 1191, New
Civil Code. Having been in default and acted in bad faith, he is not entitled to the new period
of 90 days from entry of judgment within which to pay petitioner the balance of P11, 434.44
with interest due on the purchase price of P12, 325.00 for the two lots. To allow and grant
respondent an additional period for him to pay the balance of the purchase price, which
balance is about 92% of the agreed price, would be tantamount to excusing his bad faith and
sanctioning the deliberate infringement of a contractual obligation that is repugnant and
contrary to the stability, security and obligatory force of contracts. Moreover, respondent's
failure to pay the succeeding 116 monthly installments after paying only 4 monthly
installments is a substantial and material breach on his part, not merely casual, which takes
the case out of the application of the benefits of pa paragraph 3, Art. 1191, New Civil Code.
Pursuant to Art. 1191 of the New Civil Code, petitioner is entitled to rescission with payment
of damages which the trial court and the appellate court, in the latter's original decision,
granted in the form of rental at the rate of P60.00 per month from August, 1955 until
respondent shall have actually vacated the premises, plus P2,000.00 as attorney's fees. The
Court affirmed the same to be fair and reasonable. The Court also sustained the right of the
petitioner to the possession of the land, ordering thereby respondent to vacate the same and
remove his house therefrom.

16. PACIFIC OXYGEN & ACETYLENE CO. VS CENTRAL BANK

Facts:
Plaintiff Pacific Oxygen and Acetylene Co. applied on Sept. 21, 1961 with the Philippine
Trust company, an agent of the Central Bank, for commercial credit in the amount of
$63,964.00 in favor of the Independent Engineering Co., Inc., O'Fallon. The application was

SALES CASE DIGEST COMPILATION 175


approved on October 4, 1961, with the Philippine Trust Company establishing an irrevocable
letter of credit at the free market rate of P3.01875 to every dollar, the letter of credit, expiring
on February 1, 1962.
On October 5, 1961, the Philippine Trust Company applied with the Central Bank for the
purchase of forward exchange in the amount of $63,694.00 to cover its U.S. dollar
commitment against the letter of credit opened under free market rate for the plaintiff.
Then on October 6, 1961, the Central Bank in turn executed a forward exchange contract for
the sale of foreign exchange in the said amount to be delivered on January 2, 1962.
On January 17, 1962, the Philippine Trust Company applied for the purchase of forward
exchange with the Central Bank in the amount of $71,617.00 of which $67,874.00 would
cover its U.S. dollar commitments against the letter of credit opened under free market rate
for the plaintiff. Then the next day the Central Bank executed the corresponding forward
exchange contract for the same amount to be delivered on March 17, 1962.
On January 21, 1962, the Central Bank suspended the margin levy. On February 8, 1962,
the Independent Engineering Co., Inc., O'Fallon, Illinois, U.S.A., the beneficiary, drew two
drafts against said letter of credit in the sums of $19,277.41 and the Continental Illinois
National Bank and Trust Company of Chicago, Chicago, Illinois, correspondent of the
Philippine Trust Company, Manila, honored the first draft on February 9, 1962, and the
second draft on February 13, 1962, as shown by the debit advices on the same dates
addressed to Philippine Trust Company.
On February 18 and 23, 1962, the Philippine Trust Company sent to the plaintiff statements
of account on the importation in which were included the 15% margin fee.On March 14,
1962, the plaintiff paid under protest to the Central Bank, thru the Philippine Trust Company,
the amounts of P22,058.00 and P8,780.65, or a total of P30,839.49, representing the 15%
margin fee, the amount sought to be recovered.
The applicable law is Republic Act No. 2609, which insofar as pertinent, empowers the
Central Bank "in respect of all sales of foreign exchange by [it] and its authorized agent
banks, . . . to establish a uniform margin of not more than 40% over the bank's selling rates
stipulated by the Monetary Board . . .."
Issue: Whether or not there is a perfected contract of sale despite no specific stipulation on
the margin fee percentage
Ruling:
Yes. It is well-settled in our law that a contract of sale exists from the moment "one of the
contracting parties obligates himself to transfer the ownership of and to deliver a determinate
thing, and the other to pay therefor a price certain in money or its equivalent." There is a
perfection of such a contract "at the moment there is a meeting of minds upon the thing
which is the object of the contract and upon the price" from which moment, "the parties may
reciprocally demand performance, subject to the provisions of the law governing the form of
contracts." It is a fair restatement of the prevailing principle in American law that an
agreement by one party to sell and deliver, and by the other to purchase at a mentioned
price and terms certain personal property on or before a specified future date is a contract of
sale and not an option. With the categorical finding in the decision appealed from that the
purchase of the forward exchange by the Central Bank occurred on January 17, 1962, prior
to the suspension of the margin levy on January 21, 1962, it cannot be denied that
deference must be paid to the legal provision calling for a margin fee "in respect of all sales
of foreign exchange by the Central Bank and its authorized agents . . . ."

17) VILLONCO REALTY CO. VS BORMACHECO

Francisco Cervantes (President of Bormaheco) and his wife, Rosario Cervantes, are the
owners of 3 lots.

SALES CASE DIGEST COMPILATION 176


Bormaheco, Inc., through Cervantes, made a written offer to Romeo Villonco for the sale of
the properties. The offer reads:
(1) That we are offering to sell to you the above property at the price of P400.00 per square
meter;
(2) That a deposit of P100,000.00 must be placed as earnest money on the purchase of the
above property which will become part payment of the property in the event that the sale is
consummated;
(3) That this sale is to be consummated only after I shall have also consummated my
purchase of another property located at Sta. Ana, Manila;
(4) That if my negotiations with said property will not be consummated by reason beyond my
control, I will return to you your deposit of P100,000 and the sale of my property to you will
not also be consummated; and
(5) That final negotiations on both properties can be definitely known after 45 days.
Cervantes and Teofilo Villonco had a final conference. As a result of that conference
Villonco Realty Company, through Teofilo Villonco made a revised counter-offer for the
purchase of the property:
1. (same purchase price and deposit as first offer)
2. This sale shall be cancelled, only if your deal with another property in Sta. Ana shall not
be consummated and in such case, the P100,000-00 earnest money will be returned to us
with a 10% interest p.a. However, if our deal with you is finalized, said P100,000.00 will
become as part payment for the purchase of your property without interest
3. The manner of payment shall be as follows:
a. P100,000.00 earnest money and 650,000.00 as part of the down payment, or
P750,000.00 as total down payment
If the above terms and conditions are acceptable to you, kindly sign your conformity
hereunder. Enclosed is our check for ONE HUNDRED THOUSAND (P100,000.00) PESOS
as earnest money.
The check for P100,000 mentioned in the letter-contract was delivered to Bormaheco, Inc.
Then, Cervantes returned the earnest money, with interest. Cervantes alleged that the forty-
five day period had already expired and the sale to Bormaheco, Inc. of the Sta. Ana property
had not been consummated. Villonco Realty Company filed the complaint for specific
performance against Bormaheco, Inc.
Issue: Was there a perfected contract of sale between Bormaheco and Villonco?
Ruling:
Yes, there is a perfected contract of sale.
Bormaheco's acceptance of Villonco Realty Company's offer to purchase the Buendia
Avenue property, as shown in Teofilo Villonco's letter dated March 4, 1964 (Exh. D),
indubitably proves that there was a meeting of minds upon the subject matter and
consideration of the sale. Therefore, on that date the sale was perfected.
Bormaheco, Inc. and the Cervantes spouses contend that the sale was not perfected
because Cervantes allegedly qualified his acceptance of Villonco's revised offer and,
therefore, his acceptance amounted to a counter-offer which Villonco Realty Company
should accept but no such acceptance was ever transmitted to Bormaheco, Inc. which,
therefore, could withdraw its offer.
Villonco Realty Company paid, and Bormaheco, Inc. accepted, the sum of P100,000 as
earnest money or down payment. That crucial fact implies that Cervantes was aware that
Villonco Realty Company had accepted the modifications which he had made in Villonco's

SALES CASE DIGEST COMPILATION 177


counter-offer. Had Villonco Realty Company not assented to those insertions and
annotations, then it would have stopped payment on its check for P100,000.
It is deducible from the tenor of those statements that the consummation of the sale of the
Buendia lots to Villonco Realty Company was conditioned on Bormaheco's acquisition of the
Nassco land.
The term of forty-five days was not a part of the condition that the Nassco property should
be acquired.It is simply a surmise that after forty-five days (in fact when the forty-five day
period should be computed is not clear) it would be known whether Bormaheco, Inc. would
be able to acquire the Nassco property and whether it would be able to sell the Buendia
property.

18. FIRME VS BUKLOD ENTERPRISES AND DEV CORP

Facts: Spouses Constante and Azucena Firme are the registered owners of a parcel of land
located on Dahlia Avenue, Fairview Park, Quezon City. Renato de Castro, the vice president
of Bukal Enterprises and Development Corporation authorized his friend, Teodoro Aviles, a
broker, to negotiate with the Spouses Firme for the purchase of the Property. On 28 March
1995, Bukal Enterprises filed a complaint for specific performance and damages with the
trial court, alleging that the Spouses Firme reneged on their agreement to sell the Property.
The complaint asked the trial court to order the Spouses Firme to execute the deed of sale
and to deliver the title to the Property to Bukal Enterprises upon payment of the agreed
purchase price. On 7 August 1998, the trial court rendered judgment against Bukal
Enterprises, dismissing the case and ordering Bukal Enterprises to pay the Spouses
Constante and Azucena Firme (1) the sum of P335,964.90 as and by way of actual and
compensatory damages; (2) the sum of P500,000.00 as and by way of moral damages; (3)
the sum of P100,000.00 as and by way of attorney’s fees; and (4) the costs of the suit.

The trial court held there was no perfected contract of sale as Bukal Enterprises failed to
establish that the Spouses Firme gave their consent to the sale of the Property; and that
Aviles had no valid authority to bind Bukal Enterprises in the sale transaction. Bukal
Enterprises appealed to the Court of Appeals, which reversed and set aside the decision of
the trial court. The appellate court ordered the Spouses Firme to execute the Deed of
Absolute Sale transferring the ownership of the subject property to Bukal Enterprises
immediately upon receipt of the purchase price of P3,224,000.00 and to perform all such
acts necessary and proper to effect the transfer of the property covered by TCT 264243 to
Bulak Enterprises; and directed Bukal Enterprises to deliver the payment of the purchase
price of the property within 60 days from the finality of the judgment. The Court of Appeals
held that the lack of a board resolution authorizing Aviles to act on behalf of Bukal
Enterprises in the purchase of the Property was cured by ratification; inasmuch as Bukal
Enterprises ratified the purchase when it filed the complaint for the enforcement of the sale.
The spouses Firme filed the petition for review on certiorari before the Supreme Court.

Issue: Whether there was a perfected contract between the Spouses Firme and Bukal
Enterprises, the latter allegedly being represented by Aviles.

Held: There was no consent on the part of the Spouses Firme. Consent is an essential
element for the existence of a contract, and where it is wanting, the contract is non-existent.
The essence of consent is the conformity of the parties on the terms of the contract, the
acceptance by one of the offer made by the other. The Spouses Firme flatly rejected the
offer of Aviles to buy the Property on behalf of Bukal Enterprises. There was therefore no
concurrence of the offer and the acceptance on the subject matter, consideration and terms
of payment as would result in a perfected contract of sale. Further, there was no approval
from the Board of Directors of Bukal Enterprises as would finalize any transaction with the

SALES CASE DIGEST COMPILATION 178


Spouses Firme. Aviles did not have the proper authority to negotiate for Bukal Enterprises.
Aviles testified that his friend, De Castro, had asked him to negotiate with the Spouses
Firme to buy the Property. De Castro, as Bukal Enterprises’ vice president, testified that he
authorized Aviles to buy the Property. However, there is no Board Resolution authorizing
Aviles to negotiate and purchase the Property on behalf of Bukal Enterprises. It is the board
of directors or trustees which exercises almost all the corporate powers in a corporation.
Under Sections 23 and 36 of the Corporation Code, the power to purchase real property is
vested in the board of directors or trustees. While a corporation may appoint agents to
negotiate for the purchase of real property needed by the corporation, the final say will have
to be with the board, whose approval will finalize the transaction. A corporation can only
exercise its powers and transact its business through its board of directors and through its
officers and agents when authorized by a board resolution or its by-laws. Aviles, who
negotiated the purchase of the Property, is neither an officer of Bukal Enterprises nor a
member of the Board of Directors of Bukal Enterprises. There is no Board Resolution
authorizing Aviles to negotiate and purchase the Property for Bukal Enterprises. There is
also no evidence to prove that Bukal Enterprises approved whatever transaction Aviles
made with the Spouses Firme. In fact, the president of Bukal Enterprises did not sign any of
the deeds of sale presented to the Spouses Firme. Even De Castro admitted that he had
never met the Spouses Firme. Considering all these circumstances, it is highly improbable
for Aviles to finalize any contract of sale with the Spouses Firme. Furthermore, the Court
notes that in the Complaint filed by Bukal Enterprises with the trial court, Aviles signed the
verification and certification of non-forum shopping. The verification and certification of non-
forum shopping was not accompanied by proof that Bukal Enterprises authorized Aviles to
file the complaint on behalf of Bukal Enterprises. The power of a corporation to sue and be
sued is exercised by the board of directors. “The physical acts of the corporation, like the
signing of documents, can be performed only by natural persons duly authorized for the
purpose by corporate by-laws or by a specific act of the board of directors.” The purpose of
verification is to secure an assurance that the allegations in the pleading are true and correct
and that it is filed in good faith. True, this requirement is procedural and not jurisdictional.
However, the trial court should have ordered the correction of the complaint since Aviles was
neither an officer of Bukal Enterprises nor authorized by its Board of Directors to act on
behalf of Bukal Enterprises.

19. ASUNCION VS CA

FACTS:
Petitioners allege that they are tenants or lessees of residential and commercial spaces
owned by defendants in Ongpin Street, Binondo, Manila since 1935 and that on several
occasions before October 9, 1986, defendants informed plaintiffs that they are offering to sell
the premises and are giving them priority to acquire the same. During the negotiations,
Bobby Cu Unjieng offered a price of P6-million while petitioners made a counter offer of P5-
million. On October 24, 1986, petitioners asked the respondents to specify the terms and
conditions of the offer to sell.
Petitioners now raise that since respondents failed to specify the terms and conditions of the
offer to sell and because of information received that the latter were about to sell the
property, plaintiffs were compelled to file the complaint to compel defendants to sell the
property to them.The trial court found that the respondents‘ offer to sell was never accepted
by the petitioners for the reason that they did not agree upon the terms and conditions of the
proposed sale, hence, there was no contract of sale at all. The Court of Appeals affirmed the
decision of the lower court. This decision was brought to the Supreme Court by petition for
review on certiorari which subsequently denied the appeal on May 6, 1991.

SALES CASE DIGEST COMPILATION 179


On November 15, 1990, while case was pending consideration by this Court, the Cu Unjieng
spouses executed a Deed of Sale transferring the property in question to herein respondent
Buen Realty and Development Corporation, for P15,000,000.00. On July 1,1991, respondent
as the new owner of the subject property wrote a letter to the petitioners demanding that the
latter vacate the premises. On July 16, 1991, the petitioners wrote a reply to respondent
corporation stating that the latter brought the property subject to the notice of lis pendens
regarding Civil Case No. 87-41058 annotated on TCT No. 105254/T-881 in the name of the
Cu Unjiengs. The lessees filed a Motion for Execution dated August 27, 1991 of the ecision
in Civil Case No. 87-41058 as modified by the Court of Appeals in CA-G.R. CV No. 21123.
On August 30, 1991, the RTC ordered the Cu Unjiengs to execute the necessary Deed of
Sale of the property in litigation in favor of plaintiffs Ang Yu Asuncion, Keh Tiong and Arthur
Go for the consideration of P15 Million pesos in recognition of petitioners‘ right of first refusal
and that a new Transfer Certificate of Title be issued in favor of the buyer.
ISSUE:
Whether or not Buen Realty can be bound by the writ of execution by virtue of the notice of
lis pendens, carried over on TCT No. 195816 issued in the name of Buen Realty, at the time
of the latter‘s purchase of the property on 15 November 1991 from the Cu Unjiengs.
Whether or not there is a perfected contract of sale.
Ruling:
We affirm the decision of the appellate court. A negotiation is formally initiated by an offer.
An imperfect promise (policitacion) is merely an offer. Public advertisements or solicitations
and the like are ordinarily construed as mere invitations to make offers or only as proposals.
These relations, until a contract is perfected, are not considered binding commitments. Thus,
at any time prior to the perfection of the contract, either negotiating party may stop the
negotiation. The offer, at this stage, may be withdrawn; the withdrawal is effective
immediately after its manifestation, such as by its mailing and not necessarily when the
offeree learns of the withdrawal.
Where a period is given to the offeree within which to accept the offer, the
following rules generally govern:
(1) If the period is not itself founded upon or supported by a consideration, the offeror is still
free and has the right to withdraw the offer before its acceptance, or, if an acceptance has
been made, before the offeror's coming to know of such fact, by communicating that
withdrawal to the offeree. The right to withdraw, however, must not be exercised whimsically
or arbitrarily; otherwise, it could give rise to a damage claim under Article 19 of the Civil
Code which ordains that "every person must, in the exercise of his rights and in the
performance of his duties, act with justice, give everyone his due, and observe honesty and
good faith."
(2) If the period has a separate consideration, a contract of "option" is deemed perfected,
and it would be a breach of that contract to withdraw the offer during the agreed period. The
option, however, is an independent contract by itself, and it is to be distinguished from the
projected main agreement (subject matter of the option) which is obviously yet to be
concluded. If, in fact, the optioner-offeror withdraws the offer before its acceptance (exercise
of the option) by the optionee-offeree, the latter may not sue for specific performance on the
proposed contract ("object" of the option) since it has failed to reach its own stage of
perfection. The optioner-offeror, however, renders himself liable for damages for breach of
the option. In these cases, care should be taken of the real nature of theconsideration given,
for if, in fact, it has been intended to be part of the consideration for the main contract with a
right of withdrawal on the part of the optionee, the main contract could be deemed perfected;
a similar instance would be an "earnest money" in a contract of sale that can evidence its
perfection (Art. 1482, Civil Code).

SALES CASE DIGEST COMPILATION 180


In the law on sales, the so-called ―right of first refusal‖ is an innovative juridical relation.
Needless to point out, it cannot be deemed a perfected contract of sale under Article 1458 of
the Civil Code.
In a right of first refusal, while the object might be made determinate, the exercise of the
right, however, would be dependent not only on the grantor‘s eventual intention to enter into
a binding juridical relation with another but also on terms, including the price, that obviously
are yet to be later firmed up. Prior thereto, it can at best be so described as merely
belonging to a class of preparatory juridical relations governed not by contracts (since the
essential elements to establish the vinculum juris would still be indefinite and inconclusive)
but by, among other laws of general application, the pertinent scattered provisions of the
Civil Code on human conduct. The final judgment in Civil Case No. 87-41058, it must be
stressed, has merely accorded a ―right of first refusal‖ in favor of petitioners. The
consequence of such a declaration entails no more than what has heretofore been said. In
fine, if, as it is here so conveyed to us, petitioners are aggrieved by the failure of private
respondents to honor the right of first refusal, the remedy is not a writ of execution on the
judgment, since there is none to execute, but an action for damages in a proper forum for
the purpose. Furthermore, Buen Realty, not having been impleaded in Civil Case No. 87-
41058, cannot be held subject to the writ of execution issued by respondent Judge, let alone
ousted from the ownership and possession of the property, without first being duly afforded
its day in court.

20) ALCANTARA DAUS VS. DE LEON

Facts:

This is a complaint for annulment of documents and title, ownership, possession, injunction,
preliminary injunction, restraining order and damages.

Spouses Hermoso de Leon and Soccoro de Leon (respondents) alleged in their complaint
that they are the owners of a parcel of land.

Hermoso de Leon inherited from his father Marcelino de Leon by virtue of a Deed of extra-
judicial partition. Sometime in the early 1960s, respondents engaged the services of the late
Atty. Florencio Juan to take care of the documents of the properties of his parents. Atty.
Juan let them sign voluminous documents. After the death of Atty. Juan, some documents
surfaced and most revealed that their properties had been conveyed by sale or quitclaim to
respondent Hermosos brothers and sisters, to Atty. Juan and his sisters, when in truth and in
fact, no such conveyances were ever intended by them. His signature in the Deed of Extra-
judicial Partition with Quitclaim made in favor of Rodolfo de Leon was forged. They
discovered that the land in question was sold by Rodolfo de Leon to petitioner Aurora
Alcantara. They demanded annulment of the document and reconveyance but defendants
refused.

Aurora Alcantara-Daus averred that she bought the land in question in good faith and for
value on December 6, 1975. She has been in continuous, public, peaceful, open possession
over the same and has been appropriating the produce thereof without objection from
anyone.

ISSUE: Whether or not the Deed of Absolute Sale dated December 6, 1975 executed by
Rodolfo de Leon (deceased) over the land in question in favor of petitioner was perfected
and binding upon the parties therein?
Ruling:

SALES CASE DIGEST COMPILATION 181


A contract of sale is consensual. It is perfected by mere consent, upon a meeting of the
minds on the offer and the acceptance thereof based on subject matter, price and terms of
payment. At this stage, the sellers ownership of the thing sold is not an element in the
perfection of the contract of sale.
The contract, however, creates an obligation on the part of the seller to transfer
ownership and to deliver the subject matter of the contract. It is during the delivery that the
law requires the seller to have the right to transfer ownership of the thing sold. In general, a
perfected contract of sale cannot be challenged on the ground of the sellers non-ownership
of the thing sold at the time of the perfection of the contract.
Further, even after the contract of sale has been perfected between the parties, its
consummation by delivery is yet another matter. It is through tradition or delivery that the
buyer acquires the real right of ownership over the thing sold.
Undisputed is the fact that at the time of the sale, Rodolfo de Leon was not the owner of
the land he delivered to petitioner. Thus, the consummation of the contract and the
consequent transfer of ownership would depend on whether he subsequently acquired
ownership of the land in accordance with Article 1434 of the Civil Code. Therefore, we need
to resolve the issue of the authenticity and the due execution of the Extrajudicial Partition
and Quitclaim in his favor.

XVII. CONSUMMATION STAGE


1) SPOUSES SANTOS V. CA

Facts:
The spouses Santos owned the house and lot which was mortgaged with the Rural Bank of
Salinas, Inc., to secure a loan of P150,000.00 maturing on June 16, 1987. On June 16,
1984, the bank sent Rosalinda Santos a letter demanding payment of P16,915.84 in unpaid
interest and other charges. Since the Santos couple had no funds, Rosalinda offered to sell
the house and lot to Carmen. The Casedas gave an initial payment of P54,100.00 and
immediately took possession of the property, which they then leased out. They also paid in
installments, P81,696.84 of the mortgage loan. The Casedas, however, failed to pay the
remaining balance of the loan because they suffered bankruptcy in 1987. In January 1989,
the Santoses, seeing that the Casedas lacked the means to pay the remaining installments
and/or amortization of the loan, repossessed the property. The Santoses then collected the
rentals from the tenants. Caseda then approached petitioners and offered to pay the balance
of the purchase price for the house and lot. The parties, however, could not agree, and the
deal could not push through because the Santoses wanted a higher price. Respondents
insist that there was a perfected contract of sale, since upon their partial payment of the
purchase price, they immediately took possession of the property as vendees, and
subsequently leased it, thus exercising all the rights of ownership over the property. This
showed that transfer of ownership was simultaneous with the delivery of the realty sold,
according to respondents.
Issue: WON there was a perfected contract of sale
Ruling : We are far from persuaded that there was a transfer of ownership simultaneously
with the delivery of the property purportedly sold. The records clearly show that,
notwithstanding the fact that the Casedas first took then lost possession of the disputed
house and lot, the title to the property, has remained always in the name of Rosalinda
Santos. Note further that although the parties agreed that the Casedas would assume the
mortgage, all amortization payments made by Carmen Caseda to the bank were in the name
of Rosalinda Santos. The foregoing circumstances categorically and clearly show that no
valid transfer of ownership was made by the Santoses to the Casedas. We agree with
petitioner's averment that the agreement between Rosalinda Santos and Carmen Caseda is
a contract to sell. If the vendor should eject the vendee for failure to meet the condition

SALES CASE DIGEST COMPILATION 182


precedent, he is enforcing the contract and not rescinding it. When the petitioners in the
instant case repossessed the disputed house and lot for failure of private respondents to pay
the purchase price in full, they were merely enforcing the contract and not rescinding it
.
2) DY JR. VS CA

Facts: Sometime in 1979, Wilfredo Dy purchased a truck and a farm tractor through
financing extended by Libra Finance and Investment Corporation (Libra). Both truck and
tractor were mortgaged to Libra as security for the loan.The petitioner; Perfecto wrote a
letter to Libra requesting that he be allowed to purchase from Wilfredo Dy the said tractor
and assume the mortgage debt of the latter. In a letter dated August 27, 1979, Libra
approved the petitioner's request. Thus, on September 4, 1979, Wilfredo Dy executed a
deed of absolute sale in favor of the petitioner over the tractor in question. At this time, the
subject tractor was in the possession of Libra Finance due to Wilfredo Dy's failure to pay the
amortizations. Payment having been effected through an out-of-town check, Libra insisted
that it be cleared first before Libra could release the chattels in question. Meanwhile, Civil
Case No. R-16646 entitled "Gelac Trading, Inc. v. Wilfredo Dy", a collection case to recover
the sum of P12,269.80 was pending in another court in Cebu. On the strength of an alias
writ of execution issued on December 27, 1979, the provincial sheriff was able to seize and
levy on the tractor which was in the premises of Libra in Carmen, Cebu. The tractor was
subsequently sold at public auction where Gelac Trading was the lone bidder. It was only
when the check was cleared on January 17, 1980 that the petitioner learned about GELAC
having already taken custody of the subject tractor. Consequently, the petitioner filed an
action to recover the subject tractor against GELAC Trading with the Regional Trial Court of
Cebu City.RTC rendered judgment in favor of the petitioner. CA reversed. The respondents
claim that at the time of the execution of the deed of sale, no constructive delivery was
effectedsince the consummation of the sale depended upon the clearance and encashment
of the check which was issued in payment of the subject tractor.
Issue: WON Articles 1498 and 1499 are applicable
Held: The Supreme Court agree with the petitioner that Articles 1498 and 1499 are
applicable in the case at bar. In the instant case, actual delivery of the subject tractor could
not be made. However, there was constructive delivery already upon the execution of the
public instrument pursuant to Article 1498 and upon the consent or agreement of the parties
when the thing sold cannot be immediately transferred to the possession of the vendee.
While it is true that Wilfredo Dy was not in actual possession and control of the subject
tractor, his right of ownership was not divested from him upon his default. Neither could it be
said that Libra was the owner of the subject tractor because the mortgagee can not become
the owner of or convert and appropriate to himself the property mortgaged. Said property
continues to belong to the mortgagor. Undeniably, Libra gave its consent to the sale of the
subject tractor to the petitioner. It was aware of the transfer of rights to the petitioner. Where
a third person purchases the mortgaged property, he automatically steps into the shoes of
the original mortgagor. His right of ownership shall be subject to the mortgage of the thing
sold to him. In the case at bar, the petitioner was fully aware of the existing mortgage of the
subject tractor to Libra. In fact, when he was obtaining Libra's consent to the sale, he
volunteered to assume the remaining balance of the mortgage debt of Wilfredo Dy which
Libra undeniably agreed to. The sale of the subject tractor was consummated upon the
execution of the public instrument on September 4, 1979. At this time constructive delivery
was already effected. Hence, the subject tractor was no longer owned by Wilfredo Dy when
it was levied upon by the sheriff in December, 1979. Well settled is the rule that only
properties unquestionably owned by the judgment debtor and which are not exempt by law
from execution should be levied upon or sought to be levied upon. For the power of the court
in the execution of its judgment extends only over properties belonging to the judgment
debtor.

SALES CASE DIGEST COMPILATION 183


3) ADDISON VS FELIX

Facts:
By a public instrument dated June 11, 1914, the plaintiff sold to the defendant Marciana
Felix, four parcels of land. The defendant Felix paid, at the time of the execution of the deed,
the sum of P3,000 on account of the purchase price, and bound herself to pay the remainder
in installments, the first of P2,000 on July 15, 1914, and the second of P5,000 thirty days
after the issuance to her of a certificate of title under the Land Registration Act. It was further
stipulated that the purchaser was to deliver to the vendor 25 per centum of the value of the
products that she might obtain from the four parcels “from the moment she takes possession
of them until the Torrens certificate of title be issued in her favor.” In January, 1915, the
vendor, Addison, filed suit in Court of First Instance of Manila to compel Marciana Felix to
make payment of the first installment of P2,000. The defendant answered that the plaintiff
had absolutely failed to deliver to the defendant the lands that were the subject matter of the
sale, notwithstanding the demands made upon him for thispurpose.
Issue: WON Addison is obliged to deliver to the defendants the lands in question
Ruling: Addison is obliged to deliver the 4 parcels of land. The record shows that the plaintiff
did not deliver the thing sold. With respect to two of the parcels of land, he was not even
able to show them to the purchaser; and as regards the other two, more than two-thirds of
their area was in the hostile and adverse possession of a third person. The Code imposes
upon the vendor the obligation to deliver the thing sold. The thing is considered to be
delivered when it is placed "in the hands and possession of the vendee." (Civ. Code, Art.
1462.) It is true that the same article declares that the execution of a public instruments is
equivalent to the delivery of the thing which is the object of the contract, but, in order that
this symbolic delivery may produce the effect of tradition, it is necessary that the vendor
shall have had such control over the thing sold that, at the moment of the sale, its material
delivery could have been made. It is not enough to confer upon the purchaser the ownership
and the right of possession. The thing sold must be placed in his control. When there is no
impediment whatever to prevent the thing sold passing into the tenancy of the purchaser by
the sole will of the vendor, symbolic delivery through the execution of a public instrument is
sufficient. But if, notwithstanding the execution of the instrument, the purchaser cannot have
the enjoyment and material tenancy of the thing and make use of it himself or through
another in his name, because such tenancy and enjoyment are opposed by the interposition
of another will, then fiction yields to reality — the delivery has not been effected.

MIGUEL

4) DANGUILAN V. AIC (November 28, 1988)

Facts:
 A parcel of lot owned by Domingo Melad was being claimed by petitioner Felix
Danguilan and respondent Apolonia Melad.
 Apolonia Melad contends that she acquired the property when Dominggo Melad sold it
to her when she was just 3 years old in which her mother paid the consideration.
(Evidence: Deed of sale dated December 4, 1943 with a sum consideration of P80.00.)
 Apolonia contended that she just moved out of the farm only in 1946 when Felix
Danguilan approached her and asked permission to cultivate the land and to stay
therein.
 Dangguilan, on the other hand, presented for his part 2 documents executed in
September 14, 1941 and December 18, 1943, to prove his claim that the properties
were given to him by Dominggo Melad through an onerous donation. The onerous part
of the donation includes the taking care of the farm and the arrangement of the burial
of Dominggo.

SALES CASE DIGEST COMPILATION 184


 RTC ruled in favor of Danguilan. CA reversed RTC’s ruling. It ruled that there was a
donation, which was void for failing to comply with the formalities.

Issues:
1. Who has the better right between parties? Petitioner Danguilan.
2. WON there was delivery in favor of respondent for the alleged sale? NO.

Held:

Domingo Melad intended to donate the property to petitioner Danguilan


It is our view, considering the language of the two instruments, that Domingo Melad did
intend to donate the properties to the petitioner Danguilan. We do not think, however, that
the donee was moved by pure liberality. While truly donations, the conveyances were
onerous donations as the properties were given to petitioner Danguilan in exchange for his
obligation to take care of the donee for the rest of his life and provide for his burial.

Hence, it was not covered by the rule in Article 749 of the Civil Code requiring donations of
real properties to be effected through a public instrument, and the 2 private documents
remain valid.

Assuming there was a valid deed of sale, PR Melad failed to show that it was
consummated (no actual delivery + no possession)
At any rate, even assuming the validity of the deed of sale, the record shows that Apolonia
Melad did not take possession of the disputed properties and indeed waited until 1962 to file
this action for recovery of the lands from petitioner Danguilan. If she did have possession,
she transferred the same to Danguilan in 1946, by her own sworn admission, and moved out
to another lot belonging to her step-brother.

Her claim that the petitioner was her tenant (later changed to administrator) was disbelieved
by the trial court, and properly so, for its inconsistency. In short, she failed to show that she
consummated the contract of sale by actual delivery of the properties to her and her actual
possession thereof in concept of purchaser-owner.

No constructive delivery allowed if property is in actual and adverse possession of a


third person
In our jurisdiction, it is a fundamental and elementary principle that ownership does not pass
be mere stipulation but only by delivery and the execution of a public document does not
constitute sufficient delivery where the property involved is in the actual and adverse
possession of third persons.

Therefore, in our Civil Code it is a fundamental principle in all matters of contracts and a
well- known doctrine of law that "non mudis pactis sed traditione dominia rerum
transferuntur".

In conformity with said doctrine as established in paragraph 2 of article 609 of said code,
that "the ownership and other property rights are acquired and transmitted by law, by gift, by
testate or intestate succession, and, in consequence of certain contracts, by tradition".

In accordance with such disposition and provisions the delivery of a thing constitutes a
necessary and indispensable requisite for the purpose of acquiring the ownership of the
same by virtue of a contract.

One who is in possession is presumed to be the owner

SALES CASE DIGEST COMPILATION 185


In this case, there no dispute that it is Danguilan and not Melad who is in actual possession
of the litigated properties. And even if the claim of petitioner and respondent are weak,
judgment must be in favor of the Danguilan for one who is in possession is presumed to be
the owner, and cannot be obliged to show or prove a better right.

5) NAVAL vs. ENRIQUEZ


(3 Phil 669); April 12, 1904

FACTS: Don Jorge Enriquez as heirs of his deceased parents (Don Antonio Enriquez and
Dona Ciriaca Villanueva , whose estates were at that time still undistributed, by public
document sold to Don Victoriano Reyes his interest in both estates. The deed was executed
before Don Enrique Barrera, a notary public. Another instrument was executed before the
same notary public where Don Victoriano Reyes sold to Dona Carmen the interest in the
estates which he had acquired from Don Jorge Enriquez. The purchaser, Dona Carmen was
the wife of Don Francisco Enriquez (defendant) who was the executor and administrator of
the testamentary estate of Don Antonio Enriquez at the time the two deeds were executed.
The plaintiffs demand that these deeds be declared null and void, as well as the
contracts evidenced thereby. Apparently solely so far as they refer to the estate of Don
Antonio Enriquez, no mention being made of the estate of Dona Ciriaca Villanueva in the
complaint.
The plaintiffs contended that the deeds in question were consummated and were
executed for the purpose of deceiving and defrauding Don Jorge Enriquez and his family.
The conclusion of the plaintiffs was that as such executor Don Francisco was unable to
acquire by his own act or that of any intermediary the said hereditary portion of Don Jorge
under the provisions of Article 1459, paragraph 3 of the Civil Code.

ISSUE: WON Don Francisco Enriquez as executor and administrator of estate of Don
Antonio is incapacitated to acquire by purchase the hereditary right of Jorge Enriquez.

Held: NO! The thing sold in the two contracts of sale mentioned in the complaint was the
hereditary right of Don Jorge Enriquez, which evidently was not in charge of the executor
Don Francisco Enriquez. Executors, even in those cases in which they administer the
property pertaining to estate, do not administer the hereditary rights of any heirs. This right is
vested entirely in the heirs who retain it or transmit it in whole or in part, as they may deem
convenient, to some other person absolutely, whatever powers the testator may have
desired to confer upon him, do not and cannot under any circumstances in the slightest
degree limit the power of the heirs to dispose of the said right at will. That right does not form
part of the property delivered to the executor for administration.
Further, Article 1459 of the Civil Code has no application to the present case. The
prohibition which Article 145, paragraph 3 imposes upon executors refers to the property
confided to their care and does not extend, therefore, to property not falling within this class.
Consequently, even upon the supposition that the executor (Don Francisco) was the person
who really acquired the hereditary rights of Don Jorge, the sale in question would not for that
reason be invalid, the executor not being legally incapacitated of acquiring the hereditary
right in question as the plaintiffs erroneously suppose.
The action brought by the plaintiffs is devoid of foundation.

6) R.R. PAREDES VS. CALILUNG

Facts: Petitioners are sued as official of Caltex Philippines, Inc (CPI).


Calilung alleged that the officials made false representations of the parcels of

SALES CASE DIGEST COMPILATION 186


land that he bought from Caltex, that some of which were sold by Caltex to
DAR by a Voluntary Offer to Sell (VOS) which the CA decided as a
consummated sale. The land was originally owned by Calilung’s mother-in-
law, Vda. de Medina who executed a Deed of Assignment (with Special
Power of Attorney Coupled with Interest), in which, for and in consideration of
her unpaid obligations to CPI, she assigned all of her "rights, interests, claims
and participation from the proceeds of land compensation" for the property
she voluntarily offered to sell and transfer under the CARP. She claimed in
the same Deed that the VOS over the subject properties was already under
process for indorsement to the Landbank. Hence, she was appointing CPI as
her exclusive attorney-in-fact to follow-up the processing of the VOS papers
with the DAR and the Landbank.
Issue: Is the filing of a Voluntary Offer to Sell a consummated sale?
Held: No, a VOS, as its name implies, is a voluntary offer to sell the land to
the government so that the latter can distribute the same to qualified tenants.
While a landowner who voluntarily offered his land for sale is precluded from
withdrawing his offer except under specified circumstances, such a condition
does not make the mere offer a consummated sale. It bears to emphasize
that the offer still needs to be accepted by the DAR on behalf of the
government, and just compensation for the land determined and paid to the
landowner. The sale is deemed consummated when the landowner has
received payment or deposit by the DAR of just compensation with an
accessible bank, in cash or Landbank bonds, since only then is ownership of
the land finally transferred from the landowner to the government.
In the present case, the VOS covering the subject real properties is
still being processed by the DAR. There has so far been no express
acceptance by the DAR of the said VOS or payment of just compensation to
CPI. There being no consummated sale of the subject real properties to DAR,
CPI could not have committed a double sale of the same. It remained a co-
owner of the subject real properties, together with the other heirs of Antonio
Medina, and, thus, it could still legally sell its share or interest therein to
another person, such as respondent.

XVIII. OBLIGATIONS OF SELLER

1) TOMAS K. CHUA vs. COURT OF APPEALS and ENCARNACION VALDES-CHOY


G.R. No. 119255
April 9, 2003

FACTS: Encarnacion Valdes-Choy advertised for sale her paraphernal house and lot in
Makati. They agreed on a purchase price of P10,800,000.00. Chua gave P100,000 to
Valdes-Choy as earnest money They agreed that the balance is payable on or before 15
July 1989. Failure to pay balance on or before the said date forfeits the earnest money.
On July 13, 1989, Valdes-Choy as vendor and Chua as vendee signed two Deeds of
Absolute Sale. The first Deed of Sale covered the house and lot for the purchase price
of P8,000,000.00. The second Deed of Sale covered the furnishings, fixtures
andmovable properties contained in the house for the purchase price of P2,800,000.00. The
parties also computed the capital gains tax to amount to P485,000.00.
The next day, Valdes-Choy deposited the P485,000.00 manager's check to her account and
check to the counsel who undertook to pay the capitalgains tax. Chua showed to Valdes-
Choy a PBCom manager's check for P10,215,000.00 representing the balance of the
purchase price. Chua, however, did not give this PBCom manager's check to Valdes-Choy
because the TCT was still registered in the name of Valdes-

SALES CASE DIGEST COMPILATION 187


Choy. Chua required that the Property be registered first in his name before he would turn
over the check to Valdes-Choy. This angered Valdes-Choy who tore up the Deeds of Sale,
claiming that what Chua required was not part of their agreement. Valdes-Choy demanded
the payment of the remaining purchase balance be first deposited in her account before she
transfers the title of the property to him.
Chua filed a complaint for specific performance against Valdes-Choy.
ISSUE: Can Chua (vendee) compel Valdes-Choy (vendor) to transfer the title of the
property?
HELD:
NO. Chua’s condition that a new TCT should first be issued in his name, a found neither in
the law nor in the contract to sell as evidenced by the Receipt. Thus, at this point Chua was
not ready, able and willing to pay the full purchase price which is his obligation under the
contract to sell. Chua was also not in a position to assume the principal obligation of a
vendee in a contract of sale, which is also to pay the full purchase price at the agreed
time. Article 1582 of the Civil Code provides that –
Art. 1582. The vendee is bound to accept delivery and to pay the price of the thing sold
at the time and place stipulated in the contract.
In this case, the contract to sell stipulated that Chua should pay the balance of the purchase
price “on or before 15 July 1989.” The signed Deeds of Sale also stipulated that the buyer
shall pay the balance of the purchase price upon signing of the deeds. However, on the
agreed date, Chua refused to pay the balance of the purchase price as required by the
contract to sell, the signed Deeds of Sale, and Article 1582 of the Civil Code. Chua was
therefore in default and has only himself to blame for the rescission by Valdes-Choy of the
contract to sell.
Accordingly, since Chua refused to pay the consideration in full on the agreed date, which is
a suspensive condition, Chua cannot compel Valdes-Choy to consummate the sale of the
Property. Chua acquired no right to compel Valdes-Choy to transfer ownership of the
Property to him.

SALES CASE DIGEST COMPILATION 188


XIX. DELIVERY/SPECIAL RULES

1) Behn Meyer Vs. Yangco

38 Phil 602,606, G.R. No. 22537, December 8, 1924

 A sale of 80 drums of caustic soda was agreed between Behn, Meyer & Co. and
Teodoro Yanco. The merchandise was shipped from New York to Manila.
 However, the ship carrying the cargo was detained at Penang and the 71 of the 80
drums were removed. Respondent Yangco also refused to accept the 9 remaining and
also refused to accept the offer of Behn Meyer to have the products substituted with
other merchandise, which however were different from what was ordered.
 It must be noted that the contract provided for "c.i.f. Manila, pagadero against delivery
of documents."
 Yanco filed an action seeking for damages for alleged breach of contract.

Issue: WON Behn, Meyer & Co. should bear the burden of the loss of the merchandise?
YES

Held:

Rule as to delivery of goods by a vendor via a common carrier (If contract is silent –
delivery of seller to common carrier transfer ownership to buyer)
Determination of the place of delivery always resolves itself into a question of act. If the
contract be silent as to the person or mode by which the goods are to be sent, delivery by
the vendor to a common carrier, in the usual and ordinary course of business, transfers the
property to the vendee.

Payment of freight by the buyer = acquires ownership at the point of shipment


A specification in a contact relative to the payment of freight can be taken to indicate the
intention of the parties in regard to the place of delivery. If the buyer is to pay the freight, it is
reasonable to suppose that he does so because the goods become his at the point of
shipment.

Payment of freight by the seller = title of property does not pass until the goods have
reached their destination
On the other hand, if the seller is to pay the freight, the inference is equally so strong that the
duty of the seller is to have the goods transported to their ultimate destination and that title to
property does not pass until the goods have reached their destination.

c.i.f. means Cost, Insurance and Freight = CFI is paid by the seller
The letters "c.i.f." found in British contracts stand for cost, insurance, and freight. They
signify that the price fixed covers not only the cost of the goods, but the expense of freight
and insurance to be paid by the seller.

F.O.B. stands for Free on Board = seller bear all expenses until goods are delivered
In this case, in addition to the letters "c.i.f.," has the word following, "Manila." In mercantile
contracts of American origin the letters "F.O.B." standing for the words "Free on Board," are
frequently used. The meaning is that the seller shall bear all expenses until the goods are
delivered where they are to be "F.O.B."

SALES CASE DIGEST COMPILATION 189


According as to whether the goods are to be delivered "F.O.B." at the point of shipment or at
the point of destination determines the time when property passes. However, both the terms
"c.i.f." and "F.O.B." merely make rules of presumption which yield to proof of contrary
intention.

Delivery was to be made at Manila


Hence, we believe that the word Manila in conjunction with the letters "c.i.f." must mean that
the contract price, covering costs, insurance, and freight, signifies that delivery was to made
at Manila. If petitioner Behn Meyer has seriously thought that the place of delivery was New
York and Not Manila, it would not have gone to the trouble of making fruitless attempts to
substitute goods for the merchandise named in the contract, but would have permitted the
entire loss of the shipment to fall upon the defendant.

Behn Meyer failed to prove that it performed its part in the contract
In this case, the place of delivery was Manila and plaintiff (Behn Meyer) has not legally
excused default in delivery of the specified merchandise at that place. In resume, we find
that the plaintiff has not proved the performance on its part of the conditions precedent in the
contract.

For breach of warranty, the buyer (Yanco) may demand rescission of the contract of
sale
The warranty — the material promise — of the seller to the buyer has not been complied
with. The buyer may therefore rescind the contract of sale because of a breach in substantial
particulars going to the essence of the contract. As contemplated by article 1451 of the Civil
Code, the vendee can demand fulfillment of the contract, and this being shown to be
impossible, is relieved of his obligation. There thus being sufficient ground for rescission, the
defendant is not liable.

2) LA FUERZA, INC.
vs.
THE HON. COURT OF APPEALS and ASSOCIATED ENGINEERING CO., INC.
G.R. No. L-24069
June 28, 1968
FACTS:
The plaintiff (Associated Engineering, Co., Inc.) is a corporation engaged in the manufacture
and installation of flat belt conveyors. The defendant (La Fuerza, Inc.) is also a corporation
engaged in the manufacture of wines. Sometime in the month of January, 1960, Antonio Co,
the manager of the plaintiff corporation, who is an engineer, called the office of the
defendant told Mariano Lim, the President and general manager of the defendant that he
had just visited the defendant's plant and was impressed by its size and beauty but he
believed it needed a conveyor system to convey empty bottles from the storage room in the
plant to the bottle washers in the production room thereof.

He therefore offered his services to manufacture and install a conveyor system, which,
according to him, would increase production and efficiency of his business. Eventfully after
much consideration La Fuerza asked the plaintiff to install a conveyor system.

However, it was discovered, according to the defendant's general manager, that the
conveyor system did not function to their satisfaction as represented by the technical
manager of the plaintiff Antonio Co for the reason that, when operated several bottles
collided with each other, some jumping off the conveyor belt and were broken, causing
considerable damage.

SALES CASE DIGEST COMPILATION 190


It seems that the defects indicated by the said president and general manager of the
defendant had not been remedied so that they came to the parting of the ways with the
result that when the plaintiff billed the defendant for the balance of the contract price, the
latter refused to pay for the reason that according to the defendant the conveyor system
installed by the plaintiff did not serve the purpose for which the same was manufactured and
installed at such a heavy expense.

ISSUE:

Whether or not Associated Engineering, Co., Inc. may rescind from their contract of
installation of conveyor system from La Fuerza, Inc.

HELD:
Yes, If the thing sold has hidden faults or defects as the conveyors are claimed to have the
vendor in the case at bar, the plaintiff shall be responsible therefor and the vendee or La
Fuerza, in the present case "may elect between withdrawing from the contract
and demanding a proportional reduction of the price, with damages in either case." In the
exercise of this right of election, La Fuerza had chosen to withdraw from the contract, by
praying for its rescission.

3) Victoria Milling Co., Inc. v. CA and Consolidated Sugar Corporation


G.R. No. 117356 June 19, 2000

FACTS:
 St. Therese Merchandising regularly bought sugar from Victorias Milling Co., Inc. In the
course of their dealings, Victorias Milling issued several Shipping List/Delivery Receipts
(SLDRs) to St. Therese Merchandising as proof of purchases. Among these was SLDR
No. 1214M which covers 25,000 bags of sugar. Each bag contained 50 kilograms and
priced at P638.00 per bag. The transaction it covered was a direct sale.
 On October 25, 1989, St. Therese Merchandising sold to Consolidated Sugar Corp. its
rights in SLDR No. 1214M for P14,750,000.00. Consolidated Sugar Corp. issued checks in
payment. That same day, Consolidated Sugar Corp. wrote Victorias Milling that it had
been authorized by St. Therese Merchandising to withdraw the sugar covered by SLDR
No. 1214M.
 Consolidated Sugar Corp. surrendered SLDR No. 1214M to Victorias Milling’s NAWACO
warehouse and was allowed to withdraw sugar. However, after 2,000 bags had been
released, Victorias Milling refused to allow further withdrawals of sugar against SLDR No.
1214M because, according to it, St. Therese Merchandising had already withdrawn all the
sugar covered by the cleared checks.

ISSUE: WON the contract was one of agency or sale

HELD: Sale.
 Victorias Milling heavily relies upon St. Therese Merchandising’s letter of authority allowing
Consolidated Sugar Corp. to withdraw sugar against SLDR No. 1214M to show that the
latter was St. Therese Merchandising’s agent. The pertinent portion of said letter reads:
“This is to authorize Consolidated Sugar Corporation or its representative to withdraw for
and in our behalf (stress supplied) the refined sugar covered by Shipping List/Delivery
Receipt = Refined Sugar (SDR) No. 1214 dated October 16, 1989 in the total quantity of 25,
000 bags.”
 Art. 1868. By the contract of agency a person binds himself to render some service or to do
something in representation or on behalf of another, with the consent or authority of the
latter.

SALES CASE DIGEST COMPILATION 191


 The basis of agency is representation. On the part of the principal, there must be an actual
intention to appoint or an intention naturally inferable from his words or actions; and on the
part of the agent, there must be an intention to accept the appointment and act on it, and in
the absence of such intent, there is generally no agency. One factor which most clearly
distinguishes agency from other legal concepts is control; one person - the agent - agrees to
act under the control or direction of another - the principal.
 Victorias Milling failed to sufficiently establish the existence of an agency relation between
Consolidated Sugar Corp. and St. Therese Merchandising. The fact alone that it (St.
Therese Merchandising) had authorized withdrawal of sugar by Consolidated Sugar Corp.
“for and in our (St. Therese Merchandising’s) behalf” should not be eyed as pointing to the
existence of an agency relation. Further, Consolidated Sugar Corp. has shown that the
25,000 bags of sugar covered by the SLDR No. 1214M were sold and transferred by St.
Therese Merchandising to it. A conclusion that there was a valid sale and transfer to
Consolidated Sugar Corp. may, therefore, be made thus capacitating Consolidated Sugar
Corp. to sue in its own name, without need of joining its imputed principal St. Therese
Merchandising as co-plaintiff.
 Consolidated Sugar Corp. was a buyer of the SLDFR form, and not an agent of STM.
Consolidated Sugar Corp. was not subject to St. Therese Merchandising’s control. That no
agency was meant to be established by the Consolidated Sugar Corp. and STM is clearly
shown by Consolidated Sugar Corp.’s communication to petitioner that SLDR No. 1214M
had been “sold and endorsed” to it.27 The use of the words “sold and endorsed” means that
St. Therese Merchandising and Consolidated Sugar Corp. intended a contract of sale, and
not an agency.

4) ALLIANCE TOBACCO CORPORATION, INC., petitioner


PHILIPPINE VIRGINIA TOBACCO ADNMINISTRATION (PVTA), FARMER’S VIRGINIA
TOBACCO REDRYING COMPANY, INC. AND INTERMEDIATE APELLATE COURT,
respondents
FACTS:
PVTA, a government corporation entered into a contract of procuring, redrying and servicing
with FVTR for the 1963 tobacco trading operation. In June of the same year, PVTA also
entered into a merchandising loan agreement with the petitioner, a duly incorporated and
authorized tobacco trading entity, whereby PVTA agreed to lend P25,500 to petitioner for
purchase of fluecured Virgina tobacco from bona fide Virginia tobacco farmer-producers.
Following month, petitioner shipped to FVTR 96 bales of tobacco (4,800 kilos) covered by
Guia No. 1 and 167 bales (8,350 kilos) covered by Guia No. 2 to the Redrying plant. Only 89
bales from Guia No. 2 were graded, weighed and accepted by FVTR, the remaining 174
bales were not graded and weighed because some officer and employee of FVTR asked
money for separate weighing and grading of the remaining bales.
The operations of FVTR in Bauang stopped in October 1963. Plaintiff asked that its
ungraded and unweighed tobacco be withdrawn from Redrying plant. PVTA and FVTR
refused because according to the, those tobacco were subject to merchandising loaned and
owned by PVTA.
Unfortunately, the remaining 174 bales with total value of P28,382 were lost while in the
possession of FVTR. Having learned of such loss, petitioner demanded for its value and
application of the same to its merchandising loan with PVTA but both the latter and the
FVTR refused to heed said demands.

ISSUE:
WON Petitioner’s delivery of 174 bales of Tobacco to FVTR, a contractee of PVTA perfected
the contract of sale between petitioner and PVTA so as to hold the latter liable for the loss of
the said bales while in possession of FVTR.
HELD:

SALES CASE DIGEST COMPILATION 192


The court first denied the petition for lack of merit. But was set aside upon petitioner’s motion
for reconsideration.
While under an Ideal situation, PVTA's contention that the contract of sale could not have
been perfected pursuant to Article 1475 22 of the Civil Code because to determine the price
of the tobacco traded, the shipment should first be inspected, graded and weighed has
merit, said contention misplaced herein. A strict interpretation of the provision of Article 1475
may result in adverse effects to small planters who would not be paid for the lost products of
their toil.

Equity and fair dealing, the anchor of said case, must once more prevail. Since PVTA had
virtual control over the lost tobacco bales, delivery thereof to the FVTR should also be
considered effective delivery to the PVTA.

5) EQUATORIAL REALTY V. MAYFAIR (November 21, 1996)

FACTS:
Petitioner Carmelo and Bauermann Inc. leased its parcel of land with 2-storey building to
respondent Mayfair Theater Inc.
They entered a contract which provides that if the LESSOR should desire to sell the leased
premises, the LESSEE shall be given 30-days exclusive option to purchase the same.

Carmelo informed Mayfair that it will sell the property to Equatorial. Mayfair made known its
interest to buy the property but only to the extent of the leased premises.
Notwithstanding Mayfair’s intention, Carmelo sold the property to Equatorial.

ISSUE:
WON the sale of the property to Equatorial is valid.

HELD:
The sale of the property should be rescinded because Mayfair has the right of first refusal.
Both Equatorial and Carmelo are in bad faith because they knew of the stipulation in the
contract regarding the right of first refusal.

The stipulation is a not an option contract but a right of first refusal and as such the
requirement of a separate consideration for the option, has no applicability in the instant
case. The consideration is built in the reciprocal obligation of the parties.

In reciprocal contract, the obligation or promise of each party is the consideration for that of
the other. (Promise to lease in return of the right to first refusal)

With regard to the impossibility of performance, only Carmelo can be blamed for not
including the entire property in the right of first refusal. Court held that Mayfair may not have
the option to buy the property. Not only the leased area but the entire property.

6) MASICLAT vs. CENTENO (1956 )

FACTS: Defendant-respondent Centeno owned 15 sacks of rice offered for sale at her store
situated on a street near public market. In the morning of Jan. 21, 1951, a person
approached defendant and offered her to purchase the rice in question. Defendant agreed to
sell 15 sacks of rice in question at P26/ sack, which the buyer promised to pay as soon as
he would receive the price of his adobe stones which were being then unloaded from a truck
owned Francisco Tan, then parked at the opposite side of the street in front of the Union

SALES CASE DIGEST COMPILATION 193


Grocery facing the defendant’s store. Relying on this promise and upon the request of said
purchaser, the defendant ordered the rice in question loaded in the said truck, of which the
plaintiff was the caretaker, expecting that as soon as the adobe stones would be paid, said
purchaser would pay her the price of the rice. While the rice was being loaded on the truck
and even thereafter, defendant kept an eye on it waiting for the purchaser to come to pay
her. When the adobe stones were completely unloaded from the truck, the defendant looked
for the purchaser, but the latter was no found. So defendant decided to unload the rice from
the truck but to her surprise plaintiff-petitioner Masiclat objected on the ground that he has
bought it at P26/sack from a person whom he did not know and whom he met only that
morning for the first time. Defendant insisted in unloading the rice and the plaintiff objected.
Hence, defendant called a policeman to investigate the matter and the latter brought the rice
in question to the Municipal building where it was deposited pending investigation. Plaintiff
then initiated this action for recovery of possession of the rice in question

ISSUES 1. WON the contract of sale was consummated between respondent Centeno and
unknown purchaser (alleged unknown seller to plaintiff)
2. WON petitioners have a better title to the rice in question

HELD

A. NO
1) The evidence does not clearly show the identity of the person who tried to buy the rice in
question from the respondent, and neither does it show that the same person was the one
who sold the commodity to Ramon Masiclat.

2) The sale between the respondent Centeno and the unknown purchaser was not
consummated because although the former allowed the rice in question to be loaded in the
truck, she did not intend to transfer its ownership until she was paid the stipulated price; and
this is very evident from the fact that respondent continually watched her rice and demanded
its unloading as soon as the unknown purchaser was missing. Respondent thus has not lost
ownership and legal possession thereof.

B. NO The general principle of law as enunciated in A1505 CC that where one of 2 persons
must suffer the fraud of a third, the loss should fall upon him who has enabled the third
person to do the wrong, does not apply for the ff. reasons: 1) there was no definite finding
that the unknown purchaser was same person who sold the rice to Masiclat, 2) Centeno
could not have been so negligent as to allow the unknown purchaser to run away with said
rice and enable him to sell it to Masiclat, it evident that in fact Centeno kept an eye on
the rice in question.

Ratio: Although a contract of sale is perfected upon the parties having agreed as to the
thing which is the subject matter of the contract and the price, ownership is not considered
transmitted until the property is actually delivered and the purchaser has taken possession
and paid the price agreed upon.

7) Manuel Dulay Enterprises, Inc. v. CA, Torres [Aug 27, 1993]

Facts:
Manuel R. Dulay Enterprises, Inc, a domestic corporation obtained various loans for the
construction of its hotel project, Dulay Continental Hotel (now Frederick Hotel).
Manuel Dulay by virtue of Board Resolution No 18 sold the subject property to spouses
Maria Theresa and Castrense Veloso.

SALES CASE DIGEST COMPILATION 194


Maria Veloso (buyer), without the knowledge of Manuel Dulay, mortgaged the subject
property to private respondent Manuel A. Torres. #fluffypeaches Upon the failure of Maria
Veloso to pay Torres, the property was sold to Torres in an extrajudicial foreclosure sale.
Torres filed an action against the corporation, Virgilio Dulay and against the tenants of
the apartment.
RTC ordered the corporation and the tenants to vacate the building.
Petitioners: RTC had acted with GAD when it applied the doctrine of piercing the veil
of corporate entity considering that the sale has no binding effect on corporation as Board
Resolution No. 18 which authorized the sale of the subject property was resolved without
the approval of all the members of the board of directors and said Board Resolution was
prepared by a person not designated by the corporation to be its secretary.

Issue:
WON the sale to Veloso is valid notwithstanding that it was resolved without the approval
of all the members of the board of directors. (YES)

Held:
Section 101 of the Corporation Code of the Philippines provides:
Sec. 101. When board meeting is unnecessary or improperly held. Unless the by-laws
provide otherwise, any action by the directors of a close corporation without a meeting shall
nevertheless be deemed valid if:
1. Before or after such action is taken, written consent thereto is signed by all the directors,
or
2. All the stockholders have actual or implied knowledge of the action and make no prompt
objection thereto in writing; or
3. The directors are accustomed to take informal action with the express or implied acquiese
of all the stockholders, or
4. All the directors have express or implied knowledge of the action in question and none of
them makes prompt objection thereto in writing.
If a directors' meeting is held without call or notice, an action taken therein within the
corporate powers is deemed ratified by a director who failed to attend, unless he promptly
files his written objection with the secretary of the corporation after having knowledge
thereof.
Dulay Inc. is classified as a close corporation and consequently a board resolution
authorizing the sale or mortgage is not necessary to bind the corporation for the action of
its president. #fluffypeaches At any rate, corporate actiontaken at a board meeting without
proper call or notice in a close corporation is deemed ratified by the absent director
unless the latter promptly files his written objection with the secretary of the
corporation after having knowledge of the meeting which, in his case, Virgilio Dulay failed
to do.

Although a corporation is an entity which has a personality distinct and separate from its
individual stockholders or members, the veil of corporate fiction may be pierced when it
is used to defeat public convenience justify wrong, protect fraud or defend crime.

8) Balatbat v. C.A.
G.R. No. 109410
August 28, 1996

FACTS:

The lot in question covered by Transfer Certificate of Title No. 51330 was acquired
by plaintiff Aurelio Roque and Maria Mesina during their conjugal union and the house

SALES CASE DIGEST COMPILATION 195


constructed thereon was likewise built during their marital union. Out of their union, plaintiff
and Maria Mesina had four children. When Maria Mesina died on August 28, 1966, the only
conjugal properties left are the house and lot above stated of which plaintiff herein, as the
legal spouse, is entitled to one-half share pro-indiviso thereof. With respect to the one-half
share pro-indiviso now forming the estate of Maria Mesina, plaintiff and the four children, the
defendants here, are each entitled to one-fifth (1/5) share pro-indiviso.

Aurelio Roque then entered into a contract of Absolute Sale with the spouses Aurora
and Jose Repuyan. However, on August 20, 1980, Aurelio filed a complaint for Rescission of
Contract against Spouses Repuyan for the latter’s failure to pay the balance of the purchase
price. A deed of absolute sale was then executed on February 4, 1982 between Aurelio S.
Roque, Corazon Roque, Feliciano Roque, Severa Roque and Osmundo Roque and Clara
Balatbat, married to Alejandro Balatbat. On April 14, 1982, Clara Balatbat filed a motion for
the issuance of a writ of possession which was granted by the trial court on September 14,
1982 "subject, however, to valid rights and interest of third persons over the same portion
thereof, other than vendor or any other person or persons privy to or claiming any rights or
interests under it." The corresponding writ of possession was issued on September 20,
1982.

The lower court then rendered judgment in favor of the Spouses Repuyan and
declared the Deed of Absolute Sale as valid. On appeal by petitioner Balatbat, the Court of
Appeals affirmed the lower court’s decision.

ISSUE:

Whether or not the delivery of the owner’s certificate of title to spouses Repuyan by
Aurelio Roque is for convenience or for validity or enforceability.

HELD:

The Supreme Court found that the sale between Aurelio and the Spouses Repuyan
is not merely for the reason that there was no delivery of the subject property and that
consideration/price was not fully paid but the sale as consummated, hence, valid and
enforceable.

The non-delivery of the possession of the subject property to the private respondent,
suffice it to say that ownership of the thing sold is acquired only from the time of delivery
thereof, either actual or constructive. Article 1498 of the Civil Code provides that when the
sale is made through a public instrument, the execution thereof shall be equivalent to the
delivery of the thing which is the object of the contract, if from the deed the contrary does not
appear or cannot be inferred. The execution of the public instrument, without actual delivery
of the thing, transfers the ownership from the vendor to the vendee, who may thereafter
exercise the rights of an owner over the same. In the instant case, vendor Roque delivered
the owner's certificate of title to herein private respondent. It is not necessary that vendee be
physically present at every square inch of the land bought by him, possession of the public
instrument of the land is sufficient to accord him the rights of ownership. Thus, delivery of a
parcel of land may be done by placing the vendee in control and possession of the land
(real) or by embodying the sale in a public instrument (constructive). The provision of Article
1358 on the necessity of a public document is only for convenience, not for validity or
enforceability. It is not a requirement for the validity of a contract of sale of a parcel of land
that this be embodied in a public instrument.

A contract of sale being consensual, it is perfected by the mere consent of the


parties. Delivery of the thing bought or payment of the price is not necessary for the
perfection of the contract; and failure of the vendee to pay the price after the execution of the

SALES CASE DIGEST COMPILATION 196


contract does not make the sale null and void for lack of consideration but results at most in
default on the part of the vendee, for which the vendor may exercise his legal remedies.
Tthe petition for review is hereby dismissed for lack of merit.

9) CAOIBES, JR., et al. vs. CAOIBES-PANTOJA Case Digest


JOSE CAOIBES, JR., et al. v. CORAZON CAOIBES-PANTOJA
496 SCRA 273 (2006), THIRD DIVISION (Carpio Morales J.)

FACTS: In 1982, Jose Caoibes Jr., et al. (Caoibes, Jr., et al.) and Corazon Caoibes-Pantoja
(Pantoja) entered to a contract of sale stating that a certain lot will be transferred, ceded and
conveyed by the former in favour of the latter in consideration for a sum of money. The
agreement included the stipulation that Pantoja will be subrogated or substituted to whatever
rights, interests or representations Caoibes Jr., et al. may have pending land registration
proceeding.

Fourteen years after the execution of the parties, Pantoja filed a motion to intervene and
be substituted as applicant in the Land Registration Court. The Land Registration Court
denied the motion. Pantoja filed a complaint before the Regional Trial Court (RTC) for
specific performance of the agreement. Caoibes, Jr., et al. opposed on the grounds of
prescription. The RTC ruled in favor of Caoibes, Jr., et al. On appeal, the Court of Appeals
(CA) reversed the RTC, holding that prescription had not yet set in.

ISSUE: Whether or not the action of for prescription on Pantoja started from the time of the
agreement of the parties

Held: The law does not require that the application for registration be amended by
substituting the "buyer" or the "person to whom the property has been conveyed" for the
applicant. Neither does it require that the "buyer" or the "person to whom the property has
been conveyed" be a party to the case.

He may thus be a total stranger to the land registration proceedings. The only requir
ements of the law are: (1) that the instrument be presented to the court by the interested
party together with a motion that the same be considered in relation with the application;
and (2) that prior notice be given to the parties to the case.

The agreement of the parties is analogous to a deed of sale in favour of Pantoja, it


having transferred ownership for and in consideration of her payment of the loan.. The
agreement having been made through public instrument, the execution was equivalent to the
delivery of the property to Pantoja.

The agreement is of course in consonance with Sec. 22 of P.D. 1529 (Property Registration
Decree which became effective on June 11, 1978). In light of the law and jurisprudence, the
substitution by Pantoja of Caoibes, Jr., et al. as applicant in the land registration case over
Lot 2 is not even necessary. All Pantoja has to do is to comply with the requirements under
the above-quoted Sec. 22 of the Property Registration Decree. It was unnecessary for
Pantoja to file the case for specific performance subject of the present petition against
Caoibes, Jr., et al. to honor their agreement allowing her to be substituted in their stead as
applicant in the land registration proceeding.

SALES CASE DIGEST COMPILATION 197


BANDILA
10) G.R. No. 80201. November 20, 1990
ANTONIO GARCIA, JR., Petitioner, v. COURT OF APPEALS, LASAL DEVELOPMENT
CORPORATION, Respondents.

Facts: On April 15, 1977, the Western Minolco Corporation (WMC) obtain from the
Philippine Investments System Organization two loans for Php 2,500,000.00 and Php
1,000,000.00 for which it issued the corresponding promissory note payables on May
30,1977. On the same date, Antonio Garcia and Ernest Kahn executed a surety agreement
binding themselves jointly and severally for the payment of the said loan.

Upon failure of WMC to pay after repeated demands, demand was made on Garcia pursuant
to the surety agreement. Garcia also failed to pay. Hence, on April 5, 1983, Lasal
Development Corporation (to which the credit had been assigned earlier by PISO) sued
Garcia for recovery of the debt in the RTC Makati.
On May 18, 1983, Garcia moved to dismiss on the grounds that: (a) the complaint stated no
cause of action; (b) the suit would result in unjust enrichment of the plaintiff because he had
not received any consideration from PISO; (c) the surety agreement violated the doctrine of
the limited liability of corporations; and (d) the principal obligation had been novated.

Issue: Whether or not there was a delivery of consideration in the case at bar.
Held: Yes. The peculiar nature of a surety agreement is that it is regarded as valid despite
the absence of any direct consideration received by the surety either from the principal
obligor or from the creditor. A contract of surety, like any other contract, must generally be
supported by a sufficient consideration. However, the consideration necessary to support a
surety obligation need not pass directly to the surety; a consideration moving to the principal
alone will suffice.
It has been held that if the delivery of the original contract is contemporaneous with the
delivery of the surety’s obligation, each contract becomes completed at the same time, and
the consideration which supports the principal contract likewise supports the subsidiary one.

11) G.R. No. 98382 May 17, 1993


PHILIPPINE NATIONAL BANK, petitioner, vs.THE COURT OF APPEALS and EPIFANIO
DE LA CRUZ, respondents.
Facts: Two parcels of land under the common names of the respondent Epifanio dela
Cruz, his brother and sister were mortgaged to the Petitioner Philippine National Bank. The
lots were mortgaged to guarantee the by three promissory notes. The first two were not paid
by the respondent. The third is disputed by the respondent who claims that the correct date
is June 30, 1961;however, in the bank records, the note was really executed on June 30,
1958.
PNB presented under Act No. 3135 a foreclosure petition of the mortgaged lots. The
lots were sold or auctioned off with PNB as the highest bidder. A Final Deed of Sale and a
Certificate of Sale was executed in favor of the petitioner. The final Deed of Sale was
registered in Registry of Property. Inasmuch as the respondent did not buy back the lots
from PNB, PNB sold on the same in a "Deed of Conditional Sale". The Notices of Sale of
foreclosed properties were published on March 28, April 11 and April 12, 1969 in a
newspaper.
Respondent brought a complaint for the reconveyance of the lands, which the
petitioner allegedly unlawfully foreclosed. The petitioner states on the other hand that the
extrajudicial foreclosure, consolidation of ownership, and subsequent sale were all valid
The CFI rendered its Decision; the complaint against the petitioner was dismissed.
Unsatisfied with the judgment, respondent interposed an appeal that the lower court erred in

SALES CASE DIGEST COMPILATION 198


holding that there was a valid compliance in regard to the required publication under Sec. 3
of Act. 3135. Respondent court reversed the judgment appealed from by declaring void, inter
alia, the auction sale of the foreclosed pieces of realty, the final deed of sale, and the
consolidation of ownership. Hence, the petition with SC for certiorari and intervention.
Issue: Whether or not the required publication of The Notices of Sale on the
foreclosed properties under Sec. 3 of Act 3135 was complied.
Held: No. The first date falls on a Friday while the second and third dates are on a
Friday and Saturday, respectively. Section 3 of Act No. 3135 requires that the notice of
auction sale shall be "published once a week for at least three consecutive weeks".
Evidently, petitioner bank failed to comply with this legal requirement. The Supreme Court
held that: The rule is that statutory provisions governing publication of notice of mortgage
foreclosure sales must be strictly complied with, and those even slight deviations there from
will invalidate the notice and render the sale at least voidable.

12) G.R.No.L-2684.March15,1907.
THE FIDELITY AND DEPOSIT COMPANY OF MARYLAND, Plaintiff-Appellant, v.
WILLIAMWILSON,ETAL., Defendants-Appellees.
Facts: On October 1st1902, defendant Wilson was a disbursing officer of the Bureau
of Coast Guard and Transportation. The plaintiff company and another company named
American Surety Company of New York became sureties on the official bond of Wilson for
he sum of $15,000.00. Wilson defaulted the sum of $8,931.80 and the two surety companies
were compelled to pay to the government after the government’s demand. They paid the
sum of $4,465.90 each. Wilson was captured in Montreal, Canada for the trial of his
defalcation of money. When Wilson was captured he had the sum of $785 with him, which
was turned over to Mr. Branagan whom is the insular treasurer. Petitioner filed a petition
against Wilson for the sum of $4,464.90 which is the same amount the petitioner paid to the
government under Wilson’s surety bond and to have the right over the sum of $785 which
was found in the possession of Wilson. On October 17, a depositary was named by the court
to take care of the $785 of Wilson.
On October 26, 1904, H.D. Terrell intervened in the case. He alleges that Wilson had
already transferred his rights of the $785 to him for payment of professional services that
Wilson owes him. Terrell claims he has notified Mr. Bragan prior to his appointment as
depositary in the principal case. The court ruled in favor of Terrell for the sum of $785hence
this review.
ISSUE: Whether or not there was a valid transfer of ownership of the funds to Terrell.
Held: No. the transfer by itself, and afterwards the notification of the same of Treasurer
Branagan, did not produce nor could it produce the effect of transfer to Terrell of the
ownership of the funds so transferred and which were then in the possession of the said
Treasurer. To have this effect, it would have been necessary that the delivery of the funds
had been made directly Terrell, which fact has not been proved at any time.
In the case at bar, The funds were in the possession of Branagan and afterwards were
transferred to the possession of the depositary appointed, by the court where such funds
now are, and this without their ever having been taken possession of the intervenor Terrell. It
is not alleged, nor it is claimed by Terrell, that the delivery of the funds was ever made in any
manner recognized by the law. He claims the right of ownership from the mere fact of having
derived the same, not from the fact of any delivery, but from the very fact of the transfer and
of his subsequent notification to Treasurer Branagan, it being, in addition, very clear that
such notification does not constitute, in any manner, the fact of delivery as established by
articles 1462, 1463, and 1464 of the Civil Code, all of which cover, in full this subject-matter.

SALES CASE DIGEST COMPILATION 199


13) G.R. No. 119745. June 20, 1997
POWERCOMMERCIAL AND INDUSTRIACORPORATION, petitioner, vs. COURT OF
APPEALS, SPOUSES REYNALDO and ANGELITA R. QUIAMBAO and PHILIPPINE
NATIONAL BANK, respondents.

Facts: Petitioner is an industrial asbestos manufacturer, they entered into a contract


of sale with the private respondents. The contract involved a 612-sq. m. parcel of land
located at the Makati City. The parties agreed that petitioner would pay private respondents
P108,000.00 as down payment, and the balance of P295,000.00 upon the execution of the
deed of transfer of the title over the property. Further, petitioner assumed, as part of the
purchase price, the existing mortgage on the land. In full satisfaction thereof, he paid P
79,145.77 to Respondent Philippine National Bank.
Respondent spouses mortgaged again said land to PNB to guarantee a loan of
P145,000.00, P80,000.00 of which was paid to respondent spouses. Petitioner agreed to
assume payment of the loan. Subsequently, the parties executed a Deed of Absolute Sale
with Assumption of Mortgage for some terms and condition. PNB informed respondent
spouses that, for petitioners failure to submit the papers necessary for approval pursuant to
the formers letter dated January 15, 1980, the application for assumption of mortgage was
considered withdrawn; that the outstanding balance of P145,000.00 was deemed fully due
and demandable; and that said loan was to be paid in full within fifteen (15) days from
notice. Petitioner paid PNB P41,880.45 on June 24, 1980 and P20,283.14 on December 23,
1980, payments which were to be applied to the outstanding loan.
Petitioner filed an action for recession against respondent spouses and they also demanded
the return of the payments it made on the ground that its assumption of mortgage was never
approved. On May 31, 1983, while this case was pending, the mortgage was foreclosed.
The property was subsequently bought by PNB during the public auction.
The trial court ruled that the failure of respondent spouses to deliver actual possession to
petitioner entitled the latter to rescind the sale, and in view of such failure and of the denial
of the latters assumption of mortgage, PNB was obliged to return the payments made by the
latter. CA reversed the decision. It held that the deed of sale between respondent spouses
and petitioner did not obligate the former to eject the lessees from the land in question as a
condition of the sale, nor was the occupation thereof by said lessees a violation of the
warranty against eviction. Hence, there was no substantial breach to justify the rescission of
said contract or the return of the payments made.
Issue: Whether or not delivery was effected through the execution of the Deed of
Absolute Sale.
Held: Yes. Under Article 1498 par (1), When the sale is made through a public
instrument, the execution thereof shall be equivalent to the delivery of the thing which is the
object of the contract. In the case at bar, Considering that the deed of sale between the
parties did not stipulate or infer otherwise, delivery was effected through the execution of
said deed. The lot sold had been placed under
the control of petitioner. Prior physical delivery or possession is not legally required and the
execution of the deed of sale is deemed equivalent to delivery. This deed operates as a
formal or symbolic delivery of the property sold and authorizes the buyer to use the
document as proof of ownership.
.

14) G.R. No. 151322 October 11, 2006


MARIO L. COPUYOC, petitioner, vs. ERLINDA DE SOLA, respondent
Facts: Petitioner and his spouse are holders of a Contract to Sell dated September 6,
1995, between them as buyers and the Bank of Commerce (formerly The Overseas Bank of
Manila) as seller, while Erlinda de Sola (respondent) has Transfer Certificate of Title (TCT)
No. 87569 in her name.The complaint contained the allegations that, respondent is the
owner of a parcel of land known as Lot 25, Block 6 of the Xavierville Estate Subdivision
located at No. 6 Guerrero St., Xavierville I, Loyola Heights, Quezon City, by virtue of TCT

SALES CASE DIGEST COMPILATION 200


No. 87569; respondent has been in actual possession of the property since June 7, 1993,
when the same was sold to her by Christine C. Quesada, as evidenced by an Absolute Deed
of Sale of Real Estate; petitioner commenced construction of a house on the property
without respondent’s consent and despite demands, petitioner refused to stop the
construction and vacate the premises.
Denying respondent’s allegations and asserting that: he is the lawful possessor of
the property, by virtue of a Contract to Sell executed on September 6, 1995 in his favor by
the Bank of Commerce; respondent’s title is forged and the property described therein is
located in TandangSora, not Xavierville; the Bank of Commerce is a holder of duly
reconstituted title [TCT No. RT-114371 (265907)]** covering the subject property, including
other properties in Phase I of the Xavierville Subdivision, and had possessed the same for
43 years; a syndicate was able to procure forged titles after the office of the Register of
Deeds was burned during the fire that gutted the Quezon City Hall.
Issue: Whether or not petitioner has the right over the property.
Held: Yes. Though the Absolute Deed of Sale of Real Estate between respondent
and Christine Quesada was executed on June 7, 1993, prior to the Contract to Sell between
petitioner and the Bank of Commerce on September 6, 1995, it should be emphasized that
the execution of a deed of sale is merely a prima faciepresumption of delivery of possession
of a piece of real property, which is destroyed when the delivery is not effected because of a
legal impediment. Said constructive or symbolic delivery, being merely presumptive, may be
negated by the failure of the vendee to take actual possession of the land sold, as in
respondent’s case. In the case at bar, records show that respondent never occupied the
property from the time it was allegedly sold to her on June 7, 1993 or at any time thereafter.
On the other hand, petitioner was able to establish his actual physical possession over the
property, having started construction thereon soon after the property was turned over to him
by virtue of the Contract to Sell on September 6, 1995.

15) 38 Phil 404 August 1918


Addison vs. Felix
Facts: The defendants-appellees spouses Maciana Felix and BalbinoTioco
purchased from plaintiff-appellant A.A. Addison four parcels of land to which Felix paid, at
the time of the execution of the deed, the sum of P3,000 on account of the purchase price.
She likewise bound herself to the remainder in installments, the first of P,2000 on July 15,
1914, the second of P5,000 thirty days after the issuance to her of a certificate of title under
the Land Registration Act, and further, within ten years from the date of such title, P10 for
each cocoanut tree in bearing and P5 for each such tree not in bearing that might be
growing on said parcels of land on the date of the issuance of title to her, with the condition
that the total price should not exceed P85,000. It was further stipulated that Felix was to
deliver to the Addison 25% of the value of the products that she might obtain from the four
parcels "from the moment she takes possession of them until the Torrens certificate of title
be issued in her favor," and that within 1 year from the date of the certificate of title in her
favor, Marciana Felix may rescind the contract of purchase and sale.
In January 1915, Addison , filed suit in the CFI of Manila to compel Felix to pay the first
installment of P2,000, demandable, in accordance with the terms of the contract of sale. The
defendants Felix and her husband Tioco contended that Addison had absolutely failed to
deliver the lands that were the subject matter of the sale, notwithstanding the demands they
made upon him for this purpose. The evidence adduced shows Addison was able to
designate only two of the four parcels, and more than two-thirds of these two were found to
be in the possession of one Juan Villafuerte, who claimed to be the owner of the parts he so
occupied. The trial court held the contract of sale to be rescinded and ordered Addison to
return to Felix the P3,000 paid on account of the price, together with interest thereon at the
rate of 10% per annum.
Issue: Was there a delivery made and, therefore, a transfer of ownership of the thing
sold.

SALES CASE DIGEST COMPILATION 201


Held: The thing is considered to be delivered when it is placed "in the hands and
possession of the vendee." It is true that the same article declares that the execution of a
public instrument is equivalent to the delivery of the thing which is the object of the contract,
but, in order that this symbolic delivery may produce the effect of tradition, it is necessary
that the vendor shall have had such control over the thing sold that, at the moment of the
sale, its material delivery could have been made. Symbolic delivery through the execution of
a public instrument is sufficient when there is no impediment whatever to prevent the thing
sold passing into the tenancy of the purchaser by the sole will of the vendor. But if,
notwithstanding the execution of the instrument, the purchaser cannot have the enjoyment
and material tenancy of the thing and make use of it himself or through another in his name,
because such are opposed by a third person’s will, then the delivery has not been effected.
In the case at bar, therefore, it is evident, that the mere execution of the instrument was not
a fulfillment of the vendor's obligation to deliver the thing sold, and that from such non-
fulfillment arises the purchaser's right to demand, as she has demanded, the rescission of
the sale and the return of the price.

16) G.R. No. L-38085 November 13, 1933


ANGELA MONTENEGRO, plaintiff-appellee, vs.CONSUELO ROXAS DE GOMEZ, ET
AL., defendant-appellants
Facts: The defendant Consuelo Roxas de Gomez was and is the present owner of
the property known at the "Paris Building" situated at Nos. 26 to 36 Escolta, Manila. It was
leased by B.A. Green but his contract of leased expired on May 31, 1930. On June 5th of the
same year, the plaintiff and the defendant, represented by her attorney in fact Manuel
Martinez Llanos, executed the building in question to the plaintiff for the period of three (3)
years from June 1, 1930, at a monthly rental of P4,000, payable at the end of every calendar
month. At that time the building was being occupied by various tenants among whom were
B.A. Green, G.C. Sellner and Sta. Ana Subdivision of the Manila Improvement Co., Inc., of
which said B.A. Green was the president who, for a long time had leased and occupied the
rooms of the upper floor facing the Escolta, and a small room between the lavatory and the
main stairway. The defendant's attorney in fact should have delivered to the said lessee until
the 20th of the said month. On this last date, however, the whole building was not delivered
to the plaintiff herein because B.A. Green, G.C. Sellner and Manila Improvement Co., Inc.,
stubbornly opposed the delivery of the rooms they were then occupying.
Considering Green's attitude as a disturbing factor, the plaintiff herein instituted ejectment
proceedings against him in the municipal court of the City of Manila. She likewise instituted
similar actions against other tenants thereof. Notwithstanding all these difficulties, the
plaintiff herein, desiring perhaps not to violate the terms of her contract of lease, continued
paying the monthly rental of P4,000 corresponding to the months of June, 1930, to October,
1931, inclusive, to the defendants herein was filed did she fail to pay the rent corresponding
to the months of November and December, 1931, amounting to P8,000. The defendant
herein, or rather her attorney in fact, also instituted judicial proceedings against the plaintiff
herein for the purpose of ejecting her from the building in question and recovering the rents
due and unpaid.
Issue: Whether or not the whole building under lease was delivered to the lessee
Held: The records show that the greater portion of the building in question was
delivered to and received by the lessee not later than June 20, 1930, but that the rooms or
offices occupied by B.A. Green, G.C. Sellner and Manila Improvement Co., Inc., have never
been placed in the possession of the said lessee. If this is true, it is evident that the plaintiff
herein has the right to the remedy of rescission prayed for in her complaint in accordance
with the express provisions of article 1556 of the Civil Code which grants to the lessee,
among other things, the right to rescind the contract of lease when he is not placed in the
material possession of the property which is the subject matter of the lease.
The appellants, however, contend that the execution of the contract is equivalent to delivery
of the possession thereof. This would be true if the records did not show that in reality the

SALES CASE DIGEST COMPILATION 202


lessee did not obtain the material possession of the whole building. It should be noted that
the Civil Code does not provide that the execution of the deed is a conclusive presumption
of the delivery of possession, but confines itself to providing that the execution thereof shall
be equivalent to delivery, which means that the disputable presumption established therein
can be rebutted by means of clear and convincing evidence, as in the case under
consideration.

17) G.R. No. L-32811 March 31, 1980


FELIPE C. ROQUE, petitioner, vs. NICANOR LAPUZ and THE COURT OF APPEALS,
respondents
Facts: plaintiff and defendant entered into an agreement of sale covering Lots 1, 2 and 9,
Block 1, of said property, with an aggregate area of 1,200 square meters, payable in 120
equal monthly installments at the rate of P16.00, P15.00 per square meter, respectively.
Defendant proposed to plaintiff modification of their previous contract to sell because he
found it quite difficult to pay the monthly installments on the three lots, and besides the two
lots he had chosen were better lots, being corner lots. In addition, it was agreed that the
purchase price of these two lots would be at the uniform rate of P17.00 per square (meter)
payable in 120 equal monthly installments, with interest at 8% annually on the balance
unpaid. Pursuant to this new agreement, defendant occupied and possessed Lots 4 and 12,
Block 2 of the approved subdivision plan, and enclosed them, including the portion where his
house now stands, with barbed wires and adobe walls.
Plaintiff demanded upon defendant not only to pay the stipulated monthly installments in
arrears, but also to make up-to-date his payments, but defendant, instead of complying with
the demands, kept on asking for extensions, promising at first that he would pay not only the
installments in arrears but also make up-to-date his payment, but later on refused altogether
to comply with plaintiff's demands. Defendant was likewise requested by the plaintiff to sign
the corresponding contract to sell in accordance with his previous commitment. Again,
defendant promised that he would sign the required contract to sell when he shall have
made up-to-date the stipulated monthly installments on the lots in question, but
subsequently backed out of his promise and refused to sign any contract in noncompliance
with what he had represented on several occasions. And plaintiff relied on the good faith of
defendant to make good his promise because defendant is a professional and had been
rather good to him. Later on, plaintiff demanded upon defendant to vacate the lots in
question and to pay the reasonable rentals thereon at the rate of P60.00 per month. Plaintiff
filed an action for rescission and cancellation of the agreement of sale between them
involving the two lots in question and prayed that judgment be rendered ordering the
rescission and cancellation of the agreement of sale,
Issue: Whether or not ownership of the property remained with plaintiff for so long as
the installments have not been fully paid.
Held: No. In the case at bar, there is no writing or document evidencing the
agreement originally entered into between petitioner and private respondent except the
receipt showing the initial deposit of P150.00 as shown and the payment of the 4- months
installment made by respondent corresponding to July, 1954 to October, 1954 in the sum of
P740.56 as shown. Neither is there any writing or document evidencing the modified
agreement when the 3 lots were changed to Lots 4 and 12 with a reduced area of 725 sq.
meters, which are corner lots. This absence of a formal deed of conveyance is a very strong
indication that the parties did not intend immediate transfer of ownership and title, but only a
transfer after full payment of the price.

18) G.R. No. L-44169 July 16, 1937


GUILLERMO PUATO Y CONSTANTINO, Plaintiff-Appellee, vs. FILOMENA MENDOZA
and VALENTIN DAVID Defendants
Facts: The parties agree that the land described in the complaint has been legally
mortgaged to the plaintiff as property of the defendants to answer for the sum of P37,500
forming a part of the P39,000 for which said land was sold to the defendants by the plaintiff.

SALES CASE DIGEST COMPILATION 203


After some time, the plaintiff attempted to prove that the land had been purchased by
Alejandra Padilla for the sum of P38,560, subject to a mortgage lien in favor of the National
Bank, in the sum of P6,000. That about the year 1928, when the defendants mortgaged said
land to him, palay was quoted at P4 a cavan, and when he filed his complaint for foreclosure
of mortgage, about the month of November, 1932, palay was quoted at P1.50 a cavan.
On the other hand, the defendant himself applied for the registration of the deed and
the transfer of the title to his name, later signing the mortgage deed, that the defendants
began to work on the land in March 1929, placing 41 tenants thereon and harvesting a total
of one thousand cavans because only ten hectares were clean; that they began to clean the
land, harvesting 900 cavans in the second year and 700 cavans in the third; that the
defendants decided to purchase the land because the plaintiff told them that it was good and
promised to help them by not pressing them for payment of the installments and by not
attaching the land; that they did not visit the land before buying it because they had no time
and because they relied on what the plaintiff had told them.
Issue: Whether or not the sale was absolute, therefore, plaintiff is obliged to make
delivery of the possession.
Held: Yes. The sale under consideration herein, as any other contract, was
perfected from the moment the plaintiff consented to sell the land in question and the
defendants agreed to purchase it for the sum of P39,000, or from the moment there was
consent and as said defendants had partially complied with their obligation by paying a part
of the price, and the plaintiff had complied with his by making constructive delivery of the
possession of the land by means of the execution of the deed, and the transfer and
registration of the certificate of title in the name of said defendants (article 1462, Civil Code),
the sale was consummated and became absolute and irrevocable. The same is true with the
mortgage deed,

19) NORTH NEGROS SUGAR VS CIA GEN DE TABACOS


NOTE: CASE NOT FOUND AMIDST ALL EFFORT

20) G.R. No. 117187. July 20, 2001


UNION MOTOR CORPORATION, petitioner-appellant, vs. THE COURT OF APPEALS,
JARDINE-MANILA FINANCE, INC., SPOUSES ALBIATO BERNAL and MILAGROS
BERNAL, respondents-appelles.
Facts: On September 14, 1979, the respondent Bernal spouses purchased from
petitioner Union Motor Corporation one Cimarron Jeepney for Thirty Seven Thousand Seven
Hundred Fifty Eight Pesos and Sixty Centavos (P37,758.60) to be paid in installments. For
this purpose, the respondent spouses executed a promissory note and a deed of chattel
mortgage in favor of the petitioner. Meanwhile, the petitioner entered into a contract of
assignment of the promissory note and chattel mortgage with Jardine-Manila Finance, Inc.
Through Manuel Sosmea, an agent of the petitioner, the parties agreed that the respondent
spouses would pay the amount of the promissory note to Jardine-Manila Finance, Inc., the
latter being the assignee of the petitioner.
. To effectuate the sale as well as the assignment of the promissory note and chattel
mortgage, the respondent spouses were required to sign a notice of assignment, a deed of
assignment, a sales invoice, a registration certificate, an affidavit, and a disclosure
statement. Upon the respondent spouses tender of the down payment worth Ten Thousand
Thirty-Seven Pesos (P10,037.00), and the petitioners acceptance of the same, the latter
approved the sale. Although the respondent spouses have not yet physically possessed the
vehicle, Sosmea required them to sign the receipt as a condition for the delivery of the
vehicle.
The respondent spouses continued paying the agreed installments even if the subject motor
vehicle remained undelivered inasmuch as Jardine-Manila Finance, Inc. promised to deliver
the subject jeepney. The respondent spouses have paid a total of Seven Thousand Five

SALES CASE DIGEST COMPILATION 204


Hundred Seven Pesos (P7,507.00) worth of installments before they discontinued paying on
account of non-delivery of the subject motor vehicle.
Issue: whether or not there has been a delivery, physical or constructive, of the
subject motor vehicle.
Held: No. the issuance of a sales invoice does not prove transfer of ownership of the
thing sold to the buyer; an invoice is nothing more than a detailed statement of the nature,
quantity and cost of the thing sold and has been considered not a bill of sale. The
registration certificate signed by the respondent spouses does not conclusively prove that
constructive delivery was made nor that ownership has been transferred to the respondent
spouses. Like the receipt and the invoice, the signing of the said documents was qualified by
the fact that it was a requirement of petitioner for the sale and financing contract to be
approved.
In all forms of delivery, it is necessary that the act of delivery, whether constructive or actual,
should be coupled with the intention of delivering the thing. The act, without the intention, is
insufficient. The critical factor in the different modes of effecting delivery which gives legal
effect to the act, is the actual intention of the vendor to deliver, and its acceptance by the
vendee. Without that intention, there is no tradition

21) G.R. No. 8675 September 18, 1914


BARTOLOME TABLANTE, plaintiff-appellee, vs. JOSE AQUINO, defendant-appellant.
Facts: On May 9, 1904, the said sheriff publicly announced that pursuant to a
judgment rendered by the justice of the peace of Cabanatuan against Paulino Mendiola and
his property, a lot and a warehouse situated in Sumacab of the case municipality. During
that time, the said property was sold to the defendant, their present possessor. However,
defendant alleged that prior to the auction sale, there was a sale executed between
Mendiola and a certain Bautista way back 1895. However, it is not Ciriaco Bautista, the last
owner recorded in the registry, but Bartolome Tablante who now intervenes for the recovery
of the said property. The latter averred in his complaint that he purchased it from the former
and presented as the only proof of such purchase his, which is a letter addressed to him by
Ciriaco Bautista.
The Court of First Instance of Nueva Ecija rendered judgment in the case by sentencing
Jose Aquino to deliver the plaintiff, Bartolome Tablante, the property in litigation and to pay
to the said plaintiff the sum of P387.50 with interest thereon at the rate of 6 per cent per
annum.
Issue: Whether or not defendant is obliged to deliver the property.
Held: Yes. The ownership of things is not transferred from one person to another by
mere consent in the contract, but through the delivery of the thing that is the subject of the
contract. In the present case, it is admitted by the appellee that there was no material
delivery of the lot and warehouse by Ciriaco Bautista to Bartolome Tablante, as up to now
no proof has been presented of a contract of sale made between Bautista and Tablante.
Nevertheless, the law prescribes that the "the placing of the titles of ownership in the
possession of the vendee or the use which he may make of his right with the consent of the
vendor shall be considered as a delivery." (Civil Code, art. 1464.) The title deeds form the
plaintiff' and the use of his right by the purchaser who in his complaint lays claim to the lot
and the warehouse, appear to have been consented to by the vendor, by means of the
aforementioned It is the same as though Ciriaco Bautista were the intervener, and if he had
been. Therefore, Bartolome Tablante being the owner of the lot and warehouse described in
the complaint, the defendant, Jose Aquino, is sentenced to restore them to the plaintiff,

22) G.R. No. 91029 February 7, 1991


NORKIS DISTRIBUTORS, INC., petitioner, vs.THE COURT OF APPEALS & ALBERTO
NEPALES, respondents.

SALES CASE DIGEST COMPILATION 205


Facts: Petitioner Norkis Distributors, Inc. (Norkis for brevity), is the distributor of
Yamaha motorcycles in Negros Occidental with office in Bacolod City with Avelino Labajo as
its Branch Manager. On September 20, 1979, private respondent Alberto Nepales bought
from the Norkis-Bacolod branch a brand new Yamaha Wonderbike motorcycle Model
YL2DX with Engine No. L2-329401K Frame No. NL2-0329401, Color Maroon, then
displayed in the Norkis showroom.
The price of P7,500.00 was payable by means of a Letter of Guaranty from the Development
Bank of the Philippines (DBP), Kabankalan Branch, which Norkis' Branch Manager Labajo
agreed to accept. Hence, credit was extended to Nepales for the price of the motorcycle
payable by DBP upon release of his motorcycle loan. As security for the loan, Nepales
would execute a chattel mortgage on the motorcycle in favor of DBP. Branch Manager
Labajo issued Norkis Sales Invoice No. 0120 showing that the contract of sale of the
motorcycle had been perfected. Nepales signed the sales invoice to signify his conformity
with the terms of the sale. In the meantime, however, the motorcycle remained in Norkis'
possession. On November 6, 1979, the motorcycle was registered in the Land
Transportation Commission in the name of Alberto Nepales.
Sometime on January 22, 1980, the motorcycle was delivered to a certain Julian Nepales
who was allegedly the agent of Alberto Nepales but the latter denies it. The record shows
,however, that respondent and his agent presented the unit to DBP’s appraiser-Investigator
Arriesta at the DBP Office in Kabankalan, Negros Occidental Branch.On February 1980, the
motorcycle met an accident at Binalbagan, Negros Occidental while being driven by a
certain Payba. The unit was a total wreck, was returned and stored inside Norkis’
warehouse.
DBP released the proceeds of respondent’s motorcycle loan to Norkis in the total sum of P
7,500, As the price of the motorcycle later increased to P7,828. Nepales paid the difference
of P328 and demanded the delivery of the motorcycle. Petitioner failed to deliver the unit,
and Nepales filed an specific performance with RTC of Himamaylan, Negros Occidental.
Petitioner answered that the motorcycle had already been delivered to private respondent
before the accident, hence, he should bear the risk of loss or damage as owner of the unit.
Issue: Whether or not the seller shall bore the loss.
Held: Yes. Article 1496 of the Civil Code which provides that "in the absence of an express
assumption of risk by the buyer, the things sold remain at seller's risk until the ownership
thereof is transferred to the buyer," is applicable to this case, for there was neither an actual
nor constructive delivery of the thing sold, hence, the risk of loss should be borne by the
seller, Norkis, which was still the owner and possessor of the motorcycle when it was
wrecked. This is in accordance with the well-known doctrine of res perit domino.

RYAN

23) David W. Bien Co vs CTA ( 99 Vt. 78)


_____CASE NOT FOUND AMIDST ALL EFFORT _____

24) G.R. No. L-20601 February 28, 1966


BUTUAN SAWMILL, INC., petitioner, vs.HON. COURT OF TAX APPEALS, ET
AL., respondents.
Facts: During the period from January 31, 1951 to June 8, 1953, it sold logs to
Japanese firms at prices FOB Vessel Magallanes, Agusan; that the FOB prices included
costs of loading, wharfage stevedoring and other costs in the Philippines; that the quality,
quantity and measurement specifications of the logs were certified by the Bureau of
Forestry; that the freight was paid by the Japanese buyers; and the payments of the logs

SALES CASE DIGEST COMPILATION 206


were effected by means of irrevocable letters of credit in favor of petitioner and payable
through the Philippine National Bank or any other bank named by it.
Petitioner points out that the "FOB" feature of the sales contract was made only to fix
its price and not to fix the place of delivery; that the requirement of certification of quality,
quantity, and measurement specifications of the logs by local authorities was done to comply
with local laws, rules, and regulations, and was not a part of the sales arrangement; that the
payment of freight by the Japanese buyers is not an uncommon feature of "FOB" shipments;
and that the payment of prices by means of irrevocable letters of credit is but a common
established business practice to secure payment of the price to the seller. It also insists that,
even assuming that the "FOB" feature of the disputed sales determines the situs of transfer
of ownership, the same is merely a prima facie presumption which yields to contrary proof
such as that the logs were made deliverable to the "order of the shipper" and the logs were
shipped at the risk of the shipper, which circumstances, if considered, would negate the
above implications.

Issue: Whether or not the goods deliverable to the order of the seller or his agent
negate the passing of title to the goods upon delivery to the carrier.

Held: No. Article 1503 (par.2)of the Civil Code of the Philippines provides that, Where
goods are shipped, and by the bill of lading the goods are deliverable to the seller or his
agent, or to the order of the seller or of his agent, the seller thereby reserves the ownership
in the goods. But, if except for the form of the bill of lading, the ownership would have
passed to the buyer on shipment of the goods, the sellers's property in the goods shall be
deemed to be only for the purpose of securing performance by the buyer of his obligations
under the contract.

25) G.R. No. 95536 March 23, 1992 ANICETO G. SALUDO, JR., MARIA SALVACION
SALUDO, LEOPOLDO G. SALUDO and SATURNINO G. SALUDO, petitioners,
vs.HON. COURT OF APPEALS, TRANS WORLD AIRLINES, INC., and PHILIPPINE
AIRLINES, INC., respondents.

Facts: The mother of the petitioners died in Chicago, Illinois. Pomierski Funeral
Home of Chicago made the necessary preparations and arrangements for the shipment of
the remains to the Philippines. Pomierski brought the remains to Continental Mortuary Air
Services (CMAS) at the Chicago Airport which made the necessary arrangements. CMAS
booked the shipment with PAL.PAL Airway Bill was issued wherein the requested routing
was from Chicago to San Francisco onboard Trans World Airline (TWA) and from
San Francisco to Manila on board PAL. Salvacion, one of the petitioners, upon arrival at San
Francisco, went to the TWA to inquire about her mother’s remains. But she was told they did
not know anything about it. She then called Pomierski that her mother’s remains were not at
the West Coast terminal.
Pomierski immediately called CMAS which informed that the remains were on a
plane to Mexico City, that there were two bodies at the terminal, and somehow they were
switched. Petitioners filed a complaint against TWA and PAL for the erroneous shipment and
delay of the cargo. Petitioners alleged that private respondents received the casketed
remains of the deceased on October 26, 1976, as evidenced by the issuance of PAL Airway
Bill and from said date, private respondents were charged with the responsibility to exercise
extraordinary diligence so much so that the alleged switching of the caskets on October 27,
the latter must be liable. PAL contended that it was October 28 when they received the
physical delivery of the body, thus, it is not liable for the switching which happened the day
before.
Issue: Whether or not there was a delivery of the cargo upon issuance of Airway bill.

SALES CASE DIGEST COMPILATION 207


Held: No. While delivery of the goods to the carrier normally precedes the issuance
of the bill, or delivery of the goods and issuance of the bill are regarded in commercial
practice as simultaneous acts, there is nothing to prevent an inverse order of events. It is a
general rule to the parties to a contract of carriage of goods where a bill of lading is issued,
that the recital being in essence a receipt alone, is not conclusive but may be explained,
varied, or contradicted by parol or other evidence. Here, the explanation of private
respondents that the Airway Bill was issued, not as evidence of receipt of delivery but merely
as a confirmation of the book made sufficiently overcomes the presumption relied on by
petitioners that the remains of their mother were delivered to and received by private
respondents on October 26. The Court is convinced that private respondent received the
physical delivery of the body only on October 28 as evidenced by the Interline Freight
Transfer Manifest of the American Airline Freight System. It was from that date that private
respondents became responsible for the agreed cargo under their undertakings in PAL
Airway Bill. Consequently, for the switching of caskets prior thereto which was not caused by
them, private respondents cannot be held liable.

26) G.R. No. L-55684 December 19, 1984

THE HONORABLE COURT OF APPEALS and SAMBOK MOTORS CO.


(BACOLOD), respondents,

Facts: Petitioner is a domestic corporation engaged in the assembling and sale of


motor vehicles and other automotive products. Respondent Sambok Motors Co., a general
partnership, during the period relevant to these proceedings, was its dealer for automotive
products with offices at Bacolod (Sambok, Bacolod) and Iloilo (Sambok, Iloilo).
On October 2, 1970, Sambok, Bacolod, ordered from petitioner various automotive products
worth P30,909.61, payable in 45 days; that on November 25, 1970, petitioner delivered said
products to its forwarding agent, Allied Brokerage Corporation, for shipment; that Allied
Brokerage loaded the goods on board the M/S Doña Florentina, a vessel owned and
operated by Negros Navigation Company, for delivery to Sambok, Bacolod; that when
petitioner tried to collect from the latter the amount of P31,037.56, representing the price of
the spare parts plus handling charges, Sambok, Bacolod, refused to pay claiming that it had
not received the merchandise; that petitioner also demanded the return of the merchandise
or their value from Allied Brokerage and Negros Navigation, but both denied any liability.In
its Answer, Sambok, Bacolod, denied having received from petitioner or from any of its co-
defendants, the automotive products referred to in the Complaint, and professed no
knowledge of having ordered from petitioner said articles.

Issue: WON Sambok Bacolod bears the loss of the cargo for which it is liable in
damages to Chrysler.

Held: No, Sambok Bacolod cannot be faulted for not accepting or refusing to accept
the shipment from Negros Navigation four years after shipment. It was found out that upon
receipt of the Bill of Lading, Sambok Bacolod initiated, but did not pursue steps to take
delivery as they were advised by Negros Navigation that because some party were missing,
they would just be informed as soon as the missing parts were located. It was only four
years later that the said parts were found in their off-shore bodega but were already
deteriorated and valueless. The evidence is clear that Negros Navigation could not produce
the merchandise nor ascertain its whereabouts at the time Sambok, Bacolod, was ready to
take delivery. Where the seller delivers to the buyer a quantity of goods less than he
contracted to sell, the buyer may reject them.
From the evidentiary record, Negros Navigation was the party negligent in failing to deliver
the complete shipment either to Sambok, Bacolod, or to Sambok, Iloilo, but as the Trial
Court found, petitioner failed to comply with the conditions precedent to the filing of a judicial

SALES CASE DIGEST COMPILATION 208


action. Thus, in the last analysis, it is petitioner that must shoulder the resulting loss. The
general rule that before, delivery, the risk of loss is home by the seller who is still the owner,
under the principle of "res petit domino", is applicable in petitioner's case.

27) G.R. No. L-4080 September 21, 1953


JOSE P. MARTINEZ, as administrator of the Instate Estate of Pedro Rodriguez,
deceased, plaintiff-appellant,
vs.
PHILIPPINE NATIONAL BANK, defendant-appellee.
Facts: On February 1942, the estate of Pedro Rodriguez was indebted to the
defendant Philippine National Bank in the amount of P22,128.44 which represented the
balance of the crop loan obtained by the estate upon its 1941-1942 sugar cane crop.
Sometime in February 1942, Mrs. Amparo R. Martinez, late administrator of the estate upon
request of the defendant bank through its Cebu branch endorsed and delivered to the said
bank two (2) quedans according to plaintiff-appellant issued by the Bogo-Medellin Milling co.
where the sugar was stored covering 2,198.11 piculs of sugar belonging to the estate,
although according to the defendant-appellee, only one quedan covering 1,071.04 piculs of
sugar was endorsed and delivered. During the last Pacific war, sometime in 1943, the sugar
covered by the quedan or quedans was lost while in the warehouse of the Bogo-Medellin
Milling Co. In the year 1948, the indebtedness of the estate including interest was paid to the
bank, according to the appellant, upon the insistence of land pressure brought to bear by the
bank.
Under the theory and claim the sometime in February 1942, when the invasion of the
Province of Cebu by the Japanese Armed Forces was imminent, the administrator of the
estate asked the bank to release the sugar so that it could be sold at a god price which was
about P25 per picul in order to avoid its possible loss due to the invasion, but that the bank
refused that request and as a result the amount of P54,952.75 representing the value of said
sugar was lost, the present action was brought against the defendant bank to recover said
amount.
Issue: Whether or not the endorsement and delivery of the quedan or quedans to the
bank transferred the ownership of the sugar to said bank so that as owner,
Held: No. where a warehouse receipt or quedan is transferred or endorsed to a
creditor only to secure the payment of a loan or debt, the transferee or endorsee does not
automatically become the owner of the goods covered by the warehouse receipt or quedan
but he merely retains the right to keep and with the consent of the owner to sell them so as
to satisfy the obligation from the proceeds of the sale, this for the simple reason that the
transaction involved is not a sale covered by the quedans of warehouse receipts is lost
without the fault or negligence of the mortgagee or pledgee or quedan, then said goods are
to be regarded as lost on account of the real owner, mortgagor or pledgor.1âwphïl.nêt

28) G.R. NO. 150066 April 13, 2007


SPS. EMMANUEL (deceased) and EDNA CHUA and SPS. MANUEL and MARIA
CHUA, Petitioners,
vs.
MSGR. VIRGILIO SORIANO. Substituted by Sister Mary Virgilia Celestino
Soriano, Respondent
Facts: Sometime in the early months of 1988, Soriano’s first cousin and godson,
Emmanuel C. Celestino, Sr. (Celestino) asked Soriano to lend him TCT No. 363471 as a
security for a loan to be used in the business operation of Celestino’s company, Digital

SALES CASE DIGEST COMPILATION 209


Philippines, Inc. Acceding to Celestino’s request, Soriano executed on March 29, 1988 a
Special Power of Attorney (SPA) authorizing Celestino to mortgage said property.
Then came the June 11, 1988 fire that gutted a portion of the Quezon City Hall and
destroyed in the process the original copy of TCT No. 363471 on file with the Registry of
Deeds of Quezon City.On August 22, 1988, Soriano executed a SPA authorizing Celestino
and one Carlito Castro to initiate administrative reconstitution proceedings of TCT No.
363471.On April 17, 1990, the reconstituted title, TCT No. RT-3611 (363471) PR 1686, was
issued.During the pendency of the administrative reconstitution proceedings, Soriano asked
Celestino whether there was any truth to the spreading rumor that he had already sold the
subject property. Celestino denied the rumor but informed Soriano that the subject property
was mortgaged with a foreign bank. Dissatisfied with Celestino's explanation, Soriano made
inquiries with the Registry of Deeds of Quezon City and discovered, to his dismay, that TCT
No. 363471 had been canceled by TCT No. 14514 in the name of spouses Emmanuel and
Edna Chua and spouses Manuel and Maria Chua (Chuas). By virtue of a SPA11 dated March
9, 1989 with Soriano's purported signature, Celestino sold to the Chuas the property in an
Absolute Deed of Sale12 dated July 4, 1989 for ₱500,000.00.
Claiming that his signature in the SPA is a forgery, Soriano a complaint against Celestino
and the Chuas for annulment of deed of sale and special power of attorney, cancellation of
title and reconveyance with damages
Issue: Whether or not the Chuas are purchasers in good faith.
Held: Yes. When the document under scrutiny is a special power of attorney that is
duly notarized, we know it to be a public document where the notarial acknowledgment
is prima facie evidence of the fact of its due execution. A purchaser presented with such a
document would have no choice between knowing and finding out whether a forger lurks
beneath the signature on it. The notarial acknowledgment has removed the choice from him
and replaced it with a presumption sanctioned by law that the affiant appeared before the
notary public and acknowledged that he executed the document, understood its import and
signed it.
Thus, the fact that Soriano’s signature in the SPA dated March 9, 1989 was subsequently
declared by the trial court to have been falsified would not revoke the title subsequently
issued title in favor of the Chuas. With the property in question having already passed to the
hands of purchasers in good faith, it is now of no moment that some irregularity attended the
issuance of the SPA
29) 126 Atl. 902
INTERNATIONAL BREEDING CO.
VS.
TERMINAL WAREHOUSE CO.
_____ CASE NOT FOUND AMIDST ALL EFFORT ____

30) (44 Phil. 874)


Smith Bell Company
vs.
Sotelo Matti
GR No. 16570, March 9, 1922

FACTS
Plaintiff Smith, Bell & Co and the defendant Mr. Vicente Sotel entered into a contract.
Plaintiff has to deliver (1) two steel tanks shipped from New York to Manila within three or
four months, (2) two expellers shipped from SanFrancisco in the month of September 1918
or as soon as possible, and (3) two electric motors with “approximate delivery within
ninety days. – This is not guaranteed.”

SALES CASE DIGEST COMPILATION 210


The tanks arrived at Manila on 27 April 1919; the expellers on 26 October 1918; and the
motors on 27 February 1919. Upon notification from plaintiff, defendant refused to receive
any of the goods or to pay for their price. Plaintiff alleged that the expellers and motors were
in good condition.

Plaintiff filed a complaint against the defendant. The defendant, Mr Sotelo and intervenor,
Manila Oil Refining and By-Products Co., Inc., denied the plaintiff’s allegations. They allege
that due to plaintiff’s delay in the delivery of goods, the intervenor suffered damages
.
The lower court absolved the defendants from the complaint insofar as the tanks and the
electric motors were concerned, but rendered judgment against them ordering them to
receive expellers and pay the sum of P50,000, with legal interest and cost.

Both parties appealed to the Court.

ISSUE
What period was fixed for the delivery of the goods?
Whether Smith Bell incurred delay in the delivery of goods to Sotelo.

HELD
In all these contracts, there is a final clause as follows:
“The sellers are not responsible for delays cause by fires, riots on land or on
the sea, strikes or other causes known as ‘force majeure’ entirely beyond the control
of the sellers or their representatives.
Under these stipulations, it cannot be said that any definite date was fixed for the delivery of
the goods. xxx. From the record it appears that thee contracts were executed at the time of
the world war when there existed rigid restrictions on the export from the united States xxx;
hence clauses were inserted in the contracts, regarding “Government regulations,
railroading embargoes, lack of vessel space, the exigencies of the requirements of the
United States Government” xxx. At the time of the execution of the contracts, the parties
were not unmindful of the contingency of the United States Government not allowing the
export of the goods xxx.

We cannot but conclude that the term which parties attempted to fix is so uncertain that once
cannot tell just whether, as a matter of fact, those articles could be brought to manila or not.
The obligation must be regarded as conditional. The delivery was subject to a condition
the fulfillment of which depended not only upon the effort of the plaintiff, but upon the will of
third persons who could in no way be compelled to fulfill the condition.

It is sufficiently proven in the record that the plaintiff has made all the efforts it could possibly
be expected to make under the circumstances, to bring the goods in question to Manila, as
soon as possible. Xxx it is obvious that the plaintiff has complied with its obligation.

When the time of delivery is not fixed in the contract, time is regarded unessential. In such
cases, the delivery must be made within a reasonable time. Xxx Reasonable time for the
delivery of the goods by the seller is to be determined by circumstances attending the
particular transactions. Whether of not the delivery of the machinery in litigation was
offered to the defendant within a reasonable time, is a question to be determined by
the court. Xxx The plaintiff has not been guilty of any delay in the fulfillment of its
obligation.

SALES CASE DIGEST COMPILATION 211


31) (38 Phil 602,606, G.R. No. 22537, December 8, 1924)
BEHN, MEYER & CO. (LTD.), plaintiff-appellant,
vs.
TEODORO R. YANCO, defendant-appellee

Facts:

A sale of 80 drums of caustic soda was agreed between Behn, Meyer & Co. and
Teodoro Yanco. The merchandise was shipped from New York to Manila. However,
the ship carrying the cargo was detained at Penang and the 71 of the 80 drums were
removed. Respondent Yangco also refused to accept the 9 remaining and also refused
to accept the offer of Behn Meyer to have the products substituted with other
merchandise, which however were different from what was ordered. It must be noted
that the contract provided for "c.i.f. Manila, pagadero against delivery of documents."
Yanco filed an action seeking for damages for alleged breach of contract.

Issue: WON Behn, Meyer & Co. should bear the burden of the loss of the merchandise?

Held: YES.

Rule as to delivery of goods by a vendor via a common carrier (If contract is silent –
delivery of seller to common carrier transfer ownership to buyer)
Determination of the place of delivery always resolves itself into a question of act. If the
contract be silent as to the person or mode by which the goods are to be sent, delivery by
the vendor to a common carrier, in the usual and ordinary course of business, transfers the
property to the vendee.

Payment of freight by the buyer = acquires ownership at the point of shipment


A specification in a contact relative to the payment of freight can be taken to indicate the
intention of the parties in regard to the place of delivery. If the buyer is to pay the freight, it is
reasonable to suppose that he does so because the goods become his at the point of
shipment.

Payment of freight by the seller = title of property does not pass until the goods have
reached their destination
On the other hand, if the seller is to pay the freight, the inference is equally so strong that the
duty of the seller is to have the goods transported to their ultimate destination and that title to
property does not pass until the goods have reached their destination.

c.i.f. means Cost, Insurance and Freight = CFI is paid by the seller
The letters "c.i.f." found in British contracts stand for cost, insurance, and freight. They
signify that the price fixed covers not only the cost of the goods, but the expense of freight
and insurance to be paid by the seller.

F.O.B. stands for Free on Board = seller bear all expenses until goods are delivered
In this case, in addition to the letters "c.i.f.," has the word following, "Manila." In mercantile
contracts of American origin the letters "F.O.B." standing for the words "Free on Board," are
frequently used. The meaning is that the seller shall bear all expenses until the goods are
delivered where they are to be "F.O.B."

According as to whether the goods are to be delivered "F.O.B." at the point of shipment or at
the point of destination determines the time when property passes. However, both the terms

SALES CASE DIGEST COMPILATION 212


"c.i.f." and "F.O.B." merely make rules of presumption which yield to proof of contrary
intention.

Delivery was to be made at Manila


Hence, we believe that the word Manila in conjunction with the letters "c.i.f." must mean that
the contract price, covering costs, insurance, and freight, signifies that delivery was to made
at Manila. If petitioner Behn Meyer has seriously thought that the place of delivery was New
York and Not Manila, it would not have gone to the trouble of making fruitless attempts to
substitute goods for the merchandise named in the contract, but would have permitted the
entire loss of the shipment to fall upon the defendant.

Behn Meyer failed to prove that it performed its part in the contract
In this case, the place of delivery was Manila and plaintiff (Behn Meyer) has not legally
excused default in delivery of the specified merchandise at that place. In resume, we find
that the plaintiff has not proved the performance on its part of the conditions precedent in the
contract.

For breach of warranty, the buyer (Yanco) may demand rescission of the contract of
sale
The warranty — the material promise — of the seller to the buyer has not been complied
with. The buyer may therefore rescind the contract of sale because of a breach in substantial
particulars going to the essence of the contract. As contemplated by article 1451 of the Civil
Code, the vendee can demand fulfillment of the contract, and this being shown to be
impossible, is relieved of his obligation. There thus being sufficient ground for rescission, the
defendant is not liable.

32) 200 App. Div. 52


PERCY KENT CO.
V.
SILVERTEIN

______ CASE NOT FOUND AMIDST ALL EFFORT ______

33) G.R. No. L-11109 January 7, 1918


AMADEO MATUTE, Plaintiff-Appellant,
vs.
CHEONG BOO, Defendant-Appellant.

Facts:
On January 14, 1915, a contract was made between the plaintiff, Amadeo Matute, a
resident of Davao, Philippine Islands, and the defendant Cheong Boo, a resident of
Zamboanga, Philippine Province of Zamboanga, Philippine Islands that the former should
deliver and the latter should receive within the month of February of the same year a
quantity of more than 300 and less than 500 piculs of mastic (almaciga) at the price of
P8.50/picul. Matute performed his part and delivered on February 22, 1915 the almaciga to
the defendant but he refused to accept delivery. At the hearing the court found that the
contract was made, as alleged; that the almaciga was merchantable quality, and that the
defendant had violated his contract in refusing to receive it.

SALES CASE DIGEST COMPILATION 213


The plaintiff thereupon stored the almaciga in a warehouse and he went to the court
to file a case claiming for damages plus interest for not accepting the almaciga and the
expenses of storing the almaciga in a warehouse.

Issue:

Whether or not the defendant is liable for not accepting the almaciga upon its
delivery.

Ruling: Yes. In the second paragraph of article 1124 of the Civil Code it is said that the
seller, after having elected to enforce compliance with the contract, may ask for rescission
when performance becomes impossible. It follows that, having first made the election and
cannot convert his action into an action for rescission unless performance becomes
impossible. Therefore if the seller had sold this almaciga as the court had decreed specific
performance, the seller would have been unable to comply with that judgment. As to the
right of the plaintiff to recover all of the expenses incident to the storage of
the almaciga pending this litigation, it is to be observed that under the last paragraph of
section 332 of the Code of Commerce it is expressly stated that the expenses incident to the
deposit of the goods shall be charged to the delinquent party. It will be noted that the
contract in question calls for the delivery of a quantity of almaciga of less than 500 piculs.
This would justify the delivery of a quantity fractionally below that amount, It appears that the
quantity actually tendered was 500 piculs. As the law takes no account of trifles ( de minimis
non curat lex), it is obvious that the discrepancy may be disregarded; and the delivery of 500
piculs is a sufficient compliance with the contract to deliver an amount less than 500.chanrob

34) G.R. No. L-15716 March 31, 1962

TALIGAMAN LUMBER CO., INC., petitioner,


vs.
THE COLLECTOR OF INTERNAL REVENUE, respondent.

FACTS:

Petitioner herein, a domestic corporation with principal office in the City of Manila and
branch offices in Grace Park, Caloocan, Rizal, and Butuan City, Agusan, is a duly licensed
forest and timber concessionaire. As such, it is engaged in the business of cutting logs in its
concessions and converting said logs into lumber, as well as buying logs from other
concessionaires. Most of the lumber cut in its sawmill in Agusan were sent to its branch
office in Grace Park, Caloocan, Rizal, to be sold to different lumber dealers in Manila and
suburbs.

Taligaman argued that the tax assessment done by the government for the years 1948 and
1949 had already prescribed pursuant to Section 331 of the Revised Internal Revenue Law
since it provided for a 5 year prescriptive period.

The Collector, meanwhile countered that Taligaman had done fraud in preparing its tax
returns, so Section 332 would apply. It provided that in cases of fraud, falsity or omission,
the government has a 10 year prescriptive period.

ISSUE/S:

SALES CASE DIGEST COMPILATION 214


Whether or not the export sales made by the Butuan City branch to Japanese buyers were
consummated in the Philippines and subject to sales tax in the Philippines.

HELD: Yes.
. ID.; ID.; EVIDENCE SHOWING INTENT OF PARTIES TO PASS OWNERSHIP OF
GOODS IN THE PHILIPPINES. — Where the agreed price of the logs being sold by the
company to Japanese buyers was "F.O.A. Agusan", it may be assumed, although prima
facie, that the parties intended the title to pass to the buyer upon delivery of the logs in
Agusan, on board the vessels that took the goods to Japan. Consequently, the said export
sales were subject to sales tax in the Philippines.

In this connection, petitioner’s president testified that the real intent of the parties to the
aforementioned sales was to effect the transfer of title to the buyers in Japan.
However, petitioner did not introduce in evidence the corresponding contracts of sale. Upon
the other hand, it is admitted that the agreed price was "F.O.B. Agusan", thus indicating,
although prima facie, that the parties intended the title to pass to the buyer upon delivery of
the logs in Agusan, on board the vessels that took the goods to Japan.

35) EN BANC
[G.R. No. L-8717. November 20, 1956.]
GENERAL FOODS CORPORATION, Plaintiff-Appellant,
vs.
NATIONAL COCONUT CORPORATION, Defendant-Appellee.
Facts:
Appellant General Foods Corporation is a foreign corporation organized under the
laws of the State of Delaware, U. S. A., and licensed to do business in the Philippines; chan
roblesvirtualawlibrarywhile Appellee National Coconut Corporation (otherwise called
NACOCO), was, on the date of the transaction in question, a corporation created by
Commonwealth Act No. 518, but later abolished and place in liquidation by Executive Order
No. 3727 dated November 24, 1950. On September 23, 1947, Appellee sold
to Appellant 1,500 (later reduced to 1,000) long tons of copra, at $164 (later reduced to
$163) per ton of 2,000 pounds. From November 14 to December 3, 1947, Appellee shipped
1054.6278 short tons of copra to Appellant on board the S. S. “Mindoro”. The weighing of
the cargo was done by the Luzon Brokerage Co., in its capacity as agent of the General
Superintendence Co., Ltd., of Geneva, Switzerland, by taking the individual weight of each
bag of copra and summing up the total gross weight of the shipment, then weighing a certain
number of empty bags to determine the average tare of the empty bags, which was
subtracted from the gross weight of the shipment to determine the net weight of the cargo.
Upon arrival in New York, the net cargo was reweighed by Appellant and was found to weigh
only 898.792 short tons. Deducting from the value of the shortage the sum of $8,092.02
received by Appellant from the insurer for 58.25 long tons lost.
Appellant filed the present action in the Court of First Instance of Manila to recover
from Defendant-Appellee the amount of $24,154.49 and the 17 per cent exchange tax
thereon which, under the provisions of Republic Act 529, had to be paid in order to remit
said amount to the United States, plus attorney’s fees and costs. The Court a quo found for
the Defendant and dismissed the complaint; arhence, this appeal by Plaintiff.
Issue:
Whether or not there was delivery to the buyer.

SALES CASE DIGEST COMPILATION 215


Held: Yes.
Plaintiff-Appellant’s theory is that although the sale between the parties quoted a CIF New
York price, the agreement contemplated the payment of the price according to the weight
and quality of the cargo upon arrival in New York, the port of destination, and that therefore,
the risk of the shipment was upon the seller. Defendant-Appellee, on the other hand, insists
that the contract in question was an ordinary C. I. F. agreement wherein delivery to the
carrier is delivery to the buyer, and that the shipment having been delivered to the buyer and
the latter having paid its price, the sale was consummated.
There is no question that under an ordinary C.I.F. agreement, delivery to the buyer is
complete upon delivery of the goods to the carrier and tender of the shipping and other
documents required by the contract and the insurance policy taken in the buyer’s. There is
equally no question that the parties may, by express stipulation or impliedly (by making the
buyer’s obligation depend on arrival and inspection of the goods), modify a CIF contract and
throw the risk upon the seller until arrival in the port of destination.
While the risk of loss was apparently placed on the Appellant after delivery of the
cargo to the carrier, it was nevertheless agreed that the payment of the price was to be
according to the “net landed weight”. The net landed or outturn weight of the cargo, upon
arrival in New York, was 898.692 short tons. the Appellee had the burden of proof to show
that the shortage in weight upon arrival in New York was due to risks of the voyage and not
the natural drying up of the copra while in transit, or to reasonable allowances for errors in
the weighing of the gross cargo and the empty bags in Manila. In the absence of such proof
on the part of the shipper-Appellee, we are constrained to hold that the net landed weight of
the shipment in New York should control, as stipulated in the agreement, and that therefore,
the Appellee should be held liable for the amount of $24,154.59 which it had overdrawn
from Appellant’s letter of credit.

SALES CASE DIGEST COMPILATION 216


SALES CASE DIGEST COMPILATION 217
ROMDRI
36)[G.R.No.16936.June13,1922.

WARNER, BARNES & CO., LTD., Plaintiff-Appellee, v. DIONISIO INZA, Defendant-


Appellant.

1. PERFECTION OF CONTRACT OF PURCHASE AND SALE; CIVIL CODE. — The


contract of purchase and sale is perfected upon the parties having agreed as to the thing
which is the subject matter of the contract and the price. (Art. 1450, Civil Code.)

2. PURCHASE AND SALE; TIME FOR PAYMENT; DUTY OF VENDOR; CIVIL AND
MERCANTILE SALE. — As to whether or not delivery of the thing sold should be made
before payment, even regarding the sale as civil and not mercantile, it must not be
overlooked that in the case under consideration a term for the payment has been stipulated,
which brings the case within the exception provided in article 1466 of the Civil Code, that is,
that the thing which is the object of the sale must be delivered even before the price is paid.
If the sale is held mercantile, it is still the duty of the vendor, before he can demand payment
of the price, to deliver the thing sold to the buyer or have it at the letter’s disposal within the
twenty-four hours following the making of the contract, and only after such delivery, or in
default thereof, its judicial deposit does the obligation of the buyer to pay the price begin.
(Arts. 337 and 339, Code of Commerce.)

3. RESCISSION OF SALE; DEFAULT OF PAYMENT WITHIN THE TIME STIPULATED. —


The mere default on the part of the buyer to pay the price within the period stipulated is no
ground for the rescission of the sale, where such period was not coupled with the condition
that the default of payment will operate to rescind the contract.

About April 9, 1220, the firm of Figueras Hermanos, acting as agent of Dionisio Inza, sold
through its manager, E. Sunyer, to the partnership of Warner, Barnes & Co., Ltd., the latter
acting through its agent I. Robinson, 4,000 piculs of centrifugal sugar of 96 degrees,
belonging to said Dionisio Inza, at P37.50 per picul. Neither delivery of the sugar, nor
payment of its price, was then made.

Two days later, that is, on the 12th of the same month, in pursuance to the contract, Dionisio
Inza handed to Warner, Barnes & Co., Ltd., several quedans covering 2,862.23 piculs of the
sugar sold, together with his bill, which is Exhibit 1. On that same day the partnership of
Warner, Barnes & Co., Ltd., through its agent Robinson, proposed to Dionisio Inza to pay
the price of the sugar at a future date but not later than the 15th day of the following month
of May, with interest at the rate of 8 per cent per annum on the price of the sale.

According to Warner, Barnes & Co., Ltd., this proposition was accepted in its entirety by
Dionisio Inza. The latter, however, claims that while it is true that such term was agreed
upon, yet he imposed the condition that if no payment was made to him before the 15th day
of May, the date stipulated, he would be free to dispose of the sugar.

About the 13th day of that same month of April, Dionisio Inza took back from Warner,
Barnes & Co., Ltd., the quedans which he had sent on the preceding day, and the quedans
never again came into the possession of Warner, Barnes & Co., Ltd.

SALES CASE DIGEST COMPILATION 218


On May 17, 1920, Warner, Barnes & Co., Ltd., sent Dionisio Inza a check for P108,286.22 in
payment of the 2,862.23 piculs of sugar covered by the aforesaid quedans, and the interest
on the price of said quantity of sugar at 9 per cent per annum, the rate having been raised
by Warner, Barnes & Co., Ltd., of its own accord in view of the fact that it was the interest
then charged by the banks at Iloilo. In the same letter of remittance of this check, Warner,
Barnes & Co. asked that said quedans be sent them.

Dionisio Inza refused to receive the check and to send the quedans, alleging that the sale
was rescinded by the failure of Warner, Barnes & Co., Ltd., to pay the price on the 15th day
of May, as stipulated.

On the 18th of that same month of May, Warner, Barnes & Co. again made demand on
Dionisio Inza for the surrender of the quedans, not only of those returned to him but of all the
quedans covering the 4,000 piculs of sugar which were sold. This Dionisio Inza again
refused to do.

On the 26th of that same month, the attorneys, Montinola, Montinola & Hontiveros, on behalf
of Warner, Barnes & Co., made demand on Dionisio Inza who persisted in his refusal.

On June 1, 1920, Warner Barnes & Co,. filed a complaint, which was amended on the 26th
following alleging that Dionisio Inza had not fulfilled the contract, and that it suffered
damages in the sum of P66,000, which was the difference between the value of the 4,000
piculs of sugar at fifty-four pesos (P54) per picul (the price of this commodity on June 1,
1920), and the value of this same sugar at the agreed price of thirty-seven pesos and fifty
centavos (P 37.50) per picul; and praying that judgment be rendered in its favor and against
Dionisio Inza for damages in the said amount.

ISSUE: W/N the sale of picul between Inza and Warner Bros Inc is perfected and not
deemed rescinded?

SC: Yes, the defendant contends that the sale in question is governed by the provisions of
the Civil Code, and that under article 1171 of the said Code, the plaintiff was under the
obligation to pay the defendant the price of the sugar before the delivery thereof by the
latter, the payment to be made by the plaintiff at the domicile of the defendant.

As to whether or not the delivery of the thing sold was a condition precedent to the payment
of the price, it must not be overlooked that, even if we regard the sale as of a civil, and not of
a mercantile, nature, a period was stipulation for the making of payment, and this brings the
case with the exception provided in article 1466 of the Civil Code, that is, that the sugar
should have been delivered even before its price was paid (Florendo v. Foz, 20 Phil., 388). If
we hold the sale in question to be mercantile, still it was the duty of the plaintiff to deliver the
sugar before he could demand payment of its price, and only after such delivery, or, in
default thereof, its judicial deposit, would the plaintiff have been under the obligation to pay
the price (arts. 337 and 339, Code of Commerce).

In the case before us, the defendant was bound to place the sugar at the disposal of the
plaintiff (art. 1462, Civil Code), or have it at the latter’s disposal within twenty-four hours after
the contract (art. 337, Code of Commerce.)

The evidence shows that the defendant did not place the sugar at the disposal of the
plaintiff, nor have it at the latter’s disposal even within the twenty-four hours following April
13, 1920, the date on which he took back from the plaintiff the quedans covering a part of
the sugar sold. It cannot be said that the sugar referred to in said quedans remained in the

SALES CASE DIGEST COMPILATION 219


possession of the plaintiff, notwithstanding that they were taken back by the defendant, who,
as a matter of fact, disposed of them without the consent of the plaintiff.

It was, therefore, incumbent upon the defendant to deliver the sugar sold to the plaintiff, and
not having done so, he was in default in the fulfillment of his obligation as vendor.

the defendant alleges that the evidence should have been held sufficient to establish the fact
that the payment of the price on or before the 15th day of May was a condition precedent to
the perfection of the sale.

Before to have been agreed upon for the payment of the price of the sugar lasted until May
15, 1920, or the end of that month, it should be noted that the sale became perfected on the
day of the making of the contract, before the delivery of the thing sold, or the payment of the
price, upon the mere fact of the parties having agreed as to the thing which was the subject-
matter of the contract and as to the price (art. 1450, Civil Code).

With reference to the term agreed upon for the payment of the price of the sugar, the
defendant says that he imposed the condition that if his bill (for the sugar covered by the
quedans which he had taken back) was not paid, he would sell the sugar where he could.
This is denied by the plaintiff’s evidence. It was incumbent upon the defendant to prove it, as
it was a part of his affirmative defense, and the record does not afford any justification for
finding that such a condition was sufficiently established.

As to whether or not the term for the payment of the price lasted until the 15th of May, or the
end of said month, the preponderance of evidence shows that it lasted until the 15th of May,
1920, but as we have stated, such term was not coupled with the condition that default of
payment would cause the rescission of the sale

37) Katigbak vs. Court of Appeals, No. L-16480, 4 SCRA 243 , January 31, 1962

ARTEMIO KATIGBAK, petitioner,


vs.
COURT OF APPEALS, DANIEL EVANGELISTA and V. K. LUNDBERG, respondents.

Benjamin J. Molina for petitioner.


Jesus B. Santos for respondent V. K. Lundberg.
Ledesma, Puno, Guytingco, Antonio and Associates for respondent Daniel
Evangelista.

PAREDES, J.:

This case arose from an agreed purchase and sale of a Double Drum Carco Tractor Winch.
Artemio Katigbak upon reading an advertisement for the sale of the winch placed by V. K.
Lundberg, owner and operator of the International Tractor and Equipment Co., Ltd., went to
see Lundberg and inspected the equipment. The price quoted was P12,000.00. Desiring a
reduction of the price, Katigbak was referred to Daniel Evangelista, the owner. After the
meeting, it was agreed that Katigbak was to purchase the winch for P12,000.00, payable at
P5,000.00 upon delivery and the balance of P7,000.00 within 60 days. The condition of the
sale was that the winch would be delivered in good condition. Katigbak was apprised that
the winch needed some repairs, which could be done in the shop of Lundberg. It was then
stipulated that the amount necessary for the repairs will be advanced by Katigbak but
deductible from the initial payment of P5,000.00. The repairs were undertaken and the total
of P2,029.85 for spare parts was advanced by Katigbak for the purpose. For one reason or

SALES CASE DIGEST COMPILATION 220


another, the sale was not consummated and Katigbak sued Evangelista, Lundberg and the
latter's company, for the refund of such amount.

Evangelista, on his part, claimed that while there was an agreement between him and
Katigbak for the purchase and sale of the winch and that Katigbak advanced the payment for
the spare parts, he (Katigbak) refused to comply with his contract to purchase the same; that
as a result of such refusal he (Evangelista) was forced to sell the same to a third person for
only P10,000.00, thus incurring a loss of P2,000.00, which amount Katigbak should be
ordered to pay, plus moral damages of P5,000.00 and P700.00 for attorney's fees.

LOWER COURT

WHEREFORE, judgment is hereby rendered ordering the defendants Daniel Evangelista


and V. K. Lundberg to pay plaintiff the sum of P2,029.85, with legal interest thereon from the
filing of the complaint until fully paid, plus the sum of P300.00 as attorney's fees, and the
costs." .

CA—

Notwithstanding the breach of contract committed by him, we may concede appellee's right
to a refund of the sum of P2,029.85, but equally undeniable is appellant Evangelista's right
to recover from him his loss of P2,000.00, which is the difference between the contract price
for the sale of the winch between him and appellee and the actual price for which it was sold
after the latter had refused to carry out his agreement. As held in the above-cited case of
Hanlon, if the purchaser fails to take delivery and pay the purchase price of the subject
matter of the contract, the vendor, without the need of first rescinding the contract judicially,
is entitled to resell the same, and if he is obliged to sell it for less than the contract price, the
buyer is liable for the difference. This loss, which is the subject matter of Evangelista's main
counterclaim, should therefore be set off against the sum claimed by appellee, which would
leave in favor of the latter a balance of P29.85.

Considering our finding that it was appellee who committed a breach of contract, it follows
that the present action was unjustified and he must be held liable to appellant Evangelista
for attorney's fees in the sum of P700.00.

Lastly, inasmuch as, according to the evidence appellant Lundberg was merely an agent of
his co-appellant, it is obvious that he cannot be held liable to appellee in connection with the
refund of the sum advanced by the latter.

ISSUE: W/N the buyer ( petitioner Katigbak) is liable for the balance of the winch ( 2,000)
that the vendor lost as he sold it to a third party for only 10,000?

HELD: YES

We quote from the Hanlon case:

.... In the present case the contract between Hanlon and the mining company was executory
as to both parties, and the obligation of the company to deliver the shares could not arise
until Hanlon should pay or tender payment of the money. The situation is similar to that
which arises every day in business transactions in which the purchaser of goods upon an
executory contract fails to take delivery and pay the purchase price. The vendor in such
case is entitled to resell the goods. If he is obliged to sell for less than the contract price, he
holds the buyer for the difference; if he sells for as much as or more than the contract price,
the breach of contract by the original buyer is damnum absque injuria. But it has never been
held that there is any need of an action of rescission to authorize the vendor, who is still in

SALES CASE DIGEST COMPILATION 221


possession, to dispose of the property where the buyer fails to pay the price and take
delivery... (40 Phil. 815) .

The facts of the case under consideration are identical to those of the Hanlon case. The
herein petitioner failed to take delivery of the winch, subject matter of the contract and such
failure or breach was, according to the Court of Appeals, attributable to him, a fact which We
are bound to accept under existing jurisprudence. The right to resell the equipment,
therefore, cannot be disputed. It was also found by the Court of Appeals that in the
subsequent sale of the winch to a third party, the vendor thereof lost P2,000.00, the sale
having been only for P10,000.00, instead of P12,000.00 as agreed upon, said difference to
be borne by the supposed vendee who failed to take delivery and/or to pay the price.

Of course, petitioner tried to draw a distinction between the Hanlon case and his case. The
slight differences in the facts noted by petitioner are not, however, to our mode of thinking,
sufficient to take away the case at bar from the application of the doctrine enunciated in the
Hanlon case.

38) Villanueva vs CA
G.R. No. 107624

Subject: Sales
Doctrine: meeting of the minds as to price is essential
Facts:
This is a petition assailing the decision of the CA dismissing the appeal of the petitioners. CA
rendered that there was no contract of sale.
– In 1985, Gamaliel Villanueva (tenant) of a unit in the 3-door apartment building owned by
defendants-spouses (now private respondents) Jose Dela Cruz and Leonila dela Cruz
located at Project 8, Quezon City.
– About February of 1986, Dela Cruz offered said parcel of land with the 3-door apartment
building for sale and plaintiffs, son and mother, showed interest in the property.
– Because said property was in arrears(overdue) in the payment of the realty taxes, dela
Cruz approached Irene Villanueva and asked for a certain amount to pay for the taxes so
that the property would be cleared of any incumbrance.
– Irene Villanueva gave P10,000.00 on two occasions. It was agreed by them that said
P10,000.00 would form part of the sale price of P550,000.00.
– Dela Cruz went to plaintiff Irene Villanueva bringing with him Mr. Ben Sabio, a tenant of
one of the units in the 3-door apartment building and requested Villanueva to allow said
Sabio to purchase one-half (1/2) of the property where the unit occupied by him pertained to
which the plaintiffs consented, so that they would just purchase the other half portion and
would be paying only P265,000.00, they having already — given an amount of P10,000.00
used for paying the realty taxes in arrears.
– Accordingly the property was subdivided and two (2) separate titles were secured by
defendants Dela Cruz. Mr. Ben Sabio immediately made payments by installments.
– March 1987 Dela Cruz executed in favor of their co-defendants, the spouses Guido Pili
and Felicitas Pili, a Deed of Assignment of the other one-half portion of the parcel of land
wherein plaintiff Gamaliel Villanueva’s apartment unit is situated, purportedly as full payment
and satisfaction of an indebtedness obtained from defendants Pili.
– the Transfer Certificate of Title No. 356040 was issued in the name of defendants Pili on
the same day.
– The plaintiffs came to know of such assignment and transfer and issuance of a new
certificate of title in favor of defendants Pili.
– plaintiff Gamaliel Villanueva complained to the barangay captain of Bahay Turo, Quezon
City, on the ground that there was already an agreement between defendants Dela Cruz and

SALES CASE DIGEST COMPILATION 222


themselves that said portion of the parcel of land owned by defendants Dela Cruz would be
sold to him. As there was no settlement arrived at, the plaintiffs elevated their complaint to
this Court through the instant action.
– RTC rendered its decision in favor of Dela Cruz. CA affirmed.
ISSUE: WON there was a perfected sale between Villanueva and Dela Cruz.
HELD:No
– Petitioners contend that private respondents’ counsel admitted that “P10,000 is partial or
advance payment of the property.” Necessarily then, there must have been an agreement as
to price, hence, a perfected sale. They cite Article 1482 of the Civil Code which provides that
“(w)henever earnest money is given in a contract of sale, it shall be considered as part of the
price and as proof of the perfection of the contract.”
– Private respondents contradict this claim with the argument that “(w)hat was clearly agreed
(upon) between petitioners and respondents Dela Cruz was that the P10,000.00 primarily
intended as payment for realty tax was going to form part of the consideration of the sale if
and when the transaction would finally be consummated.” Private respondents insist that
there “was no clear agreement as to the true amount of consideration.”
– Dela Cruz’ testimony during the cross-examination firmly negated any price agreement
with petitioners because he and his wife quoted the price of P575,000.00 and did not agree
to reduce it to P550,000.00 as claimed by petitioner.
– Villanueva on cross-examination: “After the Deed of Sale relative to the purchase of the
property was prepared, Mr. dela Cruz came to me and told me that he talked with one of the
tenants and he offered to buy the portion he was occupying if I will agree and I will cause the
partition of the property between us.” Villanueva said that he agreed and that the price
550,000 was to be divided into two. (Sabio and Villanueva) *The contract which the
appellant is referring to was not presented to the court and the appellant did not use all effort
to produce the said contract.
– SC: “The price of the leased land not having been fixed, the essential elements which give
life to the contract were lacking. It follows that the lessee cannot compel the lessor to sell the
leased land to him. The price must be certain, it must be real, not fictitious. A contract of sale
is not void for uncertainty when the price, though not directly stated in terms of pesos and
centavos, can be made certain by reference to existing invoices identified in the agreement.
In this respect, the contract of sale is perfected. The price must be certain, otherwise there is
no true consent between the parties. There can be no sale without a price.
– In the instant case, however, what is dramatically clear from the evidence is that there was
no meeting of mind as to the price, expressly or impliedly, directly or indirectly.
– Sale is a consensual contract. He who alleges it must show its existence by competent
proof. Here, the very essential element of price has not been proven.
– Lastly, petitioners’ claim that they are ready to pay private respondents is immaterial and
irrelevant as the latter cannot be forced to accept such payment, there being no perfected
contract of sale in the first place.

39) Republic of the Philippines


SUPREME COURT
Manila
EN BANC
G.R. No. L-17825 June 26, 1922
In the matter of the Involuntary insolvency of U. DE POLI.
FELISA ROMAN, claimant-appellee,
vs.
ASIA BANKING CORPORATION, claimant-appellant.

SALES CASE DIGEST COMPILATION 223


Wolfson, Wolfson and Schwarzkopf and Gibbs, McDonough & Johnson for appellant.
Antonio V. Herrero for appellee.
OSTRAND, J.:
FACTS:
It is hereby stipulated and agreed by and between Felisa Roman and Asia Banking
Corporation, and on their behalf by their undersigned attorneys, that their respective
rights, in relation to the 576 bultos of tobacco mentioned in the order of this court
dated April 25, 1921, be, and hereby are, submitted to the court for decision upon the
following:
I. Felisa Roman claims the 576 bultos of tobacco under and by virtue of the
instrument, a copy of which is hereto attached and made a part hereof and marked
Exhibit A.
II. That on November 25, 1920, said Felisa Roman notified the said Asia Banking
Corporation of her contention, a copy of which notification is hereto attached and
made a part hereof and marked Exhibit B.
III. That on November 29, 1920, said Asia Banking Corporation replied as per copy
hereto attached and marked Exhibit C.
IV. That at the time the above entitled insolvency proceedings were filed the
576 bultos of tobacco were in possession of U. de Poli and now are in possession of
the assignee.
V. That on November 18, 1920, U. de Poli, for value received, issued a quedan,
covering aforesaid 576 bultos of tobacco, to the Asia Banking Corporation as per
copy of quedan attached and marked Exhibit D.
VI. That aforesaid 576 bultos of tobacco are part and parcel of the
2,777 bultospurchased by U. de Poli from Felisa Roman.
VII. The parties further stipulate and agree that any further evidence that either of the
parties desire to submit shall be taken into consideration together with this
stipulation.
In the left margin of the face of the receipts, U. de Poli certifies that he is the sole owner of
the merchandise therein described. The receipt is endorced in blank "Umberto de Poli;" it is
not marked "non-negotiable" or "not negotiable."
Exhibit B and C referred to in the stipulation are not material to the issues and do not appear
in the printed record.
Though Exhibit A in its paragraph (c) states that the tobacco should remain in the
warehouse of U. de Poli as a deposit until the price was paid, it appears clearly from the
language of the exhibit as a whole that it evidences a contract of sale and the recitals in
order of the Court of First Instance, dated January 18, 1921, which form part of the printed
record, show that De Poli received from Felisa Roman, under this contract, 2,777 bales of
tobacco of the total value of P78,815.69, of which he paid P15,000 in cash and executed
four notes of P15,953.92 each for the balance. The sale having been thus consummated,
the only lien upon the tobacco which Felisa Roman can claim is a vendor's lien.
The order appealed from is based upon the theory that the tobacco was transferred to the
Asia Banking Corporation as security for a loan and that as the transfer neither fulfilled the
requirements of the Civil Code for a pledge nor constituted a chattel mortgage under Act No.
1508, the vendor's lien of Felisa Roman should be accorded preference over it.
ISSUE: W/N the warehours receipts was a negotiable instrument, hence the right of the Asia
banking Corp as an endorsee is superior to that vendor’s lien ( Felisa Roman?

SALES CASE DIGEST COMPILATION 224


HELD : Yes. The supreme court noted that, Where a negotiable receipts has been issued
for goods, no seller's lien or right of stoppage in transitu shall defeat the rights of any
purchaser for value in good faith to whom such receipt has been negotiated, whether such
negotiation be prior or subsequent to the notification to the warehouseman who issued such
receipt of the seller's claim to a lien or right of stoppage in transitu. Nor shall the
warehouseman be obliged to deliver or justified in delivering the goods to an unpaid seller
unless the receipt is first surrendered for cancellation.
The term "purchaser" as used in the section quoted, includes mortgagee and pledgee. (See
section 58 (a) of the same Act.)
In view of the foregoing provisions, there can be no doubt whatever that if the warehouse
receipt in question is negotiable, the vendor's lien of Felisa Roman cannot prevail against
the rights of the Asia Banking Corporation as the indorse of the receipt. The only question of
importance to be determined in this case is, therefore, whether the receipt before us is
negotiable.
The matter is not entirely free from doubt. The receipt is not perfect: It recites that the
merchandise is deposited in the warehouse "por orden" instead of "a la orden" or "sujeto a la
orden" of the depositor and it contain no other direct statement showing whether the goods
received are to be delivered to the bearer, to a specified person, or to a specified person or
his order.
We think, however, that it must be considered a negotiable receipt. A warehouse receipt, like
any other document, must be interpreted according to its evident intent (Civil Code, arts.
1281 et seq.) and it is quite obvious that the deposit evidenced by the receipt in this case
was intended to be made subject to the order of the depositor and therefore negotiable. That
the words "por orden" are used instead of "a la orden" is very evidently merely a clerical or
grammatical error. If any intelligent meaning is to be attacked to the phrase "Quedan
depositados en estos almacenes por orden del Sr. U. de Poli" it must be held to mean
"Quedan depositados en estos almacenes a la orden del Sr. U. de Poli." The phrase must
be construed to mean that U. de Poli was the person authorized to endorse and deliver the
receipts; any other interpretation would mean that no one had such power and the clause,
as well as the entire receipts, would be rendered nugatory.
Moreover, the endorsement in blank of the receipt in controversy together with its delivery by
U. de Poli to the appellant bank took place on the very of the issuance of the warehouse
receipt, thereby immediately demonstrating the intention of U. de Poli and of the appellant
bank, by the employment of the phrase "por orden del Sr. U. de Poli" to make the receipt
negotiable and subject to the very transfer which he then and there made by such
endorsement in blank and delivery of the receipt to the blank.
As hereinbefore stated, the receipt was not marked "non-negotiable." Under modern statutes
the negotiability of warehouse receipts has been enlarged, the statutes having the effect of
making such receipts negotiable unless marked "non-negotiable." (27 R. C. L., 967 and
cases cited.)
Section 7 of the Uniform Warehouse Receipts Act, says:
A non-negotiable receipt shall have plainly placed upon its face by the
warehouseman issuing it 'non-negotiable,' or 'not negotiable.' In case of the
warehouseman's failure so to do, a holder of the receipt who purchased it for value
supposing it to be negotiable may, at his option, treat such receipt as imposing upon
the warehouseman the same liabilities he would have incurred had the receipt been
negotiable.
This section shall not apply, however, to letters, memoranda, or written
acknowledgments of an informal character.
This section appears to give any warehouse receipt not marked "non-negotiable" or "not
negotiable" practically the same effect as a receipt which, by its terms, is negotiable

SALES CASE DIGEST COMPILATION 225


provided the holder of such unmarked receipt acquired it for value supposing it to be
negotiable, circumstances which admittedly exist in the present case.
We therefore hold that the warehouse receipts in controversy was negotiable and that the
rights of the endorsee thereof, the appellant, are superior to the vendor's lien of the appellee
and should be given preference over the latter.

43) OCEJO v. INTERNATIONAL BANKING CORPORATION


106Phil.491

FACTS:
This suit for mandatory injunction was instituted in the Court of First Instance of Cebu by the
Cebu. United Enterprise to compel Jose Gallofin, as Collector of Customs, Cebu Port, to
release and deliver to the plaintiff two imported shipments of 7,834 bales of over-issue
newspapers purchased by the latter from the United States. As ancillary relief during the
pendency of the action, the plaintiff prayed for the issuance of a writ of preliminary
mandatory injunction, which was granted by the court after the plaintiff posted a bond in
the amount of P60,000.00 in favor of the defendant. Thereafter, the goods were released
to the plaintiff, it appearing further that the advance sales tax due on the same had
been duly paid upon arrival of the merchandise at port.

The importation of the aforesaid shipments was made under and by virtue of
an Import Control Commission License No. 1225 issued by the defunct Import Control
Commission. Under the terms of the license, the plaintiff could import, on a no-
dollar remittance basis, over-issue newspapers up to the amount or value of $118,000.00.

The refusal of the defendant to deliver the imported items is premised on his contention
that while the five bills of lading covering the two shipments of the over issue newspapers
were all dated at Los Angeles, U.S.A. December 17, 1953, or one day before the
expiration of the import license in question, the vessels M/S Ventura and M/S Bataan,
carrying on board the said merchandise, actually left the ports of embarkation,
Los Angeles, and San Francisco, on January 12 and January 16, 1954
respectively. Hence, according to the defendant, the importation must be considered as
having been made without a valid import license, because under the regulations issued
by the Central Bank and the Monetary Board, "all shipments that left the port of origin after
June 30, 1953, and are covered by ICC licenses, may be released by the Bureau of
Customs without the need of a Central Bank release certificate; provided they left the port
of origin within the period of validity of the licenses". No Central Bank certificate for the
release of the goods having been shown or presented to the defendant, the latter
refused to make the delivery.

ISSUE: W/N the lower court erred in favouring the petitioner as opposed to defendants claim
to refuse to deliver imported items?

HELD: No, The lower court was thus confronted with the issue of determining
whether the valid period of the license in question should be counted up to the time when
the vessels carrying the imported items left the ports of origin on January 12 and January
16, 1954, or when the corresponding bills of lading were dated, or December
17, 1953. The court chose the latter date, and held:

"IN VIEW THEREFORE, this Court pronounces judgment making the writ of preliminary
mandatory injunction issued against defendant permanent, with orders for the cancellation of
plaintiff's bond, this after whatever advance sales tax or any taxes, surcharges and so forth
might be due on the goods shall have been paid, without costs."

SALES CASE DIGEST COMPILATION 226


The authority of the appellee to import was contained in the Import
Control Commission License No. 17225, validated on June 18, 1953, and under
Resolution 70 of the Commission (adopted March 27, 1952), the same had a six-month
period of validity counted from the said date of June 18, 1953. This license states-
, among other conditions, that

"Commodities covered by this license must be skipped from the country of origin before
the expiry date of the license, and are subject to sec. 13 of Republic Act No. 650."
Although Republic Act No. 650, creating the Import Control Commission, expired on July
31, 1953, it is to be conceded that its duly executed acts can have valid effects
even beyond the life span of said governmental agency.

What is important to consider only is the legal connotation of the word "shipped" as the
term was used in the license. Defendant maintains that it is when the vessel leaves the
port of embarkation, while plaintiff holds that it is the dates of the bills of lading, which are
usually issued after the cargo is placed on board the vessel. That the date of the shipment
is the date when the goods for dispatch are loaded on board the vessel, and not
necessarily when the ship puts to sea, is clearly implied from our ruling in the case of U.S.
Tobacco Corporation vs. Rufino Luna, et al., (87 Phil., 4), wherein we said:

"By section 6 of Act No. 426, all goods including leaf tobacco have been placed under
control. Petitioner's merchandise left the port of departure before the passage of that Act
but arrived in Manila after its approval. For the purpose of enforcing or applying said
Section 6, there can only be one date of importation. Which was the date? The date the
goods were ordered the date they were put on board vessel, or the date they reached the
port of destination? We are of the opinion that the date of importation is. the date of
shipment and not the date of arrival in Manila." (Italics supplied)
The issuance of the bill of lading, furthermore, presupposes or carries the presumption
that the goods were delivered to the carrier for immediate shipment (13 C. J. S. sec. 123
(2). pp. 235, and cases cited therein). It does not appear here that the bill of lading
specified any designated day on which the vessels were to lift anchor, nor was it shown that
plaintiff had any knowledge that the vessels M/S Ventura and M/S Bataan were not to
depart soon after he placed his cargo on board and the corresponding bills of lading issued
to him. From this latter time, the goods, in contemplation of law, are deemed already in
transit (New Civil Code, Arts. 1531 and 1736).

It should also be considered that it is entirely outside the shipper's hands to fix the dates of
departure, route or arrival of a vessel (unless he charters the whole ship [see Art. 656,
Code of Commerce]).

Defendant's reliance upon Central Bank regulations that the shipment licensed must have
"left the port of origin within the period of validity of the license" is not maintainable in the
present case, because the regulations came into effect only on July 1, 1953 already
after issuance of the appellee's license and can not be "read into the same (see 49 Off.
Gaz. No. 6, p. 2189).

The Solicitor General's contention that, assuming the six months are counted up to the date
the imported goods were placed on board the vessels for shipment the period of validity
had likewise already elapsed because, legally, six months mean 180 days, which in this
case expired on December 15, cannot now be entertained because the defendant-
appellant, under paragraph 3 of his Answer to the "Complaint, expressly admitted that the
date appearing on the bills of lading (December 17, 1953) as the date of loading on board
the vessels " is one day before the expiration of the validity of the import license". What he

SALES CASE DIGEST COMPILATION 227


only questioned in the court below is the legal connotation of the word "shipped" under the
import license.

44) SEMIRA V. CA 230 SCRA 577


FACTS: Gutierrez was the owner of a parcel of land. This parcel was sold to Buenaventura
An. He entered the premises based on the boundaries stated in the deed of sale. He then
bought two additional parcels of land. On a relevant date, he sold the first parcel to his
nephew who also entered the premises based on the boundaries stated in the deed. The
deed also stated the same boundaries and area of the lot, which was larger in actuality. This
nephew then sold the land to petitioner. The deed this time reflected a different area, the
actual area of the land. The land was found to be larger than what was stated in the previous
documents. Semira entered then the premises based on the boundaries and began
construction of a rice mill. Buenaventura then filed an action for forcible entry against
Semira, alleging that latter illegally encroached on the other parcel of land previously bought
by the former and that the land that was supposed to be occupied by the latter was smaller
than the land he was actually occupying.
HELD: In the case at bar, the issue of possession cannot be decided independently of the
question of ownership. Private respondent claimed constructive possession of the parcel of
land he alleged to be encroached by Semira. Likewise, Semira based his occupancy of the
land by virtue of the Ramirez’s sale of the land to him. The question of prior possession may
only be resolved in answering the question of who is the real owner of the disputed portion.
Where land is sold for a lump sum and not so much per unit of measure, the boundaries of
the land stated in the contract determines the effects and scope of the sale, not the area
thereof. The vendor is thus obligated to deliver the land included within the boundaries
regardless of whether the land is greater or lesser than the area stipulated in the sale.

44) OJECO, PEREZ, AND CO. 37Phil.631

FISHER, J.:
FACTS: On the 7th day of March, 1914, Chua Teng Chong of Manila, executed and
delivered to the International Banking Corporation, hereinafter referred to as "the bank," a
promissory note, payable one month after date, for the sum of P20,000. Attached to this
note was another private document, signed by Chua Teng Chong, in which it was stated that
he had deposited with the bank, as security for the said note, 5,000 piculs of sugar, which in
said document were said to be stored in a warehouse situated at No. 1008, Calle Toneleros,
Binondo, Manila. It appears from the evidence, assuming that the sugar was in the
warehouse on that date, that the bank did not take possession of it when the document was
executed and delivered, and that Chua Teng Chong continued to retain the sugar in his
possession and control. The bank made no effort to exercise any active ownership over said
merchandise until the 16th of April, when it discovered that the amount of sugar stored in the
said warehouse was much less than the 5,000 piculs mentioned in the contract. The
agreement between the bank and Chua Teng Chong with respect to the alleged pledge of
the sugar was never recorded in a public instrument. It does not appear from the evidence
that the promissory note represents money delivered by the bank on the date of its
execution, although it is stated therein that it was executed for value received.
On the 24th day of March, 1914, the plaintiff partnership Ocejo, Perez & Co., entered into a
contract with Chua Teng Chong for the sale to him of a lot of sugar. It was agreed that
delivery should be made in the month of April, the sugar to be weighed in the buyer's
warehouse. It appears that this sugar was brought to Manila by a steamer in the month of
April, and 5,000 piculs were delivered by plaintiff to Chua Teng Chong. The delivery was

SALES CASE DIGEST COMPILATION 228


completed April 16, 1914, and the sugar was stored in the buyer's warehouse situated at No.
119, Muelle de la Industria. On April 17, 1914, plaintiff partnership presented, for collection,
its account for the purchase price of the sugar, but the buyer refused to make payment, and
up to the present time the sellers have been unable to collect the purchase price of the
merchandise in question.
On the same date as that on which the 5,000 piculs of sugar were delivered into the
warehouse on Muelle de la Industria, the bank sent an employee to inspect the sugar
described in the pledge agreement, and which, as therein stated, should have been stored in
the Calle Toneleros warehouse. The bank's representative then discovered that the amount
of sugar in that warehouse did not exceed 1,800 piculs, whereas the amount which should
have been there, according to the contract, was 5,000 piculs. Upon making this discovery,
the bank's representative, accompanied by a lawyer, went immediately to see Chua Teng
Chong, and the latter informed him that the rest of the sugar covered by the pledge
agreement was stored in the warehouse at No. 119, Muelle de la Industria. The bank's
representative immediately went to this warehouse and upon arrival there found some 3,200
piculs of sugar, of which he took immediate possession, closing the warehouse with the
bank's padlocks. It is admitted that the sugar seized by the bank in the Muelle de la Industria
warehouse is the same sugar which the plaintiff firm delivered to Chua Teng Chong. On the
date on which the bank took possession of the sugar the promissory note executed March
17, 1914, had fallen due and was unpaid.
In the written contract by which the plaintiff firm undertook to sell the sugar in question to
Chua Teng Chong nothing was said concerning the time and place for payment. The court
below found that the delivery of the sugar by plaintiff to Chua Teng Chong was made upon
the mutual understanding that the price was to be paid in cash "upon the completion of
delivery." The plaintiff firm proved that in sales of this kind it is the custom among merchants
in Manila for the seller to deliver the merchandise into the warehouse of the buyer, for
inspection and verification of weights, and that as soon as this operation is completed, the
price is payable on demand. After Chua Teng Chong had refused to pay the bill for the price
of the sugar which the plaintiff firm presented to him, the day after" its delivery, an attempt
was made by plaintiff to recover possession of the sugar, and to that end, on April 24, 1914,
the plaintiff made a demand on the bank for the delivery of the sugar, to which demand the
bank refused to accede. On April 24, 1914, the buyer Chua Teng Chong was judicially
declared to be insolvent, and Francisco Chua Seco was appointed as assignee of the
insolvency. On the same date, and a few minutes after the insolvency proceedings "were
commenced, the plaintiff partnership filed a complaint, upon which this action was
commenced, naming the bank as defendant, alleging that said defendant was unlawfully
holding some 4,711 pilones of sugar, the property of the plaintiff firm, which the bank had
received from Chua Teng Chong, and prayed for judgment for the possession of said sugar.
A few days after, the plaintiff firm took advantage of those provisions of the procedural law
which permit a plaintiff to replevin personal property. Subsequently, by agreement of the
parties, the sugar was sold and the proceeds of the sale deposited in bank, subject to the
order of the court upon the final disposition of the case. After the answer of the defendant
bank was filed, a complaint in intervention was filed by Chua Seco, in which he asserts a
preferential right to the sugar, or to the proceeds of its sale, upon the ground that the
delivery of the sugar by plaintiff, by virtue of which it passed into the possession and control
of Ghua Teng Chong, had the effect of transmitting the title of the sugar to the latter, and
that, on the other hand, the pledge asserted by the bank was null and void. Upon these
allegations the intervener contends that the sugar is the property of the insolvent estate
represented by him.
ISSUE:W/N the assignee Chong is entitled to the product of the sale of the suger in question
(10,826.76) reserving to the seller the right to file his claim in the insolvency proceedings.

SALES CASE DIGEST COMPILATION 229


HELD: Yes. We believe that the contention of the defendant bank cannot be sustained. In
the first place, even giving all possible effect to the contract evidenced by the private
document exhibited by the bank (Exhibit No. 1), it is evident that the sugar therein mentioned
is not the same as that here in dispute. By this document, which bears date March 4, 1919,
an attempt was made to pledge the lot of sugar deposited in warehouse No. 1008, Calle
Toneleros, Manila. The sugar in dispute has never been in that warehouse, as the seller
delivered it into the bodega at No. 119, Muelle de la Industria. The sugar here in question
could not possibly have been the subject matter of the contract of pledge which the parties
undertook to create by the private document dated March 7, 1914, inasmuch as it was not at
that time the property of the defendant, and this constitutes an indispensable requisite for
the creation of a pledge. (Civil Code, art. 1857.) It does not appear from the record that any
effort was made to pledge the sugar which is the subject matter of this case. It is true that it
appears that in the afternoon of the day the sugar was delivered, the buyer gave the bank's
representative the keys of the warehouse on the Muelle de la Industria in which the sugar
was stored, but it also appears from the testimony of the bank's witness, Grey, to whom the
keys of the warehouse were delivered, that this was not done because of an agreement
concerning the pledge of the sugar now in dispute. Grey testified that on the afternoon of
April 16, 1914, he ascertained, after an inspection of the warehouse on Calle Toneleros, that
the sugar therein stored was not more than 1,800 piculs, instead of five .thousand, as stated
in the document of pledge; that upon observing this shortage he asked the debtor to account
for it, whereupon the latter stated "that the rest was in the warehouse at No. 119, Muelle de
la Industria;" that upon receiving this reply the witness went to the warehouse at No. 119,
Muelle de la Industria, demanded the keys from the person in charge, and then closed the
warehouse with the bank's own padlocks. From these statements it appears that no attempt
was made to enter into any agreement for the pledge of the sugar here in question. The
bank took possession of that sugar under the erroneous belief, based upon the false
statement of Chua Teng Chong, that it was a part of the lot mentioned in the private
document dated March 7, 1914. But even if it were assumed that on the afternoon of April
16, 1914, an attempt was made to pledge the sugar and that delivery, was made in
accordance with the agreement, the pledge so established would be void as against third
persons. Article 1865 of the Civil Code provides that a pledge is without effect as against
third persons "if the certainty of the date does not appear by public instrument." In the case
of Tec Bi & Co. vs. Chartered Bank of India, Australia and China, 16 Off. Gaz., 908 decided
February 5, 1916, this court held that when the contract of pledge is not recorded in a public
instrument, it is void as against third persons; that the seller of the thing pledged, seeking to
recover the purchase price thereof, is a third person within the meaning of the article cited;
and that the fact that the person claiming as pledgee has taken actual physical possession
of the thing sold will not prevent the pledge from being declared void as against the seller.
The court held that the principle established by article 1865 of the Civil Code is not adjective
in its character, but that "it prescribes a condition without which the contract of pledge cannot
adversely affect third persons." Applying the doctrine of the decision cited, it is evident that
the pledge asserted by the International Bank is inefficacious.
In the brief filed on behalf of the bank it is argued that in no case may a revindicatory action
be maintained when the plaintiff attempts to exercise the right to rescind the sale for
nonpayment of the purchase price and that therefore a replevin suit will not lie. But as it is
held that the bank has no interest in this matter, as its alleged contract of pledge is utterly
unavailing, it is evident that the question of procedure does not affect it. It appears that by
reason of the insolvency of the buyer Chua Teng Chong an insolvency proceeding was
commenced in a court of competent jurisdiction and in that proceeding Francisco Chua Seco
was appointed assignee of the property of the insolvent. As such assignee Chua Seco filed
a complaint in intervention in this suit, in which he contends that by reason of its sale and
delivery by plaintiff to the insolvent, title to the sugar passed to the latter and that the pledge
set up by the bank is void as to third persons. Standing in the place of the insolvent buyer,
the assignee asks that he be recognized in his representative capacity as the owner of the
sugar in question. The voluntary intervention of the assignee of the insolvent buyer cures the

SALES CASE DIGEST COMPILATION 230


defect of nonjoiner of the latter as a party defendant, and all parties in interest have been
heard in this proceeding.
The judgment of the court below awards the plaintiff the product of the sale of the sugar, it
having been so disposed of by agreement by the parties during the pendency of the suit.
The intervener excepted to the decision and joined in the bank's appeal. In his brief in this
court the intervener raises a question as to the sufficiency of the complaint to support the
decision of the court below, adopting the argument of the bank upon this point. That is,
assuming that by reason of the nonpayment of the purchase price, the seller is entitled to
elect to rescind the sale, is the rescission effected ipso facto by such election, or is it
necessary for him to bring an action of rescission? The action of replevin, the intervener
contends, is based (Code of Civil Procedure, sec. 263) upon the assumption that the plaintiff
at the time of bringing the action is either the owner of the thing which is the subject matter
of the suit or entitled to its possession. But the question presented is whether, in cases in
which title has passed by delivery and in which the buyer has failed to pay the purchase
price on demand, title is revested in the seller by the mere fact that he has mentally
determined to elect to rescind? In its brief the plaintiff partnership contends for the
affirmative, saying that the acts of the seller the filing of its complaint imply that it has made
the election. But the intervener, adopting the argument of the bank, contends that the party
to whom article 1124 of the Civil Code grants the right to rescind "must apply to the court for
a decree for the rescission of the contract * * *" (Scaevola, vol. 19, p. 673) ; and this
conclusion is supported by the last paragraph of the article cited. Of course, if the action of
the court is necessary in order to effectuate the rescission of the sale, such rescission does
not follow ipso jure by reason of nonpayment and the determination of the seller to elect to
rescind. Consequently, the action of replevin cannot be, maintained. The right to rescind a
sale, established by article 1506, in no wise differs. from that which is established, in general
terms, with respect to reciprocal obligations, by article 1124 in "the event that one of the
obligors fails to perform the obligation incumbent upon him." But the right so conferred is not
an absolute one. The same article provides that "the court shall decree the rescission
demanded, unless there are causes. which justify him in allowing a term."
Therefore, it is the judgment of the court and not the mere will of the plaintiff which produces
the rescission of the sale. This being so, the action of replevin will not lie upon the theory
that the rescission has already taken place and that the seller has recovered title to the thing
sold.
If the buyer himself had intervened, instead of the assignee in the bankruptcy suit, we might
perhaps have said that all the parties in interest having been heard, we would overlook the
matter of procedure and proceed to adjudicate the rights of the parties upon the evidence
submitted. But as the buyer has been declared insolvent, it is clear that his creditors have an
interest in this question, and that if this interest is discussed in the bankruptcy proceedings,
they will have an opportunity to be heard. In the present condition of the case, the only thing
we can do is to decide that the title to the sugar having passed to the buyer and no action for
rescission having been commenced against him before the declaration of insolvency, the
assignee, standing in the shoes of the buyer, has a better right to its possession or to the
product of its sale during the pendency of this action. We cannot apply section 126 of the
Code of Civil Procedure, because one of the material averments of the complaint is that
Chua Teng Chong unlawfully took possession of the sugar. The evidence shows, on the
contrary, that it was delivered to him by plaintiff. Strictly speaking the mission of the court
ends at this point, but following the. practice adopted in other cases, for the purpose of
avoiding an unnecessary multiplicity of suits, and bearing in mind the fact that the assignee
of the bankruptcy is. a party to this proceedings, we deem it advisable to indicate that we are
of the opinion that the rights of the seller are protected by section 48 of Act No. 1956,
inasmuch as the sugar in question had not passed by an "irrevocable title" when the buyer
was declared insolvent. Attention is also invited to the decision of the court overruling the
motion for a rehearing in the case of Tec Bi & Co. vs. Chartered Bank of India, Australia &

SALES CASE DIGEST COMPILATION 231


China, cited above.

45) [ G. R. No. L-1724, October 12, 1950 ]


NIEVES VDA. DE GONZALEZ MONDRAGON, PLAINTIFF AND APPELLANT, VS.
ROMAN SANTOS, DEFENDANT AND APPELLEE.
G.R. No. L-1724 October 12, 1950

NIEVES VDA. DE GONZALES DE MONDRAGON, plaintiff-appellant,


vs.
ROMAN SANTOS, defendant-appellee.

Vicente J. Francisco for appellant.


Sixto de la Costa and Severiano B. Orlina for appellee.

TUASON, J.:

FACT

It appears that Don Joaquin Gonzales Mondragon, who died on December 16, 1940 in
Manila, left a large tract of land known as Hacienda Esperanza, situated in three
municipalities of Pangasinan and covered by five certificates of titles. The deceased had
executed a will and codicil in which he provided for the distribution and disposition of his
estate among his widow, Doña Nieves Balmori Vda. de Gonzales Mondragon, the plaintiff
herein, and various children. To his widow, the testator devised 33/34 of the hacienda,
among other legacies.

In 1941, the widow and her children made a partition of the inheritance, allotting to each heir
separate and specific portions but leaving pro-indiviso the residential lots and roads in the
barrios situated within the estate. They employed a surveyor, and a sub-division plan,
introduced in evidence as Exhibit 10, was drawn, on which the area of the widow’s
approximately one-third share was stated to be 1,023 hectares.

Subsequent to the partition, negotiations were started, or resumed, for the purchase by Don
Roman Santos, the defendant, of the plaintiff’s share and those of her children who were
willing to sell. Offers and counter-offers were made until, finally, the parties closed the deal
and executed the deed.

Sometime after the sale, a new survey was made and the new plan gave the area of the
plaintiff’s approximately one-third share of the hacienda as 1,091.24 instead of 1,023.

It was the restoration of the difference between these two figures or the payment of its
equivalent in cash that the first complaint was filed, it being alleged that the plaintiff had sold
her land on the basis of P450 per hectare. Explaining why she signed the deed without
objecting to the form in which it was written, the plaintiff declared that she did not read the
document because she was then sick suffering from a heart ailment. The defendant
countered with the allegation that he bought all the plaintiff’s right and interest to and in the
hacienda for lump sum and not for a specified price for each hectare, as the plaintiff claims.

SALES CASE DIGEST COMPILATION 232


The last preceding paragraph states in a nutshell the pivotal issue, the resolution of which
will decide the rest, except the question as to the inclusion or non-inclusion in the sale of lot
No. 4397-A and barrio lots and roads, question will be taken up separately.

It is admitted that if the contract is to be construed by the language used in the deed of
conveyance, the plaintiff can not recover. It is also admitted that “as a general rule, by virtue
of section 22 of Rule 123 of the Rules of Court, Exhibit A may be considered as containing
the real agreement between the parties.”

ISSUE: But it is contended that “Exhibit A does not express the true intent and agreement of
the parties therein and that the appellant’s consent thereto was given through mistake and
error,” in that she believed “that in signing that deed she was conveying 1,023 hectares
only.”

HELD:The plaintiff has the burden of proof to overcome the strong presumption that the
document she and her co-sellers signed, expressed their true intention. Our view of the
plaintiff’s evidence is that it is neither predominant nor conclusive. The best that can be said
in its favor is that it does not rule out the opposite theory. Much less does it establish, in
order to show that the mistakes was mutual, that the buyer shared the vendor’s intention and
belief that the sale was by the hectare and not for a sum in gross as stated in the document
of sale.

The plaintiff’s evidence being as it is, the integrity of the document Exhibit A will, of
necessity, have to be maintained and equitable relief denied. This would be true even if
there were doubts. Decisions of this court and of American courts abound in favor of the
salutary doctrine that contracts solemnly and deliberately entered into may not be
overturned by inconclusive proof or by reason of mistakes of one of the parties to which the
other in no way has contributed.

Moran’s comments on the Rules of Court, Vol. III, p. 195, summing up the rulings laid down
in various decisions of the court and one of the United States Supreme Court, says: “Relief
by way of reformation of a written agreement will not be granted unless the proof of mutual
mistake is of the clearest and most satisfactory character. The amount of evidence
necessary to sustain a prayer for relief where it is sought to impugn a fact in a document is
always more than a mere preponderance of the evidence.”

In the case of Joaquin vs. Mitsumine (34 Phil., 858), this court held that “An alleged defect in
a contract perfectly valid and binding on its face, must be conclusively proved. The validity
and fulfillment of contracts can not be left to the will of one of the parties.”

In the case of Irureta Goyena vs. Tambunting (1 Phil., 490), it appeared that the defendant
bought a piece of land and agreed to pay $3,200 for it. It so happened that the land was less
than what the parties supposed, and the buyer refused to pay the price agreed upon unless
the corresponding reduction was made.

It is to be noted that in the last-cited case, the mistake was caused, intentionally or
innocently, by the agent of the plaintiff who was favored by the shortage, whereas in the
case at bar the error was in the plain of the plaintiff herself who was prejudiced by the
excess.

SALES CASE DIGEST COMPILATION 233


Similarly, the plaintiff had exclusive right to sell her share in the residential lots and roads in
the barrios within the Hacienda although they were still pro-indiviso. As a matter of fact, she
was selling only her right and interest in these lots and roads and not any specific parts
thereof.

The judgment dismissing the complaint will be affirmed with costs.

47) VALENTIN PACIA v. ISIDORO LAGMAN=


63 Phil. 361

DIAZ, J.:
FACTS:
The spouses Valentin Pacia and Toribia Lagman filed a petition for the confirmation and
registration in the registry of deeds of their alleged title to the lands described as parcels
Nos. 1, 2, 3, 4 and 5, in the plans Exhibits A and B, and in the sheet of technical descriptions
attached thereto, referred to in the record as Exhibit B-1. The petition was opposed on one
side by Anacleta Lagman, Felisa Lagman, Gaudencio Santos, Emiliana Santos,
Carmen Santos and Juan Garcia; on another side by Isidoro Lagman, with respect to a
strip of land extending from east to west along the northern side of parcel No. 1, plan Exhibit
A, with an area of 1 hectare, 29 ares and 51 centiares; on another side by Tiburcia Buan,
with respect to parcel No. 4 and the creek separating it from parcel No. 1, and on still
another side by Cristino Lagman, as to Batasan Creek along the southern side of
parcel No. 2, plan Exhibit B.

The petitioners, agreeing to Tiburcia Buan's opposition, excluded parcel No. 4 from their
application.

After the judicial proceedings, the lower court rendered judgment confirming the title of the
petitioners as to parcels Nos. 1, 2, 3 and 5, except the strip of land alongside parcel No. 1,
claimed by Isidoro Lagman; sustaining Isidoro Lagman's opposition with respect to said strip
and that of Tiburcia Buan as to parcel No. 4 and the creek separating it from parcel No.
1; declaring Cristino Lagman's opposition unfounded on the ground that the Batasan
Creek was not included in the petitioners' plan, and ordering the petitioners to file an
amended plan in accordance with said judgment.

The petitioners appealed from the judgment of the lower court depriving them of the strip of
land alongside parcel No. 1, claimed by Isidoro Lagman; and the oppositors headed by
Anacleta Lagman also appealed because their opposition was entirely overruled. The
petitioners base their appeal upon the alleged error committed by the lower court in
sustaining Isidoro Lagman's opposition in spite of their evidence, and in denying their
motion for a new trial based upon their allegation that the decision was contrary to law and
the weight of the evidence.

The oppositors Anacleta Lagman and her nephews and nieces alleged in their opposition
that they were the exclusive owners of the lands in question. During the trial, and after
seeing the documentary evidence, Exhibits E and F, for the petitioners, the genuineness of
which they could not deny, they adopted another theory saying that they had sold to
the petitioners only a part of parcel No. 1, with an area of three balitas, and another
part of parcel No. 2, with an area of two balitas. Sometime later, however, they adopted
another new theory in view of the fact that they could not prove occupation thereof, stating
that they allowed the petitioners to occupy said lands because the latter had requested

SALES CASE DIGEST COMPILATION 234


their permission to occupy said lands for at least two years in order to be indemnified for
the expenses incurred by them for having made improvements thereon. The truth, however,
is that aside from the testimony of their witnesses Mauricio Duenas, husband of the
oppositor Anacleta Lagman, Juan Garcia, nephew of said oppositor, and Cristino Lagman,
a close relative of them all, they presented no other evidence in support of their
claim. They did not even declare said lands as their own for land tax purposes from the
time they sold said lands with pacto de retro in 1902.

ISSUE: W/N the opposition of Anacleta Lagman would prosper?

HELD: NO, the Supreme Court noted that it will be seen that the lands referred to therein
are exactly the same. If Anacleta Lagman and the predecessors of her co-oppositors had
reserved for themselves some portion of the lands in question upon executing the deeds of
transfer above mentioned, we would necessarily find evidence thereof in said documents
because it would have been indicated therein that the land sold adjoined another piece of
land or other lands belonging to the grantors, and it is a fact that it does not so appear
therein. There is therefore no doubt that the opposition of Anacleta Lagman andher
nephews and nieces was unfounded and that the decision of the lower court as to said
question was in accordance with law and the evidence.

As to the petitioners' appeal, it should be stated that all the members of the court, with the
exception of one who was of the opinion that the preponderance of the evidence is in favor
of the petitioners; that Isidoro Lagman failed to prove his alleged possession; that he
neither indicated the boundaries of the land in question on the south nor proved that there
were fences, hedges or dikes separating it from that of the petitioners, declared it
unfounded. They hold with the lower court that the strip of land disputed by the parties,
with an area of 1 hectare, 29 ares and 51 centiares, originally belonged to German
Lagman, Isidoro Lagman's father; that after German Lagman's death, which took place in
1914, Isidoro Lagman succeeded him in the occupation and cultivation of the land in
question under claim of ownership; that as such he declared it for land tax purposes for the
first time in 1916, and thereafter he had been regularly paying the corresponding tax every
year.

The petitioners certainly proved that they also occupied the strip of land in question and
availed themselves of the fruits thereof from the time they acquired it in 1902 from Isidoro
Lagman's cousins named Gregorio, Anacleta, Maria, and Segunda Lagman; and that, as
said oppositor, they had it assessed for taxation purposes in their name in 1906, paying the
corresponding land taxes uninterruptedly. This court, however, following the ruling laid
down in the cases of Baltazar vs. Alberto (33 Phil., 336); Lim Soco vs. Roxas (26 Phil.,
609); United States vs. Rice (27 Phil., 641); and United States vs. Melad (27 Phil., 488),
and taking into consideration the fact that the lower court personally heard and saw all
the witnesses testify during the trial and, furthermore, had the opportunity to observe them
and pass upon their greater or less credibility, does not feel justified in altering the
conclusions of said court.

SALES CASE DIGEST COMPILATION 235


ROXANNE

49) LUMBRES v. TABLADA

FACTS:

On January 9, 1995, Spring Homes entered into a pro forma Contract to Sellwith the
respondent spouses. The prepared typewritten contract, with the blank spaces therein
merely filled up, contains the designation of the parcel sold, the price per square meter and
the stipulation as to payment.

On January 16, 1996, after having been paid the sum total of ₱179,500.00, which the
respondents claim to be the full purchase price of the subject lot, Spring Homes executed a
Deed of Absolute Sale in favor of the respondents.

Respondents later declared the subject lot for taxation purposes under Tax
Declaration No. 019-1342 and paid the corresponding real property taxes thereon. Using
their own funds, they caused the construction thereon of a residential house, which they
presently occupy, the costs of which amounted to ₱356,516.50. On June 9, 1996, a
Certificate of Occupancy was issued to them by the Office of the Building Official and the
house was declared in their names.

The respondent spouses sent a demand letter dated May 4, 1996 to Spring Homes
for the transfer and release to them of the original or owner's copy of TCT.

On September 3, 1997, the RTC issued an order attaching all of Spring Homes
properties, including Lot 8, Block 3. Premiere Development Bank subsequently intervened in
Civil Case No. 2194-95-C because all said properties had been mortgaged to it.

Meanwhile, due to the respondents’ alleged failure to pay the ₱230,000.00 unpaid
balance as per the Contract to Sell earlier adverted to despite demands, the subject lot was
sold by Spring Homes to the petitioners, again by way of a Deed of Absolute Sale executed
on December 22, 2000 for and in consideration of the sum of ₱157,500.00. The mortgage
on the lot was released by Premiere Development Bank on January 20, 2001. Subsequently,
on January 30, 2001, TCT No. T-473055 covering the subject lot was issued in petitioners'
favor

ISSUE:

Who has a better right over the property?

HELD:

The respondents have the better right.

Notwithstanding the fact that the petitioners, as the second buyer, registered their
Deed of Absolute Sale, in contrast to the Deed of Sale of the respondents which was not
registered at all precisely because of Spring Homes’ failure to deliver the owner’s copy of
TCT No. T-284037, the respondents’ right could not be deemed defeated as the petitioners
are in bad faith. Petitioners cannot claim good faith since at the time of the execution of the
Compromise Agreement in Civil Case No. 2194-95-C, they were indisputably and
reasonably informed that the subject lot was previously sold to the respondents. In fact, they

SALES CASE DIGEST COMPILATION 236


were already aware that the respondents had constructed a house thereon and are
presently in possession of the same.

Knowledge gained by the second buyer of the first sale defeats his rights even if he
is the first to register the second sale because such knowledge taints his prior registration
with bad faith. For the second buyer to displace the first, he must show that he acted in good
faith throughout (i.e. in ignorance of the first sale and of the first buyer's rights) from the time
of acquisition until the title is transferred to him by registration.

The respondents are the buyers entitled to the physical possession of the subject lot
since the prevailing doctrine is that as between the buyer who is in possession of a Torrens
title but who has acquired it in bad faith and the first buyer who failed to have his title
recorded in the Registry of Property, the first buyer must prevail

50) BEHN MEYER v. YANCO

FACTS:

A sale of 80 drums of caustic soda was agreed between Behn, Meyer & Co. and
Teodoro Yanco. The merchandise was shipped from New York to Manila. However, the
ship carrying the cargo was detained at Penang and the 71 of the 80 drums were removed.
Respondent Yangco also refused to accept the 9 remaining and also refused to accept the
offer of Behn Meyer to have the products substituted with other merchandise, which
however were different from what was ordered.

It must be noted that the contract provided for "c.i.f. Manila, pagadero against
delivery of documents." Yanco filed an action seeking for damages for alleged breach of
contract.

ISSUE:

Whether or not Behn, Meyer & Co. should bear the burden of the loss of the merchandise?
YES

HELD:

Rule as to delivery of goods by a vendor via a common carrier (If contract is silent –
delivery of seller to common carrier transfer ownership to buyer)

Determination of the place of delivery always resolves itself into a question of act. If
the contract be silent as to the person or mode by which the goods are to be sent, delivery
by the vendor to a common carrier, in the usual and ordinary course of business, transfers
the property to the vendee.

Payment of freight by the buyer = acquires ownership at the point of shipment

A specification in a contact relative to the payment of freight can be taken to indicate


the intention of the parties in regard to the place of delivery. If the buyer is to pay the freight,
it is reasonable to suppose that he does so because the goods become his at the point of
shipment.

SALES CASE DIGEST COMPILATION 237


Payment of freight by the seller = title of property does not pass until the goods have
reached their destination

On the other hand, if the seller is to pay the freight, the inference is equally so strong
that the duty of the seller is to have the goods transported to their ultimate destination and
that title to property does not pass until the goods have reached their destination.

c.i.f. means Cost, Insurance and Freight = CFI is paid by the seller

The letters "c.i.f." found in British contracts stand for cost, insurance, and freight.
They signify that the price fixed covers not only the cost of the goods, but the expense of
freight and insurance to be paid by the seller.

F.O.B. stands for Free on Board = seller bear all expenses until goods are delivered

In this case, in addition to the letters "c.i.f.," has the word following, "Manila." In
mercantile contracts of American origin the letters "F.O.B." standing for the words "Free on
Board," are frequently used. The meaning is that the seller shall bear all expenses until the
goods are delivered where they are to be "F.O.B."

According as to whether the goods are to be delivered "F.O.B." at the point of


shipment or at the point of destination determines the time when property passes. However,
both the terms "c.i.f." and "F.O.B." merely make rules of presumption which yield to proof of
contrary intention.

Delivery was to be made at Manila

Hence, we believe that the word Manila in conjunction with the letters "c.i.f." must
mean that the contract price, covering costs, insurance, and freight, signifies that delivery
was to made at Manila. If petitioner Behn Meyer has seriously thought that the place of
delivery was New York and Not Manila, it would not have gone to the trouble of making
fruitless attempts to substitute goods for the merchandise named in the contract, but would
have permitted the entire loss of the shipment to fall upon the defendant.

Behn Meyer failed to prove that it performed its part in the contract

In this case, the place of delivery was Manila and plaintiff (Behn Meyer) has not
legally excused default in delivery of the specified merchandise at that place. In resume, we
find that the plaintiff has not proved the performance on its part of the conditions precedent
in the contract.

For breach of warranty, the buyer (Yanco) may demand rescission of the contract of sale

The warranty — the material promise — of the seller to the buyer has not been
complied with. The buyer may therefore rescind the contract of sale because of a breach in

SALES CASE DIGEST COMPILATION 238


substantial particulars going to the essence of the contract. As contemplated by article 1451
of the Civil Code, the vendee can demand fulfillment of the contract, and this being shown to
be impossible, is relieved of his obligation. There thus being sufficient ground for rescission,
the defendant is not liable.

XX. DOUBLE SALES

1) CARBONELL v. CA

FACTS:

Respondent Jose Poncio was the owner of the parcel of land located in Rizal. (Area
– more or less 195 sq. m.). The said lot was subject to mortgage in favor of the Republic
Savings Bank for the sum of P1,500.00.

Carbonell and respondent Emma Infante offered to buy the said lot from Poncio.
Poncio offered to sell his lot to Carbonell excluding the house on which he and his family
stayed. Carbonell accepted the offer and proposed the price of P9.50/sq. m.. Poncio
accepted the price on the condition that from the purchase price would come the money to
be paid to the bank.

January 27, 1995: The parties executed a document in the Batanes dialect which is
translated as: CONTRACT FOR ONE HALF LOT WHICH I (Poncio) BOUGHT FROM.

Carbonell asked a lawyer to prepare the deed of sale and delivered the document,
together with the balance of P400, to Jose Poncio. (Note: Carbonell already paid P200 for
the mortgage debt of Poncio + obligated herself to pay the remaining installments.)

However, when she went to Poncio, the latter informed her that he could no longer
proceed with the sale as the lot was already sold to Emma Infante and that he could not
withdraw with the sale. Poncio admitted that on January 30, 1995, Mrs. Infante improved her
offer and he agreed to sell the land and its improvements to her for P3,535.00.

In a private memorandum agreement, Poncio bound to sell to Infante the lot for the sum of
P2,357.52, with Infante still assuming the mortgage debt of P1,177.48. (Note: The full
amount of mortgage debt was already paid by the Infantes)

February 2, 1995: A deed of sale was executed between Poncio and Infante.

February 8, 1995: Knowing that the sale to Infante has not been registered, Carbonell filed
an adverse claim.

February 12, 1995: The deed of sale was registered but it has an annotation of the adverse
claim of Carbonell.

Thereafter, Emma Infante took possession of the lot, built a house and introduced
some improvements.

In June 1995, Carbonell filed a complaint praying that she be declared the lawful
owner of the land, that the subsequent sale to spouses Infante be declared null and void,
and that Jose Poncio be ordered to execute the corresponding deed of conveyance of said
land in her favor

SALES CASE DIGEST COMPILATION 239


RTC ruled that the sale to spouses Infante was null and void. After re-trial, it reversed
its ruling. CA ruled in favor of Carbonell but after a MfR, it reversed its ruling and ruled in
favor of the Infantes.

ISSUE:

Whether or not Carbonell has a superior right over Emma Infante. YES

HELD:

Article 1544 provides that for double sale of an immovable property, the ownership
shall belong to the person who first acquired it in good faith and recorded it in the Registry of
Property. Article 1544, New Civil Code, which is decisive of this case, recites:

If the same thing should have been sold to different vendees, the ownership shall be
transferred to the person who may have first taken possession thereof in good faith, if it
should movable property.

Should it be immovable property, the ownership shall belong to the person acquiring
it who in good faith first recorded it in the Registry of Property.

Should there be no inscription, the ownership shall pertain to the person who in good
faith was first in the possession; and, in the absence thereof, to the person who presents the
oldest title, provided there is good faith.

The buyer must act in good faith in registering the deed of sale.

It is essential that the buyer of realty must act in good faith in registering his deed of
sale to merit the protection of the second paragraph of said Article 1544.

Unlike the first and third paragraphs of said Article 1544, which accord preference to
the one who first takes possession in good faith of personal or real property, the second
paragraph directs that ownership of immovable property should be recognized in favor of
one "who in good faith first recorded" his right. Under the first and third paragraph, good faith
must characterize the act of anterior registration.

Rule when there is inscription or not

If there is no inscription, what is decisive is prior possession in good faith. If there is


inscription, as in the case at bar, prior registration in good faith is a pre-condition to superior
title.

Carbonell was in good faith when she bought the lot

When Carbonell bought the lot from Poncio on January 27, 1955, she was the only
buyer thereof and the title of Poncio was still in his name solely encumbered by bank
mortgage duly annotated thereon. Carbonell was not aware — and she could not have been
aware — of any sale of Infante as there was no such sale to Infante then.

SALES CASE DIGEST COMPILATION 240


Hence, Carbonell's prior purchase of the land was made in good faith. Her good faith
subsisted and continued to exist when she recorded her adverse claim four (4) days prior to
the registration of Infantes's deed of sale.

Carbonell’s good faith did not cease when she was informed by Poncio about the
sale to Emma Infante. After learning about the second sale, Carbonell tried to talk to the
Infantes but the latter

So Carbonell did the next best thing to protect her right — she registered her
adversed claim on February 8, 1955. Under the circumstances, this recording of her adverse
claim should be deemed to have been done in good faith and should emphasize Infante's
bad faith when she registered her deed of sale four (4) days later on February 12, 1955.

The Infantes were in bad faith (5 indications of bad faith listed below)

Bad faith arising from previous knowledge by Infante of the prior sale to Carbonell is shown
by the following facts:

1. Mrs. Infante refused to see Carbonell.

Her refusal to talk to Carbonell could only mean that she did not want to listen to Carbonell's
story that she (Carbonell) had previously bought the lot from Poncio.

2. Carbonell was already in possession of mortgage passbook and copy of the mortgage
contract. (Not Poncio’s saving deposit passbook.)

Infante naturally must have demanded from Poncio the delivery to her of his
mortgage passbook and mortgage contract so that the fact of full payment of his bank
mortgage will be entered therein; and Poncio, as well as the bank, must have inevitably
informed her that said mortgage passbook could not be given to her because it was already
delivered to Carbonell.

3. Emma Infante did not inquire why Poncio was no longer in possession of the mortgage
passbook and why it was in Carbonell’s possession.

he fact that Poncio was no longer in possession of his mortgage passbook and that
the said mortgage passbook was already in possession of Carbonell, should have compelled
Infante to inquire from Poncio why he was no longer in possession of the mortgage
passbook and from Carbonell why she was in possession of the same.

4. Emma Infante registered the sale under her name after Carbonell filed an adverse claim 4
days earlier.

Here she was again on notice of the prior sale to Carbonell. Such registration of
adverse claim is valid and effective.

5. Infante failed to inquire to Poncio WON he had already sold the property to Carbonell
especially that it can be shown that he was aware of the offer made by Carbonell.

SALES CASE DIGEST COMPILATION 241


Poncio alleged in his answer that Mrs. Infante and Mrs. Carbonell offered to buy the
lot at P15/sq. m. which offers he rejected as he believed that his lot is worth at least
P20.00/sq. m. It is therefore logical to presume that Infante was told by Poncio and
consequently knew of the offer of Carbonell which fact likewise should have put her on her
guard and should have compelled her to inquire from Poncio whether or not he had already
sold the property to Carbonell

The existence of prior sale to Carbonell was duly established

From the terms of the memorandum, it tends to show that the sale of the property in
favor of Carbonell is already an accomplished act. As found by the trial court, to repeat the
said memorandum states "that Poncio is allowed to stay in the property which he had sold to
the plaintiff ..., it tends to show that the sale of the property in favor of the plaintiff is already
an accomplished act..."

There was an adequate consideration or price for the sale in favor of Carbonell

Poncio agreed to sell the same to Carbonell at P9.50 per square meter, on condition that
Carbonell:

1. should pay (a) the amount of P400.00 to Poncio and the arrears in the amount of P247.26
to the bank

2. should assume his mortgage indebtedness.

The bank president agreed to the said sale with assumption of mortgage in favor of
Carbonell an Carbonell accordingly paid the arrears of P247.26.

It is evident therefore that there was ample consideration, and not merely the sum of
P200.00, for the sale of Poncio to Carbonell of the lot in question.

The subject property was identified and described

The court has arrived at the conclusion that there is sufficient description of the lot referred
to in Exh. As none other than the parcel of lot occupied by the defendant Poncio and where
he has his improvements erected. The Identity of the parcel of land involved herein is
sufficiently established by the contents of the note Exh. 'A'.

3) DE LEON v. ONG

FACTS:

De Leon sold 3 parcels of land to Ong. The properties were mortgaged to Real
Savings and Loan Association. The parties executed a notarized deed of absolute sale with
assumption of mortgage. The deed of Assumption of mortgage shall be executed in favor of
Ong after the payment of 415K. Ong complied with it. De Leon handed the keys of to Ong
and informed the loan company that the mortgage has been assumed by Ong. Ong made
some improvements in the property. After sometime, Ong learned that the properties were
sold to Viloria and changed the locks to it. Ong went to the mortgage company and learned
that the mortgage was already paid and the titles were given to Viloria. Ong filed a complaint
for the nullity of second sale and damages. De Leon contended that Ong does not have a

SALES CASE DIGEST COMPILATION 242


cause of action against him because the sale was subject to a condition which requires the
approval of the loan company and that he and Ong only entered a contract to sell.

ISSUE:

Whether or not the parties entered into a contract of sale.

HELD:

Yes, the parties entered into a contract of sale. In a contract of sale, the seller
conveys ownership of the property to the buyer upon the perfection of the contract. The non-
payment of the price is a negative resolutory condition. Contract to sell is subject to a
positive suspensive condition. The buyer does not acquire ownership of the property until he
fully pays the purchase price. In the present case, the deed executed by the parties did not
show that the owner intends to reserve ownership of the properties. The terms and
conditions affected only the manner of payment and not the immediate transfer of
ownership. It was clear that the owner intended a sale because he unqualifiedly delivered
and transferred ownership of the properties to the respondent.

4) SAN LORENZO DEVELOPMENT CORPORATION v. CA

FACTS:

On 20 August 1986, the Spouses Lu purportedly sold two parcelsof land to


respondent Pablo Babasanta, for the price of fifteenpesos (P15.00) per square meter.
Babasanta made a downpayment of (P50,000.00) as evidenced by a memorandum receipt
issued by Pacita Lu of the same date.

Babasanta wrote a letter to Pacita Lu to demand the execution of a final deed of sale
in his favor so that he could effect full payment of the purchase price. In response, Pacita Lu
wrote aletter to Babasanta wherein she reminded Babasanta that when the balance of the
purchase price became due, he requested for areduction of the price and when she refused,
Babasanta backed out of the sale wherein petitioner San Lorenzo Development Corporation
(SLDC)filed a

Motion for Intervention

SLDC alleged that it had legal interest in the subject matter under litigation because
on 3 May1989, the two parcels of land involved had been sold to it in a Deed of Absolute
Sale with Mortgage. It alleged that it was abuyer in good faith and for value and therefore it
had a better right over the property in litigation

Respondent Babasanta, however, argued that SLDC could not have acquired
ownership of the property because it failed to comply with the requirement of registration of
the sale in good faith. He emphasized that at the time SLDC registered the sale inits favor
on 30 June 1990, there was already a notice of lis pendens annotated on the titles of the
property made as early as2 June 1989. Hence, petitioner’s registration of the sale did not
confer upon it any right.

ISSUE:

SALES CASE DIGEST COMPILATION 243


Did the registration of the sale after the annotation of the notice of lis pendens obliterate the
effects of delivery and possession in good faith which admittedly had occurred prior to
SLDC’s knowledge of the transaction in favor of Babasanta?

HELD:

NO .It must be stressed that as early as 11 February 1989, the Spouses Lu executed
the Option to Buy in favor of SLDC upon receiving P316,160.00 as option money from
SLDC. After SLDC had paid more than one half of the agreed purchase price, the Spouses
Lu subsequently executed on 3 May 1989 a Deed of Absolute Sale in favor or SLDC.

At the time both deeds were executed, SLDC had no knowledge of the prior
transaction of the Spouses Lu with Babasanta. Simply stated, from the time of execution of
the first deed up to the moment of transfer and delivery of possession of the lands to SLDC,
it had acted in good faith and the subsequent annotation of lis pendens has no effect at all
on the consummated sale between SLDC and the Spouses Lu.

A purchaser in good faith is one who buys property of another without notice that
some other person has a right to, or interest in, such property and pays a full and fair price
for the same at the time of such purchase, or before he has notice of the claim or interest of
some other person in the property.

We rule that SLDC qualifies as a buyer in good faith since there isno evidence extant
in the records that it had knowledge of theprior transaction in favor of Babasanta. At the time
of the sale of the property to SLDC, the vendors were still the registered ownersof the
property and were in fact in possession of the lands.

In assailing knowledge of the transaction between him and theSpouses Lu,


Babasanta apparently relies on the principle of constructive notice incorporated in Section
52 of the PropertyRegistration Decree (P.D. No. 1529) which reads, thus:

Sec. 52. Constructive notice upon registration.

– Every conveyance,mortgage, lease, lien, attachment, order, judgment, instrument or entry


affecting registered land shall, if registered, filed, or entered in the office of the Register of
Deeds for the province or city where the land to which it relates lies, be constructive notice to
all persons from the time of suchregistering, filing, or entering.

However, the constructive notice operates as such by the express wording of Section
52 from the time of the registration of the notice of lis pendens which in this case was
effected only on 2June 1989, at which time the sale in favor of SLDC had long been
consummated insofar as the obligation of the Spouses Lu to transfer ownership over the
property to SLDC is concerned.

5) CORONEL v. CA

FACTS:

SALES CASE DIGEST COMPILATION 244


This case is about a sale of land in Roosevelt Avenue, Quezon City by the vendor
Romulo Coronel to the vendees Conception Alcaraz and her daughter Ramona Patricia
Alcaraz with the following conditions:

The Coronel’s will immediately transfer the certificate of title in their name upon
receipt of the downpayment which is ₱50,000.

Upon the transfer in their names of the subject property, the Coronel’s will execute
the deed of absolute sale in favor of Ramona and then Ramona shall immediately pay the
Coronel’s the whole balance of ₱1,190,000.

On January 15, 1985, Conception paid the downpayment of ₱50,000 and then on
February 6, 1985, the property was now registered under the name of Coronel’s. By Feb. 18,
1985, the Coronel’s sold the property to Catalina B. Mabanag for ₱1,580,000 after she made
a ₱300,000 downpayment. This is the reason why the Coronel’s cancelled and rescind the
contract with the Alcaraz by depositing back the ₱50,000 to Ramona’s bank account.

On Feb. 22, Conception filed a complaint for specific performance against the
Coronel’s. On April, the Coronel’s executed a deed of absolute sale over the subject
property to Catalina after which on June Catalina was issued a new title over the subject
property.

ISSUE:

Whether or not the “Receipt of Down payment” embodied a perfected contract of sale or just
a mere contract to sell?

HELD:

When the “Receipt of Down Payment” document was prepared and signed by
Romulo Coronel, the parties had agreed to a conditional contract of sale the consummation
of the contract is subject only to the successful transfer of the certificate of Title.

According to Supreme Court, the receipt of down payment document manifests a


clear intent of the Coronel’s to transfer the title to the buyer, but since the title is still in the
name effect the transfer even though the buyers are able and willing to immediately pay the
purchase price. The agreement as well could not have been a contract to sell because the
seller or the Coronel’s made no express reservation of ownership or the title of the land.

On Feb. 6, 1985, the Contract of Sale between the Coronel’s and the Alcaraz’
became obligatory.

6) TORRECAMPO v. ALINDONGAN

FACTS:

May 24, 1997: spouses Jose and Lina Belmes executed a deed of sale in favor of
respondent-spouses Dennis and Heide Alindogan over a house and lot located in Legazpi
city.

July 4, 1997: Lina Belmes wrote spouses Alindogan wherein she delivered the
constructive possession of the house and lot to them. However, on July 5, 1997, before they

SALES CASE DIGEST COMPILATION 245


could take actual possession of the property, spouses Gil and Brenda Torrecampo,
petitioners, and spouses Jonathan Lozares and Jocelyn Torrecampo, entered and occupied
the premises.

Despite spouses Alindogan’s repeated demands, spouses Torrecampo failed and


refused to vacate the property.

The spouses Alindogan filed a Complaint for Recovery of Ownership, Possession


and Damages against spouses Torrecampo.

Spouses Torrecampo alleged that:

 On March 25, 1997, Belmes received from them P73,000.00 as advance payment for
the sale of the house and lot.

 On April 8, 1997, they executed a "Contract to Buy and Sell."

 That to complete the agreed partial payment of P220,000.00, spouses Torrecampo


paid spouses Belmes P130,000.00, but the latter refused to accept the amount.
Thus, on July 7, 1997, spouses Torrecampo filed with RTC a Complaint for Specific
Performance against spouses Belmes.

The RTC ruled in favor of spouses Alindogan. It ruled that the contract between
Belmes and Torrecampo was a mere contract to sell and thus, the ownership of the property
was not transferred to spouses Torrecampo. CA affirmed in toto RTC’s ruling.

Spouses Torrecampo contended that when spouses Alindogan bought the property,
they already knew that the same property was previously sold to them and being buyers in
bad faith, ownership of the property must pertain to spouses Torrecampo who, in good faith,
were first in possession.

ISSUE:

WON spouses Torrecampo has a better right over spouses Alindogan. NO

HELD:

Article 1544 is not applicable in the case at bar

The above provision does not apply to the instant case considering that the
transaction between petitioners and spouses Belmes is a mere contract to sell, not a
contract of sale.

(note: This is the only ruling related to double sales in this case. Article 1544 was cited.)

The tenor of the contract entered into by spouses Belmes and spouses Torrecampo
clearly confirms that what was executed was merely a contract to sell and not a contract of
sale

The Contract to Buy and Sell reads:

"That whereas, the vendor agreed to sell and the vendee agreed to buy the above-
described parcel of land, together with improvements therein, for the sum of Three Hundred
Fifty Thousand Pesos (P350, 000.00), Philippine currency, under the following terms and
conditions xxx"1awphi1.net

SALES CASE DIGEST COMPILATION 246


Indeed, the true agreement between petitioners and spouses Belmes is a contract to
sell. Not only did the parties denominate their contract as "Contract to Buy and Sell," but
also specified therein that the balance of the purchase price in the amount of P130,000.00 is
to be paid by petitioners upon the issuance of a certificate of title. That spouses Belmes
have in their possession the certificate of title indicates that ownership of the subject
property did not pass to petitioners.

The obligation of Belmes to transfer ownership and delivery the lot to spouses
Torrecampo did not arise since there was no full payment of the purchase price

Indeed, in contracts to sell the obligation of the seller to sell becomes demandable
only upon the happening of the suspensive condition, that is, the full payment of the
purchase price by the buyer. It is only upon the existence of the contract of sale that the
seller becomes obligated to transfer the ownership of the thing sold to the buyer. Prior to the
existence of the contract of sale, the seller is not obligated to transfer the ownership to the
buyer, even if there is a contract to sell between them.

7) ADALIN V. CA

FACTS:

Appellee-Vendors sold their 5-door commercial building to Appellants Yu and Lim located in
front of Imperial Hotel in Cotabato City.

Since there are lessees in the property, the vendors offered it first to them twice but they
refused both offers. As such, appellee-vendors and appellants executed a deed of
conditional sale. The contract states that they appellants will pay the down payment of 300K
first and the remaining balance after the appellee-vendors completely evicted the lessees
occupying the property.

After the vendors and the tenants made known their intention to buy the property for a higher
price. As such, the vendors executed three deeds of sale of registered land in favor of the
lessees.

The vendors offered to return the downpayment paid by the appellants but the latter refused.
The vendors contend that they can rescind the contract because the condition to evict the
tenants was not completed.

HELD:

Although the contract was a conditional sale, what was subject to the condition is the
payment of the balance. Both parties have their respective obligations yet to be fulfilled, the
seller the eviction of the tenants and the buyer, the payment of the balance of the purchase
price. The choice of who to sell the property to, however, had already been made by the
sellers and is thus no longer subject to any condition nor open to any change. In that sense,
the sale to the appellants was definitive and absolute. A clear breach of contract was made
by the vendors.

SALES CASE DIGEST COMPILATION 247


A case double sale occurred when the vendors sold the property to the tenants. When the
tenants bought the property, they are fully aware of its prior sale to the appellants. Though
the second sale to the said tenants was registered, such prior registration cannot erase the
gross bad faith that characterized such second sale, and as such, there is no legal basis to
rule that such second sale prevails over the first sale of the said property.

8) LEUNG YEE v. F.I. MACHINERIES

FACTS:

In 1913, Compania Agricola Filipina (CAF) was indebted to two personalities: Leung
Yee and Frank L. Strong Machinery Co. CAF purchased some rice cleaning machines from
Strong Machinery. CAF installed the machines in a building. As security for the purchase
price, CAF executed a chattel mortgage on the rice cleaning machines including the building
where the machines were installed. CAF failed to pay Strong Machinery, hence the latter
foreclosed the mortgage – the same was registered in the chattel mortgage registry.

CAF also sold the land (where the building was standing) to Strong Machinery.
Strong Machinery took possession of the building and the land.

On the other hand, Yee, another creditor of CAF who engaged in the construction of
the building, being the highest bidder in an auction conducted by the sheriff, purchased the
same building where the machines were installed. Apparently CAF also executed a chattel
mortgage in favor Yee. Yee registered the sale in the registry of land. Yee was however
aware that prior to his buying, the property has been sold in favor of Strong Machinery –
evidence is the chattel mortgage already registered by Strong Machinery (constructive
notice).

ISSUE:

Who is the owner of the building?

HELD:

Strong Machinery has a better right to the contested property. Yee cannot be
regarded as a buyer in good faith as he was already aware of the fact that there was a prior
sale of the same property to Strong Machinery.

The SC also noted that the Chattel Mortgage Law expressly contemplates provisions
for chattel mortgages which only deal with personal properties. The fact that the parties dealt
the building as if it’s a personal property does not change the nature of the thing. It is still a
real property. Its inscription in the Chattel Mortgage registry does not modify its inscription
the registry of real property.

9) SANTIAGO v. CA

SALES CASE DIGEST COMPILATION 248


FACTS:

Paula Arcega was the registered owner of that certain parcel of land covered by
Transfer Certificate of Title No. T-115510. Her residential house stood there until 1970 when
it was destroyed by a strong typhoon. On December 9, 1970, she executed what purported
to be a deed of conditional sale over the land in favor of Josefina Arcega and the spouses
Regalado Santiago and Rosita Palabyab, for and in consideration of P20,000.00

Subsequently, on July 18, 1971, supposedly upon payment of the remaining balance,
Paula Arcega executed a deed of absolute sale of the same parcel of land in favor of
petitioners.

On April 10, 1985, Paula Arcega died single and without issue, leaving as heirs her
two brothers, Narciso Arcega and private respondent Quirico Arcega. Defore Paula Arcega
died, a house of four bedrooms with a total floor area of 225 square meters was built over
the parcel of land in question and was occupied by Paula Arcega until her death despite the
execution of the alleged deed of absolute sale. The three other bedrooms, smaller than the
master's bedroom, were occupied by the petitioners who were the supposed vendees in the
sale.

ISSUE:

Whether or not there has been a valid deed of sale.

HELD:

No.

While petitioners were able to occupy the property in question, they were relegated
to a small bedroom without bath and toilet, while Paula Arcega remained virtually in full
possession of the completed house and lot using the big master's bedroom with bath and
toilet up to the time of her death on April 10, 1985. If, indeed, the transaction entered into by
the petitioner's and the late Paula Arcega on July 18, 1971 was a veritable deed of absolute
sale, as it was purported to be, then Ms. Arcega had no business whatsoever remaining in
the property and, worse, to still occupy the big master's bedroom with all its amenities until
her death on April 10, 1985. Definitely, any legitimate vendee of real property who paid for
the property with good money wil not accede to an arrangement whereby the vendor
continues occupying the most favored room in the house while he or she, as new owner,
endures the disgrace and absurdity of having to sleep in a small bedroom without bath and
toilet as if he or she is a guest or a tenant in the house. In any case, if petitioners really
stood as legitimate owners of the property, they would have collected rentals from Paula
Arcega for the use and occupation of the master's bedroom as she would then be a mere
lessee of the property in question. However, not a single piece of evidence was presented to
show that this was the case. All told, the failure of petitioners to take exclusive possession of
the property allegedly sold to them, or in the alternative, to collect rentals from the alleged
vendee Paula Arcega, is contrary to the principle of ownership and a clear badge of
simulation that renders the whole transaction void and without force and effect.

The conceded fact that subject deed of absolute sale executed by Paula Arcega in
favor of petitioners is a notarized document does not justify the petitioners' desired
conclusion that said sale is undoubtedly s true conveyance to which the parties thereto are
irrevocably and undeniably bound. To be considered with great significance is the fact that
Atty. Luis Cuvin who notarized the deed disclaimed the truthfulness of the document when

SALES CASE DIGEST COMPILATION 249


he testified that "NO MONEY WAS INVOLVED IN THE TRANSACTION."Furthermore,
though the notarization of the deed of sale in question vests in its favor the presumption of
regularity, it is not the intention nor the function of the notary public to validate and make
binding an instrument never, in the first place, intended to have any binding legal effect upon
the parties thereto. The intention of the parties still is and always will be the primary
consideration in determining the true nature of a contract. Here, the parties to the "Kasulatan
ng Bilihang Tuluyan ng Lupa," as shown by the evidence and accompanying circumstances,
never intended to convey the property thereto from one party to the other for valuable
consideration. Rather, the transaction was merely used to facilitate a loan with the SSS with
petitioners-mortgagors using the property in question, the title to which they were able to
register in their names through the simulated sale, as collateral.

The fact that petitioners were able to secure a title in their names, TCT No. 148989,
did not operate to vest upon petitioners ownership over Paula Arcega's property. That act
has never been recognized as a mode of acquiring ownership. As a matter of fact, even the
original registration of immovable property does not vest title thereto. The Torrens system
does not create or vest title. It only confirms and records title already existing and vested. It
does not protect a usurper from the true owner. It cannot be a shield for the commission of
fraud. It does not permit one to enrich himself at the expense of another. Where one does
not have any rightful claim over a real property, the Torrens system of registration can
confirm or record nothing.

10) LIAO v. CA

FACTS:

On March 5, 1986, Estrella Mapa filed with the Regional Trial Court, Quezon City,
Branch 99 a petition for reconstitution of documents and issuance of certificates of title over
certain parcels of land covered by OCT 614. She claimed that on June 16, 1913, the
Director of Lands issued certificates of sales to Vicente Salgado over the parcels of land
covered by OCT 614. The sale involves four (4) parcels of land. The four lots form part of the
Piedad Estate.

Unfortunately, the above titles were in conflict with several existing certificates of title,
resulting in the filing of several actions with the Regional Trial Court, Quezon City for
quieting, and an investigation into the matter by the National Bureau of Investigation.
Meantime, Estrella Mapa assigned the parcels of land covered by T.C.T. No. 348291 and
T.C.T. No. 348292 in favor of Palmera Agricultural Realty Development Corporation, which
is a family corporation headed by Lourdes Angeles, Estrella Mapas daughter.

On March 28, 1990, I.C. Cruz Construction, Inc. (ICC) filed with the Court of Appeals
a petition for the annulment of the Order dated June 30, 1986 of the Regional Trial Court,
Quezon City. I.C. Cruz alleged that the title issued by the Register of Deeds of Quezon City
pursuant to said order encompassed property which had been registered and titled in its
name. On July 3, 1990, Arle Realty Development Corporation (hereafter Arle) filed a similar
petition with the Court of Appeals praying for the annulment of the same order of the
Regional Trial Court. Arle claimed ownership of six (6) lots covered by TCT No. 263984 to
263989, which had overlapped TCT No. 348292 covering Lot No. 777.

On February 17, 1988, respondents Susan A. Foronda, Iluminada R. Dionisio, Azucena Q.


Pua, and Lucia Pua Liok Bin filed a complaint or "Annulment of Title, Reconveyance of
Property, Damages and Injunction with Restraining Order" against Estrella Mapa, Lourdes
Angeles, Serafin B. Riosa, Ernie M. Palmos, Palmera Agricultural Realty Corporation and

SALES CASE DIGEST COMPILATION 250


Maristel Y. Angeles, involving lots titled in the names of the complainants but also covered
by TCT No. 348292 (Lot No. 777).

11) PO SUN TUN vs. W.S. PRICE and PROVINCIAL GOVERNMENT OF LEYTE

FACTS:

On November 29, 1921, Gabino Po Ejap was the owner of a certain parcel of land
situated in the Tacloban, Leyte. On the same date, he sold the land to Po Tecsi for the sum
of P8,000 and the latter gave general power of attorney to the former including the right to
sell. Gabino and Po Tecsi are brothers, while Gabino and Po Sun Tun are father and son.

On June 21, 1923, Po mortgaged the land to W. S. Price in the amount of P17,000.
The mortgage was duly noted in the office of the RD on August 18th of the same year.

Acting under this power, Gabino sold the land on November 22, 1923, for P8,000 to
Jose H. Katigbak. On this document there appears on the upper right-hand margin the
following: "Register of Deeds, Received, Dec. 15, 1923, Province of Leyte."

On December 17, 1924, Po executed a deed of sale of the land to Price in


consideration of P17,000. This sale was recorded with the RD on January 22, 1925.

On February 16, 1927, Price with the consent of his wife, sold the land to the
Province of Leyte for P20,570. On March 17, 1927, the OCT was issued in the name of the
spouses Price. Later, the proper transfer certificate of title was provided for the Province of
Leyte. On October 12, 1927, Katigbak transferred the property to Po Sun Tun for P8,000.

Presently, the possession of the property has been under the control of Price and the
Provincial Government and has not been under the material control of Po Sun Tun. The
latter filed an action to gain the possession of the property before the CFI and decided in
favor of Price.

On the appeal, it was found out that the deed in favor of Katigbak had not been
registered in the corresponding registry of property.

ISSUE:

Whether the deed in favor of Katigbak with the note “ Register of Deeds, Received
December 23, 1923, Province of Leyte” can it be said to be recorded in the Registry of
Deeds.

HELD:

No. the term "To register" it has been said that it means to "enter in a register; to
record formally and distinctly; to enroll; to enter in a list"

The mere presentation to the office of the register of deeds of a document on which
acknowledgment of receipt is written is not equivalent to recording or registering the real
property. Escriche says that registration, in its juridical aspect, must be understood as the
entry made in a book or public registry of deeds.

SALES CASE DIGEST COMPILATION 251


If any doubt remained on the subject, it would be dispelled by turning to Act No. 2837
amendatory of section 194 of the Administrative Code, and recalling that it is therein
provided that "No instrument or deed establishing, transmitting, acknowledging, modifying or
extinguishing rights with respect to real estate not registered under the provisions of Act No.
496, entitled 'The Land Registration' and its amendments, shall be valid, except as between
the parties thereto, until such instrument or deed has been registered, in the manner
hereinafter prescribed, in the office of the register of deeds for the province or city where the
real estate lies."

Hence, since the deed made by Gabino in favor of Katigbak was not only not first
recorded in the registry of deeds but never legally so recorded, and since the purchaser who
did record his deed was Price, who secured a Torrens title and transferred the same to the
Province of Leyte, that Po Sun Tun, the holder of a defeasible title, has no legal rights as
against Price and the Province of Leyte, the holders of indefeasible titles. Further, it could
beruled that within the meaning of section 38 of the Land Registration Law, Price and the
Province of Leyte are innocent purchasers for value of the disputed property.

TROY

12) SALVORO V. TAÑEGA


G.R. No. L-32988
December 29, 1978

FACTS:

The Salvoro spouses, herein referred to as plaintiffs, mortgaged a parcel of land to the
Development Bank of the Philippines and having failed to pay the loan, the Bank gave notice
to foreclose the mortgage. On June 7, 1955, the plaintiffs executed a deed of absolute sale
in favor of the Tañega spouses, the first vendees and defendants herein. The Tañega
spouses immediately took possession of the said property and assumed the mortgage
executed by the plaintiffs.

On August 9, 1959, another Deed of Absolute Sale was executed by plaintiffs whereby they
conveyed absolutely and unconditionally in favor of the defendants the ownership of the
property. On August 25, 1959, or 16 days after plaintiffs executed the said second Deed of
Absolute Sale, the said Bank foreclosed the mortgage and it was the sole and highest
bidder. On August 26, 1960, the plaintiffs redeemed the property from the Development
Bank. Thereafter, plaintiffs executed a deed of sale in favor of the Tismo spouses, the
second vendees herein, over the same property.

On August 27, 1960, the defendants tendered payment but the plaintiffs refused to accept
the same. On September 5, 1960 the plaintiffs commenced in the Court of First Instance of
Leyte an action principally to annul a deed of sale of land executed by them in favor of the
Tañega spouses on the ground that the latter failed to comply with certain resolutory
conditions imposed in the contract.

On September 15, 1960, the defendants filed a notice of lis pendens with the Register of
Deeds of Leyte. Notwithstanding said notice, the defendants-in- counterclaim, the Tismo
spouses, were able to register the sale in their favor on December 19, 1960, and to secure
Transfer Certificate of Title.

The trial court dismissed the complaint. Dissatisfied, the plaintiffs appealed from the said
judgment to the Court of Appeals. But the appellate Court affirmed the judgment of the trial

SALES CASE DIGEST COMPILATION 252


court and ordered the Register of Deeds of the Province of Leyte to cancel the Transfer
Certificate of Title in the name of Juan and Dolores Tismo, and, in lieu thereof, to issue a
new Transfer Certificate of Title in the name of appellees, Pablo and Josefa Tañega.

ISSUE:

When real property is sold to two different persons by the same vendor, who shall have a
better right over the property under Article 1544 of the Civil Code of the Philippines, the first
vendee who immediately took possession of the property as owner but neglected to register
the sale or the second vendee who had the document in his favor duly registered?

RULING:

Spouses Tañega, are the owners of the land in question inasmuch as they, in good
faith, were first in possession of said land.

Article 1544 of the Civil Code of the Philippines provides that if the same thing should have
been sold to different vendees, the ownership shall be transferred to the person who may
have first taken possession thereof in good faith, if it should be movable property.

Should it be immovable property, the ownership shall belong to the person acquiring it who
in good faith first recorded it in the Registry of Property.

Should there be no inscription, the ownership shall pertain to the person who in good faith
was first in the possession; and, in the absence thereof, to the person who presents the
oldest title, provided there is good faith.

This Court has held in one case that the basic premise of the preferential rights established
by Article 1544 is good faith. To enjoy the preferential right, the second vendee must not
only have a prior recording of his sale but must, above all, have acted in good faith, that is,
without knowledge or notice of the previous and existing alienation made by his vendor to
another. This Court has also ruled that the rights given under this law do not accrue with the
mere inscription of the deed of conveyance unless such inscription is done in good faith.

The trial court as well as the appellate court has both held that when the Tismo spouses
registered the deed of sale executed in their favor of the property previously sold to the
Tañega spouses on December 19, 1960, they could not have failed to know the existence of
the lis pendens then annotated on the title of the property. In short, when they were about to
register the deed of sale in their favor, they acquired knowledge that the land had been
previously sold to the Tañega spouses. Indubitably there was bad faith on the part of the
Tismo spouses when they went ahead with the registration despite such knowledge.

This Court had occasions to rule that if a vendee in a double sale registers the sale after he
has acquired knowledge that there was a previous sale of the same property to a third party,
or that another person claims said property in a previous sale, the registration will constitute
a registration in bad faith and will not confer upon him any right. It is as if there had been no

SALES CASE DIGEST COMPILATION 253


registration, and the vendee who first took possession of the real property in good faith shall
be preferred.

Applying the foregoing rulings to the present case, this Court held that the defendants-
appellees, Spouses Tañega, are the owners of the land in question inasmuch as they, in
good faith, were first in possession of said land. Since the Tismo spouses were registrants in
bad faith, the situation is as if there was no registration at all. Therefore, the vendees who
first took possession of the property in good faith shall be preferred.

Hence, the petition is hereby denied and the decision of the Court of Appeals sought to be
reviewed is affirmed.

13) CAMPILLO VS. COURT OF APPEALS


129 SCRA 513

FACTS:
Tomas de Vera was the owner of two parcels of land in Tondo, Manila. In 1961, de Vera
sold the lands to Simplicio Santos. Santos however did not register the sale in the Registry
of Deeds, which means that the land was still under de Vera’s name.
On the other hand, de Vera was indebted to Sostenes Campillo. Campillo obtained a
favorable judgment for sum of money against de Vera. De Vera’s 3 parcels of land, including
those sold to Santos, were levied in 1962 in favor of Campillo.
Campillo acquired the land and he was able to have the lands be registered under his name.

ISSUE:
Who has better right over the property: Santos who first bought it without registering it or
Campillo who subsequently purchased it at a public auction and have it registered under his
name?

RULING:

Campillo has the better right over the said properties.


It is settled in this jurisdiction that a sale of real estate, whether made as a result of a private
transaction or of a foreclosure or execution sale, becomes legally effective against third
persons only from the date of its registration. Santos purchase of the two parcels of land
may be valid but it is not enforceable against third persons for he failed to have it registered.
Campillo is a purchaser in good faith as he was not aware of any previous sale for Santos
never caused the annotation of the sale.
Section 51, PD No. 1529, otherwise known as the Property Registration Decree, provides as
follows:

Section 51. Conveyance and other dealings by registered owner. – An owner of


registered land may convey, mortgage, lease, charge or otherwise deal with the
same in accordance with existing laws. He may use such forms of deeds, mortgages,
leases or other voluntary instruments as are sufficient in law. But no deed, mortgage,
lease or other voluntary instrument except a will purporting to convey or affect

SALES CASE DIGEST COMPILATION 254


registered land shall take effect as a conveyance or bind the land, but shall operate
only as a contract between the parties and as evidence of authority to the Register of
Deeds to make registration.
“The act of registration shall be the operative act to convey or affect the land insofar
as third persons are concerned, and in all cases under this Decree, the registration
shall be made in the office of the Register of Deeds for the province or city where the
land lies.

The purchaser (Campillo) in the execution sale of the registered land in suit, acquires such
right and interest as appears in the certificate of title unaffected by prior lien or
encumbrances not noted therein. This must be so in order to preserve the efficacy and
conclusiveness of the certificate of title which is sanctified under our Torrens system of land
registration.

14) SORIANO vs. HEIRS OF DOMINGO MAGALI


G.R. No. L-15133
July 31, 1963

FACTS:
On July 28, 1939, spouses Domingo Magali and Modesta Calimlim sold the aforesaid
Northeastern portion of said parcel of land to the spouses Tomas Cerezo and Segunda
Soriano, but said deed of sale was never registered and annotated on said O.C.T. No.
51878.
Said spouses Tomas Cerezo and Segunda Soriano in turn sold the same Northeastern
portion to Maximino Mamorno on January 7, 1941 (Exh. B). This deed of sale was also not
registered and annotated on said O.C.T. No. 51878.
On April 5, 1946 Francisca Reyes as surviving spouse of said Maximino Mamorno sold her
one-half pro indiviso share in said Northeastern portion to the spouses Emigdio Soriano and
Beatriz de Vera, and on the same day she, as guardian of her minor children, sold the
remaining one-half pro indiviso of said Northeastern portion to the same spouses Emigdio
Soriano and Beatriz de Vera (Exhibits C and D).
These two deeds of sale were on April 9, 1946 registered under the provision of Act 3344
and were, consequently, also not registered and annotated on O.C.T. No. 51878.
Plaintiffs Emigdio Soriano and Beatriz de Vera filed the present action for reivindicacion of
said Northeastern portion, the defendants Modesta Calimlim and Lamberto Magali being
now in possession of said Northeastern portion and claim ownership thereof.
Defendants' evidence on the other hand shows that in January, 1944 Maximino Mamorno in
turn sold said Northeastern portion back to Modesta Calimlim, but instead of executing a
formal deed of sale, merely delivered to said Modesta Calimlim the muniment of title over
said land, among which were the original of the deed of sale of July 28, 1939 by the spouses
Domingo Magali and Modesta Calimlim in favor of the spouses Tomas Cerezo and Segunda
Soriano in, and the original of the deed of sale of January 7, 1951 by the spouses Tomas
Cerezo and Segunda Soriano in favor of said Maximino Mamorno.
Consequently, defendant Modesta Calimlim, one of the registered owners of said parcel of
land covered by O.C.T. No. 51878, and her co-defendants Manuel, Consorcia, Luis, and
Tomas, all surnamed Magali, surviving children of the other registered owner, Domingo
Magali, are now the lawful owners of said parcel of land, including the aforesaid
Northeastern part thereof.

SALES CASE DIGEST COMPILATION 255


ISSUE:
Who has better right over the subject property?
RULING:
Appellees (Heirs of Magali) obtained possession of the land in good faith since 1944;
appellants never did so.
This case, therefore, should be resolved in the light of the law governing double sale of the
same property. Article 1473 of the old Civil Code, now Article 1544, provides that if
immovable property is sold to different vendees the ownership shall belong to the person
acquiring it who in good faith first recorded it in the registry of property; and should there be
no inscription the ownership shall pertain to the person who in good faith was first in the
possession.
Appellees obtained possession of the land in good faith in 1944; appellants never did so.
The registration by appellants of the sale in their favor was made under Act 3344, which
refers to properties not registered under the Land Registration Act, and hence was not
effective for purposes of Article 1544 of the Civil Code. Registration of instruments, in order
to affect and bind the land, must be done in the proper registry (Secs. 50 and 51, Act 496).

15) DBP VS. MANGAWANG


120 PHIL. 402

FACTS:
On November 24, 1941, Amposta sold the land to Santos Camacho surrendering to him
Original Certificate of Title No. 100, and because of this transfer said title was cancelled and
transfer Certificate of Title No. 5506 was issued in the name of Camacho.
Meanwhile, or on June 11, 1947, Gavino Amposta again sold the same property to Lazaro
and Arsenio Mangawang for the sum of P2,000.00, the vendees executing a mortgage on
the land to secure the payment of the balance.
On March 17, 1948, the vendees paid the balance of the purchase price, and an absolute
deed of sale was executed in their favor. As a consequence of their purchase of the land,
the Mangawang brothers took possession thereof.

ISSUE:
Who of the two buyers should be considered as the rightful owner of the land?

RULING:
The sale made by Amposta to Santos Camacho is the valid one.
On this score, it is important to consider the facts that led to the sale of the land to the
parties herein.
Note that Amposta first sold the land to Santos Camacho on November 24, 1941, who
registered it in his name on the same date. And seven years thereafter, or on March 17,
1948, Amposta again sold the land to the Mangawang brother, who also registered it in their
name on the same date.
Since both purchasers apparently have acted in good faith, as there is nothing in the
evidence to show that they did otherwise, we cannot but conclude that the sale made by

SALES CASE DIGEST COMPILATION 256


Amposta to Santos Camacho is the valid one considering that when Amposta sold the same
land to the Mangawang brothers he had nothing more to sell even if the title he surrendered
to them is one issued covering the same property. In legal contemplation, therefore,
Amposta sold a property he no longer owned, and hence the transaction is legally
ineffective.

16) GRANADOS vs.MONTON


G.R. No. L-1698
April 8, 1950

FACTS:
In 1943, Angela Trias who had inherited from Tomasa Trias, lot No. 1956 of the San
Francisco de Malabon Estate, for the sum of P1,300 which she received from Mariano
Granados, she turned over to him the said lot.
On 1944, Angela Trias filed a suit against Granados, to recover from him the possession of
said lot and to compel Granados to receive the above-stated amount, which she had
deposited with the clerk of court. Angela Trias contended that Granados delivered that sum
to her as a loan secured by mortgage on the property, but Granados refused to accept that
amount tendered by her (Exhibits 3 and 3-E).
On the other hand, Granados, in his defense, alleged that the sum of P1,300 was received
by Angela Trias in full payment of the purchase price of the lot in question; that pending the
issuance in her name of the corresponding certificate of title, the formal deed of transfer of
ownership had not been executed, and therefore, he prayed that judgment be rendered
against her, ordering her to convey to him the property in question (Exhibit 3-A). On
September 29, 1944, judgment was rendered in favor of Granados (Exhibit 3-F).
In the meanwhile, and pursuant to a deed of extrajudicial partition executed on April 8, 1944,
transfer certificate of title No. A-1462 of the office of the register of deeds of Cavite, and
marked Exhibit 1, covering said lot No. 1956 was, on May 10, 1944, issued in the name of
Angela Trias.
On August 8, 1944, she sold that lot, for the sum of P20,000 to Esteban Santiago (Exhibit 2-
A) who, two days later, or on August 10, 1944, upon cancellation of Exhibit 1, secured the
issuance in his name of certificate of title No-A-1858 (Exhibit 2).
On the other hand, by virtue of a document dated November 26,1944 (Exhibit 1-A),
acknowledged before a notary public on December 8,1944, and filed with the office of the
register of deeds of Cavite on December 11, 1944, Esteban Santiago sold said lot No. 1956
for P145,000 to Celedonio Monton. The cancellation on December 11, 1944 of Exhibit
2(transfer certificate of title No. A-1858) became necessary for the issuance of transfer
certificate of title No. 2164 in the name of Celedonio Monton.
On 1945, on the ground that he is the owner of lot No. 1956, Monton filed with the Court of
First Instance, an action to compel Mariano Granados to put him in possession of said lot,
which has been withheld from him by the latter, who had also seized the products thereof.
Granados assured with a general denial and specifically alleged by way of special defense,
that he is the owner of lot No. 1956, which he purchased from Angela Trais, and that
whatever subsequent conveyance she may have made thereof is fictitious and made in bad
faith.

ISSUE:

SALES CASE DIGEST COMPILATION 257


Whether Celedonio Monton, who had purchased from Santiago, lot No. 1956 of the San
Francisco de Malabon Estate, and in whose favor, transfer certificate of title No. 2164 was
issued, acted in good faith when he filed with the office of the register of deeds of Cavite, the
deed of sale.

RULING:
Monton cannot be held to have acted in bad faith, even if he had known of the
previous sale made by Angela Trias to Mariano Granados.
Article 1473 of the Civil Code provides that —
If the same should have been sold to different vendees, the ownership shall be
transferred to the person who may have first taken possession thereof in good faith,
if it should be personal property.
Should it be real property, it shall belong to the purchaser who first recorded it in the
Registry of Deeds.
Should it not be recorded, the property shall belong to the person who first took
possession of it in good faith; or, in default of possession, to the person who presents
the oldest title, provided there is good faith.
On December 11, 1944, Monton submitted to the Office of the Register of Deeds, the deed
of sale of the land in question, and, upon cancellation of transfer certificate of title No. A-
1858, transfer certificate of title No. 2164 was issued in his name.
The above-quoted article 1473 provides that the inscription in the registry of real property,
the ownership of which is claimed by different parties, shall have the effect of transferring
ownership thereof to the person who, in good faith, first recorded it in the register of deeds.
Did Celedonio Monton act in bad faith when he filed with the register of deed of Cavite the
deed of sale, Exhibit 1-A executed in his favor by Esteban Santiago, notwithstanding the fact
that according to the record, when, on or before December 8, 1944, he visited the land in
question, Valentin Cabrera, the tenant of Mariano Granados, informed him that the property
was under litigation between Granados, on the one hand, and Angela Trias, on the other?
Article 433 of the Civil Code says that "one who is unaware of any flaw in his title, or mode of
acquisition, by which it is invalidated, shall be deemed a possessor in good faith." But "one
who is aware of such law is deemed a possessor in bad faith."
Santiago, former owner of the land in question, testifying in this case, said that he bought lot
No. 1956 in good faith for value, relying upon the transfer certificate of title (Exhibit 1) issued
in favor of Angela Trias, which did not have any annotation thereon relative to the previous
sale made by her to Mariano Granados, and without knowledge of such transaction.
Assuming, therefore, that said lot was sold to different vendees, pursuant to the provisions of
the above-quoted article 1473, the same became the property of Esteban Santiago.
Under section 39 of Act No. 496, as amended by Act No. 2011, "every person receiving a
certificate of title in pursuance of a decree of registration, and every subsequent purchaser
of registered land who takes a certificate of title for value in good faith shall hold the same
free of all encumbrances except those noted on said certificate, and any of the following
encumbrances which may be subsisting, ...."
Inasmuch as there was no "flaw" in the title of Esteban Santiago, it necessarily follows that,
Celedonio Monton rightly believed that he could, and did, acquire a, likewise, flawless title
from Santiago. As aptly remarked by Mr. Justice Concepcion who penned the decision for
the Court of Appeals —
... Indeed, as a result of the deed of conveyance (Exhibit 1-A) executed by the latter,
Monton stepped into the shoes of Santiago, and became entitled to all the defenses
available to him, including those arising from the acquisition of the property in good

SALES CASE DIGEST COMPILATION 258


faith and for value. Viewed from the strictly legal angle, Monton can not be held,
therefore, to have acted in bad faith, even if he had known of the previous sale made
by Angela Trias to Mariano Granados.
In De la Cruz vs. Fabie (supra), wherein this Court clearly defined the indefeasibility of a
certificate of title in the hands of a purchaser for value in good faith, it was held that an
innocent holder of a title for value, under section 55 of Act No. 496 becomes the absolute
owner of the land.

16) BEATRIZ vs. MARTIN CEDERIA, ET AL.


G.R. No. L-17703
February 28, 1962

FACTS:

The land sought to be partitioned was originally owned by Fausta Alvaro. She sold it in 1935
to Cecilio Babia who also sold it on February 15, 1937 to Isaac Papa.

In turn Isaac Papa sold it to the plaintiffs and their sister, Maria Beatriz, on April 19, 1937.
After the sale the three vendees jointly possessed the land and enjoyed its products in
common.

Isaac Papa sold the land to Martin Cederia, on November 4, 1947 (Exhibit 3). Martin
Cederia, however, testified that the sale actually took place in 1937, although Isaac Papa did
not execute the deed of sale until 1947.
On the other hand, Isaac Papa confirmed having sold the land to the plaintiffs and Maria
Beatriz. He also revealed that Martin Cederia had knowledge of the sale, because he
accompanied Maria Beatriz and in fact it was he who paid to him her 1/3 share of the price.

ISSUE:
Who has better right over the property?

RULING:
The land in question belongs in common to the plaintiffs and defendants.
The foregoing testimony of Isaac Papa is worthy of full belief. As vendor of the property, he
knows to whom he really sold it. As a disinterested party, his testimony carries truthfulness
and sincerity. Moreover, he was corroborated by Martin Cederia himself who admitted
before the barrio lieutenant of San Isidro that the land was in fact the common property of
the plaintiffs and his deceased wife and that he had not paid Isaac Papa the purchase price.
It is thus clear that the land in question belongs in common to the plaintiffs and defendants,
the latter as heirs of Maria Beatriz, in the proportion of 1/3 each. As co-owners the plaintiffs
have the right to demand partition of the said property.
Plaintiffs good faith in registering the former instrument is not disputed, it is obvious that
plaintiffs' claim must be upheld, upon the authority of Article 1544 of the Civil Code of the
Philippines reading:

SALES CASE DIGEST COMPILATION 259


If the same thing should have been sold to different vendees, the ownership shall be
transferred to the person who may have first taken possession thereof in good faith, if it
should be movable property.
Should it be immovable property, the ownership shall belong to the person acquiring
it who in good faith first recorded it in the Registry of Property.
Should there be no inscription, the ownership shall pertain to the person who in good
faith was first in the possession; and, in the absence thereof, to the person who
presents the oldest title, provided there is good faith.

18) CARBONELL VS.. CA, PONCIO


G.R. No. L-29972
January 26, 1976

FACTS:

Jose Poncio was the owner of the parcel of land situated at San Juan, Rizal, subject to
mortgage in favor of the Republic Savings Bank for the sum of P1,500.00. Rosario
Carbonell, a cousin and adjacent neighbor of Poncio lived in the adjoining lot at 177 V. Agan
Street. Emma Infante lives just behind the houses of Poncio and Rosario Carbonell.

Both respondents Rosario Carbonell and Emma Infante offered to buy the said lot from
Poncio.

Poncio, unable to keep up with the installments due on the mortgage, approached Carbonell
one day and offered to sell to the latter the said lot, excluding the house wherein Poncio
lived. Carbonell accepted the offer and proposed the price of P9.50 per square meter.
Poncio accepted the price proposed by Carbonell, on the condition that the money to be
paid to the bank will come from the purchase price.

Upon arriving at Jose Poncio's house, however, the latter told Carbonell that he could not
proceed any more with the sale, because he had already given the lot to Emma Infante.
Poncio admitted that Mrs. Infante improved offer and he agreed to sell the land and its
improvements to her for P3,535.00.

Poncio indeed bound himself to sell to Emma Infante the property for the sum of P2,357.52,
with Emma Infante still assuming the existing mortgage debt in favor of Republic Savings
Bank in the amount of P1,177.48.

Jose Poncio executed the formal deed of sale in favor of respondent Mrs. Infante in the total
sum of P3,554.00 and on the same date, the latter paid Republic Savings Bank the
mortgage indebtedness of P1,500.00. The mortgage on the lot was eventually discharged.

Informed that the sale in favor of Emma Infante had not yet been registered, Atty. Garcia
prepared an adverse claim for Carbonell, who signed and swore to and registered the same
on February 8, 1955.

The deed of sale in favor of Mrs. Infante was registered only on February 12, 1955. As a
consequence thereof, a Transfer Certificate of Title was issued to her but with the annotation
of the adverse claim of Carbonell Rosario Carbonell.

SALES CASE DIGEST COMPILATION 260


ISSUE:
Whether the second sale was valid.

RULING:
Article 1544, New Civil Code, which is decisive of this case, recites:

If the same thing should have been sold to different vendees, the ownership shall be
transferred to the person who may have first taken possession thereof in good faith,
if it should movable property.

Should it be immovable property, the ownership shall belong to the person acquiring
it who in good faith first
recorded it in the Registry of Property.

Should there be no inscription, the ownership shall pertain to the person who in good
faith was first in the possession; and, in the absence thereof, to the person who
presents the oldest title, provided there is good faith.

It is essential that the buyer of realty must act in good faith in registering his deed of sale to
merit the protection of the second paragraph of said Article 1544.
Unlike the first and third paragraphs of said Article 1544, which accord preference to the one
who first takes possession in good faith of personal or real property, the second paragraph
directs that ownership of immovable property should be recognized in favor of one "who in
good faith first recorded" his right. Under the first and third paragraph, good faith must
characterize the act of anterior registration.

If there is no inscription, what is decisive is prior possession in good faith. If there is


inscription, as in the case at bar, prior registration in good faith is a pre-condition to superior
title.

GOOD FAITH OF CARBONELL

When Carbonell bought the lot from Poncio on January 27, 1955, she was the only buyer
thereof and the title of Poncio was still in his name solely encumbered by bank mortgage
duly annotated thereon. Carbonell was not aware — and she could not have been aware —
of any sale of Infante as there was no such sale to Infante then. Hence, Carbonell's prior
purchase of the land was made in good faith. Her good faith subsisted and continued to exist
when she recorded her adverse claim four (4) days prior to the registration of Infantes's deed
of sale. Carbonell's good faith did not cease after Poncio told her on January 31, 1955 of his
second sale of the same lot to Infante. Because of that information, Carbonell wanted an
audience with Infante, which desire underscores Carbonell's good faith. With an aristocratic
disdain unworthy of the good breeding of a good Christian and good neighbor, Infante
snubbed Carbonell like a leper and refused to see her. So Carbonell did the next best thing
to protect her right — she registered her adversed claim on February 8, 1955. Under the
circumstances, this recording of her adverse claim should be deemed to have been done in
good faith and should emphasize Infante's bad faith when she registered her deed of sale
four (4) days later on February 12, 1955.

BAD FAITH OF INFANTE

SALES CASE DIGEST COMPILATION 261


Bad faith arising from previous knowledge by Infante of the prior sale to Carbonell is shown
by the following facts:
1. Mrs. Infante refused to see Carbonell, who wanted to see Infante after she was
informed by Poncio that he sold the lot to Infante but several days before Infante
registered her deed of sale. This indicates that Infante knew — from Poncio and
from the bank — of the prior sale of the lot by Poncio to Carbonell. Ordinarily, one
will not refuse to see a neighbour.

2. Carbonell was already in possession of the mortgage passbook and Poncio's


copy of the mortgage contract, when Poncio sold the lot Carbonell who, after
paying the arrearages of Poncio, assumed the balance of his mortgaged
indebtedness to the bank, which in the normal course of business must have
necessarily informed Infante about the said assumption by Carbonell of the
mortgage indebtedness of Poncio.

3. The fact that Poncio was no longer in possession of his mortgage passbook and
that the said mortgage passbook was already in possession of Carbonell, should
have compelled Infante to inquire from Poncio why he was no longer in
possession of the mortgage passbook and from Carbonell why she was in
possession of the same.

4. Carbonell registered on February 8, 1955 her adverse claim, which was


accordingly annotated on Poncio's title, four [4] days before Infante registered on
February 12, 1955 her deed of sale executed on February 2, 1955. Here she was
again on notice of the prior sale to Carbonell. Such registration of adverse claim
is valid and effective.

5. In his answer to the complaint filed by Poncio, as defendant in the Court of First
Instance, he alleged that both Mrs. Infante and Mrs. Carbonell offered to buy the
lot at P15.00 per square meter, which offers he rejected as he believed that his
lot is worth at least P20.00 per square meter. It is therefore logical to presume
that Infante was told by Poncio and consequently knew of the offer of Carbonell
which fact likewise should have put her on her guard and should have compelled
her to inquire from Poncio whether or not he had already sold the property to
Carbonell.

19) CARAM VS. LAURETA


G.R. No. L-28740
February 24, 1981

FACTS:

On June 25, 1959, Claro L. Laureta filed in the Court of First Instance of Davao an action for
nullity, recovery of ownership and/or reconveyance with damages and attorney's fees
against Marcos Mata, Codidi Mata, Fermin Z. Caram Jr. and the Register of Deeds of Davao
City.

On June 10, 1945, Marcos Mata conveyed a large tract of agricultural land covered by
Original Certificate of Title No. 3019 in favor of Claro Laureta, plaintiff, the respondent
herein. The deed of absolute sale in favor of the plaintiff was not registered because it was
not acknowledged before a notary public or any other authorized officer. At the time the sale

SALES CASE DIGEST COMPILATION 262


was executed, there was no authorized officer before whom the sale could be acknowledged
inasmuch as the civil government in Tagum, Davao was not as yet organized. However, the
defendant Marcos Mata delivered to Laureta the peaceful and lawful possession of the
premises of the land together with the pertinent papers thereof such as the Owner's
Duplicate Original Certificate of Title No. 3019, sketch plan, tax declaration, tax receipts and
other papers related thereto. Since June 10, 1945, the plaintiff Laureta had been and is still
in continuous, adverse and notorious occupation of said land, without being molested,
disturbed or stopped by any of the defendants or their representatives. In fact, Laureta had
been paying realty taxes due thereon and had introduced improvements worth not less than
P20,000.00 at the time of the filing of the complaint.

However, the said property was sold to Fermin Caram, Jr., the petitioner, by Marcos Mata on
May 5, 1947. And was able to declare the ODOCT in the possession of Laureta null and
void, after Mata filed for an issuance of new ODOCT before the RD of Davao on the ground
of loss of the said title.

The Trial Court ruled infavor of Laureta, stating that Caram, Jr. was not a purchaser in good
faith, and the Court of Appeals thenafter affirmed the decision of the lower court.

PETITIONER’S CONTENTION:

The petitioner assails the finding of the trial court that the second sale of the property was
made through his representatives, Pedro Irespe and Atty. Abelardo Aportadera. He argues
that Pedro Irespe was acting merely as broker or intermediary with the specific task and duty
to pay Marcos Mata the sum of P1,000.00 for the latter's property and to see to it that the
requisite deed of sale covering the purchase was properly executed by Marcos Mata; that
the identity of the property to be bought and the price of the purchase had already been
agreed upon by the parties; and that the other alleged representative, Atty. Aportadera,
merely acted as a notary public in the execution of the deed of sale.

ISSUE: Whether petitioner have acted in bad faith through his agents action.

RULING:

In the case at bar, the court found that the Attorneys Irespe and Aportadera had knowledge
of the circumstances, and knew that Mata's certificate of title together with other papers
pertaining to the land was taken by soldiers under the command of Col. Claro L. Laureta.
Added to this is the fact that at the time of the second sale Laureta was already in
possession of the land. Irespe and Aportadera should have investigated the nature of
Laureta's possession. If they failed to exercise the ordinary care expected of a buyer of real
estate they must suffer the consequences. The rule of caveat emptor requires the purchaser
to be aware of the supposed title of the vendor and one who buys without checking the
vendor's title takes all the risks and losses consequent to such failure.

The principle that a person dealing with the owner of the registered land is not bound to go
behind the certificate and inquire into transactions the existence of which is not there
intimated 18 should not apply in this case. It was of common knowledge that at the time the
soldiers of Laureta took the documents from Mata, the civil government of Tagum was not

SALES CASE DIGEST COMPILATION 263


yet established and that there were no officials to ratify contracts of sale and make them
registrable. Obviously, Aportadera and Irespe knew that even if Mata previously had sold the
disputed property such sale could not have been registered.cdrep

There is no doubt then that Irespe and Aportadera, acting as agents of Caram, purchased
the property of Mata in bad faith. Applying the principle of agency, Caram, as principal,
should also be deemed to have acted in bad faith.

Article 1544 of the New Civil Code provides that:

"Art. 1544. If the same thing should have been sold to different vendees, the
ownership shall be transferred to the person who may have first taken possession
thereof in good faith, if it should be movable property.

"Should it be immovable property, the ownership shall belong to the person acquiring
it who in good faith first recorded it in the Registry of Property.

"Should there be no inscription, the ownership shall pertain to the person who in
good faith was first in the possession; and, in the absence thereof, to the person who
presents the oldest title, provided there is good faith. (1973)".

Since Caram was a registrant in bad faith, the situation is as if there was no registration at
all.

20) BERNAS vs. BOLO


81 Phil 16

FACTS:

The land in litigation was transferred twice by its former owner, Lucio Botor, now deceased:
first, in favor of Arcadio M. Bolo on December 19, 1929, through a private document of sale,
and, then in favor of Simeon B. Paz, made ten years later, or on April 29, 1939, by means of
a deed of cession ratified before a notary public and later on registered in the register of
deeds of Camarines Sur.

The lower court, on July 14, 1943, rendered a decision in favor of Simeon B. Paz, because
the deed of cession was registered in the register of deeds, while the deed of sale of 1929
was not and cannot be registered, it not being extended in a public document.

Arcadio M. Bolo, as appellant, among other things, maintained that Simeon B. Paz, in whose
favor the deed of cession of April 29, 1939, was executed by Lucio Botor, acted in bad faith,
because he knew that the land in question had already been sold by Lucio Botor to Arcadio
M. Bolo.

ISSUE:
Who has better right over the property?

RULING:

SALES CASE DIGEST COMPILATION 264


Arcadio. M. Bolo is declared the owner of the land in question.

As evidence, appellant points to the cross-complaint (Exhibit 2), filed by Simeon B. Paz, as
attorney of Lucio Botor in civil case No. 6693 (Exhibit K), where said Simeon B. Paz made
the following allegation:jgc:chanrobles.com.ph

". . . plaintiff (Pedro Babilonia) not being satisfied of the fraudulent transaction with
defendant, the former approached defendant (Lucio Botor) insisting that in as much as the
debt is increased due to interest for several years, one Arcadio Bolo of Baao, Camarines
Sur, through supplication of defendant (Lucio Botor), paid for the latter the supposed
increased debt, and defendant (Lucio Botor) in return ceded 100 coconut trees to said
Arcadio Bolo as payment for the amount advanced by said Arcadio Bolo to plaintiff (Pedro
Babilonia)."cralaw virtua1aw library

The above fact, stated in appellant’s brief, is not disputed by Simeon B. Paz, and is fully
supported by the evidence.

Testifying as his own witness, Simeon B. Paz ratified having alleged in his cross-complaint
dated February 24, 1939, that Lucio Botor had transferred the land in question to Arcadio M.
Bolo. Simeon B. Paz’s knowledge about the transfer made by Lucio Botor in favor of Arcadio
M. Bolo is further evidenced by Exhibits N and O. the letters sent by Simeon B. Paz to Atty.
Arcadio M. Bolo, dated February 23 and 28, 1939, respectively, requesting Bolo to testify in
favor of Lucio Botor at the hearing of civil case No. 6693 about the fact that Bolo paid Lucio
Botor’s debt to Pedro Babilonia, because, as alleged in paragraph III of the complaint and
amended complaint filed in this case by Simeon B. Paz, as counsel for plaintiffs, the
agreement between Lucio Botor and Arcadio M. Bolo at the time the private document of
sale was executed on December 19, 1929, was that the price of P300 agreed upon was to
be delivered by Bolo to Pedro Babilonia, in payment of Botor’s debt.

The cross-complaint in which Simeon B. Paz made the above-quoted allegation was filed on
February 24, 1939 more than two months before the deed of cession was executed in his
favor by his client Lucio Botor on April 29, 1939.

The fundamental premise of the preferential rights established by article 1473 of the Civil
Code is good faith. There is no question that Simeon B. Paz, in acquiring the land in
question, knowing that it had already been sold to Arcadio M. Bolo, had not acted in good
faith and, therefore, the cession in his favor is null and void.

There is no need of passing on the other questions raised by appellant Arcadio M. Bolo.

The appealed decision is reversed and appellant Arcadio. M. Bolo is declared the owner of
the land in question as described in paragraph 2 of the original complaint and amended
complaint, with costs against appellees Simeon B. Paz and Maria M. Paz.

21) PAYLAGO VS. JARABE


131 Phil 16

FACTS:

The entire lot involved in this suit was originally registered in the name of Anselmo Lacatan.
After his death, the transfer certificate was issued in the name his two sons and heirs, Vidal

SALES CASE DIGEST COMPILATION 265


and Florentino Lacatan. After Vidal Lacatan died, his heirs executed a deed of sale of a
portion of their lot in favor of petitioners-spouses.

Years after, Florentino Lacatan also died. His heirs likewise executed a deed of sale in favor
of the same vendees over a portion of the same lot. Upon the registration of the two deeds
of sale, a new TCT was issued in favor of petitioners-spouses.

However, subsequent subdivision survey for the purpose of segregating the two portions of
land described in the deeds revealed that a portion of the total area purchased by petitioners
was being occupied by defendant-respondent. Hence, an action to recover possession
and ownership of the said portion was filed. The lower court rendered judgment in favor of
respondent, and was affirmed in toto by the Court of Appeals

ISSUE: Who has a better right in case of double sale of real property, the registered buyer
or the prior but unregistered purchaser?

RULING:

As held by the Court, the general rule in this matter is that, between two purchasers, the one
who has registered the sale in his favor, in good faith, has a preferred right over the other
who has not registered his title, even if the latter is in the actual possession of the
immovable property.

This is in accordance with Art 1544, providing that if the same immovable property should
have been sold to different vendees, the ownership shall belong to the person acquiring it
who in good faith first recorded it in the registry of property.

However, it was found that their acquisition and subsequent registration were
tainted with the vitiating element of bad faith; petitioners knew beforehand that the parcel of
land in question was owned by respondent. The fundamental premise of the preferential
rights established by Article 1544 of the New Civil Code is good faith. To be entitled to the
priority, the second vendee must not only show prior recording of his deed of conveyance or
possession of the property sold, but must, above all, have acted in good faith, that is to say,
without knowledge of the existence of another alienation by his vendor to a stranger.

22) LUMBRES vs.TABLADA


G.R. No. 165831
February 23, 2007

FACTS:

On January 9, 1995, Spring Homes, entered into a pro forma Contract to Sell with the
respondent spouses. On 1996, after having been paid the sum total of ₱179,500.00, which
the respondents claim to be the full purchase price of the subject lot, Spring Homes
executed a Deed of Absolute Sale in favor of the respondents. Because the anticipated Pag-
Ibig loan failed to materialize, the ₱230,000.00, which, under the Contract to Sell, was
supposed to be paid upon release of the loan, was left unpaid.

SALES CASE DIGEST COMPILATION 266


With the execution of the aforesaid Deed of Absolute Sale, the respondent spouses sent a
demand letter dated May 4, 1996 to Spring Homes for the transfer and release to them of
the original or owner's copy of TCT No. T-284037. The acting president/chairperson of
Spring Homes, Bertha L. Pasic, promised to deliver the said title and even apologized for the
delay.
However, to their great dismay, the spouses subsequently learned that TCT No. T-284037
was canceled and a new one issued to the petitioners.
On account thereof, the respondent spouses filed with the RTC a civil suit against the
petitioners, Spring Homes and the Register of Deeds of Calamba City for nullification of title,
reconveyance and damages, docketed as Civil Case No. 3117-2001-C.
On September 21, 1999, the petitioner spouses entered into a Compromise Agreement in
Civil Case No. 2194-95-C with Spring Homes and Premiere Development Bank, which was
approved by the RTC. In that Compromise Agreement, both Spring Homes and Premiere
Development Bank recognized the rights and interests of the petitioner spouses over the
parcels of land covered by twenty (20) titles and containing an aggregate area of 2,499
square meters. The subject property (Lot 8, Block 3) was among the properties covered by
the aforementioned compromise agreement that were judicially assigned, transferred and
conveyed to the petitioners.
Meanwhile, due to the respondents’ alleged failure to pay the ₱230,000.00 unpaid balance
as per the Contract to Sell earlier adverted to despite demands, the subject lot was sold by
Spring Homes to the petitioners, again by way of a Deed of Absolute Sale executed on
December 22, 2000 for and in consideration of the sum of ₱157,500.00.
While conceding that there is a double sale in this case, the RTC, in its Order, refused to
apply the provisions of Article 1544 of the Civil Code in settling the issue of possession.
Instead, it went to the extent of determining the validity and due execution of the separate
Deeds of Absolute Sale executed by Spring Homes in favor of the herein contending
parties.In holding that the petitioners have superior right on the subject lot over the
respondents, the RTC declared that there was no valid deed of absolute sale executed in
favor of the respondents for the following reasons:
1. Even if there was a perfected Contract to Sell between respondents and Spring
Homes, the former failed to pay the full purchase price in installments that gave
Spring Homes the right to cancel the contract; and
2. The execution of the Deed of Absolute Sale in favor of the respondents on
January 16, 1996 is not a transfer of ownership but merely to use it as a collateral for
a loan of P230,000.00 from the Pag-Ibig Fund which, incidentally, did not materialize.
ISSUE: Who has better right over the subject lot.
RULING:
Respondents are the buyers entitled to the physical possession of the subject lot
since the prevailing doctrine is that as between the buyer who is in possession of a
Torrens title but who has acquired it in bad faith and the first buyer who failed to have
his title recorded in the Registry of Property, the first buyer must prevail.
We find no merit in petitioners' contention that the first sale to the respondents was void for
want of consideration.
Having stated that the Deed of Absolute Sale executed in favor of the respondent spouses is
valid and with sufficient consideration, the MTCC correctly applied the provisions of Article
1544 of the Civil Code. Article 1544 reads:
Art. 1544. If the same thing should have been sold to different vendees, the
ownership shall be transferred to the person who may have first taken possession
thereof in good faith, if it should be movable property.

SALES CASE DIGEST COMPILATION 267


Should it be immovable property, the ownership shall belong to the person acquiring
it who in good faith first recorded it in the Registry of Property.
Should there be no inscription, the ownership shall pertain to the person who in good
faith was first in the possession, and, in the absence thereof, to the person who
presents the oldest title, provided there is good faith.
Notwithstanding the fact that the petitioners, as the second buyer, registered their Deed of
Absolute Sale, in contrast to the Deed of Sale of the respondents which was not registered
at all precisely because of Spring Homes’ failure to deliver the owner’s copy of TCT No. T-
284037, the respondents’ right could not be deemed defeated as the petitioners are in bad
faith. Petitioners cannot claim good faith since at the time of the execution of the
Compromise Agreement in Civil Case No. 2194-95-C, they were indisputably and
reasonably informed that the subject lot was previously sold to the respondents. In fact, they
were already aware that the respondents had constructed a house thereon and are
presently in possession of the same.
Knowledge gained by the second buyer of the first sale defeats his rights even if he is the
first to register the second sale because such knowledge taints his prior registration with bad
faith. For the second buyer to displace the first, he must show that he acted in good faith
throughout (i.e. in ignorance of the first sale and of the first buyer's rights) from the time of
acquisition until the title is transferred to him by registration.
Consequently, the respondents are the buyers entitled to the physical possession of the
subject lot since the prevailing doctrine is that as between the buyer who is in possession of
a Torrens title but who has acquired it in bad faith and the first buyer who failed to have his
title recorded in the Registry of Property, the first buyer must prevail.14
Hence, there was nothing to cede or transfer to the petitioners when the Compromise
Agreement in Civil Case No. 2194-95-C was entered into on October 28, 1999 insofar as the
subject lot is concerned as it was already sold to and fully paid for by the respondents as
early as January 16, 1996 when the Absolute Deed of Sale was executed in their favor by
Spring Homes. More so that Spring Homes has no more to sell to the petitioners when it
executed in the latter’s favor the second deed of absolute sale on December 22, 2000.

23) ULEP VS. CA

472 SCRA 241

FACTS:

Valentin Ulep sold the one-half (1/2) eastern portion of Lot 840, comprising an area of 1,635
square meters, to respondent Maxima Rodico, while the remaining one-half (1/2) western
portion with the same area, to his son Atinedoro Ulep married to Beatriz Ulep, and to his
other daughter ValentinaUlep.

On June 1971, Atinedoro Ulep, his wife Beatriz and sister Valentina Ulep sold the one-half
(1/2) portion of the area sold to them by their father to their brother Samuel Ulep and the
latter’s wife, Susana Repogia-Ulep. The sale was registered in the Office of Registry of
Pangasinan on February 1973.3. Later, an area of 507.5 square meters of the western
portion of Lot 840 was sold by the spouses Atinedoro Ulep and Beatriz Ulep to respondent
Warlito Paringit and the latter’s spouse Encarnacion Gante. All the foregoing transactions
were done and effected without an actual ground partition or formal subdivision of Lot 8405.

In June 1977, respondent Iglesia ni Cristo (INC) begun constructing its chapel on Lot 840. In
the process, INC encroached portions thereof allegedly pertaining to petitioners and blocked
their pathways. This prompted Samuel Ulep and sister Rosita Ulep to make inquiries with

SALES CASE DIGEST COMPILATION 268


the Office of the Register of Deeds of Pangasinan. They discovered from the records of said
office that a deed of sale bearing date December 21, 1954, was purportedly executed by
their brother Atinedoro Ulep his, wife Beatriz and their sister Valentina Ulep in favor of INC
over a portion of 620 square meters, more or less, of Lot 8407. They also discovered that on
July 9, 1975, an affidavit of subdivision was executed by respondents INC, Maxima Rodico
and the spouses Warlito Paringit and Encarnation Gante, on the basis of which affidavit Lot
840 was subdivided into four lots.8. The petitioners filed a complaint for Quieting of Title,
Reconveyance and Declaration of Nullity of Title and Subdivision Plan with Damages
against respondents INC, Maxima Rodico and the spouses Warlito Paringit and Encarnacion
Gante. They allege that they and the respondents are co-owners of Lot 840.9.

The spouses Atinedoro Ulep and Beatriz Ulep and their sister Valentina Ulep denied having
executed a deed of sale in favor of INC over a portionof 620 square meters of Lot 840,
claiming that their signatures appearing on the deed were forged. They claimed that at the
most, they sold to INC only 100square meters and not 620 square meters.

Samuel and Valentina Ulep also averred that lot was subdivided without their knowledge and
consent.

INC asserted that it purchased from the spouses Atinedoro Ulep and Beatriz Ulep and their
sister Valentina Ulep the portion containing 620 square meters of Lot 840 on December 21,
1954, as evidenced by a deed of sale duly registered with the Registry of Deeds of
Pangasinan.

The trial court renders judgment in favor of the petitioners, ordering INC tovacate and
surrender 520 square meters of the land in favor of the Spouses Samuel Ulep.

Upon appeal, the Court of Appeals reversed the decision.

ISSUE:

Who has a better right over the 620 square meters of land—the spouses Samuel Ulep or
INC?

RULING:
The INC has a better right over the 620 square meters.

As the Court sees it, the present controversy is a classic case of double sale. On December
21, 1954, Atinedoro Ulep, his wife Beatriz Ulep and sister Valentina Ulep sold the disputed
area (620 square-meter) of Lot 840 to INC.

Subsequently, on January 18, 1971, a second sale was executed by the same vendors in
favor of spouses Samuel Ulep and Susana Ulep. The Court is, therefore, called upon to
determine which of the two groups of buyers has a better right to the area in question.

xxx

The law provides that a double sale of immovable transfers ownership to :

(1) The first registrant in good faith;

(2) Then, the first possessor in good faith; and

(3) Finally, the buyer who in good faith presents the oldest title.

SALES CASE DIGEST COMPILATION 269


Jurisprudence teaches that the governing principle is primus tempore, potior jure(first in
time, stronger in right). Knowledge gained by the first buyer of the second sale cannot defeat
the first buyer’s rights except where the second buyer registers in good faith the second sale
ahead of the first, as provided by the aforequoted provision of the Civil Code. Such
knowledge of the first buyer does not bar him from availing of his rights under the law,
among them to register first his purchase as against the second buyer.

In converso, knowledge gained by the second buyer of the first sale defeats his rights even if
he is first to register the second sale, since such knowledge taints his prior registration with
bad faith. This is the price exacted by the same provision of the Civil Code for the second
buyer to be able to displace the first buyer; before the second buyer can obtain priority over
the first, he must show that he acted in good faith throughout (i.e. ignorance of the first sale
and of the first buyer’s rights) from the time of acquisition until the title is transferred to him
by registration, or, failing registration, by delivery of possession.13

Per records, the sale of the disputed 620 square-meter portion of Lot 840 to respondent INC
was made on December 21, 1954 and registered with the Registry of Deeds of Pangasinan
on January 5, 1955. In fact, INC was issued a title over the same portion on September 23,
1975. On the other hand, the conveyance to the spouses Samuel Ulep and Susana
Repogia-Ulep happened on January 18, 1971 and the spouses registered their document of
conveyance only on February 22, 1973.
Clearly, not only was respondent INC the first buyer of the disputed area. It was also the first
to register the sale in its favor long before petitioners Samuel’s and Susana’s intrusion as
second buyers. Although Samuel and Susana thereafter registered the sale made to them,
they did so only after 18 years from the time INC caused the registration of its own
document of sale.
"Registration" means any entry made in the books of the Registry which records solemnly
and permanently the right of ownership and other real rights.15 However, mere registration is
not sufficient. Good faith must concur with registration, else registration becomes an
exercise in futility.16
In the instant case, the registration made by respondent INC of its deed of sale more than
satisfies this requirement. The same thing cannot be said of petitioners Samuel Ulep and
Susana Ulep. Said petitioners, by their own admission, were aware that there existed an
agreement between INC and vendors Atinedoro Ulep, his wife Beatriz and sister Valentina
Ulep involving a portion of 100 square meters of Lot 840. Knowledge of such transaction
should have put the spouses Samuel Ulep and Susana Ulep upon such inquiry or
investigation as might be necessary to acquaint them with the possible defects in the title of
their vendors. They should have acted with that measure of precaution which may
reasonably be required of a prudent man in a similar situation. After all, good faith, or the
lack of it, is, in the last analysis, a question of intention. But in ascertaining the intention by
which one is actuated on a given occasion, courts are necessarily controlled by the evidence
as to the conduct and outward acts by which the inward motive may, with safety, be
determined. So it is that ‘the honesty of intention,’ ‘the honest lawful intent,’ which constitutes
good faith implies a ‘freedom from knowledge and circumstances which ought to put a
person on inquiry.’ 17 Hence, proof of such knowledge overcomes the presumption of good
faith.
Here, the spouses Samuel Ulep and Susana Ulep were fully aware, or could have been, if
they had chosen to inquire, of the rights of INC under the deed of sale duly annotated on the
common title of the spouses Atinedoro Ulep and Beatriz Ulep and Valentina Ulep.
Verily, the sale to INC should prevail over the sale made to spouses Samuel and Susana
because INC was the first registrant in good faith.

SALES CASE DIGEST COMPILATION 270


24) MORALES DEVELOPMENT COMPANY, INC. VS. CA
137 PHIL. 307

FACTS:
Enrique P. Montinola, alleging that his owner's duplicate copy of said certificate had been
lost, Montinola succeeded in securing, from the Court, an order for the issuance of a second
owner's duplicate, with which he managed to sell the lot, on September 24, 1954, to Pio
Reyes.
Lupo Abella, married to Felisa Aguilar — hereafter referred to as the Abellas — purchased
the land from Reyes. About seven (7) months later, or on June 16, 1955, the Abellas sold
the land, for P7,000, — of which P4,500 was then paid — to the Deseos, who immediately
took possession of the property.
It appears, however, that the first owner's duplicate of TCT No. T-15687 was either never
lost or subsequently found by Montinola. Then, Montinola sold the property to Morales, for
P2,000. Morales was advised by the office of the Register of Deeds of Quezon that said TCT
No. T-15687 had already been cancelled and the property sold, first, to Pio Reyes, and,
then, to the Abellas.
Thereupon, Morales filed a petition for the annulment and cancellation of the second owner's
copy of TCT No. T-15687.

ISSUE: Who has better right over the property?

RULING:
Deseos has the better right. The ownership" of said lot "pertain(s)" — pursuant to
Article 1544 of our Civil Code 3 — to the Deseos, as the only party who took
possession thereof in good faith.
Deseos had bought the land in question for value and in good faith, relying upon the transfer
certificate of title in the name of their assignors, the Abellas. The sale by the latter to the
former preceded the purchase made by Morales, by about eight (8) months, and the Deseos
took immediate possession of the land, which was actually held by them at the time of its
conveyance to Morales by Montinola, and is in the possession of the Deseos, up to the
present.
Since the object of this litigation is a registered land and the two (2) buyers thereof have so
far been unable to register the deeds of conveyance in their respective favor, it follows that
"the ownership" of said lot "pertain(s)" — pursuant to Article 1544 of our Civil Code 3 — to the
Deseos, as the only party who took possession thereof in good faith.
Had Morales, at least gone to the Office of the Register of Deeds as the Deseos did —
before purchasing the property in dispute, Morales would have found out, not only that TCT
No. T-15687 had long been cancelled, but, also, that the property had been previously
sold by Montinola to Reyes and by Reyes to the Abellas. In short, the negligence of Morales
was the proximate cause of the resulting wrong, and, hence, Morales should be the party to
suffer its consequences.

SALES CASE DIGEST COMPILATION 271


ANNA
25) Please refer to the case No. 9 on Double Sales.

26.) CORONEL vs CA ,263 SCRA 15

FACTS: The petition involves a complaint for specific performance to compel petitioners to
consummate the sale of a parcel of land with its improvements located along Roosevelt
Avenue in Quezon City entered into by the parties sometime in January 1985 for the price of
P1,240,000.00.

On January 19, 1985, defendants-appellants Romulo Coronel, et al. (Coronels) executed a


document entitled "Receipt of Down Payment" in favor of plaintiff Ramona Patricia Alcaraz
(hereinafter referred to as Ramona)
Clearly, the conditions appurtenant to the sale are the following:

1. Ramona will make a down payment P50,000.00 upon execution of the document
aforestated;
2. The Coronels will cause the transfer in their names of the title of the property registered in
the name of their deceased father upon receipt of the P50,000.00 down payment;
3. Upon the transfer in their names of the subject property, the Coronels will execute the
deed of absolute sale in favor of Ramona and the latter will pay the former the whole
balance of P1,190,000.00.

On the same date (January 15, 1985), Concepcion D. Alcaraz (Concepcion), mother of
Ramona, paid the down payment of P50,000.00.

On February 6, 1985, the property originally registered in the name of the Coronels' father
was transferred in their names under TCT No. 327043.

On February 18, 1985, the Coronels sold the property covered by TCT No. 327043 to
intervenor-appellant Catalina B. Mabanag (Catalina) for P1,580,000.00 after the latter has
paid P300,000.00. For this reason, Coronels canceled and rescinded the contract with
Ramona by depositing the down payment paid by Concepcion in the bank in trust for
Ramona Patricia Alcaraz .

On February 22, 1985, Concepcion, et al., filed a complaint for specific performance against
the Coronels and caused the annotation of a notice of lis pendens at the back of TCT No.
327403.

On April 2, 1985, Catalina caused the annotation of a notice of adverse claim covering the
same property with the Registry of Deeds of Quezon City .

On April 25, 1985, the Coronels executed a Deed of Absolute Sale over the subject property
in favor of Catalina to which a new title over the subject property was issued in her name.

ISSUE: Whether or not Article 1544 applies on this case.

RULING: Yes. The sale to the other petitioner, Catalina B. Mabanag, gave rise to a case of
double sale where Article 1544 of the Civil Code applies. Under the established facts and
circumstances of the case, the Court may safely presume that, had the certificate of title
been in the names of petitioners-sellers at that time, there would have been no reason why

SALES CASE DIGEST COMPILATION 272


an absolute contract of sale could not have been executed and consummated right there
and then.
What may be perceived from the respective undertakings of the parties to the contract is that
petitioners had already agreed to sell the house and lot they inherited from their father,
completely willing to transfer full ownership of the subject house and lot to the buyer if the
documents were then in order. That is why they undertook to cause the issuance of a new
transfer of the certificate of title in their names upon receipt of the down payment in the
amount of P50,000.00. As soon as the new certificate of title is issued in their names,
petitioners were committed to immediately execute the deed of absolute sale. Only then will
the obligation of the buyer to pay the remainder of the purchase price arise.
What is clearly established by the plain language of the subject document is that when the
said "Receipt of Down Payment" was prepared and signed by petitioners Romeo A. Coronel,
et al., the parties had agreed to a conditional contract of sale, consummation of which is
subject only to the successful transfer of the certificate of title from the name of petitioners'
father, Constancio P. Coronel, to their names.
The Court significantly notes this suspensive condition was, in fact, fulfilled on February 6,
1985. Thus, on said date, the conditional contract of sale between petitioners and private
respondent Ramona P. Alcaraz became obligatory, the only act required for the
consummation thereof being the delivery of the property by means of the execution of the
deed of absolute sale in a public instrument, which petitioners unequivocally committed
themselves to do as evidenced by the "Receipt of Down Payment."
With the foregoing conclusions, the sale to the other petitioner, Catalina B. Mabanag, gave
rise to a case of double sale where Article 1544 of the Civil Code will apply, to wit:
Art. 1544. If the same thing should have been sold to different vendees, the ownership shall
be transferred to the person who may have first taken possession thereof in good faith, if it
should be movable property. Should if be immovable property, the ownership shall belong to
the person acquiring it who in good faith first recorded it in Registry of Property. Should there
be no inscription, the ownership shall pertain to the person who in good faith was first in the
possession; and, in the absence thereof to the person who presents the oldest title, provided
there is good faith.
The above-cited provision on double sale presumes title or ownership to pass to the first
buyer, the exceptions being: (a) when the second buyer, in good faith, registers the sale
ahead of the first buyer, and (b) should there be no inscription by either of the two buyers,
when the second buyer, in good faith, acquires possession of the property ahead of the first
buyer. Unless, the second buyer satisfies these requirements, title or ownership will not
transfer to him to the prejudice of the first buyer.
.
In a case of double sale, what finds relevance and materiality is not whether or not the
second buyer was a buyer in good faith but whether or not said second buyer registers such
second sale in good faith, that is, without knowledge of any defect in the title of the property
sold.

27.) DAUZ VS. ECHAVEZ, 533 SCRA 637


.
FACTS:
Spouses Albert Oguis, Sr. and Florencia Refuerzo Oguis were the owners of a parcel of land
with an area of 10,000 square meters, more or less, located at Barangay Irisan, Tuba,
Benguet.

On January 12, 1982, spouses Oguis sold a portion of the land, consisting of 1,295 square
meters, to spouses Eligio and Lorenza Echavez, respondents. On April 1, 1982, they had
the sale registered in the Registry of Deeds of Benguet.

SALES CASE DIGEST COMPILATION 273


Sometime in July 1982, spouses Oguis, being in need of money for their travel to the United
States, sold the remaining portion of the land to the same respondents. This sale was not
registered upon request of Albert Oguis, Sr. as he and his wife intended to repurchase the
property upon their return from abroad.

When spouses Oguis returned to the Philippines sometime in May 1987, they informed
respondents that they are no longer interested in buying back the property. Hence,
respondents paid the real property taxes for the entire property corresponding to the period
from 1983 to 1987.

Sometime in September 1987, Florencia Refuerzo Oguis died. On May 28, 1988,
Albert Oguis, Sr. and his two children, executed a Deed of Extrajudicial Settlement of
Estate. On the same date, they sold to spouses Florendo and Helen Dauz, petitioners, a
portion of the land consisting of 7,616 square meters for P65,000.00. Albert Oguis, Sr.
informed petitioners that he had sold only 1,295 square meters to respondents, as shown by
the annotation of the sale at the back of TCT No. T-13728. Albert Oguis, Sr. gave a xerox
copy of this title to spouses Dauz, claiming that he lost the owners copy. And when he died ,
petitioners filed with the RTC of Baguio for Petition for Issuance of a New Duplicate Copy of
TCT No. T-13728

On January 25, 1989, respondents had the sale to them of the remaining 7,616
square meters portion of the land registered in the same Registry of Deeds. Consequently,
TCT No. T-13728 (in the names of spouses Oguis) was cancelled and in lieu thereof, TCT
No. T-24630 was issued in respondents names. This new title covers the entire
property previously owned by spouses Oguis.

Meanwhile, petitioners sold to spouses Ignacio and Francisca Reambonanza, also


petitioners, 1,000 square meters of the 7,616-square meter portion sold to them by Albert
Oguis, Sr. and his two children. The selling price was P80,000.00.

On February 2, 1989, petitioners (spouses Dauz and spouses Reambonanza) filed with the
RTC, Branch 5, Baguio City a complaint for declaration of ownership. Petitioners, however,
impute bad faith on the part of respondents since they caused the registration of the sale
dated January 25, 1989 only after learning that petitioners, spouses Dauz, filed with the RTC
a petition for the issuance of a new duplicate copy of TCT No. T-13728.

Issue: Whether or Not there was bad faith on the part of Echavez in the Registration of the
sale.

RULING:
No. There was no evidence showing that respondent acted in bad faith. In China vs.
Airlines , Ltd. V CA, it was held that bad faith does not simply connote bad judgment or
negligence.It imports dishonest purpose or some moral obliquity and conscious doing of
wrong. It means breach of a known duty through motive , interest, or ill will that partakes of
the nature of fraud. These incidents or circumstances are not present here, Respondent did
not register the sale because they waited for Spouses Oguis to repurchase the property. In
fact it was Albert Oguis, Sr. himself who requested them not to cause the registration of the
sale.
Considering that respondent registered the sale in good faith , we rule that they are the
owners of the entire property now covered by TCT No. -26430 in their names.
Here, the subject property was sold twice, first, by spouses Oguis to respondents;
and second, by Albert Oguis, Sr. and his two children to petitioners (spouses Dauz).

SALES CASE DIGEST COMPILATION 274


Article 1544 of the Civil Code is relevant, thus:

Article 1544. If the same thing should have been sold to different vendees, the
ownership shall be transferred to the person who may have first taken possession
thereof in good faith, if it should be movable property.

Should it be immovable property, the ownership shall belong to the person acquiring
it who in good faith first recorded it in the Registry of Property.

Should there be no inscription, the ownership shall pertain to the person who in good
faith was first in possession; and, in the absence thereof, to the person who presents
the oldest title, provided there is good faith.

In April 1982, respondents caused the registration of the sale of the 1,295-square meter
portion of the land; and on January 25, 1989, the sale of the remaining 7,616 square meters
in the Registry of the Deeds. Petitioners (spouses Dauz), on the other hand, failed to cause
the registration of the sale to them in the Registry of Deeds. Where both parties claim to
have purchased the same property, as in this case, Article 1544 cited above provides that as
between two purchasers, the one who registered the sale in his favor has a preferred right
over the other who has not registered his title, even if the latter is in actual possession of the
immovable property.

28.) BERNADEZ VS. CA, 533 SCRA 451

FACTS:
Aurea Paredes Vda. de Pascual and Araeli Felicia P. Sevilla are among the
registered owners of a parcel of land covered by Transfer Certificate of Title (TCT) No. RT-
18756 (224112) PR-26276 consisting of 746 square meters located at No. 315 Roosevelt
Avenue, San Francisco Del Monte, Quezon City. On the said lot stands a four-door
apartment.
Sometime in 1970, herein respondent spouses Leopoldo and Clarita Magtoto rented
a unit of the apartment and used it as a grocery store. After several years, they also rented
the adjoining unit as a junkshop.

In December 1985, Aurea, represented by her attorney-in-fact, Araceli, sold to


the Magtoto spouses the two units consisting of 154 square meters forP700,000.00 and
executed a Conditional Deed of Sale .

However, in July 1990, Araceli, as co-owner and attorney-in-fact of the other co-
owners, offered to sell the whole lot consisting of 746 square meters to herein petitioner
spouses Brilly and Olivia Bernardez. At that time, the property was mortgaged in favor
of Banco Filipino; but the mortgage was cancelled upon payment ofP754,753.26 by
the Bernardez spouses.

Meanwhile, the Magtoto spouses filed a complaint for injunction and damages
against Aurea and Araceli for their alleged refusal to honor the first Conditional Deed of Sale
disposing of the 154-square meter portion of the property. A notice of lis pendens was

SALES CASE DIGEST COMPILATION 275


accordingly inscribed at the back of TCT No. RT-18756 (224112) PR-26276 in October
1990.

In November 1990, a second Deed of Conditional Sale was executed


by Aurea and Araceli, this time over the whole lot consisting of 746 square meters, in favor of
the Bernardez spouses, for P7,000,000.00.

In an interesting twist, the notice of lis pendens was meanwhile cancelled by virtue of
an alleged court order, which the RTC later disclaimed. Upon discovery of the forgery, the
trial court immediately ordered the re-annotation of the notice of lis pendens.

Alarmed by the forgery, the Magtoto spouses informed the Bernardez spouses of
the pending case involving the disputed property and advised them to review the sale before
they make the next payment.

ISSUE:

Who between the petitioners (Bernardezes) and private respondents (Magtotos) has a
better right to the 154-square meter portion of the subject property?

HELD: The respondents ( Magtotos ) .This case involves a double sale of an immovable,
specifically the 154-square meter portion of the disputed property. In this connection, the
pertinent Article of the Civil Code provides:

Art. 1544. If the same thing should have been sold to different
vendees, the ownership shall be transferred to the person who may have first
taken possession thereof in good faith, if it should be movable property.

Should it be immovable property, the ownership shall belong to


the person acquiring it who in good faith first recorded it in the Registry
of Property.

Should there be no inscription, the ownership shall pertain to the


person who in good faith was first in the possession; and in the absence
thereof, to the person who presents the oldest title, provided there is good
faith. (Emphasis supplied.)

The law is clear that when the thing sold twice is an immovable, the one who
acquires it and first records it in the Registry of Property shall be deemed the owner. Primus
tempore, potior jure. First in time, stronger in right. However, the act of registration must be
coupled with good faith. That is, the registrant must have no knowledge of any defect in the
title of the vendor or must not have been aware of facts which should have put him upon
such inquiry and investigation as might be necessary to acquaint him with any such defect.

In this particular case, facts on record clearly show that the Bernardez spouses
knew, at the time they bought the subject property, that the 154-square meter portion of the
lot had been previously sold to the Magtoto spouses and that there was in fact a pending
litigation involving the said property.

Further, the said prior sale of the 154-square meter portion of the property to
the Magtoto spouses was precisely the reason why the Bernardez spouses entered into a
Memorandum of Agreement with the vendors in order to proceed with the sale. By their own
admissions and undertakings, the Bernardez spouses are already estopped from claiming
lack of knowledge of the prior sale. They cannot be said to have been in good faith in

SALES CASE DIGEST COMPILATION 276


registering the subject property in their name. Hence, between the Bernardez spouses and
the Magtoto spouses, we rule that the latter have a better right to the 154-square meter
portion of the property, having first registered the same in good faith.

29.) G.R. NO. 121165, September 26, 2006


HON. DOMINADOR F. CARILLO et al VS. HON. COURT OF APPEALS, MARIA PAZ
DABON AND ROSALINA DABON

FACTS: Maria Gonzales alleged that on April 26, 1988, she paid P10,000 to Priscilla as
down payment on the P400,000 purchase price of the lot with improvements, since Priscilla
had a special power of attorney from her son, Aristotle, the owner of the land. They also
agreed that the balance would be paid within three months after the execution of the deed of
sale. Yet, after the lapse of the period and despite repeated demands, Priscilla did not
execute the deed of sale. Since Priscilla failed to execute the Deed of Sale, Gonzales filed a
case for specific performance and impleaded Priscilla (not Aristotle). The latter defaulted and
judgment was rendered against her ordering the nullification of the OCT of Aristotle and the
issuance of a new certificate of title in favor of Gonzales. The Dabons thereafter surfaced
and sought to annul the judgment of the trial court asserting that they purchased the
property from Aristotle himself and they were not impleaded as the real parties in interest.

ISSUE: Who has a better title to the land, Gonzales or Dabon?

RULING:

DABON. The decision of the lower court in favor of Gonzales was void due to extrinsic fraud.
There is extrinsic fraud when a party has been prevented by fraud or deception from
presenting his case. Fraud is extrinsic where it prevents a party from having a trial or from
presenting his entire case to the court, or where it operates upon matters pertaining not to
the judgment itself but to the manner in which it is procured. The overriding consideration
when extrinsic fraud is alleged is that the fraudulent scheme of the prevailing litigant
prevented a party from having his day in court. Of the indices of fraud cited by the Court of
Appeals, the failure to comply with the notification requirement in the petition for the
cancellation of title amounts to extrinsic fraud. Under the Property Registration Decree, all
parties in interest shall be given notice. There is nothing in the records that show Gonzales
notified the actual occupants or lessees of the property. Further, the records show that
Gonzales had known of the sale of the land by Aristotle to the Dabons and despite her
knowledge, the former did not include the Dabons in her petition for the annulment of title.
Deliberately failing to notify a party entitled to notice also constitutes extrinsic fraud. This
fact is sufficient ground to annul the order allowing the cancellation of title in the name of
Gonzales. The court never acquired jurisdiction. It must be noted that the property was sold
to Gonzales in 1988, while the same was sold to the Dabons in 1989; nonetheless, the
requirements of double-sale are two-fold: acquisition in good faith and registration in good
faith. Based on the foregoing, the case is remanded to the lower court for further
proceeding.

30) Tomassi vs. Villa- Abrille, 104 Phil. 311

FACTS:

On 28 May 1948 Fernando Villa-Abrille purchased from the Surplus Property


Commission at a public auction sale. All movable goods located at CMD 3 Area, Samar

SALES CASE DIGEST COMPILATION 277


Naval Base, Guiuan, Samar. Excluding certain articles enumerated therein. Immediately, he
took possession of all the properties located within CMD 3 Area.
On 6 October 1949 Santiago Gancayco purchased from the Surplus Property
Commission at a public auction sale 1-Lot of surplus property known as Lot No. 1, per
Notice of Sale dated August 27, 1949 and Notice of Negotiated Sale of September 17, 1949,
ALL AT GUIUAN BASE excluding certain articles .

On 2 February 1950 Raymond Tomassi acquired by purchase all the rights, share
and interest of Santiago Gancayco to the properties mentioned in SPC Invoice No. 10658 ,
except those expressly excluded in the deed of sale.

ISSUE: Whether or Not there is double sale of a movable goods.

HELD: Yes.
All movable goods located at CMD 3 Area, Samar Naval Base, Guiuan, Samar, consisting
more or less of the following: 21 Revolving Cranes, etc., etc.,could and can have no other
meaning than that "All movable goods located at CMD 3 Area, Samar Naval Base, Guiuan,
Samar," were sold to the appellee Villa-Abrille. The phrase "consisting more or less of the
following" followed by an enumeration of articles sold does not mean that whatever excess
article or articles not included in the enumeration is not included in the sale to the appellee
Villa-Abrille and may be sold to another bidder by the Surplus Property Commission.
Unless otherwise specified, the quantities of the various items listed are approximate only.
Any variations between the quantity stated for any item sold on a per item basis and the
quantity of such item actually delivered to the Buyer will be adjusted on the basis of the unit
price quoted for such item or items.
On 19 May 1948 the Control Committee of the Government Enterprise Council approved the
sale "of all movable goods located at CMD 3 Area, Samar Naval Base, Guiuan, Samar,
excluding the items indicated in the letter of this office, dated April 29, 1948," in favor of the
appellee Villa-Abrille .As early as 4 August 1948, or long before the sale by the Surplus
Property Commission to Santiago Gancayco made on 6 October 1949 of a lot of surplus,
which the appellant acquired by purchase from Santiago Gancayco on 22 February 1950,
the Board of Liquidators had acknowledged "that the intention of the sale was for all the
movable goods for purposes of liquidation less the specified exclusions as indicated in the
corresponding Invoice No. 7770 dated May 28, 1948 and the frozen items covered by the
freezing order of April 2, 1948 among which is construction materials and that "It is the
intention of the sale to include all movable goods located thereat, subject to the specified
exclusions and freezing order, for purposes of liquidation.”

What had been sold to the appellee Fernando Villa-Abrille by the Surplus Property
Commission apparently was partly sold again to Santiago Gancayco who in turn sold it to
the appellant Raymond Tomassi. The transaction between the Surplus Property
Commission and Santiago Gancayco would seem to be a case of double sale, and if that
should be the fact it would be governed by paragraph 1, Article 1473 of the old Civil
Code, which provides that

If the same thing should have been sold to different vendees, the ownership shall be
transferred to the person who may have first taken possession thereof in good faith, if it
should be personal property.
As it was the appellee Fernando Villa-Abrille who in good faith first took possession of the
articles in controversy, he has a better right to ownership and possession over them than the
appellant Raymond Tomassi.

SALES CASE DIGEST COMPILATION 278


31.) VILLA REY TRANSIT VS FERRER, 25 SCRA 861

FACTS: Jose M. Villarama was an operator of a bus transportation, under the business
name of Villa Rey Transit pursuant to certificates of public convenience granted him by the
Public Service Commission (PSC) which authorized him to operate a total of 32 units on
various routes or lines from Pangasinan to Manila, and vice-versa. On 8 January 1959, he
sold the two certificates of public convenience to the Pangasinan Transportation Company,
Inc. (Pantranco), for P350,000.00 with the condition, among others, that the seller
(Villarama) "shall not for a period of 10 years from the date of this sale, apply for any TPU
service identical or competing with the buyer."

Barely 3 months thereafter, or on 6 March 1959: a corporation called Villa Rey Transit, Inc.
(the Corporation) was organized. In less than a month after its registration with the
Securities and Exchange Commission, the Corporation, on 7 April 1959, bought 5
certificates of public convenience, 49 buses, tools and equipment from one Valentin
Fernando."

The very same day that the contract of sale was executed, the parties thereto immediately
applied with the PSC for its approval, with a prayer for the issuance of a provisional authority
in favor of the vendee Corporation to operate the service therein involved. On 19 May 1959,
the PSC granted the provisional permit prayed for, upon the condition that "it may be
modified or revoked by the Commission at any time, shall be subject to whatever action that
may be taken on the basic application and shall be valid only during the pendency of said
application." Before the PSC could take final action on said application for approval of sale,
however, the Sheriff of Manila, on 7 July 1959, levied on 2 of the five certificates of public
convenience involved therein, namely, those issued under PSC cases 59494 and 63780,
pursuant to a writ of execution issued by the Court of First Instance of Pangasinan in Civil
Case 13798, in favor of Eusebio E. Ferrer against Valentin Fernando. The Sheriff made and
entered the levy in the records of the PSC. On 16 July 1959, a public sale was conducted by
the Sheriff of the said two certificates of public convenience. Ferrer was the highest bidder,
and a certificate of sale was issued in his name. Thereafter, Ferrer sold the two certificates
of public convenience to Pantranco, and jointly submitted for approval their corresponding
contract of sale to the PSC. Pantranco therein prayed that it be authorized provisionally to
operate the service involved in the said two certificates.

ISSUE: Who among Villa Rey Transit(Corporation) and Ferrer has a better rights over the
CPC’s sold in the Sheriff’s sale.

HELD: The Corporation has the better rights over the CPC .The sale between Fernando
and the Corporation is valid, such that the rightful ownership of the disputed certificates still
belongs to the plaintiff being the prior purchaser in good faith and for value thereof. In view
of the ancient rule of caveat emptor prevailing in this jurisdiction, what was acquired by
Ferrer in the sheriff's sale was only the right which Fernando, judgment debtor, had in the
certificates of public convenience on the day of the sale.

Accordingly, by the "Notice of Levy Upon Personalty" the Commissioner of Public Service
was notified that "by virtue of an Order of Execution issued by the Court of First Instance of
Pangasinan, the rights, interests, or participation which the defendant, VALENTIN A.
FERNANDO — in the above entitled case may have in the following realty/personalty is
attached or levied upon, to wit: The rights, interests and participation on the Certificates of
Public Convenience issued to Valentin A. Fernando, in Cases Nos. 59494, etc. ... Lines —
Manila to Lingayen, Dagupan, etc. vice versa." Such notice of levy only shows that Ferrer,
the vendee at auction of said certificates, merely stepped into the shoes of the judgment
debtor. Of the same principle is the provision of Article 1544 of the Civil Code, that "If the
same thing should have been sold to different vendees, the ownership shall be transferred to

SALES CASE DIGEST COMPILATION 279


the person who may have first taken possession thereof in good faith, if it should be movable
property."
There is no merit in Pantranco and Ferrer's theory that the sale of the certificates of public
convenience in question, between the Corporation and Fernando, was not consummated, it
being only a conditional sale subject to the suspensive condition of its approval by the Public
Service Commission. While section 20(g) of the Public Service Act provides that "subject to
established limitation and exceptions and saving provisions to the contrary, it shall be
unlawful for any public service or for the owner, lessee or operator thereof, without the
approval and authorization of the Commission previously had ... to sell, alienate, mortgage,
encumber or lease its property, franchise, certificates, privileges, or rights or any part
thereof, ...," the same section also provides:
... Provided, however, That nothing herein contained shall be construed to prevent
the transaction from being negotiated or completed before its approval or to prevent
the sale, alienation, or lease by any public service of any of its property in the
ordinary course of its business.
It is clear, therefore, that the requisite approval of the PSC is not a condition precedent for
the validity and consummation of the sale.

32.) RIVERA V. ONG CHE ,37 Phil. 355

FACTS:
The Lichauco Brother had offered for sale a certain old machinery and boilers which were
deposited and exposed for sale in a yard at Tanduay, in the city of Manila. The plaintiff,
Marciano Rivera, alleges that upon January 8, 1912, he purchased some of this old material
for the price of P5.500, and received a receipt from Cresanto Lichauco showing that he had
become such purchaser. These things consisted, according to said receipt, of two complete
steam-boilers, with chimneys; one steam motor (15 by 30 inches) complete; one pair of twin
rice hullers complete, and a feeding pump (donkey) for boilers.

The plaintiff, however, did not take possession of the property, which remained in the same
place. It further appears that upon February 9, 1912, the defendants, Ong Che, bought from
Lichauco Brothers a lot of old iron, machinery, and junk for the sum of P1,100. This
purchaser took immediate possession of the materials purchased by him. Later, when
Marciano Rivera appeared to take possession of the things of which he supposed himself to
the purchaser, under the receipt given by Crisanto Lichauco, he found that many of the
accessory and auxiliary parts of the boilers, motor, and rice mill were wanting; and upon
investigation it developed that these articles were held by the defendant, Ong Che, and were
claimed by him as owner by virtue of the purchase effected by him upon February 9, as
stated above.

ISSUE: Whether or Not the plaintiff is the true owner thereof.

HELD: No. The plaintiff is not the true owner.

We concur in the conclusion reached by the judge of the Court of First Instance that the
defendant, Ong Che, was a purchaser of these articles in good faith. It is furthermore
uncontroverted that he acquired possession by virtue of his purchase. He, therefore,
undoubtedly has, under article 1473 of the Civil Code, a better title than the first purchaser,
who has never had possession at all. The defendant had, in his favor, the fact that he was
purchaser in good faith and had acquired lawful possession. There is a presumption arising
from such possession that he was the owner (sec. 334 [10], Code of Civel Procedure); and

SALES CASE DIGEST COMPILATION 280


the mere fact, if such it be, that the property originally belonged to Galo Lichauco was not
sufficient, without more, to defeat a title acquired by the defendant through the house of
Lichauco.

The only doubt as to the application of that article to the present case arises from the fact
that there is some conflict in the testimony upon the question as to who was the original
owner. It is to be inferred from the testimony that the house of Lichauco consists of Faustino
Lichauco and Galo Lichauco, and it would seem that Crisanto Lichauco, who effected the
sale of Rivera, is not a member of that establishment. Crisanto testified that the property
sold by him to the plaintiff Rivera, including the articles which are now in dispute, was the
property of Galo Lichauco. There is grave doubt as to correctness of this statement,
however, as the same witness admits that the machinery sold by him to Rivera had been
taken out of an old mill owned by Lichauco Brothers in Dagupan; and it is not made clear
that Galo Lichauco had ever become its exclusive owner. Furthermore, the evidence
submitted by the defendant tends to show that the things acquitted by him, including the
articles in dispute, were bought from Faustino Lichauco as property of the house. At any rate
we find that, under the circumstances disclosed in this case, and even conceding that
property belong to Galo Lichauco, the house of Lichauco had authority to sell it. In this view
the case presented is that where two different agents of the same owner successively
negotiated sales to two different purchasers, and it is obvious that, under the article of the
Civil Code cited above, the second purchaser having acquired possession first must be
declared the true owner. In our view of the facts it was merely a case where a mistake was
made by the house of Lichauco in selling something that had already been sold.

SALES CASE DIGEST COMPILATION 281


XXI. SALE BY NON-OWNER OR BY ONE HAVING A
VOIDABLE TITLE
1.)TAGATAC VS JIMENEZ

FACTS: Trinidad Tagatac bought a car for $4,500 in the US. After 7 months, she brought
the car to the Philippines. Warner Feist, who pretended to be a wealthy man, offered to buy
Trinidad’s car for P15,000, and Tagatac was amenable to the idea. Hnece, a deed of sale
was exceuted.
Feist paid by means of a postdated check, and the car was delivered to Feist. However,
PNB refused to honor the checks and told her that Feist had no account in said bank.
Tagatac notified the law enforcement agencies of the estafa committed by Feist, but the
latter was not apprehended and the car disappeared.
Meanwhile, Feist managed succeeded in having the car’s registration certificate (RC)
transferred in his name. He sold the car to Sanchez, who was able to transfer the
registration certificate to his name.
Sanchez then offered to sell the car to defendant Liberato Jimenez, who bought the car for
P10,000 after investigating in the Motor Vehicles Office.
Tagatac discovered that the car was in California Car Exchange’s (place where Jimenez
displayed the car for sale), so she demanded from the manager for the delivery of the car,
but the latter refused.
Tagatac filed a suit for the recovery of the car’s possession, and the sheriff, pursuant to a
warrant of seizure that Tagatac obtained, seized and impounded the car, but it was delivered
back to Jimenez upon his filing of a counter-bond.
The lower court held that Jimenez had the right of ownership and possession over the car.

ISSUE: Whether or Not Jimenez was a purchaser in good faith and thus entitled to the
ownership and possession of the car.

HELD: YES.It must be noted that Tagatac was not unlawfully deprived of his car .
In this case, there is a valid transmission of ownership from true owner [Tagatac] to the
swindler [Feist], considering that they had a contract of sale (note: but such sale is voidable
for the fraud and deceit by Feist).
The disputable presumption that a person found in possession of a thing taken in the doing
of a recent wrongful act is the taker and the doer of the whole act does NOT apply in this
case because the car was not stolen from Tagatac, and Jimenez came into possession of
the car two months after Feist swindled Tagatac.
Jimenez was a purchaser in good faith for he was not aware of any flaw invalidating the title
from the seller of the car
In addition, when Jimenez acquired the car, he had no knowledge of any flaw in the title of
the person from whom he acquired it. It was only later that he became fully aware that there
were some questions regarding the car, when he filed a petition to dissolve Tagatac’s search
warrant which had as its subject the car in question.
The contract between Feist and Tagactac was a voidable contract, it can be annulled or
ratified
. . . The fraud and deceit practiced by Warner L. Feist earmarks this sale as a voidable
contract (Article 1390). Being a voidable contract, it is susceptible of either ratification or
annulment. (If the contract is ratified, the action to annul it is extinguished -Article 1392) and
the contract is cleansed from all its defects (Article 1396); if the contract is annulled, the
contracting parties are restored to their respective situations before the contract and mutual
restitution follows as a consequence (Article 1398).
Being a voidable contract, it remains valid and binding until annulled.

SALES CASE DIGEST COMPILATION 282


However, as long as no action is taken by the party entitled, either that of annulment or of
ratification, the contract of sale remains valid and binding. When plaintiff-appellant Trinidad
C. Tagatac delivered the car to Feist by virtue of said voidable contract of sale, the title to
the car passed to Feist. Of course, the title that Feist acquired was defective and voidable.
Nevertheless, at the time he sold the car to Felix Sanchez, his title thereto had not been
avoided and he therefore conferred a good title on the latter, provided he bought the car in
good faith, for value and without notice of the defect in Feist's title (Article 1506, N.C.C.).
There being no proof on record that Felix Sanchez acted in bad faith, it is safe to assume
that he acted in good faith.
NB: ART. 1506. Where the seller of goods has a voidable title thereto, but his title has not
been avoided at the time of the sale, the buyer acquires a good title to the goods provided
he buys them in good faith, for value, and without notice of the seller’s defect of title.

2.) EDCA PUBLISHING VS SPS. SANTOS


G.R. No. 80298, April 26, 1990

FACTS: On October 5, 1981, a person identifying himself as Prof. Jose Cruz ordered 406
books from EDCA Publishing. EDCA Subsequently prepared the corresponding invoice and
delivered the books as ordered, for which Cruz issued a personal check covering the
purchase price of said books. Subsequently on October 7, 1981, Cruz sold 120 of the books
to Leonor Santos who, after verifying the seller’s ownership from the invoice he showed her,
paid him P1,700.
Upon verification by EDCA, it was discovered that Cruz was not employed as professor by
De La Salle College and that he had no more account or deposit with Phil. Amanah Bank,
the bank where he allegedly drawn the payment check. Upon arrest of Cruz by the police, it
was revealed that his real name was Tomas dela Pena and that there was a further sale of
120 books to Sps. Santos.
EDCA, through the assistance of the police forced their way into the store of Sps. Santos
and threatened Leonor with prosecution for buying stolen property. The 120 books were
seized and were later turned over to EDCA.
This resulted to Sps. Santos filing a case for recovery of the books after their demand for the
return of the books was rejected.

ISSUE: Whether or not EDCA was unlawfully deprived of the property?


HELD: NO. Santos was a good faith buyer after taking steps to verify the identity of the
seller. When she was showed the invoice, she reasonably believed that he was a legitimate
seller. With regard to unlawful deprivation, EDCA was not unlawfully deprived of the property
by mere failure of consideration. There was already a perfected contract of sale. Proof was
even substantiated when EDCA gave the invoice as proof of payment upon delivery of the
books. This did not amount to unlawful taking, because by the delivery of EDCA to Cruz,
ownership of the books already transferred to him.
It would certainly be unfair now to make the SANTOSES bear the prejudice sustained by
EDCA as a result of its own negligence. We cannot see the justice in transferring EDCA's
loss to the SANTOSES who had acted in good faith, and with proper care, when they bought
the books from Cruz.

3.) AZNAR VS. YAPDIANGCO


13 SCRA 486
G.R. No. L-18536
March 31, 1965

SALES CASE DIGEST COMPILATION 283


FACTS: Teodoro Santos was selling his FORD FAIRLANE 500. One day, a certain L. De
Dios, claiming to be a nephew of Vicente Marella, said that his uncle Vicente wants to buy
the car.
Marella agreed to buy the car for P14,700.00 on the understanding that the price would be
paid only after the car had been registered in his name.
The Deed of Sale for the car was executed in Marella's favor. Afterwhich, the car in was
registered Marella's name. Up to this stage of the transaction, the purchased price had not
been paid.

ISSUE: Between Santos and Aznar, who has a better right to the possession of the disputed
automobile?

HELD: SANTOS has a better rights. Aznar accepts that the car in question originally
belonged to and was owned by Santos, and that the latter was unlawfully deprived of the
same by Marella. However, Aznar contends that upon the facts of this case, the applicable
provision of the CC is Article 1506 and not Article 559 as was held by the decision under
review. Article 1506 provides:
ART. 1506. Where the seller of goods has a voidable title thereto, but his, title has not been
voided at the time of the sale, the buyer acquires a good title to the goods, provided he buys
them in good faith, for value, and without notice of the seller's defect of title.
The contention is clearly unmeritorious. Under the aforequoted provision, it is essential
that the seller should have a voidable title at least. It is very clearly inapplicable where,
as in this case, the seller had no title at all.
Ownership is not transferred by contract merely but by tradition or delivery. Contracts only
constitute titles or rights to the transfer or acquisition of ownership, while delivery or
tradition is the mode of accomplishing the same. For the legal acquisition and transfer of
ownership and other property rights, the thing transferred must be delivered, inasmuch as,
according to settled jurisprudence, the tradition of the thing is a necessary and indispensable
requisite in the acquisition of said ownership by virtue of contract. So long as property is not
delivered, the ownership over it is not transferred by contract merely but by delivery.
Contracts only constitute titles or rights to the transfer or acquisition of ownership, while
delivery or tradition is the method of accomplishing the same, the title and the method of
acquiring it being different in our law.
Vicente Marella did not have any title to the property under litigation because the same was
never delivered to him. He sought ownership or acquisition of it by virtue of the contract.
Vicente Marella could have acquired ownership or title to the subject matter thereof only by
the delivery or tradition of the car to him.
The lower court was correct in applying Article 559 of the CC to the case at bar, for under it,
the rule is to the effect that if the owner has lost a thing, or if he has been unlawfully
deprived of it, he has a right to recover it, not only from the finder, thief or robber, but
also from third persons who may have acquired it in good faith from such finder, thief
or robber. The said article establishes two exceptions to the general rule of
IRREVINDICABILITY: when the owner (1) has lost the thing, or (2) has been unlawfully
deprived thereof. In these cases, the possessor cannot retain the thing as against the owner,
who may recover it without paying any indemnity, except when the possessor acquired it in a
public sale.

SALES CASE DIGEST COMPILATION 284


XXII. LOSS, DETERIORATION , FRUITS AND OTHER
BENEFITS
1.)CHRYSLER VS. CA 133 SCRA 567
G.R. No. 55684 ,December 19, 1984

FACTS: Petitioner Chrysler is a domestic corporation engaged in the assembling and sale of
motor vehicles and other automotive products. Respondent Sambok is a general partnership
and was a dealer for automotive products.
Chrysler filed with CFI a complaint for damages against Allied Brokerage Corp, Negros
Navigation Co, and Sambok, alleging that:
1. On Oct 2, 1970, Sambok Bacolod ordered from petitioner various automotive products
worth P30,909.61 payable in 45 days,
2. That on Nov 25, 1970, Chrysler delivered said products to its forwarding agent Allied
Brokerage for delivery to Sambok (Allied loaded the goods through vessel of Negros Nav),
3. That when Chrysler tried to collect from Sambok Bacolod the amount of P31,037.56 (price
of spare parts plus handling charges), Sambok refused to pay, claiming that it had not
received the merchandise,
4. That Chrysler also demanded the return of the merchandise or their value from Allied and
Negros Navigation, but both denied any liability.

Sambok Bacolod denied having received the automotive products and professed no
knowledge of having ordered from petitioner the said articles.
Trial Court dismissed the complaint against Allied and Negros Navigation, but found Sambok
liable for damages “in refusing to take delivery of the shipment for no justifiable reason. The
decision was reversed by CA after finding that Chrysler had not performed its part of the
obligation under the contract by not delivering the goods at Sambok, Iloilo, the place
designated in the Parts Order Form. In other words, CA found that there was misdelivery.

ISSUE: Whether or NOT Sambok Bacolod should be liable for damages.

HELD: No. To our minds, the matter of misdelivery is not the decisive factor for relieving
Sambok, Bacolod, of liability herein. While it may be that the Parts Order Form specifically
indicated Iloilo as the destination, as testified to by Ernesto Ordonez, Parts Sales
Representative of petitioner, Sambok, Bacolod, and Sambok, Iloilo, are actually one. In fact,
admittedly, the order for spare

SALES CASE DIGEST COMPILATION 285


parts was made by the President of Sambok, Pepito Ng, through its marketing consultant.
Notwithstanding, upon receipt of the Bill of Lading, Sambok, Bacolod, initiated, but did not
pursue, steps to take delivery as they were advised by Negros Navigation that because
some parts were missing. They would just be informed as soon as the missing parts were
located.
It was only four years later however, or in 1974, when a warehouseman of Negros
Navigation, Severino Aguarte, found in their off-shore bodega, parts of the shipment in
question, but already deteriorated and valueless.
Under the circumstances, Sambok, Bacolod, cannot be faulted for not accepting or refusing
to accept the shipment from Negros Navigation four years after shipment. The evidence is
clear that Negros Navigation could not produce the merchandise nor ascertain its
whereabouts at the time Sambok, Bacolod, was ready to take delivery. Where the seller
delivers to the buyer a quantity of goods less than he contracted to sell, the buyer may reject
them.
From the evidentiary record, Negros Navigation was the party negligent in failing to deliver
the complete shipment either to Sambok, Bacolod, or to Sambok, Iloilo, but as the Trial
Court found, petitioner failed to comply with the conditions precedent to the filing of a judicial
action. Thus, in the last analysis, it is petitioner that must shoulder the resulting loss. The
general rule that before, delivery, the risk of loss is home by the seller who is still the owner,
under the principle of "res perit domino", is applicable in petitioner's case.
In sum, the judgment of respondent Appellate Court, will have to be sustained not on the
basis of misdelivery but on non-delivery since the merchandise was never placed in the
control and possession of Sambok, Bacolod, the vendee. Decision of CA affirmed.

2.)LAWYER’S COOP VS TABORA

FACTS: Perfecto Tabora bought from the Lawyers Cooperative Publishing Company one
complete set of American Jurisprudence consisting of 48 volumes with 1954 pocket parts,
plus one set of American Jurisprudence, General Index, consisting of 4 volumes, for a total
price of P1,675.50 which, in addition to the cost of freight of P6.90, makes a total of
P1,682.40.
Tabora made a partial payment of P300.00, leaving a balance of P1,382.40. The books were
delivered and receipted by Tabora in his law office.
In the midnight of the same date, however, a big fire broke out in that locality which
destroyed and burned all the buildings standing on one whole block including at the law
office and library of Tabora As a result, the books bought from the company as above
stated, together with Tabora's important documents and papers, were burned during the
conflagration.
This unfortunate event was immediately reported by Tabora to the company by sending a
letter. The company replied and as a token of goodwill it sent to Tabora free of charge
volumes 75, 76, 77 and 78 of the Philippine Reports.
As Tabora failed to pay monthly installments, the company commenced the present action
for the recovery of the balance of the obligation.
It was provided in the contract that "title to and ownership of the books shall remain with the
seller until the purchase price shall have been fully paid. Loss or damage to the books after
delivery to the buyer shall be borne by the buyer." The total price of the books, including the
cost of freight, amounts to P1,682.40. Appellant only made a down payment of P300.00
thereby leaving a balance of P1,382.40.
Defendant, in his answer, pleaded force majeure as a defense, saying that since the loss
was due to force majeure he cannot be held responsible for the loss.

ISSUE: Whether or Not Tabora is absolved from liability on the ground of force majeure

RULING: No. He is liable for the loss. It is true that in the contract entered into between the
parties the seller agreed that the ownership of the books shall remain with it until the

SALES CASE DIGEST COMPILATION 286


purchase price shall have been fully paid, but such stipulation cannot make the seller liable
in case of loss not only because such was agreed merely to secure the performance by the
buyer of his obligation but in the very contract it was expressly agreed that the "loss or
damage to the books after delivery to the buyer shall be borne by the buyer."
Article 1504 of our Civil Code, which in part provides:
(1) Where delivery of the goods has been made to the buyer or to a bailee for the buyer, in
pursuance of the contract and the ownership in the goods has been retained by the seller
merely to secure performance by the buyer of his obligations under the contract, the goods
are at the buyer's risk from the time of such delivery.
The rule that an obligor should be held exempt from liability when the loss occurs thru a
fortuitous event should not apply because it only holds true when the obligation consists in
the delivery of a determinate thing and there is no stipulation holding him liable even in case
of fortuitous event. Here these qualifications are not present. The obligation does not refer to
a determinate thing, but is pecuniary in nature, and the obligor bound himself to assume the
loss after the delivery of the goods to him. In other words, the obligor agreed to assume any
risk concerning the goods from the time of their delivery, which is an exception to the rule
provided for in Article 1262 of our Civil Code.

XXIII. REMEDIES IN CASE OF BREACH


1) LEVY VS. GERVACIO

FACTS: Levy Hermanos, Inc., sold to defendant Lazaro Blas Gervacio, a Packard car.
Defendant, after making the initial payment, executed a promissory note for the balance of
P2,400, payable on or before June 15, 1937, with interest at 12 % per annum, to secure the
payment of the note, he mortgaged the car to the plaintiff.
Defendant failed to pay the note it its maturity. Thus, Levy foreclosed the mortgage and the
car was sold at public auction, at which plaintiff was the highest bidder for P1,800. The
present action is for the collection of the balance of P1,600 and interest.

ISSUE: WON plaintiff still may collect the balance and interest after it has already foreclosed
the mortgage and sold it at public auction

RULING: Yes it can still collect the balance.


Article 1454-A of the Civil Code reads as follows:
In a contract for the sale of personal property payable in installments shall confer upon the
vendor the right to cancel the sale or foreclose the mortgage if one has been given on the
property, without reimbursement to the purchaser of the installments already paid, if there be
an agreement to this effect.
However, if the vendor has chosen to foreclose the mortgage he shall have no further action
against the purchaser for the recovery of any unpaid balance owing by the same and any
agreement to the contrary shall be null and void.
In Macondray and Co. vs. De Santos, the Court held that "in order to apply the provisions of
article 1454-A of the Civil Code it must appear that there was a contract for the sale of
personal property payable in installments and that there has been a failure to pay two or
more installments." The contract, in the instant case, while a sale of personal property, is
not, however, one on installments, but on straight term, in which the balance, after payment
of the initial sum, should be paid in its totality at the time specified in the promissory note.
The transaction is not, therefore, the one contemplated in Act No. 4122 and accordingly the
mortgagee is not bound by the prohibition therein contained as to the right to the recovery of
the unpaid balance.
___________________________________________________

SALES CASE DIGEST COMPILATION 287


CARYL
2) DELTA MOTOR SALES CORPORATION, plaintiff-appellee,
vs
NIU KIM DUAN and CHAN FUE ENG, defendants-appellants.

Facts:

On 5 July 1975, Niu Kim Duan and Chan Fue Eng (defendants) purchased from
Delta Motor Sales Corporation 3 units of ‘DAIKIN’ air-conditioner all valued at P19,350.00.
The deed of sale stipulates that the defendants shall pay a down payment of P774.00 and
the balance of P18,576.00 shall be paid by them in 24 installments; that the title to the
properties purchased shall remain with Delta Motors until the purchase price thereof is fully
paid; that if any two installments are not paid by the defendants on their due dates, the
whole of the principal sum remaining unpaid shall become due, with interest at the rate of
14% per annum: and in case of a suit, the defendants shall pay an amount equivalent to
25% of the remaining unpaid obligation as damages, penalty and attorney’s fees; that to
secure the payment of the balance of P18,576.00 the defendants jointly and severally
executed in favor of the Delta Motors a promissory note.

The 3 air-conditioners were delivered to and received by the defendants. After paying
the amount of P6,966.00, the defendants failed to pay at least 2 monthly installments; that
as of 6 January 1977, the remaining unpaid obligation of the defendants amounted to
P12,920.08. Statements of accounts were sent to the defendants and the Delta Motors’
collectors personally went to the former to effect collections but they failed to do so. Because
of the unjustified refusal of the defendants to pay their outstanding account and their
wrongful detention of the properties in question, Delta Motors tried to recover the said
properties extra-judicially but it failed to do so. The matter was later referred by Delta Motors
to its legal counsel for legal action.

In its verified complaint dated 28 January 1977, Delta Motors prayed for the issuance
of a writ of replevin, which the Court granted in its Order dated 28 February 1977, after Delta
Motors posted the requisite bond. On 11 April 1977, Delta Motors, by virtue of the writ,
succeeded in retrieving the properties in question. As of 3 October 1977, the outstanding
account of the defendants is only in the amount of P6,188.29 as shown by the computation,
after deducting the interests in arrears, cover charges, replevin bond premiums, the value of
the units repossessed and the like. In view of the failure of the defendants to pay their
obligations, the amount of P6,966.00 which had been paid by way of installments were
treated as rentals for the units in question for 2 years pursuant to the provisions of
paragraph 5 of the Deed of Conditional Sale.

The trial court promulgated its decision on 11 October 1977 ordering the defendants
to pay Delta Motors the amount of P6,188.29 with a 14% per annum interest which was due
on the 3 “Daikin” air-conditioners the defendants purchased from Delta Motors under a Deed
of Conditional Sale, after the same was declared rescinded by the trial court. They were
likewise ordered to pay Delta Motors P1,000.00 for and as attorney’s fees.

Niu Kim Duan and Chan Fue Eng appealed. The case was elevated to the Supreme
Court by the Court of Appeals, in its Resolution of 20 May 1982, on a pure question of law.

The Supreme Court set aside the judgment of the trial court in Civil Case 25578 and
dismissed the complaint filed by Delta Motor Sales Corporation; without costs.

SALES CASE DIGEST COMPILATION 288


1. Treatment of installment payments as rentals not unconscionable (even if it
approximates 1/3 of cost of the 3 airconditioners)

Defendants cannot complain that their downpayment of P774.00 and installment payments
of P5,655.92 were treated as rentals, even though the total amount of P6,429,92 which they
had paid, approximates one-third (1/3) of the cost of the 3 air-conditioners. A stipulation in a
contract that the installments paid shall not be returned to the vendee is valid insofar as the
same may not be unconscionable under the circumstances is sanctioned by Article 1486 of
the New Civil Code. The monthly installment payable by defendants was P774.00. The
P5,655.92 installment payments correspond only to 7 monthly installments. Since they admit
having used the air-conditioners for 22 months, this means that they did not pay 15 monthly
installments on the said air-conditioners and were thus using the same FREE for said
period, to the prejudice of Delta Motors. Under the circumstances, the treatment of the
installment payments as rentals cannot be said to be unconscionable.

2. Remedies available to vendor in a sale of personal property payable in installments

The vendor in a sale of personal property payable in installments may exercise one of three
remedies, namely, (1) exact the fulfillment of the obligation, should the vendee fail to pay; (2)
cancel the sale upon the vendee’s failure to pay two or more installments; (3) foreclose the
chattel mortgage, if one has been constituted on the property sold, upon the vendee’s failure
to pay two or more installments. The third option or remedy, however, is subject to the
limitation that the vendor cannot recover any unpaid balance of the price and any agreement
to the contrary is void (Art. 1484).

3. Remedies alternative, not cumulative

The 3 remedies are alternative and NOT cumulative. If the creditor chooses one remedy, he
cannot avail himself of the other two.

4. Air-conditioning units repossessed, bars action to exact payment for balance of the
price

Delta Motors had taken possession of the 3 air-conditioners, through a writ of


replevin when defendants refused to extra-judicially surrender the same. The case Delta
Motors filed was to seek a judicial declaration that it had validly rescinded the Deed of
Conditional Sale. Delta Motors thus chose the second remedy of Article 1484 in seeking
enforcement of its contract with defendants. Having done so, it is barred from exacting
payment from defendants of the balance of the price of the three air-conditioning units which
it had already repossessed. It cannot have its cake and eat it too.

XXIV. SALE OF MOVABLES ON INSTALLMENTS

1) G.R. No. L-10789 May 28, 1957


AMADOR TAJANLANGIT, ET AL., plaintiff-appellants,
vs.
SOUTHERN MOTORS, INC., ET AL., defendants-appellees.

FACTS:

SALES CASE DIGEST COMPILATION 289


Amador Tajanlangit and his wife Angeles, bought from the Southern Motors Inc. of
Iloilo two tractors and a thresher. In payment for the same, they executed the promissory
note whereby they undertook to satisfy the total purchase price of P24,755.75 in several
instalments (with interest) payable on stated dates from May 18, 1953 December 10, 1955.
The note stipulated that if default be made in the payment of interest or of any installment,
then the total principal sum still unpaid with interest shall at once become demandable etc.

The spouse failed to meet any installment. Wherefore, they were sued for the
amount of the promissory note. The spouses defaulted, and the court, after listening to the
Southern
Motors' evidence entered Judgment for it in the total sum of P24,755.75 together with
interest. Carrying out the order of execution, the sheriff levied on the same machineries and
farm implements which had been bought by the spouses; and later sold them at public
auction to the highest bidder — which turned out to be the Southern Motors itself.

Petitioner spouses objected to the sale contending that (1) They had returned the
machineries and farm implements to the Southern Motors Inc., the latter accepted them, and
had thereby settled their accounts and (2) as the Southern Motors Inc. had repossessed the
machines purchased on installment (and mortgaged) the buyers were thereby relieved from
further responsibility, in view of the Recto Law, now article 1484 of the New Civil Code.

ISSUE: Whether or not the vendor of a movable on instalment is limited only to go after the
proceeds of the sale

RULING: NO

ART. 1484. In a contract of sale of personal property the price of which is payable in
installments, the vendor
may exercise of the following remedies:

(1) Exact fulfillment of the obligation, should the vendee fail to pay;
(2) Cancel the sale, should the vendee's failure to pay cover two or more
installments;
(3) Foreclose the chattel mortgage on the thing sold, if one has been constituted,
should the vendee's failure to pay cover two or more installments. In this case, he
shall have no further action against the purchaser to recover any unpaid balance of
the price. Any agreement to the contrary shall be void. (New Civil Code.)

Appellants would invoke the last paragraph. But there has been no foreclosure of the
chattel mortgage nor a foreclosure sale. Therefore the prohibition against further collection
does not apply. It is true that there was a chattel mortgage on the goods sold. But the
Southern Motors elected to sue on the note exclusively, i.e. to exact fulfillment of the
obligation to pay. It had a right to select among the three remedies established in Article
1484. In choosing the option to exact fulfilment of the obligation, it was not thereby limited to
the proceeds of the sale, on execution, of the mortgaged good.

SALES CASE DIGEST COMPILATION 290


2) [G.R. No. 109966. May 31, 1999]
ELISCO TOOL MANUFACTURING CORPORATION, petitioner, vs. COURT OF
APPEALS, ROLANDO LANTAN, and RINA LANTAN, respondents.

FACTS:
Private respondent Rolando Lantan was employed at the Elisco Tool Mfg. Corp. On
Jan. 1980, he entered into a car plan with the company, which constitutes a lease with
option to buy for a period of 5 years. The agreement provides that Lantan shall pay a
monthly rental of P 1010.65 to be deducted from his salary or a total of P60, 639.00 at the
end of 5 years. The agreement provides that at the 60th month of payment he may exercise
his option to buy and all monthly rentals shall be applied to the payment of the full purchase
price of the car.

In 1981 Elisco Tool ceased operations, and Rolando Lantan was laid off.
Nonetheless, as of December 4, 1984, private respondent was able to make payments for
the car in the total amount of P61, 070.94.

On 1986 Elisco filed a complaint for replevin plus sum of money against Rolando
Lantan for the latter‘s alleged failure to pay the monthly rentals as of May 1986. Elisco
prayed for the following:

1. The payment of Lantan of the sum of the monthly rentals due as of May 1986 plus
legal interest;
2. The issuance of writ of replevin to gain possession of the car; and
3. On the alternative, should the delivery of the car not be possible, that Lantan be
ordered to pay the actual value of the car in the amount of 60,000 plus the accrued
monthly rentals thereof with interest until fully paid.
Both the trial court and the CA decided in favor of Lantan, declaring the latter the
lawful owner of the car and sentencing Elisco to pay for actual damages caused to
the private respondents, thus this petition.

ISSUE:
1) WON the lease with option to buy is in reality an installment sale so as to apply the Recto
Law under Art. 1484.

2)WON Elisco is entitled to any of the remedies under Art. 1484.

HELD

1. Yes.

The agreement between Elisco and the Lantans is in reality an installment sale of
personal property. However, the remedies under Article 1484 are alternative, not cumulative.

2. No.

There was already full payment. In the case at bar, although the agreement provides
for the payment of monthly rentals, it also provides the option to purchase upon the payment
of the 60th monthly rental and that all monthly rentals shall be applied to the payment of the
full purchase price of the car. Clearly the transaction is a lease in name only and so Articles
1484 and 1485 apply.

It is noteworthy that the remedies provided for in Art. 1484 are alternative, not

SALES CASE DIGEST COMPILATION 291


cumulative. The exercise of one bars the exercise of the others. It was held that in choosing
to deprive the defendant of possession of the leased vehicles, the plaintiff waived its right to
bring an action to recover unpaid rentals on the said vehicles.
Furthermore, both the trial court and the CA correctly ruled that Elisco is not entitled to any
of the remedies under Art. 1484 as there has already been full payment.

The agreement does not provide for the payment of interest on unpaid monthly
"rentals" or installments. The 2% surcharge is not provided for in the agreement.
Consequently, the total amount of P 61, 070.94 already paid is more than sufficient to cover
the full purchase price of the car which only amounts to
P 60, 639.

XXV. SALE OF REAL ESTATE ON INSTALLMENT

1) [G.R. No. 130347. March 3, 1999]


ABELARDO VALARAO, GLORIOSA VALARAO and CARLOS VALARAO, petitioners,
vs.
COURT OF APPEALS and MEDEN A. ARELLANO, respondents.

Facts:

On September 4, 1987, spouses Abelardo and Gloriosa Valarao, thru their son Carlos
Valarao as their attorney-in-fact, sold to [Private Respondent] Meden Arellano under a Deed
of Conditional Sale a parcel of land with an area of 1,504 square meters, for the sum of
P3,225,000.00 payable under a schedule of payment stated therein.

Stipulations:

 The [private respondent] vendee obligated herself to encumber by way of real estate
mortgage in favor of [petitioners] vendors her separate piece of property with the
condition that upon full payment of the balance of P2,225.000.00, the said mortgage
shall become null and void and without further force and effect.

 Should the vendee fail to pay three (3) successive monthly installments or anyone
year-end lump sum payment within the period stipulated, the sale shall be
considered automatically rescinded without the necessity of judicial action and all
payments made by the vendee shall be forfeited in favor of the vendors by way of
rental for the use and occupancy of the property and as liquidated damages. All
improvements introduced by the vendee to the property shall belong to the vendors
without any right of reimbursement.

[Private respondent] appellant alleged that as of September, 1990, she had already paid
the amount of P2,028,000.00, although she admitted having failed to pay the installments
due in October and November, 1990. Petitioner, however, [had] tried to pay the installments
due [in] the said months, including the amount due [in] the month of December, 1990 on
December 30 and 31, 1990, but was turned down by the vendors-[petitioners] thru their
maid, Mary Gonzales, who refused to accept the payment offered. [Private respondent]
maintains that on previous occasions, the same maid was the one who [had] received

SALES CASE DIGEST COMPILATION 292


payments tendered by her. It appears that Mary Gonzales refused to receive payment
allegedly on orders of her employers who were not at home.

[Private respondent] tried to get in touch with [petitioners] over the phone and was able
to talk with [Petitioner] Gloriosa Valarao who told her that she [would] no longer accept the
payments being offered and that [private respondent] should instead confer with her lawyer,
a certain Atty. Tuazon.

When all her efforts to make payment were unsuccessful, [private respondent] sought
judicial action by filing this petition for consignation on January 4, 1991.

On the other hand, vendors-[petitioners], thru counsel, sent [private respondent] a letter
dated 4 January 1991 (Exh. "C") notifying her that they were enforcing the provision on
automatic rescission as a consequence of which the Deed of Conditional Sale [was deemed]
null and void, and . . . all payments made, as well as the improvements introduced on the
property, [were] thereby forfeited. The letter also made a formal demand on the [private
respondent] to vacate the property should she not heed the demand of [petitioners] to sign a
contract of lease for her continued stay in the property.

In reply, [private respondent] sent a letter dated January 14, 1991 (Exh. "D"), denying that
she [had] refused to pay the installments due [in] the months of October, November and
December, and countered that it was [petitioners] who refused to accept payment, thus
constraining her to file a petition for consignation before the Regional Trial Court of Quezon
City.

Petitioners, through counsel, sent the private respondent another letter dated
January 19, 1991 denying the allegations of her attempts to tender payment on December
30 and 31, 1990, and demanding that [private respondent] vacate and turnover the property
and pay a monthly compensation for her continued occupation of the subject property at the
rate of P20,000.00, until she shall have vacated the same.

Issues:

1.) Whether the Answer [— (a)] categorically indicating willingness to accept the amount
already due if the [private respondent] would update the account, [(b)] praying that "if she
fail[ed] to do so immediately, . . . the Deed of Conditional Sale be declared rescinded,
pursuant to the second paragraph of Section 3 thereof, with costs against the [private
respondent], [(c)] ordering the latter to vacate and turnover possession of the premises to
the [petitioners], and to pay the latter attorney's fees in the amount of P50,000.00 and the
expenses of litigation" [—] is tantamount to a judicial demand and notice of rescission under
Art. 1592 of the Civil Code.

2.) Whether the action for consignation may prosper without actual deposit [in court] of the
amount due . . . [so as] to produce the effect of payment.

Ruling:

1.) We believe, however, that the issue of whether the requirement of a judicial demand or a
notarial act has been fulfilled is immaterial to the resolution of the present case. Article 1592
of the Civil Code. states:

Art. 1592. In the sale of immovable property, even though it may have been stipulated that
upon failure to pay the price at the time agreed upon the rescission of the contract shall of
right take place, the vendee may pay, even after the expiration of the period, as long as no
demand for rescission of the contract has been made upon him either judicially or by notarial
act. After the demand, the court may not grant him a new term.

SALES CASE DIGEST COMPILATION 293


It is well-settled that the above-quoted provision applies only to a contract of sale, 8 and not
to a sale on installment or a contract to sell. In the present case, the Deed of Conditional
Sale is of the same nature as a sale on installment or a contract to sell, which is not covered
by Article 1592.

2.) True, there is no showing that she deposited the money with the proper judicial authority
which, taken together with the other requisites for a valid consignation, would have released
her from her obligation to pay. However, she does not deny her obligation and, in fact, is
willing to pay not only the three monthly installments due but also the entire residual amount
of the purchase price. Verily, she even filed a Motion to Deposit the said entire balance with
the trial court, which however denied said motion upon opposition of the petitioners.

It would be inequitable to allow the forfeiture of the amount of more than two million pesos
already paid by private respondent, a sum which constitutes two thirds of the total
consideration. Because she did make a tender of payment which was unjustifiably refused,
we hold that petitioners cannot enforce the automatic forfeiture clause of the contract.

Application of the Maceda Law

In any event, the rescission of the contract and the forfeiture of the payments already made
could not be effected, because the case falls squarely under Republic Act No. 6552, 22
otherwise known as the "Maceda Law." Section 3 of said law provides:

Sec. 3. In all transactions or contracts involving the sale or financing of real estate on
installment payments, including residential condominium apartments but excluding industrial
lots, commercial buildings and sales to tenants under Republic Act. Numbered Thirty-eight
hundred Forty-four as amended by Republic Act Numbered Sixty-three hundred eighty-nine,
where the buyer has paid at least two years of installments, the buyer is entitled to the
following rights in case he defaults in the payment of succeeding installments:

(a) To pay, without additional interest, the unpaid installments due within the
total grace period earned by him, which is hereby fixed at the rate of one month
grace period for every year of installment payments made: Provided, That this right
shall be exercised by the buyer only once in every five years of the life of the contract
and its extensions, if any.

(b) If the contract is cancelled, the seller shall refund to the buyer the cash
surrender value on the payments on the property equivalent to fifty percent of the
total payments made and, after five years of installments, an additional five percent
every year but not to exceed ninety percent of the total payments made: Provided,
That the actual cancellation of the contract shall take place after thirty days from
receipt by the buyer of the notice of cancellation or the demand for rescission of the
contract by a notarial act and upon full payment of the cash surrender value to the
buyer.

Down payments, deposits or options on the contract shall be included in the


computation of the total number of installments made.

Hence, the private respondent was entitled to a one-month grace period for every
year of installments paid, which means that she had a total grace period of three months
from December 31, 1990. Indeed, to rule in favor of petitioner would result in patent injustice
and unjust enrichment.

SALES CASE DIGEST COMPILATION 294


2) G.R. No. 147695 September 13, 2007
MANUEL C. PAGTALUNAN, petitioner,
vs.
RU FINA DELA CRUZ VDA. DE MANZANO, respondent.

FACTS:

Patricio Pagtalunan, petitioner’s stepfather, entered into a Contract to Sell with


private respondent Rufina Manzano over a house and lot for P17,000 to be paid in the
following manner: P1,500 as downpayment upon execution of the Contract and the balance
to be paid in equal monthly installments of P150 on or before the last day of each month
until fully paid.

The contract provides that while respondent could immediately occupy the house
and lot, in case of default in the payment of any of the installments for 90 days after its due
date, the contract would be automatically rescinded without need of judicial declaration;
all payments made and all improvements done on the premises by respondent would be
considered as rentals for the use and occupation of the property or payment for damages
suffered; and that respondent should peacefully vacate the premises and deliver the
possession thereof back to the vendor.

Petitioner Pagtalunan alleged that Manzano stopped paying after December 1979
without any justification or explanation and that the latter paid only P12,950. Pagtalunan
asserted that when respondent ceased paying her installments, her status of buyer was
automatically transformed to that of a lessee.

Pagtalunan issued a demand letter for Manzano to vacate the premises of the
property but Manzano ignored the same. Thus, Pagtalunan filed a Complaint for unlawful
detainer against respondent with the Municipal Trial Court (MTC) of Guiguinto, Bulacan.

The MTC found for Pagtalunan, holding that failure to pay not a few installments
caused the resolution or termination of the Contract to Sell. After her last payment of the
installment, Manzano’s right of possession ipso facto ceased to be a legal right, and
became possession by mere tolerance of Patricio and his successors-in-interest. Said
tolerance ceased upon demand on respondent to vacate the property.

On appeal, the RTC of Malolos, Bulacan reversed the decision of the MTC and
dismissed the case for lack of merit, ruling that the agreement could not be automatically
rescinded since there was delivery to the buyer. A judicial determination of rescission
must be secured by petitioner as a condition precedent to convert the possession de
facto of respondent from lawful to unlawful.

The Court of Appeals affirmed the RTC’s decision but held that the parties, as well
as the MTC and RTC, failed to advert to and to apply the Maceda Law (RA 6522). It
ruled that the Contract to Sell was NOT VALIDLY cancelled or rescinded under Sec. 3
(b) of the Maceda Law, and recognized respondent’s right to continue occupying
unmolested the property subject of the contract to sell.

ISSUE: WON the contract has been automatically rescinded pursuant to the agreement
when Manzano defaulted in the payment of her installments

RULING:

SALES CASE DIGEST COMPILATION 295


NO, the contract has not been automatically rescinded.

While the Court agrees with petitioner that the cancellation of the Contract to Sell
may be done outside the court particularly when the buyer agrees to such cancellation, the
cancellation of the contract by the seller must be in accordance with Sec. 3 (b) of the
Maceda Law, which requires: (1) a notarial act of rescission and (2) the refund to the
buyer of the full payment of the cash surrender value of the payments on the property.
Actual cancellation of the contract takes place after 30 days from receipt by the buyer
of the notice of cancellation or the demand for rescission of the contract by a notarial
act AND upon full payment of the cash surrender value to the buyer.

Here, the Contract to Sell was not validly cancelled or rescinded under Sec. 3
(b) of the Maceda Law. First, Patricio, the vendor in the Contract to Sell, died on September
17, 1992 without canceling the Contract to Sell. Second, petitioner also failed to cancel
the Contract to Sell in accordance with law.

Meanwhile, petitioner asserts that his demand letter should be considered as the
notice of cancellation or demand for rescission by notarial act and that the cash surrender
value of the payments on the property has been applied to rentals for the use of the house
and lot after respondent stopped payment after January 1980.

The Court, however, found that the letter merely made formal demand upon
respondent to vacate the premises in question. Clearly, the demand letter is not the
same as the notice of cancellation or demand for rescission by a notarial act required
by the Maceda Law.

Moreover, petitioner cannot insist on compliance with the requirement by assuming


that the cash surrender value payable to the buyer had been applied to rentals of the
property after respondent failed to pay the installments due. Sec. 3 (b) of the Maceda Law
does not provide a different requirement for contracts to sell which allow possession of
the property by the buyer upon execution of the contract like the instant case but the refund
of the cash surrender value of the payments on the property to the buyer before
cancellation of the contract.

There being no valid cancellation of the Contract to Sell, the Court held that CA
correctly recognized respondent’s right to continue occupying the property subject of the
Contract to Sell and affirmed the dismissal of the unlawful detainer case by the RTC.
Consequently, it is only right and just, under the Maceda Law, to allow respondent to
pay her arrears and settle the balance of the purchase price, subject to interests.

XXVI. CONDITIONS
1) G.R. No. 146839 March 23, 2011
ROLANDO T. CATUNGAL, JOSE T. CATUNGAL, JR., CAROLYN T. CATUNGAL and
ERLINDA CATUNGAL-WESSEL, Petitioners,
vs.
ANGEL S. RODRIGUEZ, Respondent.
Facts:

Agapita T. Catungal (Agapita) owned a parcel of land (Lot 10963) situated in the
Barrio of Talamban, Cebu City. Agapita, with the consent of her husband Jose, entered into
a Contract to Sell with respondent Rodriguez which subsequently purportedly “upgraded”
into a Conditional Deed of Sale. Both the Contract to Sell and the Conditional Deed of Sale
were annotated on the title.

SALES CASE DIGEST COMPILATION 296


The provisions of the Conditional Deed of Sale pertinent to the present dispute are
quoted below:

1. Price of 25 Million pesos


2. Down payment of 500 Thousand
3. Balance shall be paid after the VENDEE have successfully negotiated, secured
and provided a Road Right of Way . If however said Road Right of Way could not be
negotiated, the VENDEE shall give notice to the VENDOR for them to reassess and
solve the problem by taking other options and should the situation ultimately prove
futile, he shall take steps to rescind or cancel the herein Conditional Deed of Sale.

On August 31, 1990 the spouses Catungal requested an advance of P5,000,000.00


on the purchase price for personal reasons. Rodriquez allegedly refused on the ground that
the amount was substantial and was not due under the terms of their agreement. The
Catungal’s rescinded the contract.

Rodriguez filed a complaint against the Catungal’s for arbitrarily rescinding the contract. In a
Decision dated May 30, 1992, the trial court ruled in favor of Rodriguez, finding that: (a)
under the contract it was complainant (Rodriguez) that had the option to rescind the sale; (b)
Rodriguez’s obligation to pay the balance of the purchase price arises only upon successful
negotiation of the road right of way; (c) he proved his diligent efforts to negotiate the road
right of way; (d) the spouses Catungal were guilty of misrepresentation which defeated
Rodriguez’s efforts to acquire the road right of way; and (e) the Catungals’ rescission of the
contract had no basis and was in bad faith.
During the pendency of the case with the Court of Appeals, Agapita Catungal passed
away and thus, her husband, Jose, filed on February 17, 1999 a motion for Agapita’s
substitution by her surviving children.

The Catungals alleged that the conditional deed of sale was void ab initio because it
violates the mutuality of contract in view of Article 1308 of the New Civil Code. Petitioners
rely on Article 1308 of the Civil Code to support their conclusion regarding the claimed nullity
of the aforementioned provisions. Article 1308 states that “the contract must bind both
contracting parties; its validity or compliance cannot be left to the will of one of them.”

Article 1182 of the Civil Code, in turn, provides:

Art. 1182. When the fulfillment of the condition depends upon the sole will of the
debtor, the conditional obligation shall be void. If it depends upon chance or upon the
will of a third person, the obligation shall take effect in conformity with the provisions
of this Code.

Issue: Whether or not the Conditional Deed of Sale violate the principle of mutuality of
contracts under Article 1308 of the Civil Code?

Held:

No. In the past, this Court has distinguished between a condition imposed on the perfection
of a contract and a condition imposed merely on the performance of an obligation. While
failure to comply with the first condition results in the failure of a contract, failure to comply
with the second merely gives the other party the option to either refuse to proceed with the

SALES CASE DIGEST COMPILATION 297


sale or to waive the condition. This principle is evident in Article 1545 of the Civil Code on
sales, which provides in part:

Art. 1545. Where the obligation of either party to a contract of sale is


subject to any condition which is not performed, such party may refuse to
proceed with the contract or he may waive performance of the condition x x x.

Paragraph 1(b) of the Conditional Deed of Sale, stating that respondent shall pay the
balance of the purchase price when he has successfully negotiated and secured a road right
of way, is not a condition on the perfection of the contract nor on the validity of the entire
contract or its compliance as contemplated in Article 1308. It is a condition imposed only on
respondent’s obligation to pay the remainder of the purchase price. In our view and applying
Article 1182, such a condition is not purely potestative as petitioners contend. It is not
dependent on the sole will of the debtor but also on the will of third persons who own the
adjacent land and from whom the road right of way shall be negotiated.

The condition is a mixed condition which is allowed by Article 1182. Potestative


condition is imposed not on the birth of the obligation but on its fulfillment, only the condition
is avoided, leaving unaffected the obligation itself.

With respect to petitioners’ argument that paragraph 5 of the Conditional Deed of


Sale likewise rendered the said contract void, we find no merit to this theory.

Reading paragraph 5 in its entirety will show that Rodriguez’s option to rescind the
contract is not absolute as it undeniably only limited to the contingency that Rodriguez shall
not be able to secure the road right of way. .

In sum, Rodriguez’s option to rescind the contract is not purely potestative but rather
also subject to the same mixed condition as his obligation to pay the balance of the
purchase price – i.e., the negotiation of a road right of way. In the event the condition is
fulfilled (or the negotiation is successful), Rodriguez must pay the balance of the purchase
price. In the event the condition is not fulfilled (or the negotiation fails), Rodriguez has the
choice either (a) to not proceed with the sale and demand return of his downpayment or (b)
considering that the condition was imposed for his benefit, to waive the condition and still
pay the purchase price despite the lack of road access. This is the most just interpretation
of the parties’ contract that gives effect to all its provisions.

In any event, even if we assume for the sake of argument that the grant to Rodriguez
of an option to rescind, in the manner provided for in the contract, is tantamount to a
potestative condition, not being a condition affecting the perfection of the contract, only the
said condition would be considered void and the rest of the contract will remain valid.

2) G.R. No. L-17150 June 20, 1922

ANDRES SOLER, plaintiff-appellee,


vs.
EDWARD CHESLEY, defendant-appellant.

FACTS:

SALES CASE DIGEST COMPILATION 298


Andres Soler agreed with Wm. H. Anderson and Co. for the purchase of certain
machinery. Anderson and Co. was to deliver to Soler the coconut oil machinery which was
ordered by cable. The machinery consisted of oil expellers, rotary pumps, cookers, filter
press, and chain elevators among others. The machinery was to be invoiced at
manufacturers’ price plus all charges (freight, insurance, interest etc.) Half of the payment
through deposit was to be made upon arrival of the machinery and the balance to be paid 90
days after delivery of the machinery.

If Soler would fail to comply with the terms, it would be sufficient cause to terminate the
contract and all payments made by him would remain with Anderson and Co. The title of the
machinery was to remain in the name of Anderson and Co. until full payment had been
made after which time transfer of all right and title would be made to Soler. On Nov. 16,
1918, Soler sold to Edward Chesley all his rights and interest in the contract of sale. Soler
stated in their contract that a part of the aforesaid machinery was at that time on the
way with the other part already in Manila. With this transfer of rights and interest, Soler
was to be relieved from his contract with Anderson and Co. since Chesley was subrogated
in his place. The sale for the machinery for P100k by Chesley to Anderson and Co. was
accompanied by Soler paying the difference between the amount of the invoices and the
100k.

However, of the machinery to be delivered, only the filter press, cooker, and the chains
were in Manila on Nov. 16, 1918 but the most important parts such as the oil expellers and
the grinding mills were not yet there. The expellers arrived on Feb 13, the motors on Mar 8,
the machinery on Apr 27, and the grinding mills on Aug 23, 1919 respectively. Chesley
received and paid for them under protest since they were not delivered within the period
stipulated in the contract. Soler was informed through letter that the contract between him
and Chesley was rescinded since parts of the machinery that the former had stated were on
the way were actually not.

Soler filed suit praying that Chesley be ordered to pay him P30,546 and the price of the
machinery paid by the latter. Chesley alleged that he accepted and signed the contract on
the assertion of Soler that the machinery was either in Manila already or on the way. He
signed the contract because he was desirous of having the machinery.

ISSUE/S: WON time was an essential element as a condition of the contract

RULING: YES

The act of Chesley in insisting that the guaranty of Soler (that part of the machinery
to be delivered was on its way), his repeated complaints and protests when he made
payments as the parts arrived, and his letter to Soler leave no room for doubt that the arrival
of the machinery within a reasonably short time was one of the determining elements of his
consent.

These acts disclose the fact that he intended the arrival of the machinery to be an
essential element of the contract.. Chesley had no reason to doubt the veracity of Soler’s
assertion that the machinery was on its way and the latter even testified that he showed the
letters stating that according to information he received, the expellers had already been sent
out.

The fact that Soler had no control of the prompt transportation of the machinery to
Manila does not relieve him from making good his guaranty inserted in the contract. He who
contracts and assumed an obligation is presumed to know the circumstances under which

SALES CASE DIGEST COMPILATION 299


said obligation can be complied with. Since Soler had failed to carry out his obligation in the
contract, he has no right to compel Chesley to comply with his obligation to pay him.

3) G.R. No. 19009 September 26, 1922


E.C. MCCULLOUGH & CO., plaintiff-appelle,
vs.S. M . BERGER, defendant-appellant.

FACTS:

Plaintiff and defendant and defendant entered into an agreement by which the
defendant was to deliver plaintiff 501 bales of tobacco of New York City in good condition.
That delivery was made and the plaintiff paid the full purchase price. The tobacco arrived in
two shipments. After it was placed in the warehouse, the tobacco itself was examined as to
its condition and quality by the different buyers to whom the plaintiff had contracted to sell it,
and after such physical inspection, they refused to accept it and complete their purchase
because it was "musty." When the condition of the tobacco was discovered, plaintiff promptly
notified the defendant, who ignored the protest.

ISSUE: WON there was a breach of contract because of the condition of the tobacco when it
arrived in New York

RULING: YES

Although at the time of the making of the contract between them the plaintiff and
defendant were in Manila, the tobacco involved was on the high seas in transit to New York.
By the terms of the contract, the defendant guaranteed the arrival of the tobacco in New
York "in good condition. From necessity the plaintiff could not see or examine it and would
not know anything about its grade or quality, and, for that reason, insisted that the defendant
should make and sign the writing above quoted in which he says:

I guarantee the arrival of the tobacco in New York in good condition, subject, of
course to, to conditionsarising after its departure from Manila, which contingencies
are covered by adequate insurance.

The trial court found and the testimony is conclusive that the tobacco did not arrive in
New York "in good condition," and that , as a matter of fact, it was not "in good condition"
when it left Manila. The plaintiff and defendant had known each other for about ten years,
and had mutual confidence in each other, and were experienced business men. Defendant's
draft of the tobacco had been dishonored. Plaintiff was willing to take the tobacco and honor
the draft, with the proviso that the defendant would guarantee its arrival "in good condition."

4) G.R. No. L-18137 August 31, 1963


ROSELLER T. LIM, ET AL., plaintiffs-appellants,
vs.
PACITA DE LOS SANTOS, ET AL., defendants-appellees

SALES CASE DIGEST COMPILATION 300


FACTS:

The defendants contracted to sell to each of the several plaintiffs certain subdivision
lots embraced in the Congressional Avenue Subdivision and covered by Transfer
Certificates of Title Nos. 35882, 27016, and 34844 of the Register of Deeds of Quezon City.
Except as to the lots, the amounts for the consideration of the contracts, the parties and
corresponding particulars involved for each plaintiffs, the contracts were identical and were
executed within the months of February to April, 1960.

The plaintiffs-vendees have complied with their obligations and have begun or
contracted for the construction of their houses, however, the defendants failed,
notwithstanding the plaintiffs' demands, to construct the necessary roads that would serve
as outlets in accordance with the requirements and specifications of existing laws and
regulations; that defendant de los Santos "made plaintiffs understand that she binds herself
to, among others, construct roads, particularly outlets for entrance and egress to and from
the lots"; that "defendant represented to plaintiffs that she would have constructed adequate
outlets to and from the lots purchased"; that without these roads, the lots in question would
be uninhabitable, impassable and valueless pieces of real estate which plaintiffs would not
even venture to consider purchasing; wherefore, the plaintiffs pray for judgment ordering the
defendants to construct these roads and pay actual and moral damages and attorney's fees.

ISSUE: WON the plaintiffs have a cause of action to compel the defendant to construct
roads as a one of the conditions of the contract

RULING: YES

The allegations in the complaint that defendant-vendor made representations that


"she would have constructed (i.e., would cause to be built) adequate outlets" for the lots sold
do not strike us to be so improbable as to justify their being disbelieved de plano. After all, a
seller's duty is to deliver the thing sold in a condition suitable for its enjoyment by the buyer
for the purposes and proper access to a residence is essential to its enjoyment. Additionally,
at the time the contracts in question were made, the Subdivision Regulations of the National
Urban Planning Commission required paved roadways to be constructed by the subdivision
owner. In any event, since paragraph VI of the complaint avers a representation by
defendant herself that she would construct the roads, her failure to do so constitute by itself
a distinct and sufficient cause of action irrespective of the Regulations and Ordinance that
plaintiffs invoke in paragraph V of their complaint.

XXVII. EXPRESS WARRANTIES

1) [G.R. No. 132269. April 27, 2000]


HARRISON MOTORS CORPORATION, petitioner,
vs.
RACHEL A. NAVARRO, respondent.

SALES CASE DIGEST COMPILATION 301


FACTS:
Sometime in June of 1987 Harrison Motors Corporation through its president, Renato
Claros, sold two (2) Isuzu Elf trucks to private respondent Rachel Navarro, owner of RN
Freight Lines, a franchise holder operating and maintaining a fleet of cargo trucks all over
Luzon. Petitioner, a known importer, assembler and manufacturer, assembled the two (2)
trucks using imported component parts. Prior to the sale, Renato Claros represented to
private respondent that all the BIR taxes and customs duties for the parts used on the two
(2) trucks had been paid for.

In December of 1988 government agents seized and detained the two (2) Elf trucks
of respondent after discovering that there were still unpaid BIR taxes and customs duties
thereon. The BIR and the BOC ordered private respondent to pay the proper assessments
or her trucks would be impounded. Private respondent went to Claros to ask for the receipts
evidencing payment of BIR taxes and customs duties; however, Claros refused to comply.
Private respondent then demanded from Claros that he pay the assessed taxes and warned
him that he would have to reimburse her should she be forced to pay for the assessments
herself. Her demands were again ignored

But wanting to secure the immediate release of the trucks to comply with her
business commitments, private respondent paid the assessed BIR taxes and customs duties
amounting to P32,943.00. Consequently, she returned to petitioners office to ask for
reimbursement, but petitioner again refused, prompting her to send a demand letter through
her lawyer. When petitioner still ignored her letter, she filed a complaint for a sum of money
on 24 September 1990 with the Regional Trial Court of Makati.

Petitioner contends that private respondent should be the one to pay the internal
revenue taxes and customs duties. It claims that at the time the Memorandum Orders and
the two (2) Memoranda of Agreement took effect the two (2) Elf trucks were already sold to
private respondent, thus, it no longer owned the vehicles. Whatever payments private
respondent made to the government after the sale were solely her concern and such burden
should not be passed on to petitioner.

ISUE: WON there was an express warranty from the vendor

RULING:
It is true that the ownership of the trucks shifted to private respondent after the sale.
But petitioner must remember that prior to its consummation it expressly intimated to her that
it had already paid the taxes and customs duties. Such representation shall be considered
as a sellers express warranty under Art. 1546 of the Civil Code which covers any affirmation
of fact or any promise by the seller which induces the buyer to purchase the thing and
actually purchases it relying on such affirmation or promise. It includes all warranties which
are derived from express language, whether the language is in the form of a promise or
representation. Presumably, therefore, private respondent would not have purchased the
two (2) Elf trucks were it not for petitioners assertion and assurance that all taxes on its
imported parts were already settled.

SALES CASE DIGEST COMPILATION 302


XXVIII. IMPLIED WARRANTIES

1) G.R. No. L-42636 August 1, 1985


MARIA LUISA DE LEON ESCALER and ERNESTO ESCALER, CECILIA J. ROXAS and
PEDRO ROXAS, petitioners,
vs.
COURT OF APPEALS, JOSE L. REYNOSO, now deceased, to be substituted by his
heirs or legal representatives and AFRICA V. REYNOSO, respondents.

FACTS:
On March 7, 1958, the spouses Reynoso sold to petitioners a parcel of land situated
in Antipolo, Rizal with an area of 239,479 sqm and covered by TCT No. 57400. However, on
April 21, 1961, the Register of Deeds of Rizal and A. Doronilla Resources Development, Inc.
filed a case (Case no. 4252) for the cancellation of OCT No. 1526 issued in the name of
Angelina C. Reynoso (the predecessors-in-interest of the spouses Reynoso) on the ground
that the subject property is already covered by TCT No. 42999 issued under A. Doronilla.
The court favored A. Doronilla in this case.

Thereafter, herein petitioners filed the present case against the respondents, for the
recovery of the value of the property sold to them plus damages on the ground that the
spouses violated the vendors’ “warranty against eviction.” The lower court ruled in favor of
the petitioners, which was reversed in the CA. Hence, the present petition.

ISSUE: W/N Articles 1558 and 1559 of the Civil Code are to be strictly applied in this case.

HELD:
YES. (See Articles 1548, 1558 and 1559).
In order that a vendor’s liability for eviction may be enforced, the following requisites
must concur:
a. There must be a final judgment;
b. The purchaser has been deprived of the whole or part of the thing sold;
c. Said deprivation was by virtue of a right prior to the sale made by the vendor; and
d. The vendor has been summoned and made co-defendant in the suit for eviction at the
instance of the vendee.

In the case at bar, the fourth requisite—that of being summoned in the suit for
eviction at the instance of the vendee—is not present. All that petitioners did, per their very
admission, was to furnish respondents, by registered mail, with a copy of the opposition they
(petitioners) filed in the suit. Decidedly, this is not the kind of notice prescribed by Articles
1558 and 1559 of the Civil Code.
The term “unless he is summoned in the suit for eviction at the instance of the
vendee” means that the respondents as vendor/s should be made parties to the suit at the
instance of petitioners-vendees, either by way of asking that the former be made a co-
defendant or by the filing of a third-party complaint against said vendors. Nothing of that sort
appeared to have been done by the petitioners in the instant case.

SALES CASE DIGEST COMPILATION 303


2) [G.R. No. 119745. June 20, 1997]
POWER COMMERCIAL AND INDUSTRIAL CORPORATION, petitioner,
vs.
COURT OF APPEALS, SPOUSES REYNALDO and ANGELITA R. QUIAMBAO and
PHILIPPINE NATIONAL BANK, respondents.

FACTS:

Petitioner Power Commercial & Industrial Development Corporation (PowerCom), an


industrial asbestos manufacturer, needed a bigger office space and warehouse for its
products. PowerCom entered into a contract of sale with the respondent spouses Reynaldo
and Angelita R. Quiambao involving parcel of land in San Antonio Village, Makati City

The parties agreed that petitioner PowerCom would pay private respondents spouses
Quiambao P108,000.00 as down payment, and the balance of P295,000.00 upon the
execution of the deed of transfer of the title. Further, petitioner assumed, as part of the
purchase price, the existing mortgage on the land. In full satisfaction thereof, he paid
P79,145.77 to respondent PNB.

Later on, the respondent spouses mortgaged again said land to PNB to guarantee a loan.
PowerCom agreed to assume payment of the loan. The parties executed a Deed of
Absolute Sale With Assumption of Mortgage. On the same day, Mrs. C.D. Constantino,
then General Manager of PowerCom, submitted to PNB said deed with a formal application
for assumption of mortgage

PNB informed respondent spouses that for petitioner’s failure to submit the papers
necessary for approval pursuant to the former’s letter dated January 15, 1980, the
application for assumption of mortgage was considered withdrawn; that the outstanding
balance of P145,000.00 was deemed fully due and demandable; and that said loan was to
be paid in full within fifteen (15) days from notice

PowerCom paid PNB P41,880.45 on June 24, 1980 and P20,283.14 on December 23,
1980, payments which were to be applied to the outstanding loan. PowerCom then filed a
case against respondent spouses for rescission and damages. Petitioner demanded the
return of the payments it made on the ground that its assumption of mortgage was never
approved.

While this case was pending, the mortgage was foreclosed. The property was
subsequently bought by PNB during the public auction.

The RTC ruled that the failure of respondent spouses to deliver actual possession to
petitioner entitled the latter to rescind the sale, and in view of such failure and of the denial
of the latter’s assumption of mortgage, PNB was obliged to return the payments made by the
latter.

However, on appeal, the CA reversed the trial court. It held that the deed of sale between
respondent spouses and PowerCom did not obligate the former to eject the lessees from the
land in question as a condition of the sale, nor was the occupation thereof by said lessees a
violation of the warranty against eviction. Hence, there was no substantial breach to justify
the rescission of said contract or the return of the payments made

Petitioner PowerCom, on the other hand, contends that there was a substantial
breach of the contract between the parties warranting rescission and that CA gravely erred
in failing to consider in its decision that a breach of implied warranty under Article 1547 in
relation to Article 1545 of the Civil Code applies in the case-at-bar.

SALES CASE DIGEST COMPILATION 304


ISSUE: WON there was a substantial breach of the contract between the parties warranting
rescission

RULING: No.

It is petitioner’s failure to establish any breach of the warranty against eviction.


Despite its protestation that its acquisition of the lot was to enable it to set up a warehouse
for its asbestos products and that failure to deliver actual possession thereof defeated this
purpose, still no breach of warranty against eviction can be appreciated because the facts of
the case do not show that the requisites for such breach have been satisfied.

Requisites of Breach of Warranty Against Eviction: A breach of this warranty requires the
concurrence of the following circumstances:

(a) The purchaser has been deprived of the whole or part of the thing sold;
(b) This eviction is by a final judgment;
(c) The basis thereof is by virtue of a right prior to the sale made by the vendor; and
(d) The vendor has been summoned and made co-defendant in the suit for eviction at
the instance of the vendee.

In the absence of these requisites, a breach of the warranty against eviction under
Article 1547 cannot be declared.

As correctly pointed out by CA, the presence of lessees does not constitute an
encumbrance of the land, nor does it deprive petitioner of its control thereof.

We note, however, that petitioner’s deprivation of ownership and control finally


occurred when it failed and/or discontinued paying the amortizations on the mortgage,
causing the lot to be foreclosed and sold at public auction. But this deprivation is due to
petitioner’s fault, and not to any act attributable to the vendor-spouses.

3) [G.R. No. 148173. December 10, 2004]

SUPERCARS MANAGEMENT & DEVELOPMENT CORPORATION, represented by its


President BENIGNO CHAN, petitioner,
vs.
THE LATE FILEMON FLORES, substituted by his surviving spouse, NORA C.
FLORES,respondent.

FACTS:

Filemon Flores, respondent, purchased from Supercars Management and


Development Corporation, petitioner, an Isuzu Carter Crew Cab for P212,000.00 payable
monthly with a down payment equivalent to 30% of the price or P63,600.00. The balance
was to be financed by the Rizal Commercial Banking Corporation (RCBC). The sale was
coursed through Pablito Marquez, petitioners salesman.

A day after the vehicle was delivered, respondent used it for his family’s trip to
Bauang, La Union. While traversing the national highway in Tarlac, Tarlac, the fan belt of the

SALES CASE DIGEST COMPILATION 305


vehicle snapped. Then its brakes hardened after several stops and did not function properly;
the heater plug did not also function; the engine could not start; and the fuel consumption
increased.

Upon their return to Manila in the first week of January 1989, respondent complained
to petitioner about the defects of the vehicle. Marquez then had the vehicle repaired and
returned it to respondent that same day, assuring the latter that it was already in good
condition.
But after driving the vehicle for a few days, the same defects resurfaced, prompting
respondent to send petitioner a letter dated January 30, 1989 rescinding the contract of sale
and returning the vehicle due to breach of warranty against hidden defects. A copy of the
letter was furnished RCBC.

ISSUE: WON respondent Flores is entitled to rescind the contract because there was a
breach of warranty for hidden defects

RULING: YES

Respondents complaint filed with the RTC seeks to recover from petitioner the
money he paid for the vehicle due to the latter’s breach of his warranty against hidden
defects under Articles 1547, 1561, and 1566 of the Civil Code. The vehicle, after it was
delivered to respondent, malfunctioned despite repeated repairs by petitioner. Obviously, the
vehicle has hidden defects. A hidden defect is one which is unknown or could not have been
known to the vendee.

It is well within respondent’s right to recover damages from petitioner who committed a
breach of warranty against hidden defects. Article 1599 of the Civil Code partly provides:

Article 1599. Where there is a breach of warranty by the seller, the buyer may, at his
election: x x x

(4) Rescind the contract of sale and refuse to receive the goods, or if the goods have
already been received, return them or offer to return them to the seller and recover
the price or any part thereof which has been paid.

When the buyer has claimed and been granted a remedy in anyone of these ways,
no other remedy can thereafter be granted, without prejudice to the provisions of the
second paragraph of Article 1191.

Petitioners contention that under Article 1191 of the Civil Code, rescission can no
longer be availed of as the vehicle was already in the hands of an innocent purchaser for
value lacks merit. Rescission is proper if one of the parties to a contract commits a
substantial breach of its provisions. It creates an obligation to return the object of the
contract. It can be carried out only when the one who demands rescission can return
whatever he may be obliged to restore. Rescission abrogates the contract from its inception
and requires a mutual restitution of the benefits received. Petitioner is thus mandated by law
to give back to respondent the purchase price upon his return of the vehicle.

SALES CASE DIGEST COMPILATION 306


4) [G.R. No. 152219. October 25, 2004]
NUTRIMIX FEEDS CORPORATION, petitioner,
vs.
COURT OF APPEALS and SPOUSES EFREN AND MAURA
EVANGELISTA, respondents.

FACTS:

In 1993, private respondent spouses Evangelista procured various animal


feeds from petitioner Nutrimix Feeds Corporation. The petitioner gave the respondents
a credit period of 30-45 days to postdate checks to be issued as payment for the
feeds. The accommodation was made apparently because the company’s president
was a close friend of Evangelista. The various animal feeds were paid and covered by
checks with due dates from July 1993-September 1993.

Initially, the spouses were good paying customers. However, there were
instances when they failed to issue checks despite the delivery of goods.
Consequently, the respondents incurred an aggregate unsettled account with Nutrimix
amounting to P766,151.

When the checks were deposited by the petitioner, the same were dishonored
(closed account). Despite several demands from the petitioner, the spouses refused to
pay the remaining balance. Thereafter, Nutrimix filed a complaint against Evangelista
for collection of money with damages.

The respondents admitted their unpaid obligation but impugned their liability.
The nine checks issued were made to guarantee the payment of the purchases, which
was previously determined to be procured from the expected proceeds in the sale of
their broilers and hogs. They contended that inasmuch as the sudden and massive
death of their animals was caused by the contaminated products of the petitioner, the
nonpayment of their obligation was based on a just and legal ground.

The respondents also lodged a complaint for damages against the petitioner,
for the untimely and unforeseen death of their animals supposedly effected by the
adulterated animal feeds the petitioner sold to them.

Nutrimix alleged that the death of the respondents’ animals was due to the
widespread pestilence in their farm. The petitioner, likewise, maintained that it
received information that the respondents were in an unstable financial condition and
even sold their animals to settle their obligations from other enraged and insistent
creditors. It, moreover, theorized that it was the respondents who mixed poison to its
feeds to make it appear that the feeds were contaminated.

The trial court held in favor of petitioner on the ground that it cannot be held
liable under Articles 1561 and 1566 of the Civil Code governing “hidden defects” of
commodities sold. The trial court is predisposed to believe that the subject feeds were
contaminated sometime between their storage at the bodega of the Evangelistas and
their consumption by the poultry and hogs fed therewith, and that the contamination
was perpetrated by unidentified or unidentifiable ill-meaning mischief-maker(s) over
whom Nutrimix had no control in whichever way.

CA modified the decision of the trial court, citing that respondents were not
obligated to pay their outstanding obligation to the petitioner in view of its breach of
warranty against hidden defects. The CA gave much credence to the testimony of Dr.

SALES CASE DIGEST COMPILATION 307


Rodrigo Diaz, who attested that the sample feeds distributed to the various
governmental agencies for laboratory examination were taken from a sealed sack
bearing the brand name Nutrimix.

ISSUE: WON Nutrimix is guilty of breach of warranty due to hidden defects

HELD: NO.

The provisions on warranty against hidden defects are found in Articles 1561 and 1566
of the New Civil Code of the Philippines. A hidden defect is one which is unknown or
could not have been known to the vendee. Under the law, the requisites to recover on
account of hidden defects are as follows:

a) the defect must be hidden;

b) the defect must exist at the time the sale was made;

c) the defect must ordinarily have been excluded from the contract;

d) the defect, must be important (renders thing UNFIT or considerably decreases


FITNESS);

e) the action must be instituted within the statute of limitations

In the sale of animal feeds, there is an implied warranty that it is reasonably fit
and suitable to be used for the purpose which both parties contemplated. To be able to
prove liability on the basis of breach of implied warranty, three things must be
established by the respondents. The first is that they sustained injury because of the
product; the second is that the injury occurred because the product was defective or
unreasonably unsafe; and finally, the defect existed when the product left the hands of
the petitioner. A manufacturer or seller of a product cannot be held liable for any
damage allegedly caused by the product in the absence of any proof that the product
in question was defective. The defect must be present upon the delivery or
manufacture of the product; or when the product left the seller’s or manufacturer’s
control; or when the product was sold to the purchaser; or the product must have
reached the user or consumer without substantial change in the condition it was sold.
Tracing the defect to the petitioner requires some evidence that there was no
tampering with, or changing of the animal feeds. The nature of the animal feeds
makes it necessarily difficult for the respondents to prove that the defect was existing
when the product left the premises of the petitioner.

A review of the facts of the case would reveal that the petitioner delivered the
animal feeds, allegedly containing rat poison, on July 26, 1993; but it is astonishing
that the respondents had the animal feeds examined only on October 20, 1993, or
barely three months after their broilers and hogs had died. A difference of
approximately three months enfeebles the respondents’ theory that the petitioner is
guilty of breach of warranty by virtue of hidden defects. In a span of three months, the
feeds could have already been contaminated by outside factors and subjected to many
conditions unquestionably beyond the control of the petitioner.

SALES CASE DIGEST COMPILATION 308


Even more surprising is the fact that during the meeting with Nutrimix President
Mr. Bartolome, the respondents claimed that their animals were plagued by disease,
and that they needed more time to settle their obligations with the petitioner. It was
only after a few months that the respondents changed their justification for not paying
their unsettled accounts, claiming anew that their animals were poisoned with the
animal feeds supplied by the petitioner.

HANNAH

5. INVESTMENT AND DEVELOPMENT INC. v. COURT OF APPEALS

FACTS:

On January 14, 1965, Raymundo Gatpayat sold a parcel of agricultural land subject
matter of this case to petitioner Investment and Development, Inc. (IDI). On February 20,
1966, an OCT was issued in the name of respondent Gatpayat. On January 30, 1967, a
TCT was issued in Investment and Development, Inc.'s name.
Previously, Gatpayat bought such land from Tolentino on the condition that the annual
rental of ten cavans of palay given by tenant Ramirez would pertain to Gatpayat only after
full payment of the purchase price. Subsequently, respondent Gatpayat completed his
payments for the land and entered into an agreement with tenant Ramirez that the latter
shall sell the ten cavans of palay and give the proceeds to him.
On March 8, 1971, the petitioner sold the land to respondent Agencia de Empenos de A..
Aguirre, Inc. As a result thereof, a TCT was issued in the vendee's name.
In April 1972, tenant Ramirez was forced to stop cultivating the land in question
because of the bulldozing caused by A. Aguirre, Inc. Ramirez filed a complaint asking for
compensation for the act of A. Aguirre, Inc. In turn, IDI filed a cross-claimed against
Gatpayat in case of judgment adverse to it, while respondents Agencia and Aguirre, Inc.
filed a cross-claim against IDI.
ISSUE:
Whether or not IDI is entitled of reimbursement from Gatpayat
HELD:

No. The petitioner cannot claim reimbursement from its seller, respondent Gatpayat, on
the basis of an implied warranty against hidden faults or defects under Article 1547,
subparagraph (2) inasmuch as the term "hidden faults or defects" pertains only to those
that make the object of the sale unfit for the use for which it was intended at the time of the
sale.
In the case at bar, since the object of the sale by Gatpayat to the petitioner is an
agricultural land, the existing tenancy relationship with respect to the land cannot be a
"hidden fault or defect." It is not a lien or encumbrance that the vendor warranted did not
exist at the time of the sale. It is a relationship which any buyer of agricultural land should
reasonably expect to be present and which it is its duty to specifically look into and provide
for. Agencia saw to it that the warranty was specific when it, in turn, purchased the land.

SALES CASE DIGEST COMPILATION 309


6. MONFORT v. WILLIS (CA)

ISSUE:

Whether or not implied warranty is applicable in “as is and where is” sale

HELD:

No. The phrase “as is and where is” (which has been adopted from dispositions of army
surplus property) means nothing more than that the vendor makes no warranty as to the
quality or workable condition of the goods, and that the vendee takes them in the conditions
in which that they are found and from the place where they are located. It does not extend to
liens or encumbrances unknown to the vendee and could not be disclosed by a physical
examination of the goods sold.

7. SERFINO v. COURT OF APPEALS

FACTS:

On August 25, 1937, a parcel of land in Sagay, Negros Occidental, was patented and
registered in the name of Pacifico Casamayor. On December 14, 1945, the latter sold said
land in favor of Nemesia D. Baltazar.

Apparently, Casamayor’s original transfer certificate was lost during the war and upon
petition of Nemesia Baltazar, the Court ordered the reconstitution thereof. Pursuant thereto,
a new certificate of title was issued on January 18, 1946 in the name of Pacifico Casamayor.
On that same day, a TCT was issued in the name of Nemesia Baltazar but after the
cancellation of Casamayor’s OCT.

On August 25, 1951, Nemesia Baltazar, sold said property to Lopez Sugar Central. Upon
registration, it was found out that the same property was covered by another certificate of
title in the name of Federico Serfino.

Such discrepancy was caused by the conduct of the Provincial Treasurer of Negros of a
public auction sale of this property for tax delinquency for the period starting the year 1950.
Notice of this public auction sale was sent to Pacifico Casamayor but none to Nemesia
Baltazar and Lopez Sugar Mill. There being no public bidders on the scheduled date of sale,
the Provincial Treasurer of Negros Occidental issued a Certification of Sale of Delinquent
Real Property over the disputed land to the Province of Negros Occidental. Subsequently, a
Certificate of Repurchase of Real Property was issued and executed by the Provincial
Treasurer in favor of Federico Serfino, for and in behalf of Pacifico Casamayor.

On May 28, 1964, Serfino filed a petition for the Reconstitution of OCT in the name of
Pacifico Casamayor, upon the allegation that said title was lost. It was granted.

On November 19, 1964, the spouses Serfinos mortgaged the land to the Philippine National
Bank (PNB) to secure, thus prompting the Office of Registry of Deeds not to grant the
registration requested by the Lopez Sugar Central.

The lower court ordered Lopez Sugar Central and spouses Serfino to take the necessary
steps towards the clearing of their respective titles before a court of general jurisdiction.
Pursuant thereto, Lopez Sugar Central, on May 5, 1965, instituted an action for 1)
annulment of OCT of Baltazar of and TCT and of the mortgage executed by the Serfinos in
favor of PNB, 2) for the registration of the Deed of Sale, 3) for the issuance of a TCT in its
name and 4) for recovery of possession of the disputed land from the Serfinos.

ISSUE:

SALES CASE DIGEST COMPILATION 310


Whether or not the principle of caveat emptor applies

HELD:

YES. A purchaser of real estate at the tax sale obtains only such title as that held by the
taxpayer, the principle of caveat emptor (let the buyer beware) applies. Where land is sold
for delinquency taxes under the provisions of the Provincial Assessment Law,
rights of registered but undeclared owners of the land are not affected by the proceedings
and the sale conveys only such interest as the person who has declared the property for
taxation has therein. The sale therefore by the Province of Negros Occidental of the land in
dispute to the spouses Serfinos was void since the Province of Negros Occidental was not
the real owner of the property thus sold. In turn, the spouses Serfinos title which has been
derived from that of the Province of Negros Occidental is likewise void.

XXIX. WARRANTY AGAINST EVICTION

1. MENDOZA v. CAPARROS

FACTS:

Agapito sold to Pelejo two (2) parcels of land situated in Quezon Province for P3,650.00.

After a decade, Pelejo sold the same lots to spouses Mendoza and Tolentino. The latter
spouses died, leaving as heirs Pedro (plaintiff-appellee), Leandro and Justiniano Mendoza.
Upon conducting an extrajudicial partition as their parents-heirs, the 2 lots aforementioned
was allotted to Pedro Mendoza.

However, subsequently, Agapito obtained an OCT over the 2 parcels of land mentioned
above. Later, Agapito’s heirs, herein defendants Caparros, Socorro and Policornia (his
widow and 2 daughters, respectively) obtained a TCT over the lots in question after
conducting an extrajudicial partition.

Pedro Mendoza then instituted an action against defendants Pelejo, Caparros, Socorro and
Policornia (heirs of Agapito Ferreras) for cancellation of the TCT registered under Agapito’s
heirs and that another be issued in his name, married to Alfonsa Perez.

The trial court ruled in favor of plaintiff Mendoza although no bad faith was found either on
the part of Agapito or his heirs in the registrationof the lots in question even if such were recorded
incorrectly. The defendants, except Peleja, were ordered to pay costs.

The heirs of Agapito did not appeal.

Pelejo filed a motion for reconsideration, seeking the award of attorney’s fees, claiming that
his inclusion in the case as defendant was unfounded and malicious character.

ISSUE:

Whether or not Pelejo should be held responsible in the enforcement of warranty against
eviction

HELD:

SALES CASE DIGEST COMPILATION 311


YES. Peleja sold to Pedro Mendoza’s parents the two lots with the following condition that
“upon the sale of the lots, the seller undertook to defend the buyers against any adverse
claims over the lots sold.”

Hence, under the said condition, Peleja made himself liable to Pedro’s parents and to Pedro
as successor-in-interest, in case of eviction or in the event that the buyer or his heirs would
be deprived of the lots or part thereof by final judgment.

Even if such stipulation were not included in the deed of sale, the seller would still be
responsible in case of eviction of buyers pursuant to NCC Art. 1548 which states that:
eviction shall take place whenever by a final judgment based on a right prior to the sale or
an act imputable to the vendor, the vendee is deprived of the whole thing purchased. The
vendor shall answer for the eviction even though nothing has been said in the contract on
the subject.”

Had appellant Peleja not been included as defendant to the suit and he was made to pay
indemnity, He could raise the defense that he was not given fair opportunity to prove his title
over the 2 lots at the time he sold such to Bictoriano and Mendoza and they had
been improperly recorded.

In the case that Agapito Ferreras’ heirs sued Pedro Mendoza, demanding that possession
of the parcels of land be transferred to them and with the corresponding TCT to bolster their
claim, Peleja must be notified of the application of eviction at the instance of Mendoza in
order that the former must be bound to indemnify the latter.

2. NAKPIL v. MANILA TOWERS DEVELOPMENT CORP.

FACTS:

In 1964, Cheong Kiao Ang, leased his building to about 200 Filipino Chinese tenants
including Atty. Bonifacio Nakpil. The tenants of the building later formed the House
International Building Tenants Association, Inc. (HIBTAI).
The property was mortgaged with the Government Service Insurance System (GSIS) as
security for a loan by Ang. Upon failure to pay the loan, the GSIS had the real estate
mortgage foreclosed and the property sold at public auction, with GSIS as the winning
bidder. GSIS then sold the property to the Centertown Marketing Corporation (CMC) which
assigned all its rights to its sister-corporation, the Manila Tower Development Corporation
(MTDC). The HIBTAI protested, claiming that its members had the priority to buy the
property. The tenants refused to pay their rentals and instead remitted them to HIBTAI.
On June 29, 1981, the City Engineer wrote the MTDC, urgently requesting that the defects
of the building be corrected. However, before the MTDC could make the necessary repairs,
the HIBTAI, filed a complaint against the GSIS for injunction and damages, but it was
dismissed.
HIBTAI filed another complaint for annulment of contract and damages against the CMC,
MTDC and GSIS averring the tenants had the priority right to purchase the property. The
court dismissed the complaint. It was appealed before the CA, but was denied. According to
the Court, the tenants of the building, not the HIBTAI, were the real parties-in-interest as
parties-plaintiffs.
Years later, Atty. Samuel S. Samuela, the building administrator, wrote Architect Juan A.
Maravillas, Jr., then Officer-in-Charge (OIC), Office of the Building Official, City of Manila,
requesting for an immediate ocular inspection of the building to determine its safety. The
letter mentioned that, as far back as 1981, the City Engineer and Building Official had

SALES CASE DIGEST COMPILATION 312


ordered the building condemned after inspection. Atty. Samuela stated that when the MTDC
was about to initiate the repairs on the building, the tenants filed several suits against it; this
prevented MTDC from complying with the said order. During the pendency of these cases,
the tenants likewise took control of the building and even illegally put up structures in the
building without MTDC's consent.
At the time, Atty. Nakpil was overseas. Upon his return, he discovered his room was
destroyed. He then filed a complaint against MTDC, averring that the latter committed a
breach of contract because it failed to maintain the peaceful and adequate enjoyment of
the lease for the duration of contract. On the contrary, MTDC argued that Nakpil failed to
prove that MTDC had anything to do with the repairs, as such were done by employees
under the city engineer.

ISSUE:

Whether or not Nakpil is entitled to damages

HELD:

NO. Breach of contract is the failure without legal reason to comply with the terms of a
contract. It is also defined as the failure, without legal excuse, to perform any promise which
forms the whole or part of the contract. There is no factual and legal basis for any award for damages
to respondent.
The duty to maintain the lessee in the peaceful and adequate enjoyment of the lease for the duration of the
contract is merely a warranty that the lessee shall not be disturbed in his legal, and not physical, possession.

In the early case of Goldstein v. Roces, the court pointed out that the obligation to maintain the lessee in the
peaceful and adequate enjoyment of the leased property seeks to protect the lessee not only from acts of
third persons but also from the acts of the lessor. By his own acts, because, being the person principally
obligated by the contract, he would openly violate it if, in going back on his agreement, he should
attempt to render ineffective in practice the right in the thing he had granted to the lessee; and
by others' acts, because he must guarantee the right he created, for he is obliged to give warranty in the
manner we have set forth in our commentary on article 1553, and, in this sense, it is incumbent upon him to
protect the lessee in the latter's peaceful enjoyment.

When the act of trespass is done by third persons, it must be distinguished whether it is
trespass in fact or in law because the lessor is not liable for a trespass in fact or a mere act
of trespass by a third person.

In the Goldstein case, trespass in fact was distinguished from legal trespass, thus:
if the act of trespass is not accompanied or preceded by anything which reveals a juridical
intention on the part of the trespasser, in such wise that the lessee can only distinguish the
material fact, stripped of all legal form or reasons, we understand it to be trespass in fact
only (de mero hecho). Further, the obligation under Article 1654(3) arises only when acts, termed as legal
trespass (perturbacion de derecho), disturb, dispute, object to, or place difficulties in the way of
the lessee's peaceful enjoyment of the premises that in some manner cast doubt upon the
right of the lessor by virtue of which the lessor himself executed the
lease. What is evident in the present case is that the disturbance on the leased premises on July 19, 1996
was actually done by the employees under the City Engineer of Manila and the City Building Official on
orders of the City Mayor without the participation of the MTDC.

3. ESCALER v. CA

FACTS:

SALES CASE DIGEST COMPILATION 313


On March 7, 1958, the spouses Reynoso sold to petitioners a parcel of land situated in
Antipolo, Rizal with an area of 239,479 square meters and covered by TCT No. 57400.
However, on April 21, 1961, the Register of Deeds of Rizal and Doronilla Resources
Development, Inc. filed a case for the cancellation of OCT No. 1526 issued in the name of
Angelina Reynoso (the predecessors-in-interest of the spouses Reynoso) on the ground that
the subject property is already covered by TCT No. 42999 issued under A. Doronilla. The
court favored Doronilla in this case.

Thereafter, herein petitioners filed the present case against the respondents, for the
recovery of the value of the property sold to them plus damages on the ground that the
spouses violated the vendors’ “warranty against eviction.” The lower court ruled in favor of
the petitioners, which was reversed in the CA. Hence, the present petition.

ISSUE:

Whether or not Articles 1558 and 1559 of the Civil Code are to be strictly applied in this
case.

HELD:

YES. In order that a vendor’s liability for eviction may be enforced, the following requisites
must concur:
a. There must be a final judgment;
b. The purchaser has been deprived of the whole or part of the thing sold;
c. Said deprivation was by virtue of a right prior to the sale made by the vendor; and
d. The vendor has been summoned and made co-defendant in the suit for eviction at the
instance of the vendee.

In the case at bar, the fourth requisite—that of being summoned in the suit for eviction (Case
no. 4252) at the instance of the vendee—is not present. All that petitioners did, per their very
admission, was to furnish respondents, by registered mail, with a copy of the opposition they
(petitioners) filed in the suit. Decidedly, this is not the kind of notice prescribed by Articles
1558 and 1559 of the Civil Code.
The term “unless he is summoned in the suit for eviction at the instance of the vendee”
means that the respondents as vendor/s should be made parties to the suit at the instance
of petitioners-vendees, either by way of asking that the former be made a co-defendant or by
the filing of a third-party complaint against said vendors. Nothing of that sort appeared to
have been done by the petitioners in the instant case.

4. BAUTISTA v. LASAM

FACTS:

Gabriel Lasam and brothers Felix and Melquiades Bautista entered into a deed of sale
where the former will sell to the latter brothers 2,130 hectares. It is stated herein that Gabriel
Lasam acquired the land from the late Jacinto Lasam and the parties shall proceed to
register the land despite the insufficiency of title on the part of Jacinto. Subsequently, the
land was registered up to the extent of only 1,445 hectares. The possession of the remaining
hectares remained with the Bautista brothers. Later on, Lasam demanded the unpaid
balance of the stipulated price of the property. Meanwhile, the brothers insisted that the land
was only measured at 1,445 hectares and the amount of the remaining hectares should be
discounted from the original amount. The trial court ruled in favor of Lasam. On appeal, the
brothers alleged that they sent letter to Lasam about the non-registration of the remaining
hectares and the reason for such was due to the claim of Land and Forestry Offices. Further,

SALES CASE DIGEST COMPILATION 314


they asserted that Lasam had the obligation of warranty of eviction in connection with the
claim of the Land and Forestry. By reason of this, the brothers cannot be held in default of
the payment of the purchase price.

ISSUE:

Whether or not Lasam should respond to the assertion by the brothers of their right of
warranty against eviction

HELD:

No. It is clear that to force a seller to respond to the eviction, the following must concur: (1)
there is final judgment, (2) the buyer has been deprived in all or in part of the thing sold, (3)
that this deprivation was by virtue of a right prior to the sale made by the seller, and (4) that
the seller has been previously notified of the demand for eviction at the request of the buyer.
In this case, the 2nd and 4th requirements were not present. The brothers were not deprived
of any part of the land since they took possession of the land and continue to own it, despite
not having managed to register in the Registry. Also, it has been proven that the letters sent
to Lasam were not letters notifying Lasam about the eviction, but rather the brothers’ request for
an extension of time for the payment of their remaining obligation.

5. JOVELLANO v. LUALHATI

FACTS:

On November 6, 1911, Dionisia Solmirano, Lucio Solmirano, and Macario Solmirano sold to
Enrique Jovellano a parcel of land in Nagcarlan, Laguna, for P150. The deed of sale
contained the usual covenant against eviction. Thereafter, Jovellano entered upon the land.
In 1913, Maxima Dorado filed an action against Jovellano to recover possession of the land.
Dorado won the case. Instead of appealing, Jovellano instituted a case for the determination
of ownership of the land, but he was defeated again.

The present case was filed by Jovellano against the defendants herein for the recovery of
the price paid for the land plus other related expenses. The defendants alleged that they
were not notified as provided by law. Also, the Solmiranos were never cited to appear or
made parties. Only Dionisia Solminaro was made witness and Solmirano may have been
present in the court at the time of the trial. The trial judge argued that this was a substantial
compliance with the

ISSUE:

Whether or not Solminaros should be held liable for warranty of eviction

HELD:

NO. Article 1475 of the Code provides that eviction exists when by final judgment based
upon a right prior to the sale, the vendee is deprived of the whole or any part of the thing
purchased. The vendors shall be liable for the eviction even though the contract is silent
on the subject. Also, articles 1481 and 1482 provide for the need of giving notice to the
vendor by vendee of the suit for eviction.
In the instant case, Jovellano, the purchaser, failed to give formal notice to the Solminaros
of the suits for eviction. Thus, the former loses all recourse against him. There is no cause

SALES CASE DIGEST COMPILATION 315


of action against the vendor of real property to make him responsible for warranty in case
of eviction unless said vendor is given notice of the suit for eviction.|||

6. CANIZARES v. TORREJON

FACTS:
The plaintiff, Don Leopoldo Cañizares Tiana, purchased from the defendant Don
Jose Maria Torrejon, a lot with buildings and improvements, in Jolo, Moro Province.
At the time of sale, Torrejon warranted the title by agreeing to be responsible to
Tiana and his heirs and successors in interest in defending the such title against any
just claims.
Prior to the execution of the said deed, Torrejon filed an application of registration of
the land and its improvements. However, the Court dismissed the application and
declared the estate to be public property. Further, it appeared that the military
authorities acquired possession of the land and Tiana did not receive any payment
for it including the rents thereof. Tiana demanded that she be paid of such amount.
It also appeared that Torrejon had knowledge of the Government’s objection of his
application.
ISSUE:
Whether or not the action upon warranty of title should prosper
HELD:
YES. Article 1475 of the Civil Code speaks of three indispensable requisites: (1) final
judgment; (2) that the vendee be deprived of the whole or a part of the thing sold;
and, (3) a right prior to the sale; and, finally, another requisite is that prescribed in
article 1481 of the Civil Code: that the vendor be given notice of the suit at the
instance of the vendee. In the present case, all the foregoing requisites appear
herein. In his brief, the defendant alleges that the decision of the registration case
was not final and that the plaintiff could and should have appealed from it. Manresa
said in one of his commentaries that the right of the plaintiff does not suffer the least
impairment because he did not appeal. Also, Torrejon cannot set up the defense of
not being notified of the objection of the Government as he himself is the applicant in
the case. For these reasons, Torrejon should be made to pay the price of the estate
plus the rents Tiana had been deprived of.

7. ANGELO v. PACHECO
FACTS:

On July 14, 1920, the plaintiff sold to the defendant a parcel of land consisting of
some 659 hectares Paniqui, Tarlac, and five carabaos, for P13,500, of which the
defendant paid P5,500 and promised to pay the balance of P8,000 on or before the
month of March, 1921. It is also agreed by both parties that warranty against eviction
be waived by the defendant. In 1924, Pacheco asked the Angelo to sign a different
deed eliminating the prohibition from disposing the said property while retaining the
waiver against eviction. Prior to the sale, Angelo instituted a registration proceeding
of the subject land. Pacheco was aware of it. Later on, the registration of the land is
decreed in favor of the Angelo, the plaintiff.

SALES CASE DIGEST COMPILATION 316


Thereafter, a motion for review was filed by the Director of Lands and two groups of
opponents. Pacheco was already the owner with a certificate of title when he filed his
answer to the motion. However, the decree of registration was set aside after the
hearing was held. An appeal was taken by Pacheco and de Valle Cruz, the latter to
whom the former sold the property. The Court ordered that de Valle Cruz shall retain
lien upon the land with the right to foreclose the property. Later on, de Valle Cruz
filed a case against Pacheco and obtained favourable judgment.

ISSUE:
Whether or not the wavier still binds Angelo as the vendor

HELD:

NO. Besides acting in bad faith for having knowledge of the registration decree and
pending motions for review, the defendant did not only make a formal and express waiver
of warranty for eviction, but because he believed that he waived it entirely, he failed to
notify the Angelo, or to have her summoned for the reopening of the decree, when he
should have done so because he was in danger of losing the property, as he indeed did.
For failure of that notice, the plaintiff is not bound to warranty, according to article 1481, of
the Civil Code.

8. DE LA RIVA v. AH KEE

FACTS:
On October 26, 1928, the Panabutan Lumber & Plantation Co., Inc., executed a chattel
mortgage on certain personal and real properties in favor of the Philippine National Bank
to secure payment of the sum of P120,000.
On February 15, 1930, judgment was rendered in Ah Kee et al, vs. Panabutan Lumber, in
favor of the therein plaintiffs. In consequence thereof, certain properties of the Panabutan
Lumber were attached. Some of the attached properties were not included in the mortgage
in favor of the Philippine National Bank. However, plaintiffs could not deposit the estimated
amount of the expenses and costs and execution, the sale of the attached properties could
not be made.
During the pendency of case of Torrejon Jurika & Co., Inc., Ah Kee et al. vs. Panabutan
Lumber & Plantation Co., the chattel mortgage executed by Panabutan in favor of PNB
was declared null and void. By this time, the mortgaged properties were already sold to
PNB as the highest bidder in a public auction. All the properties of the Panabutan,
however, were not included in said mortgage.
On December 4, 1931, Philippine National Bank sold to the plaintiff-appellant
Antonio de la Riva all the properties acquired by it in the sale of forclosed the chattel
mortgage constituted by Panabutan.
On January 13, 1932, another writ of execution was issued in favor of Ah Kee in and the
properties which were not included in the mortgage as well as those which were excluded.
Dela Riva was also required to surrender all the properties of the Panabutan in his
possession which were not covered by the mortgage and which had been attached by the
sheriff by virtue of the first writ.
ISSUE:
Whether or not PNB is bound to warrant the properties sold

SALES CASE DIGEST COMPILATION 317


HELD:

YES. The vendor is bound to deliver and warrant the thing which is the subject matter of
the sale (article 1461, Civil Code); and shall be responsible to the vendee for the legal and
peaceful possession of the thing sold and for any hidden faults or defects therein (article
1474, Civil Code) in case the vendee, by a final judgment based upon a right prior to the
sale, is deprived of the whole or of any part of the thing purchased (article 1475, Civil
Code), the warranty consisting in the restitution of the value which the thing sold had at the
time of the eviction, whether it be greater or less than the price for which it was sold (article
1478, Civil Code). The declaration of nullity of the chattel mortgage in question, with
respect to the real properties included therein, is equivalent to a final judgment that the
vendor thereof, Philippine National Bank, had no right to sell them and the purchaser,
Antonio de la Riva, acquired no right to the ownership by virtue of the sale and can,
therefore, be dispossessed thereof, which is equivalent to eviction. Consequently, the
vendor, Philippine National Bank, is bound to warrant to real properties sold by restoring to
the purchaser Antonio de la Riva the value thereof at the time of the eviction.
|

9. BARRIOS v. COURT OF APPEALS

FACTS:

Lorenza Montano was the original registered owner of a large tract of land in Barrio Mabini,
Cadiz, Negros Occidental. Before registration, the area in controversy was part of the
forestal zone. Before its release from the forestal zone, Graciano Lamis entered the same
and from then on, was known to be in continuous and uninterrupted possession and
occupation thereof. Subsequently, Montano sold her land to Barrios.
Meanwhile, Lamis sold his rights over the area in controversy to a certain Virgilio Butaz.
However, Lamis repurchased the same from Butaz. and on the same day, Lamis sold all his
rights and interests in the land to respondents Villacins without any evidence of title.
Petitioner received information that his land was being sold by Valentine Botas, whom he
erroneously thought to be the vendor, to Mayor Heracleo Villacin, Sr. of Cadiz City and the
father of herein respondents. Petitioner thru his lawyer, sent a letter of Mayor Villacin
informing the later of the true ownership of the land. Despite such letter, the mayor’s men
entered the property and threatened the encargado. This prompted the the petitioner to file a
forcible entry case against the mayor but was dismissed.
ISSUE:
Whether or not the right of the Villacins should prevail over Barrios’s
HELD:
NO. Firstly, when respondents bought the land from Lamis, the latter could not and did not at
any time produce any title or application to said land. A high degree of prudence is
required of a purchaser of registered land from one who shows a Certificate of Title but who
appears not to be the registered owner. In case the vendor could not show any title, the law
requires even more caution from a purchaser of registered land Failing to exercise
caution of any kind whatsoever, as in the case of the respondents Villacins, is tantamount to
bad faith.

SALES CASE DIGEST COMPILATION 318


Secondly, petitioner sent letters to the father of the respondents informing him of the true
ownership of the aforesaid land. Normally, sons and father would have occasion to talk
about the controversy surrounding the letters. Given that the mayor’s men entered the
property, the mayor had knowledge that his sons bought the land. Also, it is unimaginable he
withheld the information of the true ownership of the land from his sons, unless he was not
acting in their interest, which is quite remote. The children could have exercised prudence
required by law, but they did nothing but took possession of the land.
Thirdly, respondents knew of the case of forcible entry brought by petitioner against their
father, Mayor Villacin, Sr., under the impression that the latter was the vendee. Such
knowledge was a warning to them that the land they bought is subject to the claim of other
parties, but again they continued in their possession of the land and planting thereon. A
party's mere refusal to believe that a defect exists or his willful closing of his eyes to the
possibility of the existence of a defect in his vendor's title will not make him an innocent
purchaser for value, if it afterwards develops that the title was in fact defective. Similarly, a
buyer of registered land who fails to act with the diligence of a prudent man cannot be a
purchaser in good faith.

10. RFC v. JAVILLINAR

FACTS:
Consuelo Agrava Vda. de Agoncillo executed a mortgage contract over his house in favor
of Paz R. de Tubangui. The house was built on a lot which at the time of the mortgage was
not owned by the mortgagor, but which was later acquired by her. The house was then
sold in an execution sale and bought by Tubangui, who later sold it to Lucio Javillonar. In
the meantime, the said lot and its improvements were mortgaged to the Rehabilitation
Finance Corporation (RFC), and later sold at a foreclosure sale to the RFC, which took
possession of the house and collected the rentals for the same.
In the instant action, the plaintiff urged that: (a) the sale be declared valid; (2) the
mortgage agreement between the RFC and Agoncillo be declared null and void ab initio;
and, (3) the plaintiff be declared the owner of the building and free from any lien or
encumbrance whatever including that of the RFC, and that rents be accounted for and
collected from the latter. The RFC filed its answer, but defendant Agoncillo did not. Upon
the facts, the lower court declared that Tubangui's lien and right over the house were
superior to those of the RFC. In this appeal, appellant RFC maintains that the levy and
execution sale in its favor should be preferred over that of TubanguiI.

ISSUE:
Whether or not the right of Javillonar is superior to that right of RFC

HELD:

YES. It appears that the RFC was apprised of this credit before the execution of the
mortgage agreement. If the RFC had acted with that measure of diligence reasonably
expected of a prudent man in a like situation, it would have furthermore learned that
Agoncillo's debt was long overdue and Tubangui has filed a case and obtained a judgment
in her favor which was not yet satisfied.
In Leung Yee vs. F. L. Strong Machinery Co., 37 Phil. 644, 611, it was stated that a party's
mere refusal to believe that a defect exists or his willful closing of his eyes to the possibility

SALES CASE DIGEST COMPILATION 319


of the existence of a defect in his vendor's title, will not make him an innocent purchaser
for value, if it afterwards develops that the title was in fact defective, and it appears that he
had such notice of the defect as would have led to its discovery had he acted with that
measure of precaution which may reasonably be required of a prudent man in a like
situation. That previous knowledge of the RFC was equivalent to registration, thereby
making Tubangui's lien superior to the mortgage lien of the said RFC over the house.

11. FONSECA v. GUTIERREZ

FACTS:

In their complaint for specific performance against the defendants herein, the petitioners-
spouses alleged, among others, that they are the owners of a 2-door Ford Sedan by virtue
of an informal sale executed between them on June 8, 1964. Upon the lapse of the 60-day
period within which the defendant had to pay the balance, the private respondent notified
the petitioners that he had rescinded the contract of sale since the corresponding customs
duties and taxes of the said automobile had not been paid.

In his answer with counterclaim, the private respondent counter-alleged, among others, that
he purchased the petitioners' automobile with the understanding that the taxes and duties
due thereon had been lawfully and fully paid. He also discovered that the automobile was a
hot car, and the related documents were unlawfully produced. For this reason, he confronted
the petitioners with their misrepresentations and demanded the rescission of their contract.
However, the petitioners refused to do so.
ISSUE:
Whether or not the contract should be rescinded
HELD:
Yes. The failure of the petitioner to show any circumstances of this case which warrant the
defendant’s not assuming that the car he is buying is not a hot car and that all taxes have
been paid should be held against the former. As stated by the respondent, the petitioners
have elected to stand or fall exclusively on the issue of jurisdiction and improper service of
judgment. It is for this reason that the petitioner must not be excused from any liability for the
warranty imbedded in the sale. As a consequence, the contract may be rescinded and the
defendant is entitled to a refund of whatever was paid by him.

JONAH

12.SANTIAGO LAND DEVELOPMENT CORPORATION, petitioner, vs. THE HONORABLE


COURT OF APPEALS and the HEIRS OF NORBERTO J. QUISUMBING, respondents.
[G.R. No. 106194. August 7, 1997]
FACTS:
Petitioner has filed a motion for reconsideration to which private respondents, heirs of
Norberto J. Quisumbing, have filed an opposition. Petitioner has in turn filed a reply.
Petitioner maintains that, as purchaser pendente lite of the land in litigation in Civil Case No.

SALES CASE DIGEST COMPILATION 320


10513 of the Makati Regional Trial Court, entitled Norberto J. Quisumbing v. Philippine
National Bank, petitioner has a right to intervene under Rule 12, 2.
ISSUE: WON Petitioner has the right to intervene
RULING:
Petitioner points out that Sen. Vicente J. Franciscos book on the Rules of Court (Vol. 1,
page 719), which the Court cited in its decision, in turn cites Moores Federal Practice (Vol. 2,
page 2307) which actually supports petitioners right to intervene. Petitioner states:
9. Prof. Moore, in his above-cited treatise, cites among others a case decided by the
Supreme Court of California for the proposition that intervention of a purchaser pendente
lite is recognized by the U.S. courts.
The Court cited Sen. Franciscos work on the Rules of Court only for the proposition, not
disputed by petitioner, that the purpose of Rule 12, 2 on intervention is to enable a stranger
to an action to become a party to protect his interest and the court to settle in the process all
conflicting claims. Since petitioner is not a stranger in the action between Quisumbing and
the PNB, petitioner in fact having stepped into the shoes of PNB in a manner of speaking, it
follows that it cannot claim any further right to intervene in the action.
Nor do we find the cases said to be cited in Moores Federal Practice supportive of
petitioners right to intervene in this case. The first three cases (Dutcher v. Haines City
Estate, 26 F.ed 669 (CCA Fla., 1928); State ex rel. Thelen v. District Court, 17 P.2d 57, 93
Mont. 149 (S.C. Mont., 1932) and Bily v. Board of Property Assessment Appeals and
Review, 44 A.2d 250, 353 Pa. 49 (S.C. Penn. 1945)) involve purchasers pendente lite in
execution or sheriffs sales, not in voluntary transactions. The difference is important. In
voluntary sales or transactions, the vendor can be expected to defend his title because of his
warranty to the vendee. No such obligation is owed by the owner whose land is sold at
execution sale. In fact the buyer at such sales takes the property subject to the superior right
of other parties. Thus, in Dutcher v. Haines City Estates, supra, in the action brought by
Dutcher against the Haines City Estates to claim a lien on certain lands, it was held that a
bank, which subsequently obtained a judgment against the Haines City Estates for a sum of
money and bought the lands being claimed by Dutcher, was entitled to intervene. It was held
that the title of the bank is superior to any lien asserted by appellants [Dutchers and
company]. ...It is immaterial that the title was acquired by the purchaser pendente lite, as it is
valid and cannot be affected by the pending litigation.
In State ex rel. Thelen v. District Court, supra, Ke-Sun Oil Co. brought a suit to quiet title
against Sunburst Oil & Refining, Oil Well Supply and Ferdig Oil Co. Oil Well Supply, which
had a lien on a property belonging to Ferdig Oil, foreclosed the lien and the property was
sold to J.N. Thelen. It was held that J.N. Thelen should have been allowed by the lower
court to intervene in order to have his rights adjudicated. (17 P.2d at 59)
And in Bily v. Board of Property Assessment Appeals, supra, it was held that in an appeal
brought by property owners to protest an assessment, a party who foreclosed a mortgage on
one of the properties and purchased the property at sheriffs sale has a right to intervene. It
was held that the right of intervention should be accorded to any one having title to property
which is the subject of litigation, provided that his rights will be substantially affected by the
direct legal operation and effect of the decision, and provided also that it is reasonably
necessary for him to safeguard an interest of his own which no other party on record is
interested in protecting. (44 A.2d at 251)
As the purchaser in those cases did not acquire the property from their owners but adverse
to them, he could expect no party in the pending suit to safeguard his interest. Hence the
necessity of allowing his intervention. In contrast, in the case at bar, petitioner himself
considers the defenses raised by PNB, its predecessor in interest, to be formidable and all

SALES CASE DIGEST COMPILATION 321


that it desires in seeking to intervene is to fortify even more such defenses (Reply to
Opposition. p. 4).
Petitioner is thus unlike the heirs in Dizon v. Romero, 26 SCRA 452 (1968) or the purchaser
pendente lite at a sheriffs sale in Bily v. Board of Property Assessment who had to be
allowed to intervene because it was reasonably necessary for him to safeguard an interest
of his own which no other party on record is interested in protecting. (44 A.2d at 251) It is
simply petitioners perfectionism or meticulousness that makes it want to intervene to further
improve the defenses of the original party (here, PNB). But otherwise there is no reasonable
necessity for its intervention.
On the other hand the last case cited in Moores Federal Practice, Miracle House Corp. v.
Haige, supra, while involving a vendee who sought to intervene in a case in which said
vendee had an interest by virtue of a contract of sale made in its favor by one of the
vendors, does not involve a purchaser pendente lite so as to be considered on all fours with
the case at bar. He was a purchaser but not pendente lite. Hence the ruling in that case
cannot be invoked by petitioner.

13.
AGAPITO BONZON, plaintiff-appellant, vs. STANDARD OIL COMPANY OF NEW YORK
and LEONARDO OSORIO, as sheriff, defendants-appellees.
[G.R. No. L-8851 March 16, 1914]

FACTS:
This is an appeal from a judgment sustaining a demurrer to the original complaint filed in this
action. The complaint appellees, in substance, that plaintiff purchased certain real estate at
an execution sale, paying therefor the sum of P2,170 to the defendant sheriff, who turned
over the purchase price to the defendant company, the execution creditor, at whose instance
the sale was had; that thereafter, plaintiff having gone into possession of the land was
evicted therefrom in judicial proceedings, wherein the court found that the land in question
was the property of certain third parties, and that neither the judgment debtor nor the
purchaser at the execution sale had any title thereto.
The prayer of the complaint is for judgment against the judgment creditor and the sheriff for
the amount of the purchase price paid at the execution sale.
The principal contention of counsel for appellee is as follows:
The only question presented by plaintiff is the interpretation to be placed upon
section 470 of the Code of Civil Procedure. Unless this section gives him the remedy
for which he contends, he has stated no cause of action, because the only other
theory upon which he could possibly require a reinbursement of the price paid by him
at the sheriff's sale is that of an implied warranty by the judgment creditor and the
sheriff. The complaint shows no compliance, as to the Standard Oil Company, with
article 1481 of the Civil Code.
ISSUE: WON Plaintiff is authorized to recover the purchase price
RULING:
We must admit that section 470 of the code of Civil Procedure leaves something to be
desired in the way of clearness. Its English text. which of course must govern in case of any
discrepancy between the English and the Spanish versions, is as follows:
"SEC. 470. If the purchaser of real property sold on execution, or his
successor in interest, be evicted therefrom in consequence of irregularities in
the proceedings concerning the sale, or of the reversal or discharge of the

SALES CASE DIGEST COMPILATION 322


judgment, he may recover the price paid, with interest, from the judgment
creditor. If the purchaser of property at such official sale, or his successor in
interest, fail to recover possession in consequence of irregularity in the
proceedings concerning the sale, or because the property sold was not
subject to execution and sale, the court having jurisdiction thereof shall, after
notice and on motion of such party in interest, or his attorney, revive the
original judgment in the name of the petitioner, for the amount paid by such
purchaser at the sale, with interest thereon from the time of payment of the
same rate that the original judgment bore; and the judgment so revived shall
have the same force and effect as would an original judgment of the date of
the revival and no more."
Plaintiff claims to be entitled to the remedy set out in the first part of the section. It is
apparent, however, that plaintiff's eviction, as set up in his complaint, is not due to any of the
causes shown in this first part of section 470. So far as appears, 'the proceedings
concerning the sale were perfectly regular, and the complaint discloses no reversal or
discharge of the judgment' upon whose execution plaintiff became a purchaser.
The second part of section 470, applied by the judge of the trial court, seems, at first glance,
not exactly applicable, because it provides for the case where the purchaser may "fail to
recover possession," while in the present case the purchaser entered into possession and
was subsequently dispossessed by others. We think, however, that the phrase "fail to
recover possession" was intended to meet such a case as this; otherwise section 470 would
not meet a case like the present, although it was obviously intended to.
We agree with counsel for the appellee that the section of the Code in question leaves
something to be desired in the way of clearness; and it may be admitted that it is only by a
liberal construction of the language used in the statute that the sale of property under
execution in which the judgment debtor has no title, can be held to be an "irregularity in the
proceedings concerning the sale."
But we are of opinion, that the section of the Code of Civil Procedure under consideration,
being remedial in its character, should be construed liberally so as to give a remedy as
broad as that to be obtained by the corresponding suit in equity, which we think, would
extend to a case where the sale of property under execution is held to be void on the ground
that the judgment had no title.
The supreme court of California, discussing the provisions of section 708 of the California
Code of Civil Procedure in the case of Merguire vs. O'Donnell (139 Cal., 6), held as follows:
We think a sale made by a sheriff on an order of the court and void execution is
"irregular" in the extreme degree, and that a sale had on a void execution is void for
the reason of "irregularity in the proceedings concerning the sale." Section 708 of the
Code of Civil Procedure being remedial in its character, should be liberally construed.
(Hitchcock vs. Caruthers, 100 Cal., 100; Cross vs. Zane, 47 Cal., 602.) The section
under consideration was intended to give a remedy by petition in the action which
had culminated in the judgment sough to be revived. There was and is a remedy by
an independent suit in equity by which similar relief may be had as is given by the
statute (Scherr vs. Himmelman, 53 Cal., 312); an this remedy ad administered in
equity extends to cases where the execution and sale under it are both held to be
void (Smith vs. Reed, 52 Cal., 345); and, giving the section the liberal construction
required, it is clear that the remedy intended to be given under it is as board as that
to be obtained in the corresponding action in equity. It is certainly necessary and
consonant with the principles of equity that a party should have relief in cases where
the execution and sale are void as well as in those cases where there is an
irregularity of such a character as to render the sale merely voidable. Indeed, it would
seem that the requirements of equity were the same in both the supposed cases,

SALES CASE DIGEST COMPILATION 323


and there is no good reason for applying the section to one of them and not to the
other.
Upon like principles we think that where sheriff, by virtue of the authority conferred upon him
by the issuance of an execution it sell the property of the judgment debtor, undertakes to sell
and does sell property or an interest in property to which the judgment debtor is n no wise
entitled, there is certainly a grave irregularity in the procedure had under color of the
authority conferred by the execution, and it would seem that in the irregularity may fairly be
held to be an "irregularity in the proceedings concerning the sale."
No sound reason suggested itself for restricting the meaning of the language of the statute
so as to exclude there from- cases such as that under consideration. While the doctrine of
caveat emptor, relied upon by counsel for appellee, has its legitimate force and effect in
precluding any idea of a warranty by plaintiff or defendant in execution or by the sheriff, it
has no application in a case where a purchaser acquires no title to the property sold, as
distinguished from a case wherein there is only a partial failure of title; and it has been
universally held that in case of failure of title a bona fide purchaser s entitled to recover the
purchase price from the officer, if the funds are still in his hands, or from the judgment
debtor.
True it is that in some jurisdiction in the United States purchasers at execution sales where
the debtor had no title to the property sold have no cause of action against the judgment
creditors, but in others, "by judicial construction or express statutory enactment," bona fide
purchaser s given a cause of action against the execution creditors as well as the judgment
debtor n case of failure of title. See text and cases cited under heading "Right and remedies
on failure of title." (17 Cyc., 1319.) And we think that such was the intention of the legislator
in enacting the section under consideration.
In this jurisdiction (even In the absence of the statute), under the general principles that one
person may not enrich himself at the expense of another, a judgment creditor would not be
permitted to retain the purchaser price of land sold as the property of the judgment debtor
after it has been made to appear that the judgment debtor had no title to the land and that
the purchaser had failed to secure title thereto, and we find no difficulty therefore in
accepting a liberal construction of the statute which arrives at the same equitable result.
The judgment in favor of the Standard Oil Company, and the execution issued thereon, gave
to that company merely the right to have the property of the judgment debtor sold in
satisfaction of the judgment. It did not and could not give the company the right to have the
judgment satisfied out of the property of any other person.
By the tortious act of the sheriff, certain property was sold to which the judgment debtor had
no title whatever; and the proceedings concerning the sale having been found to be and the
purchaser having been evicted from the property, it s clear that the company had right under
its judgment to the proceeds of the sale, and that the sale having been held to have been
void, the purchaser at the sale is equitably entitled to the return of the purchaser price. This
is precisely the result which we hold the remedial provisions of the section under
consideration were intended to secure, and it is the result which naturally and properly
follows from a liberal construction of its terms.
We think that it will help to clear up the uncertainly as to the meaning of the different
provisions of the statute if it be kept in mind that the remedy provided in cases where "the
property sold was not subject to execution and sale" was evidently intended to include cases
wherein exempted property of the judgment debtor is sold under execution, and does not
refer to cases wherein property of third persons is tortiously seized and sold, the case of
Hitchcock vs. Caruthers (100 Cal., 100), cited in appellee's brief, to the contrary
notwithstanding. We think that the reasoning on which that decision should have been based
is that set froth in the later case of Merguire vs. O'Donnell, above cited.
The plaintiff's right to recovery from the judgment creditor not being predicated on the theory
of an express or implied warranty of title, defendant's contentions based on the provisions of

SALES CASE DIGEST COMPILATION 324


article 1481 of the code need not be considered at this time. If defendant was not given an
opportunity to be heard in the eviction proceedings, it would seem that the can avail himself
in the pending action of any defense which if set up in the former action would have relieved
him from liability to reimburse the purchaser.

14.

ASPIRAS vs GALVAN

CASE NOT FOUUND AMIDST ALL EFFORT

15.ARISTON ANDAYA, ET AL., plaintiffs-appellees, vs. DR. MELENCIO MANANSALA,


defendant-appellant.

[G.R. No. L-14714 April 30, 1960]

FACTS:

1. On June 13, 1934, Isidro Fenis sold the land in question to Eustaquia Llanes, with
right of repurchase within a period of five years. After the expiry of said period, and
without repurchasing the said property, Isidro Fenis sold it again to Maria Viloria.
Then, Maria Viloria sold by way of sale with right to repurchase within a period of one
year, the said property together with another parcel of land to defendant Melencio
Manansala. Upon the expiry of the said period, Manansala registered with the
Register of Deeds an affidavit consolidating his title on the property. A year later,
Maria Viloria sold by way of absolute sale the same property to Ciriaco Casiño,
Fidela Valdez, and the plaintiff spouses Ariston Andaya and Micaela Cabrito, for
P4,800.00.

2. Then, Llanes instituted a civil case to quiet title and recover possession from Casino.
Subsequently, Manansala sold the land to Valdez and Casino for P1500. It was
stipulated that there was a warranty (i)that said land is free from all liens and
encumbrances and (ii) in case of eviction, the vendor shall answer to the vendee in
the manner provided by law.

3. Subsequently, Llanes included Manansala, Valdez, Cabrito and Andaya as co-


defendants. The case was decided in favor of Llanes. A writ of execution was issued
and the land was sold at a public auction.

4. Andaya and Cabrito instituted a case against Manansala in the CFI of Ilocos Sur for
the recovery of damages because of the breach of warranty of title and against
eviction. Manansala denied liability and stated that his co-purchasers pleaded him to
sell to them at a low price after the case instituted by Llanes against them.

5. The lower court decided that it is inequitable to hold defendant liable under ART
1555 since plaintiffs apparently knew that the warranty could not have been
intended. And that the obligation of the defendant is that of a vendor in cases of a
rescission of contract.

ISSUE: Whether Manansala shall be liable as a vendor in cases of a rescission of a contract

HELD: NO. HE is exempt from liability.

SALES CASE DIGEST COMPILATION 325


The vendor's liability for warranty against eviction in a contract of sale is waivable and may
be renounced by the vendee (ART 1548). Not having appealed from the decision of the
lower court, appellees are bound by these findings, the implication of which is that they not
only renounced or waived the warranty against eviction, but that they knew of the danger of
eviction and assumed its consequences. Therefore, the appellant is not even obliged to
restore to them the price of the land at the time of eviction, but is completely exempt from
liability whatsoever.

ART 1544: When the vendee has waived the right to warranty in case of eviction, and
eviction shall occur, the vendor shall only pay the price which the thing sold had at the time
of the eviction, unless the vendee has made the waiver with knowledge of the danger of
eviction and assumed its consequences.

Neither may appellant be condemned to return the price received from appellees on the
theory of rescission of their contract of sale, as held by the court below. In the first place, the
remedy of rescission contemplates that the one demanding it is able to return whatever he
has received under the contract; and when this can not be done, rescission can not be
carried out (Art. 1385). It is for this reason that the law on sales does not make rescission a
remedy in case the vendee is totally evicted from the thing sold, as in this case, for he can
no longer restore the thing to the vendor. It is only when the vendee loses "a part of the thing
sold of such importance, in relation to the whole, that he would not have purchased it without
said part" that he may ask for rescission, but he has "the obligation return the thing without
other encumbrances than those which it had when he acquired it" (Art. 1556). In the second
place, appellees, as already stated, assumed the risk of eviction, which stops them from
asking for rescission even were it possible for them to restore what they had received under
the contract.

On their part, appellees claim that in view of the eviction from the land in question, they are
entitled to recover from appellant more items of damages under Article 1555 than the mere
return of the price with interests as ordered by the trial court. The claim is untenable, not
only because appellant, as we have held, is exempt from any liability for appellees eviction,
but also because not having appealed from the decision of the lower court, appellees can
not ask for a modification thereof or an award of damages not included therein.

16.

LA VINA vs VELOSO (no citation. Ruling lang makita)

Effect of waiver by vendee.(1) If the waiver was only conscious, the vendor shall pay onlythe
value which the thing sold had at the time of eviction. This isa case of solutio indebiti. The
sole effect of a waiver unaccompaniedby the knowledge and assumption of the danger of
eviction is todeprive the purchaser of the benefits mentioned in Nos. 2, 3, 4, and5 of Article
1555.

17.

DIOSDADO STA. ROMANA, PETITIONER, VS. CARLOS IMPERIO. ET AL.,


RESPONDENTS.

FACTS:
On January 6, 1943, Silvio R. Viola, hereinafter referred to as the Principal, executed in
favor of his brother, Dr. Jose P. Viola, hereinafter referred to as the Agent, a power of
attorney, Exhibit A-l, vesting in the latter the authority to take charge of, manage and
administer seven (7) parcels of registered land situated in the municipality of San Miguel,
Province of Bulacan, to be converted into a "subdivision" for residential purposes, until all of
the subdivision lots therein shall have been sold. It would seem that some of these parcels

SALES CASE DIGEST COMPILATION 326


of land, one of which was known as Lot No. 622 of the Cadastral. Survey of San Miguel,
Bulacan, were covered by Transfer Certificates of Title Nos. 19556 and 19559 of said
province.
On April 26, 1946, the Principal asked the Court of First Instance of Bulacan to order the
issuance of a second owner's duplicate of said transfer certificates of title, upon the ground
that his duplicates thereof had been lost; but on June 25, 1946, he amended the motion to
exclude therefrom TCT No. 19559, his copy thereof having been seemingly located in the
meanwhile.
Soon, later, or on June 29, 1946, the court granted said motion, as amended, and ordered
the Register of Deeds to issue a second owner's duplicate of TCT No. 19556.
Meanwhile, or on June 18, 1946, the Agent had executed, in favor of Pablo Ignacio, a deed
(Exhibit A) in which he undertook to sell on installments six (6) lots covered by said TCT No.
19556, with an aggregate area of 3,804 square meters. This instrument (Exhibit A) and the
Agent's aforementioned power of attorney (Exhibit A-l) were filed with the office of the
register of deeds and annotated on said TCT No. 19556 on July 2, 1946. This
notwithstanding, four (4) months later, or on October 18, 1946, the Principal sold a land of
about thirty (30) hectares, including said Lot No. 622 to appellant.
A week later, or on October 25, 1946, the latter, in turn, conveyed said land to the appellee,
by virtue of the deed Exhibit C, which was filed with the office, of the Register of Deeds on
November 4, 1946. Thereupon, TCT No. 19556 was cancelled and, in lieu thereof, TCT No.
28946 issued in appellee's name.
On December 14, 1946, appellee sold portions of said let No. 622 to the following persons,
hereinafter referred to as occupants, who had been and were holding, as lessees thereof,
the portions respectively purchased by them, to wit:
a. 665 sq. m. to Domingo Manabat, Patricia Lopez and Calixta Bautista (to whom TCT
No. T-1635 was issued) (Exhibit D) ;
b. 600 sq. m. to Conrado Manabat and Eladio Sioson (to whom TCT No. T-1634 was
issued);
c. Lot No. 14 of Block 13 of the subdivision to Eeynaldo Salvador and Graciano Garcia
(to whom TCT No. T-1633 was issued); and
d. 682 sq. m. to Hilario de Jesus, Apolonio Pablo and Ismaela
Jimenez (to whom TCT No. T-1632 was issued).
Having failed to take possession of the land sold to him by the Agent, on April 22, 1947,
Pablo Ignacio commenced hi action in the Court of First Instance of Bulacan, against said
occupants, as well as against appellee, appellant, and the Principal, to annul the sales made
by the latter to appellant, by appellant to appellee and by appellee to said occupants, as well
as for the possession of the land in question and damages.
On May 7, 1947, a pleading was filed, purporting to be defendants' answer, alleging that
appellee had, in good faith and for value, purchased said land from appellant in whose name
the title to said land was, free from any lien or encumbrance in favor of Ignacio; that the
occupants had purchased the portions assigned to them by appellee under similar
conditions; that the sale in favor of Ignacio was fraudulent; and that Ignacio knew that said
occupants were in possession of said portions, and had a right of pre-emption thereto.
On June 20, 1947, the Principal filed his own answer alleging that the land conveyed by him
to Ignacio is different from the one covered by the sale made by the Agent to appellant and
that he (the Principal) had instituted Civil Case No. 137 of the Court of First Instance of
Bulacan against appellant to annul the aforementioned sale by the Agent.

SALES CASE DIGEST COMPILATION 327


On January 5, 1949, appellee, and said occupants filed a cross-claim against the Principal,
the Agent and appellant herein. On January 24, 1949, said occupants filed against the
appellee, a cross-claim which was amended on October 17, 1949.
The lower court rendered judgment: (I) declaring that Ignacio is the owner in fee simple of
the lots in question; (II) ordering the Principal to execute the corresponding deed of final sale
thereof to Ignacio; (III) directing appellee to surrender to the register of deeds of Bulacan the
owner's duplicate of TCT No. 28948 for its cancellation and the issuance, in lieu thereof, of
another certificate in the name of Ignacio; (IV) ordering the above-mentioned occupants to
similarly surrender to said register of deeds their respective certificates of title for
cancellation thereof; and (V) sentencing:
(a) appellee to refund P2,161.25 to Domingo Manabat, Patricia Lopez
and Calixta Batitista; P1,950 to Conrado Manabat and Eliodoro Jimenez;
P2,216.50 to Hilario de Jesus and Apolonio and Ismaela Jimenez; and
P2,135.25 to Reynaldo Salvador and Graciano Garcia; and (b) the
Principal to pay Ignacio 25% of P6,457.00 representing the price of
materials purchased by the latter for the construction of a movie house in
order to compensate for the deterioration and depreciation of said
materials, plus P5,000, by way of damages, with costs against the
defendants.

Appellee and the occupants appealed from this judgment, which was affirmed by the Court
of Appeals
On motion for reconsideration filed by appellee, the Court of Appeals rendered an amended
decision ordering appellant to reimburse the appellee in the sum of P8,463.00, representing
the aggregate amount to be refunded by him (appellee) to the aforementioned occupants,
pursuant to the original decision. Appellant maintains that the Court of Appeals has erred;
(1) in taking cognizance of this case; (2) in amending its original decision without giving him
a chance to answer appellee's motion for reconsideration and in entertaining appellee's
"supposed" cross-claim; and (3) in sustaining the same, despite the fact that apellee was in
in pari delicto.
ISSUE: WON the apellee is entitled to reimbursement
RULING:
Appellant alleges in support of the third alleged error that appellee had never filed a cross-
claim against him and that, at any rate, appellee is not entitled to reimbursement from him
because they are in part delicto.
Although it is true that the appellee has not filed a cross-claim against the appellant, it is a
fact that the occupants had filed a cross-claim against both of them; and that, upon payment
to the occupants of the amount of the cross-claim adjudged to be due them, the appellee
becomes subrogated into their rights, under said cross-claim, against the appellant.
Moreover, it is an elementary principle of law (Articles 1495, 1547 and 1555, Civil Code of
the Philippines), as well as of justice and equity, that, unless a contrary intention appears,
the vendor warrants his title to the thing sold, and that, in the event of eviction, the vendee
shall be entitled to the return of the value which the thing sold had at the time of the eviction,
be it greater or less than the price of the sale. In the case at bar, it has been established that
the land in dispute was, at the time of the eviction worth at least the sum of P83463, which is
the aggregate amount charged by the appellee from said occupants.
Appellant cites Article 1412 of the Civil Code of the Philippines, in support of the view that
appellee may not recover said amount from appellant, upon the ground that both are in pari
delicto. This provision is part of Title II of Book IV of the Civil Code, on contracts in general
and it refers to contract which are null and void ab initio, pursuant to Article 1409 of the Civil

SALES CASE DIGEST COMPILATION 328


Code. The contract between appellant and appellee does not fall, however, under this
provision, and is, accordingly, beyond the purview of the aforementioned Article 1412. Said
contract is governed by Title VI of the same Book, on Sales in particular, specially by the
aforesaid Articles 1495, 1547 and 1555, which are part of said Title VI, regarding breach of
the warranty arising from avalid contract of sale, due to the application of Art 1544 of the
same title, regulating the effects of double sales. Incidentally, these provisions suggest, also,
the remedies available to appellant herein.
18.

ENGRACIO ORENSE, plaintiff-appellee, vs. CIRILIO JAUCIAN, defendant-appellant.


[G.R. No. L-5619 March 11, 1911]
FACTS:
This is an appeal from a judgment of the Court of First Instance of the Province of Albay,
Hon Grant T. Trent presiding, in favor of the plaintiff and against the defendant for the sum
of P2,042.42, and costs.
On or about the 20th day of April, 1902, the appellant, by written instrument, duly executed,
ratified the sale to the plaintiff of a certain house and parcel of land which had been
tentatively made on the 6th day of October, 1898. By said writing the vendor warranted the
title to said premises and agreed to protect the purchaser in his possession.
Later the plaintiff was dispossessed of a part of said land by virtue of a final judgment in an
action prosecuted against him by Mariano Perfecto. The appellant in this case was duly
notified of the pendency of said action and, while he did not intervene as a suitor, he urged
the plaintiff in this case to defend the action and prosecuted an appeal from the judgment
rendered by the trial court.
ISSUES:
1. WON the court erred in fixing P650 as the value of that portion of the land lost by the
plaintiff.
2. WON the court erred in assessing as damages against the appellant the sum of P325,
fees of stenographers and an interpreter who served on the trial by interpreting and
transcribing the record of the action in which was entered the judgment depriving plaintiff in
this case of a portion of his land.
3. WON the court erred in including in its judgment the sum of P70, being expenses of
travel, etc., inccurred by the plaintiff in the action which caused the loss of his land.
4. WON The court erred in including in the judgment the sum of P850 as lawyer's fees in the
said action in which plaintiff lost said part of his property.
RULING:

As to the first issue, it appears that the plaintiff lost about two thirds or three-quarters of the
land purchased from the appellant. The appellant, in his testimony, placed the value of such
portion at about P700. A witness for the plaintiff testified that the value thereof was about
P600. The learned trial court found the correct value to be P650. There is no evidence
whatever in the record contradicting the evidence upon which this finding was made.
As to the second issue, the appellant urges that these expenses do not fall within any of the
provisions of article 1478 of the Civil Code and therefore ought not to have been allowed by
the trial court. It is not necessary to determine whether this contention is sound or not. It
appears uncontradicted in the record that the translation and transcription of the record and
the payment of P325 therefor was made at the request of the appellant and on his promise
to reimburse the plaintiff. In fact, it appears that the appellant induced the plaintiff to take the

SALES CASE DIGEST COMPILATION 329


appeal on which said record was to be used. From these facts it is clear that the allowance
of 325 was proper.
As to the third issue, the appellant asserts that such expenses are not a part of the costs as
defined in the Civil Code, article 1478, and in sections 487 to 492 of the Code of Civil
Procedure. In this the appellant is partly right. Section 492 of the Code of Civil Procedure
reads:
• SEC. 492. Costs in Courts of First Instance. — In an action pending in a Court of
First Instance, the prevailing party may recover the following costs, and no others:
• For the complaint or answer, eight pesos.
• For his own attendance, and that of his lawyer, down to and including final judgment,
twenty pesos.
• For each witness necessarily produced by him, for each day's necessary attendance
of such witness at the trial, one peso, and his lawful traveling fees.
• For each deposition lawfully taken by him, and produced in evidence, five pesos.
• For original documents, deeds, or papers of any kind produced by him, nothing.
• For official copies of such documents, deeds, or papers, the lawful fees necessarily
paid for obtaining such copies.
• The lawful fees paid by him for the service of any process in the action, and all lawful
clerk's fees paid by him."
From these provisions it is clear that the prevailing party is entitled to only P20 for the
attendance of himself and lawyer. No provision is made for expenses of travel or other
expenses of that nature. Bad faith in making the sale to the plaintiff not having been shown,
these expenses can not be allowed as damages under paragraph 5 of article 1478
aforesaid. The contention of the appellant should, therefore, be allowed, at least to the
extent of P50. But inasmuch as we shall later allow the other P20 as lawyer's fees, we now
disallow the said sum of P70 included by the learned trial court in the judgment appealed
from.
As to the last issue, we are of the opinion that the appellant is partly right in this contention.
As we have already seen from section 492 of the Code of Civil Procedure, the only
allowance for lawyer's fees to the prevailing party in an action in the Court of First Instance is
the sum of P20. Section 489 of that Code provides as follows:
• SEC. 489. Lawyer's fees as costs. — No lawyer's fees shall be taxed as costs
against the adverse party, except as herein specially provided. But this section shall
have no relation to the fees to be charged by a lawyer as against his client."
Section 494 reads in part:
• SEC. 494. Costs in Supreme Court. — In an action pending in the Supreme Court,
the prevailing party may recover the following costs, and no others:
• For his own attendance, and that of his lawyers, down to and including final
judgment, forty pesos.
This court has repeatedly held that the word "costs," as used in the law, includes no lawyer's
fees except those fees specifically prescribed therein. Therefore, the only sums to which the
plaintiff is entitled as lawyer's fees are P20 in the Court of First Instance and P40 on appeal.
It follows that the learned trial court should have included in his judgment the sum of P60 as
lawyer's fees instead of P850.
These are the only modifications which are required by the facts of record.

SALES CASE DIGEST COMPILATION 330


With the modification that the judgment against the appellant is the sum of P1,182.42
instead of P2,042.42, the judgment appealed from is affirmed, without special finding as to
costs of appeal.

19. ESQUIVEL vs PALIZA 13 car[2s] 1120 : CASE NOT FOUUND AMIDST ALL EFFORT

20. Yson vs rivera 5 car[2s] 779: CASE NOT FOUUND AMIDST ALL EFFORT

21.
ANASTACIO PINEDA, Plaintiff-Appellee, vs. MARGARITA SANTOS, in her own behalf
and as judicial administratrix of the intestate estate of Natalio Santos, Defendant-
Appellant.
[ March 3, 1932]
FACTS:
On July 14, 1925, appellant and appellee executed Exhibit D whereby the former bound
herself to sell to the latter 213,370 square meters of calcareous land in the municipality of
Montalban, Province of Rizal, belonging to the late Natalio Santos, whose judicial
administratrix she was, for the sum of P8,000, the contract to be subject to the approval of
the proper probate court.
According to Exhibit C the appellee made a part payment of P500 to the appellant. Having
obtained the judicial approval on the 30th of that month, the parties executed the deed
Exhibit A, whereby the appellant sold the land in fee simple to the appellee at the price
stipulated, receiving a check for the remainder of the agreed price, P7,500.
The appellee entrusted the draughting of the deed of sale to his lawyer, Lorenzo Sunico. In
the deed, the appellant expressly stated that she was the owner in fee simple of the
property, and that she conveyed it "free of all charge, lien, and liability."
At that time the property was registered under the Torrens system and the transfer certificate
of title No. 8986 was issued in favor of one Gabriel Cruz who had previously executed
another absolute deed of sale in favor of the late Natalio Santos, but as it had not been
presented for registration, no new transfer certificate was issued.
On that occasion the appellant exhibited the said transfer certificate to the notary public; the
latter as well as the appellee examined it and saw that it bore no annotation of any
encumbrance, or of the lis pendens.
After the deed had been signed, Attorney Sunico took it with him, together with the transfer
certificate of title, to the office of the registrar of deed for the Province of Rizal, in order to
have the first recorded and the second cancelled, and a new transfer certificate issued in
favor of the appellee.
When the lawyer went back to said office to get those documents he was surprised to see
that the lis pendens notice had been entered on the transfer certificate of title as of April 25,
1925, 8.16 a.m. He spoke to registrar Jose Tupas, and upon asking him why he had made
the annotation with an earlier date, he was told that one A. A. Addison had filed a lis
pendens notice on April 24, 1925, and that, having noted the 25th of said month on the
original certificate of title to the land, he had to do so on the transfer certificate as well.
Sunico left the office and returning to his own, informed his client, the appellee, of what had
happened, who thereupon took steps to have the deed of sale set aside, and later brought
an action to nullity.

SALES CASE DIGEST COMPILATION 331


Some time before the discovery of the lis pendens notice, the appellee had conveyed the
same property at the same price to the Pineda, Ampil Manufacturing & Co., Inc., but when
the latter found that there was such a notice recorded, it sought and obtained the rescission
of the sale.
While the appellee was in possession of the land and before he transferred it and brought
the action for nullity, he was engaged in the lime business for which he had purchased the
property, and spent the sum of P2,334.30 which is one of the sums awarded to him by the
court.
ISSUES: Whether the lis pendens notice appearing on the transfer certificate of title was
entered after the consummation of the contract of sale
Whether there was fraud in the contract vitiating the appellee's consent.
RULING:
We have carefully examined all the evidence and arrived at the conclusion that said
annotation was entered on the transfer certificate of title after the execution of the deed of
sale, and that the appellant's statement in the deed to the effect that the property was
absolutely free of all liens and encumbrances was untrue and therefore misled the appellee
into giving his consent. This conclusion is borne out both by the testimony of the appellee
and Attorney Sunico, and by the admissions of the registrar of deeds, Jose Tupas. The latter
plainly admitted during his testimony that he entered the lis pendens notice upon the transfer
certificate of title many days after the date therein set forth, probably after 1st of May, and
from this it may be reasonably inferred that this official did not exactly remember the correct
date of the annotation, which leads us to believe that the testimony of Attorney Sunico and
the appellee was not altogether improbable when they said that such annotation was made
after July 30, 1925, and only when t he transfer certificate of title was presented at the office
of the registrar together with the deed of sale.
A notice of the pendency of an action in accordance with section 79 of Act No. 496,
applicable to the case in question, constitutes a legal lien and the result of the litigation
referred to affects the parties thereto and all who might have any interest in the lands which
are the subject of such action. (Section 79, Act No. 496; Atkins, Kroll & Co. vs. Domingo, 46
Phil., 362.) The present case must not be governed by the provisions of articles 42 and 43 of
the Mortgage Law, invoked by the appellant.

22. PERALTA vs JORDANA ENTERPRISES INC 8 CAR[2s] 270: CASE NOT FOUUND
AMIDST ALL EFFORT

23.
CHANG YONG TEK, PLAINTIFF AND APPELLEE, VS. GENEROSA SANTOS,
DEFENDANT AND APPELLANT.
[ G.R. NO. 4386, FEBRUARY 24, 1909 ]

FACTS:
The plaintiff sold to the defendant a certain quantity of tobacco, amounting in all to the sum
of P894, P442 of the said amount was to be paid at the end of January, 1904, and that P452
of said amount was to be paid at the end of the month of September, 1905. No part of the
said amount of P894 having been paid, the plaintiff commenced an action on the 24th of
January, 1907, for its recovery.
The only defense presented by the defendant was that the tobacco delivered by the plaintiff
was not of good quality.The record does not show that the defendant had made any

SALES CASE DIGEST COMPILATION 332


complaint to the plaintiff concerning the quality of the tobacco or that it was not the kind of
tobacco which she had purchased, until after the present action had been commenced.
Neither does the record disclose that the plaintiff made any false representations with
reference to the quality or kind of tobacco sold.There is no attempt to show that the plaintiff
undertook to warrant the quality of the tobacco.
ISSUE:
Whether or not the buyer is liable for the price of the thing delivered?
RULING:
In the absence of an express warranty, a vendor of merchandise only warrants;
First. The legal and peaceable possession of the thing sold; and
Second. That there are no hidden faults or defects therein. (Art. 1474, Civil Code.)
It not being proven that the plaintiff made any warranty or any misrepresentations with
reference to the quality of the tobacco in question, and it having been proven that the
defendant had an opportunity to and did examine the tobacco in question at the time of
purchase and not having made any objection whatever' until after a lapse of more than three
years and not then until after an action had been brought, and making no objection whatever
as to the price agreed upon, nor as to the quantity of the tobacco delivered, in our opinion
she should be held liable for the payment of the amount agreed upon.

XXX. CONVENTIONAL REDEMPTION

1.
KINGS PROPERTIES CORP., VS. CANUTO A. GALIDO
FACTS: This case involves an action for cancellation of certificates of title, registration of
deed of sale and issuance of certificates of title filed by Canuto A. Galido before the RTC of
Antipolo City. On April 18, 1966, the heirs of Eniceo, namely Rufina and Maria Eniceo, were
awarded with Homestead Patent consisting of four parcels of land located in San Isidro,
Antipolo, Rizal. The Antipolo property with a total area of 14.8882 hectares was registered
under OCT No. 535.

The issuance of the homestead patent was subject to the following conditions:

“To have and to hold the said tract of land, with the appurtenances thereunto of right
belonging unto the said Heirs of Domingo Eniceo and to his heir or heirs and assigns
forever, subject to the provisions of sections 118, 121, 122 and 124 of Commonwealth Act
No. 141, as amended, which provide that except in favor of the Government or any of its
branches, units or institutions, the land hereby acquired shall be inalienable and shall not be
subject to incumbrance for a period of five (5) years next following the date of this patent,
and shall not be liable for the satisfaction of any debt contracted prior to the expiration of that
period; that it shall not be alienated, transferred or conveyed after five (5) years and before
twenty-five (25) years next following the issuance of title, without the approval of the
Secretary of Agriculture and Natural Resources; that it shall not be incumbered, alienated, or
transferred to any person, corporation, association, or partnership not qualified to acquire
public lands under the said Act and its amendments; x x x”

On September 1973, a deed of sale covering the Antipolo property was executed between
Rufina Eniceo and Maria Eniceo as vendors and respondent as vendee. The property was

SALES CASE DIGEST COMPILATION 333


sold to respondent for P250,000. A certain Carmen Aldana delivered the owner's duplicate
copy of OCT No. 535 to respondent.

On 1988, the Eniceo heirs registered with the Registry of Deeds a Notice of Loss of the
owner's copy of OCT No. 535.

RTC rendered a decision finding that the certified true copy of OCT No. 535 contained no
annotation in favor of any person, corporation or entity. The RTC ordered the Registry of
Deeds to issue a second owner's copy of OCT No. 535 in favor of the Eniceo heirs and
declared the original owner's copy of OCT No. 535 canceled and considered no further
value.

Petitioner states that as early as 1991, respondent knew of the RTC decision because
respondent filed a criminal case against Rufina Eniceo and Leonila Bolinas for giving false
testimony upon a material fact during the trial. They alleged that sometime in 1995, Bolinas
came to the office of Alberto Tronio Jr., petitioner's general manager, and offered to sell the
Antipolo property. During an on-site inspection, Tronio saw a house and ascertained that the
occupants were Bolina's relatives. Tronio also went to the Registry of Deeds to verify the
records on file and ascertained that OCT No. 535 was clean and had no lien and
encumbrances. After the necessary verification, petitioner decided to buy the Antipolo
property.

On March 20, 1995, the Eniceo heirs executed a deed of absolute sale in favor of petitioner
covering lots 3 and 4 of the Antipolo property for P500,000.00.

On August 17, 1995, the Secretary of Department of Environment and Natural Resources
(DENR Secretary) approved the deed of sale between the Eniceo heirs and respondent. On
January 1996, respondent filed a civil complaint with the trial court against the Eniceo heirs
and petitioner praying for the cancellation of the certificates

of title issued in favor of petitioner, and the registration of the deed of sale and issuance of a
new transfer certificate of title in favor of respondent.

The trial court rendered it's decision dismissing the case for lack of legal and factual basis.
However, the CA reversed the trial court's decision.

ISSUE: Whether the deed of sale delivered to respondent should be presumed an equitable
mortgage pursuant to Article 1602(2) and 1604 of the Civil Code.

HELD: Validity of the deed of sale to respondent

The contract between the Eniceo heirs and respondent executed on 10 September 1973
was a perfected contract of sale. A contract is perfected once there is consent of the
contracting parties on the object certain and on the cause of the obligation. In the present
case, the object of the sale is the Antipolo property and the price certain is P250,000.

The contract of sale has also been consummated because the vendors and vendee have
performed their respective obligations under the contract. In a contract of sale, the seller
obligates himself to transfer the ownership of the determinate thing sold, and to deliver the
same to the buyer, who obligates himself to pay a price certain to the seller. The execution
of the notarized deed of sale and the delivery of the owners duplicate copy of OCT No. 535
to respondent is tantamount to a constructive delivery of the object of the sale.

Petitioner invokes the belated approval by the DENR Secretary, made within 25 years from
the issuance of the homestead, to nullify the sale of the Antipolo property. The sale of the
Antipolo property cannot be annulled on the ground that the DENR Secretary gave his
approval after 21 years from the date the deed of sale in favor of respondent was executed.

SALES CASE DIGEST COMPILATION 334


Equitable Mortgage

Petitioner contends that the deed of sale in favor of respondent is an equitable mortgage
because the Eniceo heirs remained in possession of the Antipolo property despite the
execution of the deed of sale.

An equitable mortgage is one which although lacking in some formality, or form or words, or
other requisites demanded by a statute, nevertheless reveals the intention of the parties to
charge real property as security for a debt, and contains nothing impossible or contrary to
law. The essential requisites of an equitable mortgage are:

1. The parties entered into a contract denominated as a contract of sale; and


2. Their intention was to secure existing debt by way of a mortgage.

Petitioner claims that an equitable mortgage can be presumed because the Eniceo heirs
remained in possession of the Antipolo property. Apart from the fact that the Eniceo heirs
remained in possession of the Antipolo property, petitioner has failed to substantiate its
claim that the contract of sale was intended to secure an existing debt by way of
mortgage. In fact, mere tolerated possession is not enough to prove that the
transaction was an equitable mortgage.

Furthermore, petitioner has not shown any proof that the Eniceo heirs were indebted to
respondent. On the contrary, the deed of sale executed in favor of respondent was drafted
clearly to convey that the Eniceo heirs sold and transferred the Antipolo property to
respondent. The deed of sale even inserted a provision about defrayment of registration
expenses to effect the transfer of title to respondent.

The Court notes that the Eniceo heirs have not appealed the CAs decision, hence, as to the
Eniceo heirs, the CAs decision that the contract was a sale and not an equitable mortgage is
now final. Since petitioner merely assumed the rights of the Eniceo heirs, petitioner is now
estopped from questioning the deed of sale dated 10 September 1973.

2.

SPS. CARLOS AND EULALIA RAYMUNDO and SPS. ANGELITO AND JOCELYN
BUENAOBRA, Petitioners, vs. SPS. DOMINADOR and ROSALIA BANDONG,
Respondents.

FACTS:

Eulalia Raymundo was engaged in the business of buying and selling cattle.
Dominador Bandong worked as a biyahero for Raymundo for 30 years. Because of his long
years of service without any derogatory record, Raymundo no longer required him to post
any security to perform his duties.
Subsequently, Bandong incurred shortage in his cattle procurement operation costing
P70,000
Bandong executed a Deed of Sale in favor of Raymundo on a 96 sq m parcel of land in
Caloocan City which he owned
Raymundo sold said property to Buenaobra.
Buenaobra instituted an action for ejectment against Bandong. Bandong alleged that there
was no sale intended but only equitable mortgage to secure the shortage incurred by
Bandong to Raymundo.
Buenaobra alleged that she was a buyer in GF.

SALES CASE DIGEST COMPILATION 335


RTC ruled in favor of Raymundo, declaring that the Deed of Sale was valid and binding. CA
reversed, declaring that the transaction was an equitable mortgage, not a sale.
ISSUE: WON the parties entered into a Contract of Sale or Equitable Mortgage? ~ Equitable
Mortgage
HELD:
The transaction between Raymundo and Bandong was an equitable mortgage, not a sale.
Thus, Bandong can redeem the subject property.
An equitable mortgage is one that - although lacking in some formality or other requisites
demanded by a statute, nevertheless reveals the intention of the parties to charge a real
property as security for a debt and contains nothing impossible or contrary to law.
The instances when a contract, regardless of its nomenclature, may be presumed to be an
equitable mortgage are enumerated in the Civil Code as follows:
Art. 1602. The contract shall be presumed to be an equitable mortgage, in any of the
following cases:
(1) When the price of a sale with right to repurchase is unusually inadequate;
(2) When the vendor remains in possession as lessee or otherwise;
(3) When upon or after the expiration of the right to repurchase another instrument extending
the period of redemption or granting a new period is executed;
(4) When the purchaser retains for himself a part of the purchase price;
(5) When the vendor binds himself to pay the taxes on the thing sold.
(6) In any other case where it may be fairly inferred that the real intention of the parties is
that the transaction shall secure the payment of a debt or the performance of any other
obligation.
Art. 1604. The provisions of Article 1602 shall also apply to a contract purporting to be an
absolute sale.
For Articles 1602 and 1604 to apply, two requisites must concur
1. The parties entered into a contract denominated as a contract of sale
2. Their intention was to secure an existing debt by way of an equitable mortgage.
The decisive factor in evaluating such agreement is the intention of the parties, as shown not
necessarily by the terminology used in the contract but by all the surrounding circumstances.
Applying such principle, the SC ruled that in executing the said Deed of Sale, Raymundo
and Bandong never intended to transfer ownership of the subject property but to burden the
same with an encumbrance to secure the indebtedness incurred by Dominador on the
occasion of his employment with Eulalia.
Eulalia admitted that it was her customary business practice to require her biyaheros to
deliver the titles to their real properties and execute in her favor the corresponding deeds of
sale as security for the money she provided for their cattle procurement task. This defeats
the contention that Dominador easily ceded his property as payment for his obligation.
The explicit provision of Article 1602 that any of those circumstances would suffice to
construe a contract of sale to be one of equitable mortgage is in consonance with the rule
that the law favors the least transmission of property rights.
The existence of any one of the conditions under Article 1602, not a concurrence, or an
overwhelming number of such circumstances, suffices to give rise to the presumption that
the contract is an equitable mortgage.

SALES CASE DIGEST COMPILATION 336


Said transaction was not void in its entirety but an equitable mortgage, thereby merely
altering the relationship of the parties from seller and buyer, to mortgagor and mortgagee,
while the subject property is not transferred but subjected to a lien.
Furthermore, Buenaobra cannot be a buyer in GF. She admitted that she was aware that
Dominador and a certain Lourdes were in possession of the subject property.
As regards the belated filing by Bandong of an action to annul the sale, the SC held that the
one who is in actual possession of a piece of land claiming to be the owner thereof may
await to vindicate his right.
Petition DENIED.

SALES CASE DIGEST COMPILATION 337

Вам также может понравиться